Esercizi di Elettromagnetismo e Elettrodinamica

empty
Esercizi di Elettromagnetismo e
Elettrodinamica
R. Pettorino, A. Porzio, S. Solimeno
Dipartimento di Scienze Fisiche
Università di Napoli “Federico II
Queste note sono sottoposte a continue revisioni al fine di rimuovere errori e chiarire
passaggi e sviluppi spesso contorti. Di ciò gli autori si scusano con gli studenti confermando la disponibilità a chiarire attraverso contatti personali esercizi e metodi di calcolo
non sufficientemente chiari.
4
Indice
1 Metodi matematici della Elettrodinamica
1.1 Identità vettoriali . . . . . . . . . . . . . . . . . .
1.2 Operatori differenziali . . . . . . . . . . . . . . .
1.3 Distribuzioni . . . . . . . . . . . . . . . . . . . . .
1.4 Trasformate integrali . . . . . . . . . . . . . . . .
1.4.1 Trasformata di Fourier . . . . . . . . . . .
1.4.2 Trasformate di Fourier-Bessel . . . . . . .
1.4.3 Trasformata di Hilbert . . . . . . . . . . .
1.5 Integrali . . . . . . . . . . . . . . . . . . . . . . .
1.5.1 Valutazione asintotica . . . . . . . . . . .
1.5.1.1 Metodo SP della fase stazionaria
1.5.1.2 Metodo SD dello steepest descent
1.6 Metodo WKB . . . . . . . . . . . . . . . . . . . .
1.6.1 Equazione di Helmholtz . . . . . . . . . .
1.6.2 Equazione dei raggi . . . . . . . . . . . . .
1.7 Funzioni speciali . . . . . . . . . . . . . . . . . .
1.7.1 Funzioni di Bessel, Hankel e Struve . . . .
1.7.2 Funzioni di Mathieu . . . . . . . . . . . .
1.7.3 Funzione di Airy . . . . . . . . . . . . . .
1.7.4 Integrali ellittici . . . . . . . . . . . . . . .
.
.
.
.
.
.
.
.
.
.
.
.
.
.
.
.
.
.
.
2 Richiami di teoria della relatività
2.1 4-Vettori e 4-tensori dello spazio-tempo . . . . . . .
2.1.1 Trasformazioni di Lorentz . . . . . . . . . .
2.1.2 Riferimento proprio . . . . . . . . . . . . . .
2.2 4-Tensori elettromagnetici . . . . . . . . . . . . . .
2.3 Mezzi in movimento . . . . . . . . . . . . . . . . .
2.4 Invarianti del campo e.m. . . . . . . . . . . . . . .
2.5 4-Tensore degli sforzi elettromagnetici . . . . . . . .
2.6 4-vettore d’onda . . . . . . . . . . . . . . . . . . . .
2.7 4-vettore quantità di moto . . . . . . . . . . . . . .
2.7.1 Effetto Compton . . . . . . . . . . . . . . .
2.7.2 Comptonizzazione . . . . . . . . . . . . . . .
2.7.3 Effetto Sunayev-Zeldovich . . . . . . . . . .
2.7.4 Effetto Mössbauer . . . . . . . . . . . . . . .
2.8 Processi di diffusione . . . . . . . . . . . . . . . . .
2.8.1 Trasformazioni sezioni d’urto differenziali . .
2.8.2 Riferimenti utilizzati nello scattering elastico
5
.
.
.
.
.
.
.
.
.
.
.
.
.
.
.
.
.
.
.
.
.
.
.
.
.
.
.
.
.
.
.
.
.
.
.
.
.
.
.
.
.
.
.
.
.
.
.
.
.
.
.
.
.
.
.
.
.
.
.
.
.
.
.
.
.
.
.
.
.
.
.
.
.
.
.
.
.
.
.
.
.
.
.
.
.
.
.
.
.
.
.
.
.
.
.
.
.
.
.
.
.
.
.
.
.
.
.
.
.
.
.
.
.
.
.
.
.
.
.
.
.
.
.
.
.
.
.
.
.
.
.
.
.
.
.
.
.
.
.
.
.
.
.
.
.
.
.
.
.
.
.
.
.
.
.
.
.
.
.
.
.
.
.
.
.
.
.
.
.
.
.
.
.
.
.
.
.
.
.
.
.
.
.
.
.
.
.
.
.
.
.
.
.
.
.
.
.
.
.
.
.
.
.
.
.
.
.
.
.
.
.
.
.
.
.
.
.
.
.
.
.
.
.
.
.
.
.
.
.
.
.
.
.
.
.
.
.
.
.
.
.
.
.
.
.
.
.
.
.
.
.
.
.
.
.
.
.
.
.
.
.
.
.
.
.
.
.
.
.
.
.
.
.
.
.
.
.
.
.
.
.
.
.
.
.
.
.
.
.
.
.
.
.
.
.
.
.
.
.
.
.
.
.
.
.
.
.
.
.
.
.
.
.
.
.
.
.
.
.
.
.
.
.
.
.
.
.
.
.
.
.
.
.
.
.
.
.
.
.
.
.
.
.
.
.
.
.
.
.
.
.
.
.
.
.
.
.
.
.
.
.
.
.
.
.
.
.
.
.
.
.
.
.
.
.
.
.
.
.
.
.
.
.
.
.
.
.
.
.
.
.
.
.
.
.
.
.
.
.
.
.
.
.
.
.
.
.
.
.
.
.
.
.
.
.
.
.
.
.
.
.
.
.
.
.
.
.
.
.
.
.
.
.
.
.
.
.
.
.
17
17
17
21
23
23
23
24
25
25
27
27
29
29
30
31
31
34
35
35
.
.
.
.
.
.
.
.
.
.
.
.
.
.
.
.
37
37
38
44
45
50
51
51
54
55
56
58
59
59
60
60
60
6
Indice
2.8.3 Spazio delle fasi . . . . .
2.9 Moto dello spin . . . . . . . . .
2.10 Campi e.m. in sistemi rotanti .
2.10.1 Effetto Sagnac . . . . . .
2.11 Propagazione in spazi curvi . .
2.11.1 Metrica di Schwarzshild
.
.
.
.
.
.
.
.
.
.
.
.
.
.
.
.
.
.
.
.
.
.
.
.
.
.
.
.
.
.
.
.
.
.
.
.
.
.
.
.
.
.
.
.
.
.
.
.
.
.
.
.
.
.
.
.
.
.
.
.
.
.
.
.
.
.
.
.
.
.
.
.
3 Interazioni elettriche e magnetiche
3.1 Riepilogo grandezze elettriche e magnetiche . . . . .
3.2 Interazioni elettrostatiche . . . . . . . . . . . . . . .
3.2.1 Multipoli . . . . . . . . . . . . . . . . . . . .
3.2.2 Interazione elettroni-nucleo . . . . . . . . . . .
3.2.3 Interazione elettrone-elettrone in un atomo . .
3.2.4 Interazione elettrone-elettrone in una molecola
3.2.5 Elettroni di un metallo . . . . . . . . . . . . .
3.3 Interazioni nei cristalli . . . . . . . . . . . . . . . . .
3.3.1 Campo locale . . . . . . . . . . . . . . . . . .
3.3.2 Interazioni in dielettrici non omogenei . . . .
3.4 Forze di van der Waals . . . . . . . . . . . . . . . . .
3.5 Interazioni magnetiche . . . . . . . . . . . . . . . . .
3.5.1 Bobine di Helmholtz . . . . . . . . . . . . . .
3.5.2 Magnetismo nei solidi . . . . . . . . . . . . . .
3.5.3 Materiali diamagnetici . . . . . . . . . . . . .
3.5.4 Materiali ferromagnetici . . . . . . . . . . . .
3.5.5 Equazioni di Bloch . . . . . . . . . . . . . . .
3.5.6 Approssimazione di onda rotante . . . . . . .
3.5.7 Accoppiamento spin nell’NMR . . . . . . . . .
4 Onde elettromagnetiche
4.1 Teoremi di reciprocità . . . . .
4.2 Teorema dell’energia . . . . . .
4.3 Onde piane . . . . . . . . . . .
4.3.1 Funzione di Green . . .
4.4 Focalizzazione onde piane . . .
4.5 Tecnica SNOM . . . . . . . . .
4.6 Campi trasversi e longitudinali .
4.7 Fibre ottiche . . . . . . . . . . .
4.8 Guide d’onda . . . . . . . . . .
4.9 Oscillazioni di una cavità . . . .
4.10 Campi trasversi e radiali . . . .
4.11 Scattering di Mie . . . . . . . .
4.12 Potenziali di Debye . . . . . . .
.
.
.
.
.
.
.
.
.
.
.
.
.
.
.
.
.
.
.
.
.
.
.
.
.
.
.
.
.
.
.
.
.
.
.
.
.
.
.
.
.
.
.
.
.
.
.
.
.
.
.
.
.
.
.
.
.
.
.
.
.
.
.
.
.
.
.
.
.
.
.
.
.
.
.
.
.
.
.
.
.
.
.
.
.
.
.
.
.
.
.
.
.
.
.
.
.
.
.
.
.
.
.
.
.
.
.
.
.
.
.
.
.
.
.
.
.
.
.
.
.
.
.
.
.
.
.
.
.
.
.
.
.
.
.
.
.
.
.
.
.
.
.
.
.
.
.
.
.
.
.
.
.
.
.
.
.
.
.
.
.
.
.
.
.
.
.
.
.
.
.
.
.
.
.
.
.
.
.
.
.
.
.
.
.
.
.
.
.
.
.
.
.
.
.
.
.
.
.
.
.
.
.
.
.
.
.
.
.
.
.
.
.
.
.
.
.
.
.
.
.
.
.
.
.
.
.
.
.
.
.
.
.
.
.
.
.
.
.
.
.
.
.
.
.
.
.
.
.
.
.
.
.
.
.
.
.
.
.
.
.
.
60
60
62
67
68
70
.
.
.
.
.
.
.
.
.
.
.
.
.
.
.
.
.
.
.
.
.
.
.
.
.
.
.
.
.
.
.
.
.
.
.
.
.
.
75
75
76
76
79
84
86
89
91
91
96
100
103
103
104
105
105
106
107
112
.
.
.
.
.
.
.
.
.
.
.
.
.
.
.
.
.
.
.
.
.
.
.
.
.
.
.
.
.
.
.
.
.
.
.
.
.
.
.
.
.
.
.
.
.
.
.
.
.
.
.
.
.
.
.
.
.
.
.
.
.
.
.
.
.
.
.
.
.
.
.
.
.
.
.
.
.
.
.
.
.
.
.
.
.
.
.
.
.
.
.
.
.
.
.
.
.
.
.
.
.
.
.
.
.
.
.
.
.
.
.
.
.
.
.
.
.
.
.
.
.
.
.
.
.
.
.
.
.
.
.
.
.
.
.
.
.
.
.
.
.
.
.
.
.
.
.
.
.
.
.
.
.
.
.
.
.
.
.
.
.
.
.
.
.
.
.
.
.
.
.
.
.
.
.
.
.
.
.
.
.
.
.
.
.
.
.
.
.
.
.
.
.
.
.
.
.
.
.
.
.
.
.
.
.
.
.
.
115
. 115
. 116
. 117
. 118
. 121
. 123
. 125
. 127
. 127
. 128
. 132
. 134
. 137
5 Relazioni costitutive
5.1 Modello di Drude . . . . . . . . . . . . . . . .
5.2 Polarizzazione molecolare . . . . . . . . . . . .
5.3 Funzione dielettrica per elettroni delocalizzati
5.4 Oscillatori di Lorentz . . . . . . . . . . . . . .
5.5 Relazioni di dispersione di Kramers-Kronig . .
.
.
.
.
.
.
.
.
.
.
.
.
.
.
.
.
.
.
.
.
.
.
.
.
.
.
.
.
.
.
.
.
.
.
.
.
.
.
.
.
.
.
.
.
.
.
.
.
.
.
.
.
.
.
.
.
.
.
.
.
.
.
.
.
.
.
.
.
.
.
.
.
.
.
.
.
.
.
.
.
.
.
.
.
.
.
.
.
.
.
.
.
.
.
.
.
.
.
.
.
.
.
.
.
.
.
.
.
.
.
.
.
.
.
.
.
.
.
.
.
.
.
.
.
.
.
.
.
.
.
.
.
.
.
.
.
.
.
.
.
.
.
.
.
.
.
.
.
.
.
.
.
.
.
.
.
.
.
.
.
.
.
.
.
.
.
.
.
.
.
.
139
139
142
145
146
146
Indice
5.6
5.7
5.8
5.9
5.10
7
Mezzi anisotropi . . . . . . . . . . . . . . . . .
Isolatore di Faraday . . . . . . . . . . . . . . .
Mezzi periodici . . . . . . . . . . . . . . . . .
Pacchetti d’onda in mezzi dispersivi. . . . . .
Mezzi non-lineari: Effetto Kerr . . . . . . . .
5.10.0.1 compressione impulsi. Impulsi
. . . . . . .
. . . . . . .
. . . . . . .
. . . . . . .
. . . . . . .
ulttracorti .
6 Elettrodinamica
6.1 Teoria delle orbite . . . . . . . . . . . . . . . . . . . . .
6.1.1 Lenti elettrostatiche e magnetostatiche . . . . .
6.1.2 Ondulatore . . . . . . . . . . . . . . . . . . . .
6.1.3 Ciclotrone . . . . . . . . . . . . . . . . . . . . .
6.1.4 Invarianti adiabatici . . . . . . . . . . . . . . .
6.1.5 Specchi e bottiglie magnetiche . . . . . . . . . .
6.1.6 Intrappolamento nel campo magnetico terrestre
6.1.7 Elettroni in un’onda piana . . . . . . . . . . . .
6.1.8 Reazione di radiazione . . . . . . . . . . . . . .
6.1.9 Forza ponderomotrice . . . . . . . . . . . . . . .
6.1.10 Pinzette ottiche . . . . . . . . . . . . . . . . . .
6.2 Trappole . . . . . . . . . . . . . . . . . . . . . . . . . .
6.2.1 MOT . . . . . . . . . . . . . . . . . . . . . . . .
6.2.2 Trappola di Paul . . . . . . . . . . . . . . . . .
6.2.3 Trappola di Penning . . . . . . . . . . . . . . .
6.3 Campi di cariche in movimento . . . . . . . . . . . . .
6.3.1 Potenziali di Liénard-Wiechert . . . . . . . . .
6.3.2 Effetto Stewart-Tolman . . . . . . . . . . . . . .
6.3.3 Interazioni tra cariche in movimento . . . . . .
6.3.4 Densità spettrale energia e potenza irradiata . .
6.4 Interazione atomi-sciami di particelle cariche . . . . . .
6.4.1 Esperimento di Franck ed Hertz . . . . . . . . .
6.5 Radiazione Cerenkov . . . . . . . . . . . . . . . . . . .
6.6 Radiazione di frenamento (Bremsstrhalung) . . . . . .
6.7 Radiazione di sincrotrone . . . . . . . . . . . . . . . . .
6.7.1 Radiazione di ciclotrone . . . . . . . . . . . . .
6.7.2 Radiazione di ondulatore . . . . . . . . . . . . .
Indice analitico e dei nomi
.
.
.
.
.
.
.
.
.
.
.
.
.
.
.
.
.
.
.
.
.
.
.
.
.
.
.
.
.
.
.
.
.
.
.
.
.
.
.
.
.
.
.
.
.
.
.
.
.
.
.
.
.
.
.
.
.
.
.
.
.
.
.
.
.
.
.
.
.
.
.
.
.
.
.
.
.
.
.
.
.
.
.
.
.
.
.
.
.
.
.
.
.
.
.
.
.
.
.
.
.
.
.
.
.
.
.
.
148
150
151
151
153
153
.
.
.
.
.
.
.
.
.
.
.
.
.
.
.
.
.
.
.
.
.
.
.
.
.
.
.
.
.
.
.
.
.
.
.
.
.
.
.
.
.
.
.
.
.
.
.
.
.
.
.
.
.
.
.
.
.
.
.
.
.
.
.
.
.
.
.
.
.
.
.
.
.
.
.
.
.
.
.
.
.
.
.
.
.
.
.
.
.
.
.
.
.
.
.
.
.
.
.
.
.
.
.
.
.
.
.
.
.
.
.
.
.
.
.
.
.
.
.
.
.
.
.
.
.
.
.
.
.
.
.
.
.
.
.
.
.
.
.
.
.
.
.
.
.
.
.
.
.
.
.
.
.
.
.
.
.
.
.
.
.
.
.
.
.
.
.
.
.
.
.
.
.
.
.
.
.
.
.
.
.
.
.
.
.
.
.
.
.
.
.
.
.
.
.
.
.
.
.
.
.
.
.
.
.
.
.
.
.
.
.
.
.
.
.
.
.
.
.
.
.
.
.
.
.
.
.
.
.
.
.
.
.
.
.
.
.
.
.
.
.
.
.
155
155
158
175
180
181
183
185
187
191
192
193
194
196
199
202
202
202
205
207
207
210
210
218
219
221
230
231
235
8
Indice
Elenco delle figure
1.1 coordinate sferoidali . . . . . . . . . . . . . . . . . . . . . . . . . . . . .
1.2 Regioni di stabilità delle funzioni di Mathieu. Quando la coppia di parametri a, q cade nelle regioni comprese tra la curva più in basso e quella
successiva, tra la terza e la quarta, e così via l’esponente caratteristico µ
risulta reale.
. . . . . . . . . . . . . . . . . . .
1.3 Rappresentazione espansa della Fig. precedente relativa alla regione di
stabilitò a0 (q) < a < b1 (q) . . . . . . . . . . . . . . . . . . . . . . . . . .
1.4 Integrali ellittici E (ξ) (curva in basso) e K (ξ) (curva in alto) in funzione
di z . . . . . . . . . . . . . . . . . . . . . . . . . . . . . . . . . . . . . . .
2.1 Spettrometro a singolo cristallo per la determinazione dello spettro di
sorgenti di raggi gamma. . . . . . . . . . . . . . . . . . . . . . . . . . . .
2.2 Geometria dell’effetto Compton . . . . . . . . . . . . . . . . . . . . . . .
2.3 Rappresentazione schematica di una cavità a 4 specchi con due modi che
si propagano in verso orario ed antiorario. Quando la cavità è posta su
un piano che ruota rigidamente attorno ad un asse, i vettori d’onda locali dipendono localmente dallo shift Doppler, dando così luogo ad una
differenza tra le fasi accumulate in un giro completo dai due modi. . . . .
2.4 Geometria della deflessione di un raggio luminoso in prossimità di un buco
nero . . . . . . . . . . . . . . . . . . . . . . . . . . . . . . . . . . . . . .
2.5 Angolo di deflessione di un raggio luminoso da parte di una black-hole . .
. 21
. 35
. 36
. 36
. 56
. 56
. 68
. 73
. 73
3.1 Potenziale all’interno del nucleo . . . . . . . . . . . . . . . . . . . . . . . . 83
3.2 Geometria relativa al calcolo della costante di Madelung per un cristallo di
NaCl. . . . . . . . . . . . . . . . . . . . . . . . . . . . . . . . . . . . . . . 93
3.3 . . . . . . . . . . . . . . . . . . . . . . . . . . . . . . . . . . . . . . . . . . 98
3.4 Coppia di bobine di Helmholtz . . . . . . . . . . . . . . . . . . . . . . . . . 104
3.5 Andamento di 4a∆Aφ / (µ0 Iρ) lungo l’asse z/a di un quadrupolo costituito
da due spire di raggio a e distanti tra loro 2Z, per Z/a = 1.2, 0.8, 0.6, 0.4, 0.2.105
3.6 Suscettività di un campione in prossimità di una risonanza nucleare . . . . 109
4.1 Schema di principio di un apparato SNOM. Il cilindro che termina a punta
rappresenta una fibra ottica cava utilizzata per illuminare il campione. Il
campo riflesso dal campione può essere raccolto dalla stessa fibra. Il sistema può anche lavorare in trsmissione misurando l’intensità trasmessa dal
campione. . . . . . . . . . . . . . . . . . . . . . . . . . . . . . . . . . . . . 124
9
10
Elenco delle figure
4.2 Campo vicino ′′ 026A30CEx ′′ 026A30C ( sinistra ) e ′′ 026A30CEz ′′ 026A30C
(destra). Curve tratteggiate: z = 0.04 λ,.1 λ (sinistra) e z = 0.1 λ,.5 λ
(destra). Le curve a tratto continuo più interne rappresentano i campi a
z = 0, mentre quelle più esterne si riferiscono alle distribuzioni misurate con
microscopi
ideali che lasciano passare valori di ′′ 026A30Ckx ′′ 026A30C <
√
k0 / 2. . . . . . . . . . . . . . . . . . . . . . . . . . . . . . . . . . . . . . . 124
5.1 Modulo del coefficiente di riflessione per incidenza normale per un metallo
per diversi valori di γ/ω p = 0.05, 0, 1, 02, 0, 4 . . . . . . . . . . . . . . . . . 140
5.2 Parte reale (tendente a π/4 per ω → ∞) ed immaginaria dell’angolo di
Brewster per un metallo descritto dal modello di Drude con ω p =frequenza
di plasma e γ = ω p /10 . . . . . . . . . . . . . . . . . . . . . . . . . . . . . 141
6.1 Moto a rosetta di un elettrone in un’orbita ellittica di Bohr. Dopo una
rivoluzione completa attorno al nucleo l’asse maggiore risulta ruotato di un
angolo ∆φ = π (αZ/k)2 , con α costante di struttura fine. . . . . . . . . . . 157
6.2 Microscopio elettronico a scansione. Un fascio di elettroni vien messo a
fuoco sul campione. Dalla regione di impatto del fascio focalizzato vengono sia emessi elettroni secondari che riflessi parte di quelli incidenti. I
rivelatori posti a lato raccolgono questi elettroni e ne misurano le energie.
da www.astarmathsandphysics.com . . . . . . . . . . . . . . . . . . . . . . 159
6.3 Rappresentazione schematica di una lente elettrostatica . . . . . . . . . . . 160
6.4 Schemi di lenti elettrostatiche costituite (in alto) da una coppia di diaframmi a potenziali diversi e (in baso) da cilindri coessiali da encyclopedia2.thefreedictionary.com . . . . . . . . . . . . . . . . . . . . . . . . . . 160
6.5 Focalizzazione degli elettroni emessi da da un catado mediante una lente
elettronica formata da due diaframmi circolari. La famiglia di curve rappresenta le sezioni delle superfici equipotenziali da encyclopedia2.thefreedictionary.com161
6.6 Rappresentazione schematica di un quadrupolo elettrico costituito da 4
cililindri posti a potenziali alternativamente pari a V e -V . . . . . . . . . . 163
6.7 Superfici equipotenziali per un quadrupolo elettrico . . . . . . . . . . . . . 163
6.8 Rappresentazione schematica di una lente magnetica . . . . . . . . . . . . 166
6.9 Rappresentazione schematica della forza agente su un elettrone mentre
attraversa una lente magnetica . . . . . . . . . . . . . . . . . . . . . . . . . 166
6.10 Andamento dell’inversa lunghezza focale di un quadrupolo magnetico al
variare della distanza tra le due spire . . . . . . . . . . . . . . . . . . . . . 171
6.11 Sezione di un quadrupolo magnetico inserito su una linea di trasporto di
fasci di particelle cariche . . . . . . . . . . . . . . . . . . . . . . . . . . . . 171
6.12 Lente magnetica a quadrupolo trasverso da www.helmholtz-berlin.de . . . 171
6.13 Traiettorie spiraliformi di ioni ed elettroni in un campo magnetico uniforme 173
6.14 Traiettorie dell’elettrone relative ai casi a, b e c. . . . . . . . . . . . . . . . 174
6.15 Rappresentazione schematica della traiettoria di un elettrone nel campo di
un ondulatore . . . . . . . . . . . . . . . . . . . . . . . . . . . . . . . . . . 175
6.16 . . . . . . . . . . . . . . . . . . . . . . . . . . . . . . . . . . . . . . . . . . 177
6.17 Rappresentazione schematica di un ciclotrone. Al centro è riportata la
camera a vuoto a forma di una doppia D metallica, compresa tra due
elettromagneti. Alle due D è applicata una tensione oscillante che crea
un campo localizzato tra i due bordi rettilinei affacciati. Attraversando
ciclicamente questa regione gli ioni vengono accelerati. . . . . . . . . . . . 180
Elenco delle figure
11
6.18 Traiettorie spiraliformi degli ioni all’interno di un ciclotrone. Gli ioni, iniettati al centro sono sottoposti ad un campo magnetico uniforme verticali
che li obbliga a descrivere orbite circolari. Attraversando il campo elettrico
oscillante localizzato tra le due D gli ioni vengono accelerati finendo cosè
per descrivere orbite circolari spiralizzanti verso la periferia. . . . . . . . . 181
6.19 . . . . . . . . . . . . . . . . . . . . . . . . . . . . . . . . . . . . . . . . . . 185
6.20 Particelle cariche imbottigliate in una bottiglia magnetica. La forza di
Lorentz trattiene tra i due punti di massimo di B le particelle con angoli
di lancio ϑ tali che sin2 ϑ > Bmin /Bmax da physics.miami.edu . . . . . . . . 186
6.21 Rappresentazione schematica di un Tokamak in grado di confinare un plasma da fusione in una regione toroidale . . . . . . . . . . . . . . . . . . . . 186
6.22 Andamento schematico delle linee del campo magnetico terrestre . . . . . . 187
6.23 Andamento schematico della sezione d’urto di un elettrone atomico in funzione della frequenza nell’approssimazione di una risonanza singola.. Si
possono individuare tre regioni: bassa frequenza (o regione di Rayleigh) in
cui la polarizzabilità è costante e si osserva una variazione con la quarta
potenza di ω; una regione di risonanza, in cui l’elettrone viene eccitato
ad un livello risonante, ed infine una regione in alta frequenza in cui l’elettrone risponde solo al campo incidente e si comporta quindi come l’elettrone
libero della diffusione alla Thomson. . . . . . . . . . . . . . . . . . . . . . . 197
6.24 Pressione di radiazione agente su un atomo investito da due fasci contropropaganti in funzione della velocità assiale . . . . . . . . . . . . . . . . . . 197
6.25 Rappresentazione schematica di una trappola MOT. Gli atomi da intrappolare e raffreddare vengono investiti da tre coppie di fasci conrpropaganti
diretti lungo gli assi x,y e z. Il campo magnetico quadrupolare è utilizzato
per modulare spazialmente per effetto Zeeman la frequenza di transizione,
determinando così l’intrappolamento. . . . . . . . . . . . . . . . . . . . . . 198
6.26 Schema di una trappola MOT 1D. Gli atomi sono sottoposti ad una pressione di radiazione dipendente dalla posizione lungo l’asse, ottenuta variando spazialmente il detuning per effetto Zeeman. In basso sono indicati gli
spostamenti dei livelli accoppiati con i due fasci con polarizzazioni circolari
σ ± . . . . . . . . . . . . . . . . . . . . . . . . . . . . . . . . . . . . . . . . . 198
6.27 Struttura iperfine delle transizioni del Rb utilizzate nel raffreddamento laser200
6.28 Rappresentazione schematica del campo in una trappola di Paul. La trappola è circondata da una bobina che crea un campo magnetico che obbliga
le particelle cariche a spiralizzare attorno a B. A loro volta gli elettrodi
creano un potenziale di quadrupolo oscillante proporzionale a U + V cos (Ωt) .201
6.29 Rappresentazione schematica dell’interazione tra un atomo ed un elettrone
sufficientemente veloce. La nuvola di elettroni atomici viene eccitata dalle
componenti spettrali del campo prodotto dall’elettrone “proiettile” a frequenze “f” coincidenti con le risonanze dell’atomo. Perché ciò avvenga la
regione in cui si estende il campo a frequenza f deve comprendere la nuvola
elettronica che circonda l’atomo. . . . . . . . . . . . . . . . . . . . . . . . . 213
6.30 Sezione d’urto di eccitazione dell’idrogeno in funzione dell’energia degli
elettroni. Le curve continue sono teoriche (cf. H.S.W. Massey and E.H.S.
Burhop, “Electronic and Ionic Impact Phenomena” , Clarendon Press, Oxford (1952)) mentre quella tratteggiata è stata misurata da W.L. Fite et
al., Phys. Rev. vol. 116, pg. 356 (1959). . . . . . . . . . . . . . . . . . . . 214
12
Elenco delle figure
6.31 Radiazione di bremsstrhalung emessa nella collisione tra due particelle (da
www4.nau.edu) . . . . . . . . . . . . . . . . . . . . . . . . . . . . . . . .
6.32 From: Elements of X-ray Diffraction, B.D.Cullity, Addison-Wesley Publishing, Third Edition, 1967 . . . . . . . . . . . . . . . . . . . . . . . . .
6.33 Geometria relativa ad una carica che descrive un’orbita circolare. L’osservatore in P sarà investito da un impulso di radiazione emesso mentre l’elettrone percorre l’arco di traiettoria indicato in figura. Per utilizzare questa
radiazione si utilizzano anelli di accumulazione in cui, come schematizzato in (b), gli elettroni passano attraverso dei magneti curvanti, emettendo
impulsi di radiazione. . . . . . . . . . . . . . . . . . . . . . . . . . . . . .
6.34 Distribuzione delle intensità delle armoniche in funzione dell’ordine m. . .
6.35 (a) Spettro della radiazione di sincrotrone emessa lungo il piano dell’orbita;
(b) andamento temporale della radiazione vista da un osservatore. Si nota
che questa è costituita da una sequenza periodica di impulsi con periodo
pari al tempo T di circolazione dell’elettrone. Il rapporto tra T e la durata
del singolo impulso è circa uguale al numero di armoniche presenti nello
spettro. . . . . . . . . . . . . . . . . . . . . . . . . . . . . . . . . . . . .
. 219
. 220
. 222
. 225
. 226
Elenco delle tabelle
13
Prefazione
Il testo è rivolto a studenti sia della laurea triennale che di quella magistrale. Queste note
possono essere utilizzate a corredo di insegnamenti dei corsi di laurea in Fisica, Chimica,
Biotecnologie ed Ingegneria.
Le grandezze sono espresse per la maggior parte dei cai in unità del sistema MKSA1 .
L’energia è data a seconda dei casi in Joule (J), elettron-volt (eV ), cm−1 , Hertz (Hz).
Per i valori delle costanti fisiche fondamentali ci si è riferiti a quelli riportati nell’articolo The fundamental physical constants di E. R. Cohen e B. N. Taylor, apparso in
Physics Today, Aug. 1996 pp. 9-13. Maggiori informazioni sulle costanti della fisica si
possono ottenere consultando vari siti, tra i quali si segnala quello del N.I.S.T.2
1
2
Vedi R.A. Nelson, ”Guide for metric practice” , in Physics Today, pp. 15-16, Aug. 1996
http://physics.nist.gov/cuu/Constants/
15
Capitolo 1
Metodi matematici della
Elettrodinamica
1.1
Identità vettoriali
Esercizio 1.1.1 Verificare le seguenti (a) identità vettoriali:
a × (b × c) = b (a · c) − (b · a) c ,
a × (b × c) = b × (a × c) + c × (b × a) ,
(a × b) · (c × d) = (a · c) (b · d) − (a · d) (b · c)
e (b) relazioni differenziali tra vettori di
uno spazio 3D:
▽ (a · b)
▽ · (a × b)
▽ × (f a)
▽ × (a × b)
▽ × (▽ × a)
▽2 (f g)
▽2 (f a)
▽ ▽ · (f a)
▽ × ▽ × (f a)
1.2
=
=
=
=
=
=
=
=
=
a × ▽ × b+b × ▽ × a+ ( a · ▽) b+ (b · ▽) a ,
a·▽×b−b·▽×a,
f ▽ ×a + ∇f × a
a▽ · b−b▽ · a − (a · ▽) b+ (b · ▽) a ,
▽ ▽ ·a − ▽2 a ,
f ▽2 g + g ▽2 f + 2 ▽ f · ▽g ,
f ▽2 a + a ▽2 f + 2 (▽f · ▽) a ,
(▽f ) ▽ ·a + f ▽ ▽ · a+ ▽ f × ▽ × a + (▽f · ▽) a+ (a · ▽) ▽ f ,
(▽f ) × ▽ × a − a ▽2 f + (a · ▽) ▽f + f ▽ × ▽ ×a+ ▽ f ▽ ·a − (▽f · ▽) a ,
Operatori differenziali
Si consideri una connessione affine a 3 dimensioni ed un generico sistema di coordinate
uµ per definire un generico punto P (u1 , u2 , u3 ). Lo spostamento tra due punti vicini è
rappresentato da
−
→
P x1 + dx1 , x2 + dx2 , x3 + dx3 − P x1 , x2 , x3 ≡ dx = dxi ei (P )
17
18
Metodi matematici della Elettrodinamica
dove i vettori ei (xj ) formano una base di rappresentazione, dipendente dalle coordinate
xj . La dipendenza dei vettori ei (P ) dal punto è descritta dalla relazione differenziale
ei (P + dP ) − ei (P ) = Γkij (P ) ek (P ) dxj
(1.1)
in cui sono presenti i coefficienti di connessione Γkij (P ) . Nello spazio vetoriale è definito
un prodotto scalare definito dall’insieme di prodotti ove si è posto
ei · ej ≡ gij
A gij è dato il nome di tensore metrico mentre si dà il nome di tensore duale di gij e lo
sindica con g iJ il tensore che soddisfa le seguenti condizioni:
g im gmj ≡ δ ij
Se si rappresenta gij come una matrice 3 × 3 [gij ], [g im ] è l’inverso di [gij ].
La distanza tra due punti vicini è data da
−
→ −
→
dx · dx = ds2 = dxi dxj ei · ej ≡ dxi dxj gij
Dalla (1.1) discende che
gii′ ,k dxk ≡ Γkij ek · ei′ dxj + Γki′ j ei · ek dxj
Γkij gki′ + Γki′ j gik dxj
=
dove col simbolo , k posto a pedice si è indicata la derivata ∂k = ∂x∂ k rispetto alla
coordinata xk .Ne segue, e la dimostrazione vene lasciata come esercizio, che Γijk risulta
legato al tensore metrico dalla relazione:
1
Γijk = Γikj = g im (gmj,k + gmk,j − gjk,m )
2
(1.2)
A questa regola che descrive come muovere in modo legittimo un vettore lungo una curva
sulla varietà senza mutarne la direzione si dà il nome di connessione di Levi-Civita . I
coefficienti della connessione così definiti prendono il nome di coefficienti di Christoffel di
seconda specie, per distinguerli da Γijk = gim Γm
jk detti invece di prima specie.
. = V i ei la quantità
Si definisce derivata covariante di un campo vettoriale V
Dj Vji = V,ji + Γijk V k
e viene indicata con
Pertanto si ha
V;ji = V,ji + Γijk V k
Dj V. = V,ji + Γijk V k ei
In particolare la divergenza è data da:
∇ · V. = Di V i = V;ii = V,ii + Γiik V k
Tenuto conto che
g im (gmk,i − gik,m ) = 0
(1.3)
1.2 Operatori differenziali
19
si ha
1
Γiik = g im gmi,k
2
D’altra parte
k
δ ln det gij x
det gij xk + dxk
= ln
det [gij (xk )]
= ln det
gim xk
−1
gmi xk + dxk
= ln det 1 + [gim ]−1 [δgmi ]
= ln 1 + T r [gim ]−1 [δgmi ]
= T r [gim ]−1 [δgmi ]
Pertanto, si ha
−1
1 im
1 T r [gim ] [δgmi ]
g gmi,k =
2
2
dxk
1 δ ln det [gij ]
1
1 √
=
= ∂k ln g = √ ∂k g
k
2
dx
2
g
Γiik =
avendo posto g = det [gij ]. Ne discende che
1
1
√ k
√
V;ii = V,ii + √ (∂k g) V k = √ ∂k
gV
g
g
Esercizio 1.2.1 Si consideri un sistema di
tensore metrico
 2
h1
gij =  0
0
(1.4)
coordinate curvilinee ortogonali associate al

0 0
h22 0 
0 h23
Calcolare la divergenza ed il Laplaciano di un campo vettoriale
Soluzione: Dalla relazione (1.4) discende che
1
∂k h1 h2 h3 V k
h1 h2 h3
1
hi hj
=
∂k
Vk
h1 h2 h3
hk
V;ii =
Per quanto riguarda il Laplaciano
∆2 V ℓ =
1
∂k
h1 h2 h3
hi hj
∂k V ℓ
hk
Esercizio 1.2.2 Esprimere ∇, ∇·, ∇× e ∆2 in coordinate (a) cilindriche e (b) sferiche
Soluzione: (a) In coordinate cilindriche risulta
hρ = 1 , hz = 1 , hφ = ρ
20
Metodi matematici della Elettrodinamica
dimodochè
∂
∂
1 ∂
+ ez
+ eφ
∂ρ
∂z
ρ ∂φ
∂
∂ 1
1 ∂
=
ρeρ + ρez +
eφ
ρ ∂ρ
∂z
∂φ ρ


eρ ez eφ
∂
∂
1 ∂ 
=  ∂ρ
∂z
ρ ∂φ
1 1
ρ
1 ∂ ∂
∂2
1 ∂2
=
ρ + 2 + 2 2,
ρ ∂ρ ∂ρ ∂z
ρ ∂φ
∇ = eρ
∇·
∇×
∆2
·
(b) In coordinate sferiche
hr = 1 , hθ = r , hφ = r sin θ
Pertanto
∂
1 ∂
1 ∂
+ eθ
+ eφ
∂r
r ∂θ
r sin θ ∂φ
∂
∂ 1
1
∂ 2
= 2
r sin θer +
sin θeθ +
eφ ·
r sin θ ∂r
∂θ
∂φ sin θ


er eθ
eφ
∂
1 ∂
1
∂ 
=  ∂r
r ∂θ
r sin θ ∂φ
1
r
r sin θ
1
∂
1 ∂2
1 ∂ 2∂
1 ∂
+
sin θ
+
= 2 r
,
r ∂r ∂r r2 sin θ ∂θ
∂θ sin2 θ ∂φ2
∇ = er
∇·
∇×
∆2
Esercizio 1.2.3 In alcuni casi può convenir utilizzare coordinate sferoidali ξ, η, φ definite
dalle relazioni:
rA + rB
rA − rB
y
, η=
, φ = arctan
ξ =
R
R
x
(∞ ≥ ξ ≥ 1 , −1 ≤ η ≤ 1, 0 ≤ φ ≤ 2π)
(1.5)
con R distanza tra i fuochi A e B, φ anomalia rispetto all’asse z. Le superfici a ξ = const e
η = const corrispondono rispettivamente ad ellissoidi ed iperboloidi confocali di rotazione
attorno all’asse z. (a) Calcolare i fattori di scala e l’operatore Laplaciano
Soluzione: Si può facilente dimostrare che lo Jacobiano di questo sistema di coordinate
è dato da
R3 2
∂ (x, y, z)
R
=
ξ − η 2 = rA rB
∂ (ξ, η, φ)
8
2
Pertanto, un elemento di volume dV è espresso da
dV =
R
rA rB dξdηdφ = hξ hη hφ dξdηdφ
2
Ad esse corrisponde la seguente espressione per gli elementi di lunghezza:
R2
dℓ =
4
2
ξ 2 − η2 2 ξ 2 − η 2 2
dξ +
dη + ξ 2 − 1
2
2
1−η
ξ −1
1 − η2 dφ2
1.3 Distribuzioni
21
Figura 1.1: coordinate sferoidali
per cui
R
hξ =
2
R
ξ 2 − η2
, hη =
2
2
ξ −1
R
ξ 2 − η2
, hφ =
2
1−η
2
ξ 2 − 1 (1 − η2 )
Inoltre, il Laplaciano si scrive nella forma
∇2 =
1.3
4
∂
∂
∂
∂
(ξ 2 − 1)
+
(1 − η2 )
2
∂ξ
∂η
∂η
− η ) ∂ξ
2
∂
1
+ 2
(ξ − 1)(1 − η 2 ) ∂φ2
R2 (ξ 2
Distribuzioni
Formalmente la delta di Dirac viene definita dalla seguente relazione:
∞
(x)φ (x) dx = φ (0)
−∞
valida per ogni funzione continua φ (x) in un intorno dello zero.
(x) è una distribuzione che soddisfa la proprietà (x) = 0 per x = 0, e tale che:
∞
(x)dx = 1
−∞
La derivata distribuzionale della delta è la distribuzione ’ definita a partire da una
funzione di test \operatorname\phi liscia e a supporto compatto:
∞
−∞
δ ′ (x)φ (x) dx = −φ′ (0)
22
Metodi matematici della Elettrodinamica
La funzione
può essere considerata come il limite
δ(x) = lim δ n (x)
n→∞
di alcune particolari successioni di funzioni δ n (x) localmente integrabili,
∞
n (x)dx
=1
−∞
tali che ∀ε > 0, ∀a ∈ [ε, +∞]
∞
lim
n→∞
−a
n (x)dx
−∞
∞
,
n (x)dx
n (x)dx
= 1
< K
a
−∞
con K indipendente da n. Le sucessioni
−a
n (x)dx
∞
,
n (x)dx
a
−∞
convergono uniformemente a 0
Esercizio 1.3.1 Dimostrare che la funzione delta di Dirac è esprimibile come il limite
delle seguenti successioni1 :
δ (t) =
δ (t) =
δ (t) =
lim N exp −N 2 πt2
N→∞
lim N rect (Nt)
N→∞
lim N sin c (Nt)
N→∞
doce
sinc (x) =
sin (x)
,
x
(1.6)
mentre in uno spazio 2D si ha
δ (x, y) =
δ (x, y) =
δ (x, y) =
δ (x, y) =
δ (x, y) =
lim N 2 exp
N→∞
−N 2 π x2 + y 2
lim N 2 rect (N x) rect (N y)
N→∞
lim N 2 sin c (N x) sin c (Ny)
N→∞
lim π −1 N 2 circ N
N→∞
lim N
j1 2πN
N→∞
x2 + y 2
x2 + y 2
x2 + y 2
Esercizio 1.3.2 Dimostrare che
δ(at) =
1
1
δ(t)
|a|
J. W. Goodman, Introduction to Fourier Optics, McGraw-Hill, New York 1968, App. A
1.4 Trasformate integrali
23
Esercizio 1.3.3 Dimostrare che se f è una funzione derivabile e xi sono gli zeri della
funzione, allora:
δ(x − xi )
δ(f (x)) =
|f′(xi )|
i
Esercizio 1.3.4 Dimostrare che
1
δ (θ) δ (r − r0 )
sin θ
δ (3) (r−r0 ẑ) =
2πr2
1.4
Trasformate integrali
1.4.1
Trasformata di Fourier
Per una funzione integrabile secondo Lebesgue,
∞
−∞
esiste la trasformata di Fourier
|f (t)| dt < ∞ ,
∞
F {f (t)} = f˜ (ω) =
f (t) eiωt dt .
−∞
Per ω reale f˜ (ω) è limitata e continua non necessariamente integrabile e soddisfa il lemma
di Riemann—Lebesgue,
lim f˜ (z) = 0 .
|z|→∞
Quando anche f˜ (ω) , con ω reale, è integrabile la trasformata è invertibile,
f (t) =
∞
1
2π
f˜ (ω) e−iωt dω .
−∞
Esercizio 1.4.1 Trasformata di Fourier della delta
Soluzione:
δ (x) =
1
2π
∞
e−iωt dω
−∞
La delta è l’antitrasformata della funzione costante f(x) = 1
1.4.2
Trasformate di Fourier-Bessel
Si definiscono trasformate di Fourier-Bessel le traformate integrali:
∞
fF B (λ) =
f (ρ) Jn (λρ) ρdρ
0
che godono della proprietà di essere coniugate:
∞
f (ρ) =
0
fF B (λ) Jn (λρ) λdλ
24
Metodi matematici della Elettrodinamica
1.4.3
Trasformata di Hilbert
La trasformata di Hilbert di una funzione g (x) è data da e Hilbert
H [g] = gH (x) =
∞
1
V.P.
π
−∞
g (y)
dy ,
y−x
dove V.P. sta per il valor principale dell’integrale.
Esercizio 1.4.2 Dimostrare che la trasformata di Hilbert gode di due importanti proprietà
H [g] |f = − g|H [f ]
HH [g] = −g
ovvero
∞
−∞
∗
gH
(x) f (x) dx = −
∞
−∞
g ∗ (x) fH (x) dx
gHH (x) = −g (x)
Esercizio 1.4.3 Dimostrare che la trasformata di Hilbert di una funzione analitica g (x)
priva di poli per Im (x) > 0 o Im (x) < 0 è data rispettivamente da
gH (x) = ig (x) , gH (x) = −ig (x) .
Soluzione: Innanzitutto si osserva che per g (y) regolare in x si può rappresentare gH (x)
nella forma
∞+iε
1
g (y)
g (z)
gH (x) = lim
dy −
dz
π ε→0 −∞+iε y − x
Γ z−x
dove Γ indica una semicirconferenza di raggio |ε| con centro in x e che si sviluppa per
Im z > (<) 0 in accordo con sign (ε) . Se g (z) non ha poli per Im z > 0 si sceglierà ε > 0.
Nel caso opposto si sceglierà ε < 0. Poichè g (z) si annulla all’infinito si può applicare il
lemma di Riemann ottenendo così
1
gH (x) = − lim
π ε→0
Γ
g (z)
dz = −i sign (ε) g (x)
z−x
Esercizio 1.4.4 Calcolare la trasformata di Hilbert della funzione
g (x) =
1
x−z
con z una quantità complessa
Soluzione: Dall’esercizio precedente discende che
H
1
1
= −i Im [z]
x−z
x−z
1.5 Integrali
25
Esercizio 1.4.5 Calcolare la trasformata di Hilbert della funzione
g (x) =
Soluzione: Poichè
1
1 + x2
1
1
= − Im
2
1+x
x−i
si ha
H
1
1 + x2
= −H Im
= Im
1
x−i
x
i
=
x−i
1 + x2
Esercizio 1.4.6 Calcolare la trasformata di Hilbert della funzione
g (x) =
Soluzione: Poichè
x
1 + x2
1
1
= Re
2
1+x
x−i
procedendo come si è fatto nel precedente esercizio si ottine
H
1
x
=
1 + x2
1 + x2
Esercizio 1.4.7 Mostrare che le trasformate di Hilbert delle funzioni δ (x) , sinc(x) =
sin(x)
e rect (x) sono date da
x
1
πx
1 − cos (x)
H [sin c (x)] =
x
x + 12
1
rect (x) =
ln
π
x − 12
H [δ (x)] =
1.5
1.5.1
Integrali
Valutazione asintotica
Si consideri l’integrale2
a
e−ikh(s) ν (s) g (s) ds
I (k) =
0
2
per una introduzione generale alla valutazione asintotica v.p.e. I. N. Bleistein, Mathematical Methods
for Wave Phenomena, Academic Press, Orlando (1984)
26
Metodi matematici della Elettrodinamica
con h (s) una funzione monotona di s nell’intervallo (0, a) ,
ν (s) =
a
s
exp − y1 +
1
y−a
dy
a
0
exp − y1 +
1
y−a
dy
una funzione, introdotta da van der Corput, detta neutralizzatore tale che
ν (0) = 1 , ν (a) = 0
mentre le derivate tendono a 0 quando s tende ad ognuno degli estremi dall’interno di
(0, a) . Si assuma inoltre che
h (s) = h0 + sρ u (s)
g (s) = sγ v (s)
cone ρ > 0 e 0 > Re γ > −1 mentre u e v sono funzioni analitiche diverse da 0 per s = 0.
Introducendo ora la variabile
xρ = sρ u (s)
I (k) si riscrive nella forma
a
ρ
e−ikεx xγ f (x) ν (x) dx
I (k) = e−ikh0
0
con x > 0, ε = sgn u (0) e
g (s) ds
h (s) − h0 dx
Se f (x) è differenziabile N-volte, procedendo per successive integrazioni per parti si
ottiene3 :
N−1
dn
ikh0
h(−n−1) (x; k) n [f (x) ν (x)]|x(a)
+ R0,N
(1.7)
e I (k) = −
0
dx
n=0
f (x) =
dove
x(a)
RN = (−1)N
h(−N) (x; k)
0
e
dN
[f (x) ν (x)] dx
dxN
h(−n−1) (x; k)
x+∞(−iε)1/ρ
=
xn +∞(−iε)1/ρ
x1 +∞(−iε)1/ρ
dxn
x
= −
1
n!
dxn−1 · · ·
xn
x+∞(−iε)1/ρ
x
−ikεxρ0
(x0 − x)n xγ0 e
x1
ρ
xγ0 e−ikεx0 dx0
dx0
Sostituendo queste ultime espressioni nella (1.7) si ottiene
1
eikh0 I (k) =
ρ
=
3
N−1
n=0
ikh0
e I0
1Γ
n!
n+γ+1
ρ
n+γ+1
ρ
k
(k; N) + RN
π
e−i 2 ε
n+γ+1
ρ
dn
f (x)|0 + R0,N
dxn
v.p.e. S. Solimeno, B. Crosignani, P. D Porto, Guiding, Diffraction and Confinement of Optical
Radiation, Academic Press, Orlando (1986), Cap. V
1.5 Integrali
27
Si può ora dimostrare che per k → ∞ RN è un infinitesimo dello stesso ordine di
RN = O k −
N +γ+1
ρ
Introducendo ora il neutralizzatore ν a (x) che presenta le stesse proprietà di ν (x) per
x = a, si perviene ad analogo risultato. Pertanto più in generale si ha
a
I (k) = e−ikh0
e−ikh(s) g (s) ds
0
= I0 (k; N) + Ia (k; N ) + O k −
N +γ+1
ρ
(1.8)
ovvero per h (s) monotona in (0, a) I (k) è esprimibile in funzione dei contributi dei due
estemi dell’intervallo di integrazione.
1.5.1.1
Metodo SP della fase stazionaria
In molti casi h′ (s) si annulla in più punti di (0, a) e g (s) può risultare singolare. In tal
caso (0, a) va suddiviso in tanti sottointervalli contigui (0, a) = (0, a1 )∪(a1 , a2 ) · · ·∪(am , a)
in ciascuno dei quali h (s) risulta monotona e per i quali vale l’espansione (1.8).
Esercizio 1.5.1 Dimostrare che se è presente un solo punto a fase stazionaria (sadle
point) in s = sSP si ha al primo ordine in k
∞
0
e−ikh(s) g (s) ds ∼ ISP + I0
dove
ISP =
I0 =
2π
e−iεπ/4−ikh(sSP ) g (sSP ) + 0 k −1
k |h′′ (sSP )|
e−ikh(0) g (0)
√
+ 0 k −3/2
′
2πk h (0)
(1.9)
con ε = sgn h′′ (sSP ) .
1.5.1.2
Metodo SD dello steepest descent
Esercizio 1.5.2 Si analizzi al tendere di k → ∞ l’integrale
I (k) =
∞
e−ikh(s) g (s) ds
−∞
con h (s) e g (s) funzioni analitiche tali che la derivata
punto del piano complesso z
d
h (z)
dz
si annulli in un generico
d
Soluzione: Per estendre il metodo SP al caso in cui la derivata dz
h (z) si annulli in un
generico punto del piano complesso z, si utilizza il metodo SD consistente nel modificare
28
Metodi matematici della Elettrodinamica
l’intervallo di integrazione (−∞, ∞) nel cammino ΓSD che passi per il punto sella zSDP
di h (z) , supposto per il momento unico. Si pone infatti
I (k) =
g (z) e−ikh(z) dz +
+
ΓSD
rq e−ikh(sq )
ΓB
q
dove ΓB rappresenta un insieme di cammini che circondano eventuali branch-cuts di g,
mentre rq sta per il q-esimo residuo di un eventuale polo di g compreso tra il cammino
di integrazione iniziale e ΓSD + ΓB . Intanto, lungo ΓSD h (z) si può scrivere come h (z) =
Re h (zSD ) + i Im h (z) con Im h (z) che presenta un singolo massimo in zSD . Pertanto, per
ζ ∈ ΓSD si può trasformare ζ nella nuova variabile reale u tale che:
ζ → u = ± Im h (zSD ) − Im h (ζ)
In tal modo l’integrale
ΓSD
· · · diventa
2
ISD = e−ikh(zSDP )
g [ζ (u)] e−ku
ΓSD
dζ
du
du
2
Per k → ∞ il fattore e−ku decresce rapidamente allontanandosi da u = 0 ed il contributo
maggiore all’integrale viene da g (zSD ) . Pertanto,
ISD = e−ikh(zSD ) g (zSD )
dζ
du
+ 0 k −1
SD
2π
e−iπ/4−ikh(zSD ) g (zSD ) + 0 k −1
kh′′ (zSD )
=
(1.10)
Esercizio 1.5.3 Con riferimento al precedente esercizio dimostrare che integrando per
parti l’Eq. (1.10) si estende alla serie asintotica:
ISD =
2π
e−iπ/4−ikh(zSP ) g (zSD )
(zSD )
kh′′
′′′′
i h′′′ g ′ 1 h
5 h′′′2
1 g ′′
× 1+
+
+ ···
−
−
2k h′′2 g
4 h′′2 12 h′′3
h′′ g
Esercizio 1.5.4 Valutare asintoticamente un integrale del tipo di Fourier
e−ikr cos(β−θ) g (β) dβ
I (k) =
Γ
con Γ un cammino sul piano complesso β
Soluzione: Poichè
d
h (β)
dβ
si annulla per β = θ dall’Eq. (1.10) discende:
ISD =
2π −i(kr−π/4)
e
g (θ) + 0 k −1
kr
(1.11)
1.6 Metodo WKB
1.6
29
Metodo WKB
Un problema ricorrente è quello di integrare l’equazione di Helmholtz
d2
+ k 2 (x) u = 0
dx2
(1.12)
dove il vettore d’onda k (x) è funzione di x. La (1.12) può essere integrata ricorrendo al
metodo asintotico WKB4 .
Ponendo
k 2 (x) = k02 n2 (x)
La funzione
f (x) =
sin k0
x
n (x′ ) dx′ +
π
4
k0 n (x)
(1.13)
soddisfa l’equazione
d2
3n′2 − 2nn′′
2 2
+
k
n
(x)
f
=
f (x) .
0
dx2
4n2
con f ′ = df (x) /dx. Se il lato destro di questa equazione si può trascurare, f (x) è un
integrale dell’Eq.(1.12): il che accade per k0 → ∞. uniformemente nell’intervallo di
integrazione.
Se k (x) → 0 per x → x0 il lato destro tende ad esplodere e f (x) non soddisfa più la
(1.12). In questa regione la (1.12) si riduce all’equazione di Airy (v. Sez. 1.7.3)
d2
+ (x − x0 ) K0 f = 0 ,
dx2
con K0 = k02
1.6.1
d 2
g
dx
(x)
x=x0
(1.14)
, soddisfatta dalla funzione Ai (−K0 (x − x0 )) (v. Eq. (1.27)).
Equazione di Helmholtz
Esercizio 1.6.1 Estendere il metodo WKB all’integrazione dell’equazione di Helmholtz
nello spazio a 3 dimensioni
∇2 + k02 n2 (r) u (r) = 0
(1.15)
per k0 → ∞ partendo dalla serie asintotica di Luneburg e Kline5
(1.16)
Am (r)
−N
m + o ko
(ik
)
o
m=0
(1.17)
u (r) ∼ e
dove il simbolo ∼ sta ad indicare che
iko S(r)
u (r) ∼ e
∞
Am (r)
(iko )m
m=0
iko S(r)
N
con o ko−N simbolo di Landau che indica una funzione che decresce più rapidamente di
ko−N
4
v.p.e. G. Esposito, G. Marmo and G.
Sudarshan, “From Classical to Quantum Mechanics”, Cambridge U. P., Cambridge 2004, Sez. 4.8; S.
Flügge, Practical Quantum Mechanics, Springer-Verlag, Berlin 1974, Sez. E.
5
L. K. Luneburg, Mathematical Theory of Optics Univ. of Califfornia Press, Berkeley, 1964; M. Kline
and I. W. Kay, Electromagnetic Theory and Geometrical Optics, Wiley (Interscence), New York, 1965
30
Metodi matematici della Elettrodinamica
Soluzione: Inserendo i termini della serie (1.16) nella (1.15) si ottiene:
N
Qm (r)
−N+2
m = o ko
(iko )
m=0
dove
Q0 = |∇S|2 − n2
Q1 = ∇2 S + 2∇S · ∇ A0
Qm =
∇2 S + 2∇S · ∇ Am−1 + ∇2 Am−1
L’Eq. (1.17) è soddisfatta a tutti gli ordini solo se
Qm = 0 m = 0, 1, . . .
In particolare si ha
|∇S|2 = n2
∇2 S + 2∇S · ∇ A0 = 0
(1.18)
All’equazione (1.18-a) si dà il nome di equazione dell’iconale mentre (1.18-b) descrive
il trasporto dell’ampiezza A0 lungo i raggi.
1.6.2
Equazione dei raggi
Esercizio 1.6.2 Con riferimento all’esercizio precedente ricavare (a) l’equazione della
famiglia di curve perpendicolarri alle superfici a S=costante e (b) l’equazione di trasporto
dell’ampiezza A0 lungo i raggi
Soluzione: (a) Assegnata l’iconale S (r) si definiscono raggi quelle curve perpendcolari
in ogni punto alle superfici a S (r) = cos tante.
I raggi sono pertanto curve s (r) le cui tangenti sono date da:
dr
∇S
∇S
= ŝ (r) =
=
ds
|∇S|
n (r)
In particolare differenziando lungo un raggio si ha:
d
d
[n (r) ŝ (r)] = ∇S =
ds
ds
dr
1
· ∇ ∇S = (∇S · ∇) ∇S
ds
n
D’altra parte
Pertanto
2n∇n = 2 (∇S · ∇) ∇S
d
[n (r) ŝ (r)] = ∇n
ds
(b) L’Eq. (1.18-b) si pò riscrivere nella forma:
A20 ∇2 S + nŝ · ∇A20 = 0
ovvero
∇ · A20 nŝ = 0
1.7 Funzioni speciali
31
Esercizio 1.6.3 Un modo equivalente di ricavare le traiettorie dei raggi fa uso della
condizione imposta al cammino ottico L tra due generici punti PA,B
PB
L=
n (r (s)) ds
PA
di risultare stazionario (minimo o massimo) per una generica deformazione del cammino.
Ricavare l’equazione per le geodesiche utilizzando le rispettive equazioni di Eulero-Lagrange
Soluzione: Tenuto conto che
ds2 = grr dr2 + gθθ dθ2 + gφφ dφ2
L si può riscrivere nella forma
PB
L=
rB
n (r (s)) ds =
PA
n (r)
2
2
grr ṙ2 + gθθ θ̇ + gφφ φ̇ ds
rA
dove . indica la derivata rispetto a s.
Minimizzare questo funzionale equivale ad integrare le equazioni di Eulero -Lagrange
d
∂
n α
ds ∂ ẋ
1.7
1.7.1
2
2
grr ṙ2 + gθθ θ̇ + gφφ φ̇ =
∂
2
2
n grr ṙ2 + gθθ θ̇ + gφφ φ̇
α
∂x
Funzioni speciali
Funzioni di Bessel, Hankel e Struve
Le funzioni di Bessel sono integrali generali dell’equazione di Bessel
x2
d2
d
y + x y + x2 − ν 2 y = 0
2
dx
dx
è espresso dalla combinazione
y (x) = Cj Jν (x) + CY Yν (x)
di funzioni di Bessel di primaJν (x) e seconda specie Yν (x). Mentre Jν (x) è una funzione
regolare di x per arg (ν) < π, esprimibile mediante la serie
x
Jν (x) =
2
ν
∞
n=0
2n
(−1)n x2
n!Γ (ν + n + 1)
Yν (x) è singolare per x = 0.
Per ν non intero Jν (x) e J−ν (x) sono linearmente indipendenti mentre
Yν (x) =
Jν (x) cos (νπ) − J−ν (x)
sin (νπ)
Quelle di Hankel sono combinazioni di
H (1,2) (x) = Jν (x) ± iYν (x)
32
Metodi matematici della Elettrodinamica
Nel caso dell’equazione di Bessel non omogenea
d2
d
4 (x/2)ν+1
2
2
y+x y+ x −ν y = √
x
dx2
dx
πΓ ν + 12
2
la soluzione generale è data da
y (x) = Cj Jν (x) + CY Yν (x) + Hν (x)
dove
x
Hν (x) =
2
∞
ν+1
n=0
2n
(−1)n x2
Γ n + 32 Γ n + ν +
(1.19)
3
2
sta per la funzione di Struve
Esercizio 1.7.1 Dimostrare che le funzioni di bessel Jn (x) di indice n intero sono rappresentate dagli integrali
Jn (x) =
2π
1
2πin
eix cos θ+inθ dθ
(1.20)
0
Soluzione: Sviluppando eix cos θ in serie si ha
1
2πin
2π
ix cos θ+inθ
e
0
∞
1
dθ =
2πin
1
=
2π
k=0
∞
k=0
∞
=
k−n
=0
2
(ix)k
k!
1 x
i
k! 2
2π
cosk θeinθ dθ
0
k
k
m=0
x k
i2
=
k+n
k−n
!
!
2
2
2π
k
m
x
2
ei(2m−k+n)θ dθ
0
n
∞
2m
(−1)m x2
= Jn (x)
(n
+
m)!m!
m=0
Esercizio 1.7.2 Dimostrare la seguente identità di Jacobi
−ix cos θ
e
=
∞
(−i)n Jn (x) einθ
(1.21)
n=−∞
Soluzione: Essendo e−ikρ cos θ una funzione periodica in θ si può rappresesentare sotto
forma di serie di Fourier:
∞
e−ix cos θ =
Cn einθ
n=−∞
dove
2π
1
Cn =
e−ix cos θ−inθ dθ
2π 0
Tenuto conto della (1.20) ne segue la (1.21).
Esercizio 1.7.3 Dimostrare che l’onda piana eikr cos θ si può espandere in una serie di
onde sferiche del tipo
eikr cos θ =
∞
l=0
(2l + 1) il Jl (kr) Pl (cos θ)
1.7 Funzioni speciali
33
n
Esercizio 1.7.4 Dimostrare che Jn cosh
, con α reale, tende per n → ∞ asintoticaα
mente a:
n
exp [n (tanh α − α)]
√
Jn
(1.22)
≈
cosh α
2πn tanh α
Soluzione: Utilizzando la rappresentazione integrale (1.20) si ha
Jn
2π
cos θ
1
ein( cosh α +θ) dθ
n
2πi 0
= I1 (n) − I2 (n)
n
cosh α
=
dove
I1 (n) =
1
2πin
ein( cosh α +θ) dθ
cos θ
Γ
∞
1
I2 (n) = −
2πin−1
ein( cosh α +iθ) dθ
cosh θ
0
Γ è un cammino che parte da θ = 0, passe per il punto sella θSD , legato ad α dalla
relazione:
π
sin θ SD = cosh α = sin
+ iα
2
ovvero
π
θSD = + iα
2
per proseguire |θ| → ∞. L’integrale I1 (n) e I2 (n) si possono riscrivere nella forma
I1 (n) =
1
2πin
I2 (n) = −
e
cos θ
sin θ SD
in
Γ
∞
1
2πin−1
+θ
dθ
en(i cosh α −θ) dθ
cosh θ
0
Applicando il metodo dello steepest descent6 all’integrale
in
einh(θ) dθ =
Γ
e
cos θ
sin θ SD
+θ
dθ
Γ
si ha
1
2πin
Γ
einh(θ) dθ −→
n→∞
1
2πin
2π
nh′′ (θ
=
1
2πin
=
exp [n (tanh α − α)]
√
+ 0 n−1
2πn tanh α
I2 (n) −→ 0
n→∞
v. Sez. 1.5.1.2
eiπ/4+inh(θSD ) + 0 n−1
2π
eiπ/4+in(cot θSD +θSD ) + 0 n−1
−n cot (θSD )
D’altro canto
6
SD )
34
Metodi matematici della Elettrodinamica
Esercizio 1.7.5 Discutere gli andamenti delle funzioni di Bessel modificate di prima e
seconda specie.
2
Jν (ix) = iν Iν (x) , Hν(1) (ix) = i−ν−1 Kν (x) .
π
Di particolare rilevanza nel calcolo dello spettro della radiazione di sincrotrone sono le
funzioni
∞
1
3
iη x + x3 dx ,
2
3
−∞
∞
1
3
K2/3 (η) =
x exp iη x + x3 dx ,
2
3
−∞
4
K5/3 (η) =
K2/3 (η) + K1/3 (η)
3η
K1/3 (η) =
1.7.2
exp
(1.23)
Funzioni di Mathieu
Esercizio 1.7.6 Un’equazione particolarmente importante è quella di Mathieu7
d2
+ a − 2q cos (2ζ) u(ζ) = 0 ,
(1.24)
dζ 2
Discutere le proprietà degli integrali di questa equazione. In particolare discuterne la
stabilità, ovvero per quali valori dei parametri a e q le soluzioni risultano limitate per
ζ ∈ (−∞, ∞) .
Soluzione: L’equazione di Mathieu (Eq. (1.24)) ammette in generale due integrali
C(a, q, ζ) e S(a, q, ζ) rispettivamente pari e dispari rispetto a ζ rappresentabili nella forma
di Floquet
C(a, q, ζ) = eiµζ fµ (q, ζ) + e−µζ fµ (q, −ζ)
S(a, q, ζ) = eiµζ fµ (q, ζ) − e−µζ fµ (q, −ζ) ,
(1.25)
con fµ (q, ζ) = fµ (q, ζ + π). Quando l’esponente caratteristico risulta reale sia C che S
sono limitate e l’integrale dell’equazione risulta stabile. Pertanto sul piano q,a si possono
inviduare delle regioni di stabilità definendole come quelle per cui µ (q, a) risulta reale.
In particolare per una sequenza di valori a0 (q) , a1 (q) , . . . C(an (q) , q, ζ) si riduce ad
una funzione periodica cen (q, ζ) = cen (q, ζ + π). Lo stesso accade per S(bn (q) , q, ζ) =
sen (q, ζ) = sen (q, ζ + π) per la sequenza di valori a = b1 (q) , b2 (q) , . . .
L’insieme di autovalori a0 (q) < b1 (q) < a1 (q) < b2 (q) < a2 (q) < . . . ripartisce l’intervalo −∞ < a < ∞ in intervalli a0 (q) < a < b1 (q), a1 (q) < a < b2 (q) , . . . all’interno
dei quali sia C che S sono stabili.
In particolare µ (q, a) tende per assegnato q ad un intero m per a → an e ad m + 1 per
a → bn+1 In particolare per µ intero, ovvero per a = am , bp , eiµζ fµ (q, ζ) e e−iµζ fµ (q, −ζ)
non sono più soluzioni indipendenti, per cui la (1.25) non è più valida. Mentre S(bn , q, ζ) =
sen (q, ζ) è periodica, S(an , q, ζ) è invece una funzione aperiodica.
Quando il punto rappresentativo (a, q) cade in una delle regioni di Fig. 1.2 an (q) <
a < bn+1 (q) eiµζ si comporta come un fattore di fase, per cui l’integrale generale risulta
stabile8 .
7
8
N. W. McLachlan, Theory and Applications of Mathieu Functions, Oxford Univ. Press, Oxford, 1947;
M. Abramowitz and I. A. Stegun,“Handbook of Mathematical Functions”, Dover Publ., N.Y. (1970),
Cap. 20.
per calcolare le regioni di stabilità v.p.e. http://www.physics.drexel.edu/~tim/open/mat/node6.html
1.7 Funzioni speciali
35
Figura 1.2: Regioni di stabilità delle funzioni di Mathieu. Quando la coppia di parametri
a, q cade nelle regioni comprese tra la curva più in basso e quella successiva, tra la terza
e la quarta, e così via l’esponente caratteristico µ risulta reale.
1.7.3
Funzione di Airy
Le funzioni di Airy Ai (x) e Bi (x) sono integrali dell’ED
d2
+ x f (x) = 0 ,
dx2
(1.26)
In particolare Ai (x) è rappresentato dall’integrale
Ai (x) =
1
π
∞
cos
0
t3
1
+ xt dt =
3
2πi
i∞
exp
−i∞
z3
− zt dz
3
(1.27)
Ai (x) decresce esponenzialmente per x → ∞ mentre è una funzione oscillante per
x < 0:
e−(2/3)x x > 0
|x|−1/4
Ai (x) = √
2 sin 23 |x|3/2 + π4 x < 0
2 π
mentre Bi (x) esplode per x → ∞
3 |x|−1/4 (2/3)x3/2
Bi (x) = √
e
x>0
π
1.7.4
Integrali ellittici
ϕ
dt
K (ϕ, ξ) =
0
ϕ
E (ϕ, ξ) =
0
1 − ξ sin2 t
1 − ξ sin2 tdt
(1.28)
36
Metodi matematici della Elettrodinamica
Figura 1.3: Rappresentazione espansa della Fig.
stabilitò a0 (q) < a < b1 (q)
precedente relativa alla regione di
Figura 1.4: Integrali ellittici E (ξ) (curva in basso) e K (ξ) (curva in alto) in funzione di z
π
,ξ
2
π
E (ξ) = E
,ξ
2
K (ξ) = K
sono detti integrali ellittici completi.
Sviluppando in serie rispetto a ξ si ottiene:
ϕ 1
3
− sin (2ϕ) ξ +
(12ϕ − 8sin (2ϕ) + sin (4ϕ))ξ 2 + O[ξ]3
4 8
256
1
1
E (ϕ, ξ) = ϕ + (−2ϕ + sin (2ϕ))ξ +
(−12ϕ + 8sin (2ϕ) − sin (4ϕ))ξ 2 + O[ξ]3
8
256
(1.29)
K (ϕ, ξ) = ϕ +
Capitolo 2
Richiami di teoria della relatività
2.1
4-Vettori e 4-tensori dello spazio-tempo
In relatività un evento P è associato ad un punto
−
→
x = xµ eµ
di uno spazio vettoriale avente come basi i vetori eµ e componenti:
xµ = (x0 = ct, x1 = x, x2 = y, x3 = z)
Sulle basi eµ è definito un prodotto scalare

−1
 0
eµ · eν = 
 0
0
che per sistemi inerziali assume la forma:

0 0 0
1 0 0 
 = gµν
0 1 0 
0 0 1
Ne segue che il prodotto scalare di vettori generici è espresso da:
x · y = xµ y ν (eµ · eν ) = xµ y ν gµν
dove si è omesso, seguendo la convenzione di Einstein, il simbolo di somma sugli indici
che appaiono ripetuti in alto ed in basso.
Alle componenti xµ , dette controvarianti, sono associate le componenti covarianti
definite da:
xµ = xν gµν .
Pertanto si avrà
xµ yµ = xµ y µ .
I 4-vettori si possono suddividere in tre classi secondo che xµ xµ > 0, xµ xµ = 0 e xµ xµ < 0.
Nel primo caso il vettore è detto space-like, nullo nel secondo, time-like nel terzo.
Lo scalare
1
dτ =
−dxµ dxµ
(2.1)
c
associato alla distanza tra due eventi relativi ad una particella che descrive una traiettoria
nello spazio-tempo rappresenta l’incremento del tempo proprio della particella. Pertanto
37
38
Richiami di teoria della relatività
il tempo proprio è quello che vede scorrere un osservatore solidale ad ogni istante con la
particella,
In relatività si distingue tra sistema di laboratorio KL e sistema proprio KP (τ ) associato al tempo proprio di una particella. Quest’ultimo è definito come il sistema inerziale
in cui l’elettrone si trova a riposo all’istante t considerato. Inoltre KP (τ + dτ ) si ottiene
da KP (τ ) spostando parallelamente gli assi coordinati. Gli intervalli di tempo dt in KL
e dτ in KP sono legati tra loro dal fattore relativistico
1
γ=
1 − |β|2
=
dt
.
dτ
con β (t) = v (t) /c.
Particolarmente importante è il 4-vettore della velocità
u=
d µ
x eµ = uµ eµ
dτ
di quadrato pari a
uµ uµ = −c2 .
2.1.1
Trasformazioni di Lorentz
Si consideri un sistema inerziale K2 che si muove con velocità costante v = cβ rispetto al
sistema K1 e con assi a quest’ultimo paralleli. Un punto xµ1 in K1 ha in K2 coordinate
xµ2 = Pνµ xν1
con Pνµ rappresentato in notazione matriciale da
Pνµ =
γ
−γβ T
T
−γβ (γ − 1) β̂ β̂ + 1
.
(2.2)
Nel caso più generale in cui gli assi di K2 non siano paralleli a quelli di K1 si ha
xµ2 = Λµν xν1
dove
Λµν = Rαµ Pνα
risulta composto da una trasformazione pura Pνµ per una rotazione spaziale
Rνµ =
1 0
0 R
con R una matrice unitaria nello spazio x, y, z. Pertanto nel caso generale Λµν assume la
forma matriciale
γ
−γβ T
µ
T
Λν =
−γR · β R · (γ − 1) β̂ β̂ + 1
Le trasformazioni di Lorentz formano un gruppo1 .
1
v.p.e. A. O. Barut, Electrodynamics and Classical Theory of Fields, Dover Ed. N.Y. 1980 parte I sez.
5
2.1 4-Vettori e 4-tensori dello spazio-tempo
39
Esercizio 2.1.1 Verificare la seguente identità
T
(γ − 1) β̂ β̂ + 1⊥
n
T
= (γ n − 1) β̂ β̂ + 1
Soluzione: Utilizzando un sistema cartesiano con

1
T
(γ − 1) β̂ β̂ + 1 =  0
0
da cui segue l’identità (3.21).
(2.3)
l’asse ẑ parallelo a β si ha

0 0
1 0 
0 γ
Esercizio 2.1.2 Dimostrare che det Λµν = 1
Soluzione: Tenuto conto che
det [Λµν ] = det [Rµα Pνα ] = det [Rµα ] det [Pνα ]
e che
det [Rµα ] = 1
si ha:
det Λµν
γ
−γβ T
T
−γβ (γ−1) β̂ β̂ + 1
= det
T
= γ det (γ−1) β̂ β̂ + 1 − γ 2 β T · β
= γ 2 − γ 2β 2 = 1
Esercizio 2.1.3 (a) Verificare la seguente proprietà
ΛT · G · Λ = G
con G = [gµν ] e Λ = [Λµν ]. Ne
Lorentz e (c) che

i
 0
L=
 0
0
(2.4)
segue (b) che xµ y ν gµν è invariante per trasformazioni di
0
1
0
0
è una matrice initaria..
0
0
1
0


0

0 
 · [Λµν ] · 

0 
1
−i
0
0
0
0
1
0
0
0
0
1
0

0
0 

0 
1
Soluzione: Dal momento che
ΛT · G · Λ = PT ·
1 0
0 RT
1 0
0 R
·G·
· [P]
tenuto conto che
1 0
0 RT
·G·
1 0
0 R
=
1 0
0 RT
·
−1 0
0 R
=G
40
Richiami di teoria della relatività
si ha:
γ
−γβ T
T
−γβ (γ − 1) β̂ β̂ + 1
ΛT · G · Λ =
−1 0
0 1
=
−γ
γβ T
T
−γβ (γ−1) β̂ β̂ + 1
·
=G
(b) Dati due generici 4-vettori x, y per le trasformate x′ = Λ · x, y′ = Λ · y si ha
x′T · G · y′ = xT · ΛT · G · Λ · y = xT · G · y
(c) Ponendo

−i
 0
G=
 0
0
0
1
0
0
2
0
0 

0 
1
0
0
1
0
l’Eq. (2.4) si può riscrivere nella forma
L† · L = 1µν
il che prova che L è unitaria.
Esercizio 2.1.4 Comporre una trasformazione di Lorentz omogenea con una pura
′µ α α
Soluzione: Il prodotto Λ′′µ
ν = Pα Rβ Pν è dato in forma matriciale da:
−γ ′ β ′T
γ′
′ ′T

= 
=
−γ ′ β ′ (γ ′ −1) β̂ β̂
′
′
+1
·
−γ γβ − γ
γ ′′
−γ ′′ R′′ · β ′′ R′′ ·
Ne segue che:
T
−γR · β R· (γ−1) β̂ β̂ + 1
γ ′ γ 1 + β ′T · R · β
γ ′2
β ′ β ′T
γ ′ +1
−γβ T
γ
′
′
−γ ′ γβ T − γ ′ β ′T · R ·
T
+ 1 · R · β γ γβ · β +
−γ ′′ β ′′T
γ ′′2
β ′′ β ′′T + 1
γ ′′ +1
γ ′2
β ′ β ′T
γ ′ +1
γ2
ββ T
γ+1
+1 ·R·
γ ′′ = γ ′ γ 1 + β ′T · R · β
Sostituendo 1 + β ′T · R · β con γ ′′ /γ ′ γ si ha
γ ′′ β ′′ = γ
γ ′ + γ ′′
β + γ ′ RT · β ′
γ+1
e
′′ ′′T
R′′ =
γ ′′−1 − 1 β̂ β̂
·
+1
γ2
γ ′2 ′ ′T
γ ′ γ (γ + γ ′ + γ ′′ ) ′ T
R · ββ T + ′
β β · R+ ′
ββ +R
γ+1
γ +1
(γ + 1) (γ + 1)
+1
γ2
ββ T
γ+1
+1


2.1 4-Vettori e 4-tensori dello spazio-tempo
41
Esercizio 2.1.5 Una rotazione finita dell’angolo Φ intorno alla direzione n̂ può essere
rappresentata equivalentemente o dalla matrice R 3×3 o da Q 2×2 utilizzante i parametri
di Cayley-Klein. Confrontare le due rappresentazioni2
Soluzione: Una rotazione dell’angolo Φ intorno alla direzione n̂ trasforma il raggio
vettore r in:
r′ = R (n̂, Φ) · r
con R (n̂, Φ) una matrice 3×3 dipendente da 3 parametri: direzione n̂ e angolo di rotazione
Φ. Per calcolare R (n̂, Φ) risulta utile stabilire un omomorfismo tra lo spazio delle matrici
3 × 3 a elementi reali e le matrici unitarie Q (n̂, Φ) 2 × 2 a temini complessi
α
β
∗
−β −α∗
Q (n̂, Φ) =
(2.5)
unimodulari, cioè tali che
det Q (n̂, Φ) = |α|2 + |β|2 = 1.
L’omomorfismo associa il raggio vettore r alla matrice Hermitiana
P (= r) =
z
x − iy
x + iy
−z
che viene trasformata da Q per similitudine in P′ (= r′ ):
P′ (= r′ ) =
x′ − iy ′
z′
x′ + iy ′
−z ′
=Q·
z
x − iy
x + iy
−z
· Q†
(2.6)
Introducendo i parametri di Eulero, detti anche di Cayley-Klein,
α = e0 + ie3
β = e2 + ie1
si ha per una rotazione elementare dΦ intorno alla direzione n̂
1
dQ = i n̂ · σdΦ
2
dove n̂ · σ = ni σ i con σ i matrici di Pauli. Pertanto, per una rotazione finita di un agolo
Φ si ha:
Φ
Q (n̂, Φ) = exp i n̂ · σ
2
Tenendo conto delle proprietà delle σ i :
σ 21
σ1σ2
T r[σ i ]
det[σ i ]
2
=
=
=
=
σ 22 = σ 23 = σ 0 = 1
−σ 2 σ 1 = iσ 3 , σ 2 σ 3 = −σ 3 σ 2 = iσ 1 , σ 3 σ 1 = −σ 1 σ 3 = iσ 2
0
−1
v.p.e. H. Goldstein, Classical Mechanics, Addison Wesley, Reading 1980 Sez. 4.5
(2.7)
42
Richiami di teoria della relatività
si vede facilmente che
(n̂ · σ)2k = 1
(n̂ · σ)2k+1 = n̂ · σ
(2.8)
Pertantol’esponenziale della matrice i Φ2 n̂ · σ è espresso da
Φ
Q (n̂, Φ) = ei 2 n̂·σ
=
k
(−1)k
(2k)!
i
Φ
2
2k
+ in̂ · σ
k
(−1)k
(2k + 1)!
Φ
2
2k+1
Φ
Φ
= σ 0 cos + in̂ · σ sin
2
2
Pertanto i corrispondenti parametri ei sono dati da
Φ
2
Φ
e = n̂ sin
2
e0 = cos
Dalla (2.6) discende che:
r′ =
e20 − e21 − e22 − e23 + 2ee· − 2e0 e× r
ovvero
R (n̂, Φ) = [cos Φ + (1 − cos Φ) n̂n̂ · − sin Φn̂×] r
Esercizio 2.1.6 Comporre due rotazioni
Soluzione: Moltiplicando le matrici Q relative alle due rotazioni si ha:
Φ′ ′
Φ
n̂ · σ exp i n̂ · σ
2
2
′
′
Φ
Φ
Φ
Φ
=
σ 0 cos
+ in̂′ · σ sin
· σ 0 cos + in̂ · σ sin
2
2
2
2
′
′
′
Φ
Φ
Φ
Φ
Φ
Φ
+ n̂′ sin cos
·σ
= σ 0 cos cos + i n̂ sin cos
2
2
2
2
2
2
Φ′
Φ
− (n̂′ · σ) (n̂ · σ) sin sin
2
2
Q (n̂′ , Φ′ ) · Q (n̂, Φ) = exp i
D’altra parte dalle Eqq. (2.7) discende che:
(n̂′ · σ) (n̂ · σ) = n′i nj σ i σ j = n̂′ · n̂σ 0 + i (n̂′ × n̂) · σ
Pertanto
Φ′ ′
Φ
n̂ · σ exp i n̂ · σ
2
2
′′
Φ
Φ′′
′′
+ in̂ · σ sin
σ 0 cos
2
2
Q (n̂′′ , Φ′′ ) = exp i
=
(2.9)
2.1 4-Vettori e 4-tensori dello spazio-tempo
43
dove
Φ′
Φ
Φ′′
= cos cos − n̂′ · n̂
2
2
2
Φ′′
Φ
Φ′
Φ′
Φ
′′
n̂ sin
= n̂ sin cos
+ n̂′ sin cos − n̂′ × n̂
2
2
2
2
2
cos
Esercizio 2.1.7 Estendere la rappresentazione delle rotazioni spaziali mediante la matrice Q (n̂, Φ) di (2.5)ai ”boost” Pνµ . Servirsi a tal fine di matrici hermitiane unimodulari
Q (β) 2 × 2 a temini complessi3
Q (β) =
tali che
α β
β∗ δ
det Q (β) = αδ − |β|2 = 1.
ed associare il 4-vettore
xµ = (x0 = ct, x1 = x, x2 = y, x3 = z)
alla matrice Hermitiana
S=
che viene trasformata da Q (β) in S′ :
S′ =
z ′ + ct′ x′ − iy ′
x′ + iy ′ −z ′ + ct′
z + ct x − iy
x + iy −z + ct
= Q (β) ·
z + ct x − iy
x + iy −z + ct
· Q† (β)
Soluzione: Per un boost elementare dΨ lungo la direzione β̂ si ha
1
dQ = − β̂ · σdΨ
2
Pertanto per un boost di ampiezza generica β = tanh Ψ/2 si ha:
Ψ
Q (β) = exp − β̂ · σ
2
Dalle relazioni (2.8) discende che
Ψ
Q (β) = e− 2 β̂·σ
=
k
(−1)k
(2k)!
−
Ψ
2
2k
− β̂ · σ
k
(−1)k
(2k + 1)!
Ψ
2
2k+1
Ψ
Ψ
= σ 0 cosh − β̂ · σ sinh
2
2
3
v.p.e. A. O. Barut, loc. cit. p. 38 Parte I Sez. 5; H. Goldstein, loc. cit. p. 41, Sez. 7.1
(2.10)
44
Richiami di teoria della relatività
2.1.2
Riferimento proprio
Il moto di una particella che descrive un’orbita parametrizzata dal tempo proprio τ , viene
descritto assegnando ad ogni τ un sistema inerziale KP (τ ) tale che KP (τ + ∆τ ) si ottenga
applicando a KP (τ ) una trasformazione di Lorentz omogenea pura Pνµ (τ ) del tipo (2.2)
con β 21 = β (τ + ∆τ ) − β (τ ) dove β (τ ) è la velocità della particella vista in KL al tempo
τ . Alle KP (τ ) si dà il nome di riferimento proprio.
Questo però non deve far pensare che KP (τ ′ ) al tempo generico τ ′ si possa ottenere applicando a KP (τ ) una trasformazione pura. Infatti il prodotto di due generiche
trasformazioni pure è equivalente ad una pura per una rotazione:
Pαµ (τ + ∆τ , τ ) Pνα (τ ) = Rαµ (τ + 2∆τ ) Pνα (τ + 2∆τ )
Può sorprendere che mentre gli assi spaziali di KP (τ + ∆τ ) sono paralleli a quelli
di KP (τ ) e quelli di KP (τ + 2∆τ ) a quelli di KP (τ + ∆τ ), quelli di KP (τ + 2∆τ ) e
KP (τ ) in genere non lo sono. Questo significa che non vale la proprietà transitiva per il
parallelismo degli assi spaziali nello spazio-tempo.
Per rendersene conto si assuma che la particella si muova al tempo τ con velocità β
(τ ) rispetto a KL . Dal momento che KP (τ + ∆τ ) si muove con velocità δβ rispetto a
KP (τ ), con | δβ | ≪ 1, trascurando i contributi di ordine superiore a δβ si ha
1
−δβ T
−δβ
1
Pαµ (τ + ∆τ , τ ) =
.
Pertanto,
Λµν (τ + ∆τ ) = Pαµ (τ + ∆τ , τ ) Λαν (τ )
γ
−γβ T
γ2
−γβ 1+ γ+1
ββ T
=
1
−δβ T
−δβ
1
=
γ
ββ T ]
γ
−γβ T − δβ T · [1+ γ+1
γ2
−γ(β+δβ)
1+γδββ T + γ+1
ββ T
·
2
In particolare risulta
Rαµ (τ + ∆τ )−1 Λ = P (τ + ∆τ )
2
γ
γ
−γβ T − δβ T · [1+ γ+1
ββ T ]
γ2
−γ(β+δβ)
1+γδββ T + γ+1
ββ T
=
1 0
0 R−1
=
γ
γ
−γβ T − δβ T · [1+ γ+1
ββ T ]
γ2
−γR−1 · (β + δβ) R−1 · [1+γδββ T + γ+1
ββ T ]
·
2
Perchè R−1 Λ rappresenti una trasformazione pura i due elementi fuori diagonale di quest’ultima matrice debbono essere il trasposto l’uno dell’altro,
R−1 · (β + δβ) = β +
1
γ
1+
γ2
ββ T
γ+1
· δβ
(2.11)
D’altra parte R−1 deve rappresentare una rotazione infinitesima di un angolo ∆θ, per cui
R−1 = 1 + ∆θ×
2.2 4-Tensori elettromagnetici
45
con ∆θ parallelo all’asse di rotazione. Pertanto, combinando questa espressione di R−1
con (2.11) si ottiene
γ−1 1
∆θ = −
β × δβ
γ β2
Finalmente, dividendo per ∆τ si perviene alla velocità di precessione di questo movimento
ωT =
γ −1
∆θ
γ2 v × a
= − 2 2 v×a =−
∆τ
γ + 1 c2
β c
(2.12)
In conclusione, il riferimento proprio KP (τ ) visto nel sistema di laboratorio KL effettua
un movimento di precessione con la velocità angolare istantanea di Thomas (v. Fig. ??).
Precessione di Thomas di un riferimento che descrive un’orbita circolare
2.2
4-Tensori elettromagnetici
I vettori E, D, H, B di un campo e.m. in un mezzo materiale soddisfano le equazioni di
Maxwell4 :
∂
∇ × E(r, t) = − B(r, t) ,
∂t
∂
∇ × H(r, t) =
D(r, t) + J(r, t) − ∇ × M ,
∂t
∇ · D(r, t) = ρext (r, t) ,
∇ · B(r, t) = 0 .
(2.13)
dove
ρext (r; t) =
n
J(r; t) =
n
4
qn δ (3) (r − rn (t)) C m−3 ,
qn (3)
δ (r − rn (t))pn (t) A m−2 ,
mn
per una presentazione accurata delle proprietà e dei metodi di integrazione di queste equazioni v.p.e.
G. Franceschetti, “Campi Elettromagnetici”, Boringhieri, Torino 1983 Cap.6; per vari aspetti dell’elettrodinamica v.p.e. R. Stroffolini, “Lezioni di Elettrodinamica” a cura di V. Marigliano-Ramaglia
e F. Ventriglia, Bibliopolis, Napoli 2001.
46
Richiami di teoria della relatività
con qn , rn e pn relativi alla carica n-esima introdotta dall’esterno: ovvero, ρext non tiene
conto delle cariche (elettroni e nuclei dei costituenti atomici) del mezzo materiale. M(r, t)
rappresenta la densità del momento magnetico dovuto ai momenti angolari e di spin dei
nuclei e degli elettroni.
Nel vuoto ed in unità SI l’induzione magnetica B (Wb m−2 ) ed il campo elettrico E (V
−1
m ) sono legati al campo magnetico H (A m−1 ) ed allo spostamento elettrico D (C/m2 )
dalle relazioni
9
E = ε−1
0 D = 36π × 10 D ,
B = µ0 H = 4π × 10−7 H ,
(2.14)
dove ε0 (C V−1 m−1 ) e µ0 (Wb A−1 m−1 ) rappresentano la permettività elettrica e permeabilità magnetica. In particolare, ε0 µ0 = 1/c2 .
In molti casi si vuole conoscere il campo creato da una distribuzione di cariche esterne
qnext . Ad esempio qnext può rappresentare le cariche che diffondono in un semiconduttore
posto a contatto con un altro materiale. Il sistema risponde riassestando la disposizione
spaziale dei nuclei e modificando le funzioni d’onda elettroniche, dando così luogo ad una
densità di carica indotta −enind , intesa come la modifica della densità di carica elettronica
preesistente dovuta alla distribuzione qnext . Si distingue quindi tra vettore spostamento
D e campo elettrico E legati rispettivamente a qnext e qnext + ρind dalle relazioni
∇ · D(r, t) = qnext (r, t) ,
ε0 ∇ · E(r, t) = qnext (r, t) + ρind (r, t) ,
In un mezzo materiale si pone
D = P + ε0 E,
B = µ0 (H + M)
dove P e M rappresentano rispettivamente la polarizzazione e la magnetizzazione legato
ai momenti magnetici orbitali e di spin
P(r, t) = D(r, t) − ε0 E(r, t) C m−2
−1
M(r, t) = µ−1
.
0 B(r, t) − H(r, t) A m
In particolare
∇ · P (r,t) = −enind (r, t) ,
∇ · M (r,t) = ∇ · H (r,t)
Pertanto P(r, t) è dovuto al riassestamento delle cariche del materiale prodotto da next ,
Esercizio 2.2.1 Riscrivere le equazioni di Maxwell (a) per campi sinusoidali H(r, t) =
e−iωt H̃(r), B(r, t) = e−iωt B̃(r), E(r, t) = e−iωt Ẽ(r), D(r, t) = e−iωt D̃(r) e (b) per le relative
trasformate di Fourier 4-dimensionali H(r, t) = e−iωt+ik·r H̃, B(r, t) = e−iωt+ik·r B̃, E(r, t) =
e−iωt+ik·r Ẽ, D(r, t) = e−iωt+ik·r D̃
Soluzione: (a) Trasformando rispetto al tempo
Ẽ(r) =
∞
−∞
eiωt E(r, t)dt
2.2 4-Tensori elettromagnetici
47
le Eqq. (2.13) si trasformano in:
∇ × Ẽ(r) = iω B̃(r) ,
∇ × H̃(r) = −iω D̃(r) + J̃(r) ,
∇ · D̃(r) = ρ̃ext (r) ,
∇ · B̃(r) = 0 .
(2.15)
(b) Trasformando ulteriormente rispetto ad r queste ultime equazioni si trasformano
∂
in un sistema di equazioni algebriche ∂t
→ −iω, ∇ → ik nei vettori H̃, B̃, Ẽ, D̃, P̃ e
nelle densità q ñext , −eñind , tutti funzioni di (c−1 ω, k):
k × Ẽ = ω B̃ ,
k × H̃ = −ω D̃ + J̃ ,
ik · D̃ = ρ̃q ,
(2.16)
ik · B̃ = 0 .
Esercizio 2.2.2 Partendo dall’equazione di conservazione della densità di corrente elettrica J (A m−2 ) e di carica ρext (C m−3 ),
∇·J+
∂
ρ =0
∂t ext
(2.17)
dimostrare che J µ = cρext , J è un 4-vettore detto 4-corrente.
∂
Soluzione: Dal momento che l’operatore c ∂t
, ∇ si comporta da 4-vettore e che la
relazione (2.17) risulta valida in tutti i riferimenti inerziali cρext , J risulta essere un
4-vettore.
∂
∂α Jα = ∇ · J + ρext = 0 ,
(2.18)
∂t
che stabilisce un bilancio tra la variazione locale di carica ed il flusso di corrente.
Esercizio 2.2.3 Costruire con le coppie di vettori E, B e H, D le due coppie di matrici
antisimmetrice Fµν e Gµν
Fαβ =
0
1
E
c
− 1c ET
B
, Gαβ =
0 −cDT
cD
H
,
(2.19)
dove
B0000000000000000000000000000000000000000000000000000000000000000000000000000000000000000
(a) Verificare che F e G soddisfano rispettivamente le equazioni
∂γ Fαβ + ∂α Fβγ + ∂β Fγα = 0 ,
∂β Gαβ = −Jα + (∇ × M)α ,
(2.20)
48
Richiami di teoria della relatività
con Jα = (cρext , Jx , Jy , Jz ) = (cρext , J) la 4-corrente e posto ∂µ = ∂x∂µ . M(r, t) è legata ai
momenti di spin dei nuclei e degli elettroni, dipendente in genere non linearmente da B e
dalla temperatura. (b) Provare che F e G sono dei 4-tensori invarianti5 per trasformazioni
di Lorentz.
Soluzione: (a) Le componenti Fαβ di F sono date da:


0 − 1c Ex − 1c Ey − 1c Ez
 1 Ex
0
Bz
−By 
c
 ,
Fαβ = 
 1 Ey −Bz

0
B
x
c
1
E
By
−Bx
0
c z
e soddisfano le equazioni
∂
B(r, t) + ∇ × E(r, t) = 0 ,
∂t
∇ · B(r, t) = 0 .
(2.21)
Scegliendo due indici uguali, p.e. α = β, dal momento che Fαβ è antisimmetrico si ha:
∂γ Fαα + ∂α Fαγ + ∂α Fγα = 0 .
Se ad esempio α = 0, β = 1, γ = 2 si ha
1
1
F01 = − Ex , F12 = Bz , F20 = Ey
c
c
Dalle (2.21) discende che
∂2 F01 + ∂0 F12 + ∂1 F20 =
∂
1 ∂
1 ∂
Bz +
Ey −
Ex = 0
∂y
c ∂x
c ∂y
Analogamente se α = 1, β = 2, γ = 3 si ha
F12 = Bz , F23 = Bx , F31 = By
per cui
∂3 F12 + ∂1 F23 + ∂2 F31 = ∇ · B = 0 .
Per quanto riguarda le componenti Gαβ di G si ha


0 −cDx −cDy −cDz
 cDx
0
Hz
−Hy 
 ,
Gαβ = 
 cDy −Hz
0
Hx 
cDz
Hy
−Hx
0
e soddisfano le equazioni
∂
D(r, t) + J(r, t) ,
∂t
∇ · D(r, t) = ρext (r, t) ,
∇ × H(r, t) =
5
(2.22)
J.D. Jackson, Elettrodinamica Classica, Zanichelli, Bologna 2011, Sez. 11.11; C.H. Papas, Theory of
Electromagnetic Wave Propagation, Dover Pub. N. Y. 1988, Sez. 7.1; L. D. Landau e E. M.Lifshitz,
Teoria Classica dei Campi, Editori Riuniti Edizioni Mir, Roma, 1976 Sez. IV.30.
2.2 4-Tensori elettromagnetici
Per α = 0 si ha
49
∂β G0β = ∇ · D = ρext (r, t)
mentre per α = 1 risulta
∂β G1β =
∂
∂
∂
Dx + Hz − Hy = −Jx
∂t
∂y
∂z
dove si è introdotta la 4-corrente Jα = (cρext , Jx , Jy , Jz ) = (cρext , J) e posto ∂µ = ∂x∂µ .
(b) Dal momento che Jα + (∇ × M)α e ∂µ sono dei 4-vettore e che l’Eq. (2.20-b)
deve risultare valida in tutti i riferimenti inerziali Gαβ risulta essere un 4-tensore. Un
ragionamento analogo vale per Fαβ .
Esercizio 2.2.4 (a) Dimostrare che il 4-tensore Fαβ può essere espresso nella forma
Fαβ = ∂ β Aα − ∂ α Aβ
con ∂ µ = ∂x∂µ e Aα = Vc , Ax , Ay , Az il potenziale 4-vettore. (b) Esprimere nel vuoto il
quadrivettore Aµ in funzione della quadricorrente J µ
Soluzione: (a) In vista della (2.20-a) Fαβ può essere espresso nella forma
Fαβ = ∂ β Aα − ∂ α Aβ
(2.23)
con ∂ µ = ∂x∂ µ e Aα = Vc , Ax, Ay , Az il potenziale 4-vettore.
(b) Nel vuoto imponendo ad Aµ di essere un 4-vettore a divergenza nulla (gauge di
Lorentz ):
∂µ Aµ = 0 ,
(2.24)
ed esprimendo Gαβ in funzione di Fαβ tenendo conto delle relazioni (2.14) l’Eq. (2.20-b)
assume la forma
∂µ ∂ µ Aα = −µ0 [J α − (∇ × M)α ] .
(2.25)
ovvero
2
con
2
∂µ ∂ µ A(r, t) = −µ0 (J − ∇ × M) ,
2
∂µ ∂ µ V (r, t) = −
il d’Alambertiano
2
= ∂µ ∂ µ = ∇2 −
1
ρ (r, t) ,
ε0 ext
(2.26)
1 ∂2
,
c2 ∂t2
Esercizio 2.2.5 Trasformare i campi E, B e H, D da un sistema di riferimento inerziale
a riposo ad un altro che si muove con velocità v
Soluzione: Passando da un sistema di riferimento inerziale a riposo ad un altro che si
muove con velocità v il 4-tensore Fαβ si modifica in F′αβ = Λαµ Λβν Fµν . Lo stesso vale per
Gαβ . Dalle espressioni (2.19) discende che per trasformazioni pure (v. Eq. (2.2)) i campi
si modificano come segue:
E′
B′
=
D′
H′
=
γ + (1 − γ) β̂ β̂
cγβ×
γ
− c β×
γ + (1 − γ) β̂ β̂
γ
γ + (1 − γ) β̂ β̂
β×
c
−cγβ×
γ + (1 − γ) β̂ β̂
·
E
B
,
·
D
H
.
(2.27)
50
2.3
Richiami di teoria della relatività
Mezzi in movimento
Esercizio 2.3.1 Si consideri un mezzo dielettrico caratterizzato a riposo da una costante
dielettrica relativa ε̂ = 1 ed una permeabilità magnetica elativa µ̂ = 1. Discutere le
ralazioni costitutive del mezzo quando questo si muove rispetto a KL con velocità costante
v.
Soluzione: I campi E, D, B, H nel sistema Kp a riposo rispetto al mezzo sono legati a
E , D′ , B′ , H′ nel sistema di laboratorio KL dalle relazioni (2.27). Poichè in KP D = ε̂ε0 E
e B = µ0 H da (2.27) discende che
′

=
ε̂ε0 γ + (1 − γ) β̂ β̂


ε̂ε0 γcβ×
γ
−µ−1
0 c β×
µ−1
γ + (1 − γ) β̂ β̂
0
γ
γ + (1 − γ) β̂ β̂
β×
c
−γcβ×
γ + (1 − γ) β̂ β̂
D′
H′
·
E′
B′
·
Separando le componenti del campo longitudinali da quelle trasverse
E′
B′
=
E′
B′
+
D′
H′
=
D′
H′
+
E′⊥
B′⊥
D′⊥
H′⊥
si ottiene
ε̂ε0
1
−µ−1
0 c β×
ε̂ε0 0
0 µ−1
0
·
E′
B′
=
D′
H′
ε̂ε0 cβ×
µ−1
0
·
E′⊥
B′⊥
=
1
−cβ×
1 0
0 β̂×
Moltiplicando per la matrice
1 0
0 β̂×
·
Pertanto
1
−cβ×
·
ε̂ε0
ε̂ε0 cβ×
−1 1
µ0 c β µ−1
0 β̂×
ovvero
ε̂ε0
ε̂ε0 cβ×
−1 1
µ0 c β µ−1
0 β̂×
Invertendo
γ2
1
cβ
1
β
c
1
·
·
D′⊥
H′⊥
·
1
si ottiene
ε̂ε0
ε̂ε0 cβ×
1
−µ−1
β×
µ−1
0 c
0
1 0
0 β̂×
1
β×
c
1
β×
c
=
1
E′⊥
B′⊥
E′⊥
β̂ × B′⊥
ε̂ε0
−1 1
µ0 c β
1 1c β×
=
=
v
1
β
c
1
D′⊥
H′⊥
·
β̂×
1
cβ
=
β̂×
1
β×
c
1
cβ
ε̂ε0 cβ×
µ−1
0 β̂×
·
D′⊥
β̂ × H′⊥
si ottiene
1
− 1c β
−cβ
1
·
ε̂ε0
−1 1
µ0 c β
ε̂ε0 cβ×
µ−1
0 β̂×
·
E′⊥
β̂ × B′⊥
=
D′⊥
β̂ × H′⊥
2.4 Invarianti del campo e.m.
51
da cui in definitiva risulta
D′⊥
β̂ × H′⊥
2.4
ε̂ − β 2 ε0
− (ε̂ − 1) ε0 β
= γ2
c (ε̂ − 1) ε0 β
1 − β 2 ε̂ µ−1
0
·
E′⊥
β̂ × B′⊥
Invarianti del campo e.m.
Esercizio 2.4.1 (a) Discutere la forma dei seguenti invarianti dei tensori e.m. Fµν e
Gµν
(F )
I1
= c5
(F )
I2
c2 µν
=
F Fµν
4
(G)
= c−5
(G)
=
I1
I2
det [Fµν ] = c5
eαβγδ F0α F1β F2γ F3δ
det [Gµν ] = c−5
eαβγδ G0α G1β G2γ G3δ
c−2 µν
G Gµν
4
0123
per α = β = γ = δ e 0 negli altri casi.
αβγδ
(b) Verificare che altri possibili invarianti p.e. del tipo eαβγδ Fαβ Fγδ sono combinazioni di
questi invarianti
con eαβγδ = segno della permutazione
Soluzione: (a) Si verfica facilmente che
(F )
I1
(F )
I2
(G)
= cE · B , I1 = c−1 D · H
1 2 2
1 −2 2
(G)
=
c B − E2 , I1 =
c H − D2
4
4
(b) Tenuto conto che
si ha:
(2.28)
eαβγδ Fαβ = Fγδ
(F )
eαβγδ Fαβ Fγδ = Fγδ Fγδ = I2
D’altra parte
Fαβ Fβγ Fγδ Fδα = Sαγ Sγα
avendo posto
Fαβ Fβγ = Sαγ
Esercizio 2.4.2 Esprimere lo scalare Fαβ Fβγ Fγδ Fδα in funzione degli invarianti dell’eserc. 2.4.1
2.5
4-Tensore degli sforzi elettromagnetici
Moltiplicando scalarmente la prima delle Eqq. (2.13) per H e la seconda per E, ed
integrando su un volume V si ottiene
d
d
Uem = − L + Φ −
dt
dt
µ0 H ·
∂
∂
M(r, t) + E · P(r, t) d3 r ,
∂t
∂t
(2.29)
52
Richiami di teoria della relatività
dove
1
1
µ0 H2 (r, t) + ε0 E2 (r, t) d3 r ,
2
2
Uem = Ue + Um =
(2.30)
sta per l’energia e.m. del vuoto accumulata al tempo t nel volume V , L misura il lavoro
compiuto dalle correnti sul campo
d
L (t) = −
dt
mentre
Φ=
E(r, t) · J(r, t)d3 r W,
!
(2.31)
S(r, t) · n̂d2 r ,
∂V
rappresenta il flusso del vettore di Poynting
S(r, t) = E(r, t) × H(r, t) W m−2 ,
(2.32)
uscente dal volume V . Si noti che mentre S rappresenta il flusso della densità di energia
del campo, c−2 S rappresenta anche la densità del momento della quantità di moto del
campo.
Il momento esercitato dal campo sul fluido carico è dato da
ρext (r, t)r × [E(r, t) + v(r, t) × B(r, t)] d3 r
K=
Esercizio 2.5.1 Costruire col 4-tensore del campo Fµν per il vuoto (a) il 4-tensore degli
sforzi elettromagnetici utilizzando i vettori E, B del campo e (b) relazionarlo alle equazioni
di Maxwell.
Soluzione: (a) Tenuto conto che il 4-tensore degli sforzi elettromagnetici T µν è dato
da:
1
T µν = ε0 Fµα Fνα + g µν Fαβ Fαβ
4
esprimendo Fµα in funzione di E, B si ottiene
T µν =
+ 12 ε0 E2 c−1 S T
c S
−σ
1 −1 2
µ B (r, t)
2 0
−1
con S vettore di Pynting, e
σ = [σ ij ]
tensore degli sforzi
σ ij = ε0 Ei Ej +
1
Bi Bj −
µ0
1 −1 2 1
µ B + ε0 E2 δ ij
2 0
2
(b) A partire dalle equazioni di Maxwell si mostra che T µν è relazionato con Fµν e con
la quadricorrente J µ dalla relazione:
T,νµν + F µα Jα = 0
2.5 4-Tensore degli sforzi elettromagnetici
53
dove la virgola denota la derivata parziale, che esprime la conservazione dell’energia
T,ν0ν + F 0α Jα = 0
ovvero
c−1
1 −1 2
1
µ0 B (r, t) + ε0 E2 + ∇· S = −E · J
2
2
∂
∂t
e dell’impulso
T,νiν = −F iα Jα
ovvero
∂
S + ∇ · σ = ρext E + J × B
∂t
Si noti che ρext E + J × B rappresenta la forza di Lorentz che agisce sulla densità di carica
ρext
c−1
Esercizio 2.5.2 Per un campo e.m. nel vuoto (a) analizzare E′ e B′ in un sistema ottenuto per trasformazione di Lorentz omogenea pura con β̂ = ẑ e x̂ diretti rispettivamente
come E × B ed E. (b) Calcolare il valore di γ per cui E′ e B′ risultano paralleli. (c)
Discutere la forma del 4-tensore T µν in quest’ultimo sistema di riferimento. (d) Calcolare
il valore di γ per cui E′ e B′ risultano perpendicolari
Soluzione: (a) Dall’Eq. (2.27) discende:
E′
B′
ovvero
=
γ + (1 − γ) ẑ ẑ
cγβ ẑ×
γ
− c β ẑ×
γ + (1 − γ) ẑ ẑ
E′
B′
=γ
·
E x̂
B
E x̂ + cβ ẑ × B
1
Eβ ŷ + B
c
Pertanto
E′ × B′ = γ 2
= γ2
1 2
E β + EB ẑ + Eβ 2 (ẑ × B) × ŷ + cβ (ẑ × B) × B
c
1 2
E β + EB ẑ + Eβ 2 (ẑ × B) × ŷ + cβ (ẑ × B) × B
c
Tenuto conto dell’identità vettoriale
(b × c) ×a = (b · a) c − (a · c) b
si ha
E′ × B′ = γ 2
= γ2
1 2
E β + EB ẑ + Eβ 2 (ẑ × B) × ŷ + cβ (ẑ × B) × B
c
1 2
E β + EB − EBy β 2 − cβB2 ẑ
c
Tenendo conto degli invarianti del campo (v. (2.28)):
(F )
I1
(F )
I2
= cE · B
1 2 2
=
c B − E2
4
54
Richiami di teoria della relatività
si ha
(F )
E′ × B′ = γ 2 −2c−1 βI2
+ EB − EBy β 2 ẑ
D’altra parte per E′ e B′ paralleli By = 0 per cui deve risultare
(F )
−2c−1 βI2
(F )
+ EB = −2c−1 βI2
(F )
+ c−1 I1
=0
Ne segue che
(F )
β=
1 I1
2 I2(F )
Il sistema di riferimento in cui E′ e B′ risultano paralleli è detto canonico.
(c) Nel sistema canonico T µν assume la forma:


1 0 0
0
 0 1 0
1 −1 2 1
0 

T µν =
µ0 B + ε0 E2 
 0 0 −1 0 
2
2
0 0 0 −1
2.6
4-vettore d’onda
In molte situazioni i campi e le correnti sono rappresentati da onde piane
Aα (xν ) = Ãα exp (ikµ xµ )
J α (xν ) = J˜α exp (ikµ xµ )
Fαβ (xν ) = F̃αβ exp (ikµ xµ )
Gαβ (xν ) = G̃αβ exp (ikµ xµ )
dove kµ = (−c−1 ω, k) sta per il 4-vettore d’onda.
In particolare k si può esprimere nella forma
k = ñ ω, k̂
ω
c
con ñ ω, k̂ l’indice di rifrazione e k̂ direzione di propagazione dell’onda. Pertanto si ha:
kµ k µ = ñ2 ω, k̂ − 1
ω2
.
c2
Applicando a k µ = (c−1 ω, k) la trasformazione di Lorentz omogenea k µ si modifica in
(c−1 ω ′ , k′ ) , ovvero:
ω
v + (γ − 1) v̂v̂ · k
c2
= γ (ω − v · k) ≡ ω + δω .
k′ = k − γ
ω′
(2.33)
Esercizio 2.6.1 Si consideri un mezzo isotropo nel sistema KL . caratterizzato da un
indice di rifrazione ñL ω, k̂ = ñL (ω) . (a) Analizzare ñ′ = ñ ω ′ , k̂ ′ in un sistema di
riferimento inerziale generico. (b) In particolare esprimere ñ′ in funzione dell’angolo θ′
formato da k′ con v.
2.7 4-vettore quantità di moto
55
Soluzione6 : Tenendo conto della legge di trasformazione (4.4) di k si ha:
k′ =
k 2 + 2γ 2
ω
ω2 2
2 − 1) (k · v̂)2 + γ 2
k
·
v+
(γ
β
c2
c2
Un mezzo isotropo se a riposo, quando si muove con velocità v appare dotato di un
indice di rifrazione ñ′ tale da soddisfare le relazioni:
′
′ω
k = ñ
′
c
k̂ ′ = ñL (ω)
ω
ω
(1 + (γ − 1) v̂v̂) · k̂ − γ 2 v ,
c
c
ovvero risulta anisotropo. Ne segue che
ck ′
ñ = ′ =
ω
′
ñ2L + 2γ 2 ñL β cos θ + (γ 2 − 1) ñ2L cos2 θ + γ 2 β 2
γ (1 + ñL β cos θ)
(2.34)
con θ l’angolo formato da k con v. D’altra parte l’ angolo θ′ formato da k′ con v è legato
a θ dalla relazione:
γ (ñ′ cos θ′ − β)
cos θ =
,
2
′
2 ′
′2
2
′
ñ sin θ + γ (ñ cos θ − β)
mentre
ñ′ cos θ′ =
ñL cos θ + β
,
1 + ñL β cos θ
che combinate tra loro danno
1 − ñL 2 − 1 γ 2 β 2 cos2 θ ñ′2 + 2 ñL 2 − 1 γ 2 β cos θ ñ′ − γ 2 ñL 2 − β 2 = 0
Risolvendo si ottiene
′
ñ =
2.7
1 + γ 2 (ñL 2 − 1) 1 − β 2 cos2 θ ′ − γ 2 β (ñL 2 − 1) cos θ′
1 − (ñL 2 − 1) γ 2 β 2 cos2 θ′
4-vettore quantità di moto
Dalla (5.8) discende anche che in assenza di correnti
kT · ε̃ · Ẽ = 0
Ad un elettrone che si muove nel vuoto in un campo e.m descritto dal 4-vettore Aµ è
associato il 4-vettore quantità di moto
π µ = me uµ = pµ + eAµ
con pµ = 1c E, p il 4-impulso ed Aµ = 1c V, A il 4-potenziale del campo. π µ gode
dell’importante proprietà
π µ π µ = −m2e c2 ,
6
v. C. H. Papas loc. cit. 116 Sez. 7.5
56
Richiami di teoria della relatività
ovvero
1
(E + eV )2 = −m2e c2
c2
con E l’energia della particella. Pertanto l’energia E è espressa da:
|p + eA|2 −
E=
m2e c4 + c2 |p+eA|2 − eV
(2.35)
In particolare per A = 0 questa equazione si riduce a
E = me c2 γ − eV
2.7.1
(2.36)
Effetto Compton
Rutherford e Andrade utilizzarono uno spettrometro di Bragg per determinare le lunghezze
d’onda dei raggi gamma emessi dal radio B (v. Fig. 2.1). Un sottile fascio di raggi gamma
proveniente da un composto radioattivo, reso collimato facendolo passare attraverso una
sottile fenditura in un blocco di piombo, veniva riflesso da un cristallo di rocca su una
lastra fotografica. Ruotando il cristallo di circa 150 gradi, ottennero una serie di righe
sottili corrispondenti allo spettro emesso dal radio B. Questo metodo fu poi utilizzato per
misurare le lunghezze d’onda dei raggi gamma emessi da molte altre sostanze radioattive.
Con uno spettrometro a cristallo di tipo Bragg si è riusciti a misurare lunghezze d’onda
pari a 0.016 Å.
I raggi gamma sono radiazioni e.m. i cui fotoni hanno energie comprese generalmente
tra 10 keV e 10 M eV . Essi sono prodotti quando un nucleo cambia assetto per effetto di
collisioni con altri nuclei o per emissione spontanea (radioattività) di particelle alfa e beta.
In questi casi modificandosi l’energia di interazione coulombiana tra i nucleoni o riorientandosi gli spin dei nucleoni vengono emessi fotoni gamma. Un esempio di generazione
è fornito dal ciclo CNO ; un altro è quello del cobalto-60, che decade spontaneamente in
nichel-60 eccitato attraverso il decadimento beta
60
27 Co
−
→60
28 Ni* + e + ν̄ e
Figura 2.1: Spettrometro a singolo cristallo per
la determinazione dello spettro di sorgenti di
raggi gamma.
Figura 2.2: Geometria relativa
all’effetto Compton.
2.7 4-vettore quantità di moto
57
60
con ν̄ e l’antineutrino elettronico. Il nucleo eccitato 60
28 Ni* decade radiativamente in 28 Ni
emettendo raggi gamma. Questo processo è simile a quello del decadimento radiativo
di atomi eccitati. I protoni e neutroni del nucleo di 60
28 Ni* decadono ai rispettivi stati
fondamentali attraverso transizioni di multipolo sia elettrico che magnetico coinvolgenti
sia le cariche associate ai protoni che gli spin 1/2 dei nucleoni.
Per rivelare i raggi gamma oggi si utilizzano scintillatori (p.e. cristalli di BaF2 o CsI),
camere a ionizzazione e microstrisce al silicio.
Esercizio 2.7.1 Se un elettrone accelera durante l’interazione con raggi γ, o X, il campo emesso oscillerà per effetto Doppler ad una frequenza leggermente diversa da quella
dell’onda incidente. In genere questo effetto è del tutto trascurabile, ma diventa importante per fotoni con energie confrontabili con quella di riposo dell’elettrone (511 keV ).
Questo fenomeno fu osservato nel 1922 da Arthur H. Compton7 utilizzando fotoni X da
0.7 Å prodotti da un tubo Roentgen a catodo di Mo alimentato a 50 kV . Usando come
bersaglio per gli X un foglio di grafite, Compton8 osservò nello spettro trasmesso a varie
energie due picchi, di cui uno alla stessa lunghezza d’onda del fascio incidente ed un altro alla lunghezza d’onda prevista dall’ultima formula. Mentre quest’ultimo picco andava
attribuito al trasferimento di parte dell’energia agli elettroni liberi, l’altro era dovuto agli
elettroni di “core” che non subivano alcun effetto di rinculo apprezzabile, restando saldamente legati al nucleo. In esperimenti successivi Compton riuscì a rivelare gli elettroni
di rinculo dimostrando che la quantità di moto e l’energia si conservano nel processo.
Analizzare l’interazione γ-elettrone riconducendola alla collisione tra un fotone di energia
ω ed impulso k con un elettrone
Soluzione: L’interazione campo-particella può essere ricondotta alla collisione tra un
fotone di energia ω ed impulso k con un elettrone. Varranno quindi le relazioni di
conservazione dell’energia e dell’impulso (vedi Fig. 2.2)
ω i + Ei =
ki =
ω f + Ef ,
kf + pf ,
(2.37)
dove ki,f sta per il vettore d’onda iniziale e finale. Ei e Ef rappresentano rispettivamente
l’energia iniziale e finale dell’elettrone e sono legate ai rispettivi impulsi dalla relazione
relativistica
E = m2e c4 + p2 c2 .
Nello scrivere queste relazioni si è supposto che per effetto dell’interazione il campo di
vettore d’onda ki perda un fotone mentre quello di vettore d’onda kf ne acquisti uno. Se
si indica con θ l’angolo formato da kf con ki e si tratta l’elettrone relativisticamente si
può facilmente verificare che esiste tra ω i e ω f la relazione
ω f = ω i P (Eγ , θ) ,
7
8
A.H. Compton, Phys. Rev., 22:09, 1923;–, Phys. Rev., 21:483, 1923; A.H. Compton and S.K. Allison,
“X-Rays in Theory and Experiments”, Van Nostrand Company, New York, 1935; A.H. Compton and
A.W. Simon, Phys. Rev., 26:289, 1925; C.T.R. Wilson, Proc. of the Royal Society (London), 104:1,
1923.
v.p.e. E. Landi Degl’Innocenti, “Spettroscopia Atomica e Processi Radiativi”, Springer-Verlag, Milano
2009, Sez. 15.6.
58
Richiami di teoria della relatività
dove P (Eγ , θ) sta per il rapporto tra l’energia del fotone γ diffuso e quello incidente
(Eγ = ω i )
1
P (Eγ , θ) =
,
(2.38)
λC
1 + 2πc ω i (1 − cos θ)
ovvero in termini di lunghezze d’onda:
λf = λi + λC (1 − cos θ) ,
dove
λC =
h
= 2.42631058 × 10−12 m
me c
(2.39)
è la cosiddetta lunghezza d’onda di Compton per l’elettrone: essa coincide con la lunghezza
d’onda di un fotone di energia pari a quella di riposo dell’elettrone. Ne segue che l’energia
del fotone diffuso varia al variare dell’angolo di diffusione θ.
E’ importante notare che il raggio di Compton rC = λC /2π risulta legato al raggio di
Bohr ed alla costante di struttura fine dalle relazione
rC = αa0 .
Inoltre, introducendo il raggio classico dell’elettrone definito come quello di una distribuzione uniforme sferica di energia elettrostatica pari all’energia di riposo me c2 ,
re =
1
e2
= 2, 8179402894 × 10−15 m ,
4πε0 me c2
si vede facilmente che
re = αrC = α2 a0 .
2.7.2
Comptonizzazione
Esercizio 2.7.2 Si consideri lo scattering di un’onda e.m. piana da parte di un elettrone
che si muove con velocità v perpendicolare al vettore k dell’onda incidente. Analizzare
l’andamento del k′ relativo all’onda scatterata nella direzione −k̂.
Soluzione: Dal sistema (4.4) discende che
ω
v
c2
= γω ≡ ω + δω .
k′ = k − γ
ω′
Si osserva che la fequenza dell’onda back-scatterata è maggiore di quella incidente. Per
elettroni che si muovoono con velocità relatistiche questo fatto segnala la presenza di
un effetto Compton inverso. Questo è facilmente riscontrabile: quando un elettrone di
altissima energia, ad esempio generato dai raggi cosmici, interagisce con un fotone a bassa
energia quale ad esempio quello dovuto alla radiazione cosmica di fondo. Avendo carica
energetica più alta, sarà l’elettrone a cedere energia al fotone. E’ questo importantissimo
processo che permette di generare fasci di fotoni ad alta energia (centinaia di MeV).
2.7 4-vettore quantità di moto
59
2.7.3
Effetto Sunayev-Zeldovich
2.7.4
Effetto Mössbauer
Si consideri ora l’assorbimento di un fotone γ di frequenza angolare ω da parte di un
atomo di massa m inizialmente a riposo, che presenta una risonanza a frequenza angolare
|ω if |. Limitandosi a considerare una collisione collineare il sistema di equazioni (2.37) si
riduce a
ω + mc2 =
m∗2 c4 + p2 c2 ,
ω = cp ,
dove la massa m∗ dell’atomo eccitato è legata a quella dell’atomo allo stato fondamentale
dalla relazione
m∗ c2 = mc2 + |ω if | .
Quindi, la frequenza di assorbimento risonante ω a = |ω if | 1 +
di emissione ω e = |ω if | 1 −
|ωif |
2mc2
|ωif |
2mc2
differisce da quella
della quantità
ωa − ω e =
|ω if |2
.
mc2
(2.40)
Si immagini ora di misurare in funzione della temperatura l’assorbimento e l’emissione risonante di raggi gamma in un cristallo. Fintantoché la quantità ora calcolata
è più piccola del contributo dovuto all’effetto Doppler, allora si può anche non tenerne
conto. Man mano che si abbassa la temperatura si arriva alla situazione in cui questo
rapporto diventa maggiore di quello Doppler e sia l’assorbimento che l’emissione di fotoni
si riducono drasticamente. Nel 1958 Mössbauer9 , un chimico-fisico di Monaco, studiando
il comportamento di nuclei di 191 Ir bombardati a 129 keV , si accorse che raffreddando sia
la sorgente di γ che il bersaglio, l’assorbimento aumentava piuttosto che diminuire.
La ragione di questo comportamento “anomalo”, divenuto noto come effetto Mossbauer, risiedeva nel fatto che la formula (2.40) presuppone che il fotone γ interagisca
coi singoli atomi, non interagenti tra loro. Inserendo il singolo atomo in un cristallo, la
massa m che appare nella (2.40) va moltiplicata per il numero totale di nuclei presenti,
dell’ordine di 6 × 1023 , dimodoché |(ω a − ω e ) /ω if | diventa dell’ordine di 10−25 : questo
significa che sia l’assorbimento che l’emissione dei γ avvengono senza rinculo.
Mössbauer10 , abbassando la temperatura del cristallo assorbitore e della sorgente da
400 K a 88 K, non faceva altro che aumentare il numero dei nuclei che prendevano parte
collettivamente al processo di interazione con un singolo γ. Con questa scoperta s’era
trovato il modo di applicare la spettroscopia gamma ai solidi.
Per analizzare delle risonanze γ si misura l’assorbimento in funzione dell’energia dei
γ incidenti. Se si dispone di fasci molto monocromatici, ottenuti eliminando gli effetti
dovuti al rinculo, basta variarne la frequenza per ispezionare una riga di assorbimento del
materiale da studiare. Per ottenere questa scansione in frequenza si imprime alla sorgente
dei γ un moto oscillatorio che per effetto Doppler si trasforma in una modulazione in
9
10
R.L. Mössbauer, Zeit. fur Physik, 151:124, 1958; H. Frauenfelder, “The Mossbauer effect”, W.A.
Benjamin N. Y. 1962.
premio Nobel per la fisica nel 1961.
60
Richiami di teoria della relatività
frequenza. Poiché le righe d’assorbimento γ hanno delle larghezze dell’ordine dei GHz
mentre ν ≃ 1019 Hz, sono sufficienti velocità dell’ordine di v ≃ c × 10−10 ≃ 0 − 5 ×
10−2 m s−1 .
2.8
Processi di diffusione
2.8.1
Trasformazioni sezioni d’urto differenziali
2.8.2
Riferimenti utilizzati nello scattering elastico
2.8.3
Spazio delle fasi
2.9
Moto dello spin
Esercizio 2.9.1 Discutere l’equazione del moto dello spin sottoposto ad un campo magnetico BP nel suo sistema poprio (a) caso non relativistico, (b) caso relativistico
Soluzione: (a) s effettua un moto di precessione attorno a BP
ege
d
s=
s × BP + ω T × s .
dτ
2me
(b) Nel caso relativistico si deve tener conto della precessione di Thomas
d
ege
s=
s × BP + ω T × s .
dτ
2me
(2.41)
dove il termine proporzionale al vettore di Thomas ω T rappresenta il contributo dovuto
alla precessione di KP
γ2
d
ωT =
β× β
γ+1
dτ
Esercizio 2.9.2 Lo spin si può trattare come un 4-vettore sα che nel sistema proprio
KP dell’elettrone assume la forma sαP = (s,0), ovvero ha componente temporale nulla e
componente spaziale coincidente con s. Ne segue che
sα uα = 0
(2.42)
con uλ 4-velocità dell’elettrone. Assumendo dsα /dτ dipendente solo da sα , uα , u̇α =
duα /dτ e Fαβ , e lineare in sα e Fαβ , Bargmann, Michel e Telegdi11 espressero l’equazione
del moto sα in forma covariante nella forma12
d α A1 αβ
A2
A3
s = 2 F sβ + 2 sλ Fλµ uµ uα + 2 sλ u̇λ uα ,
dτ
c
c
c
Dimostrare che per ge ≃ 2 e γ ≃ 1 questa equazione si riduce a:
(2.43)
d α
e αβ
s =
F sβ .
dt
me
11
12
V. Bargmann, L. Michel and V. L. Telegdi, Phys. Rev. Lett. 2, 435 (1959); A.P.Balachandran,
G.Marmo, A.Stern, B-S.Skagerstam, Phys.Lett.89B (1980)199; – in ”Gauge Symmetries and Fibre
Bundles”, Lecture Notes in Physics ,188 Springer, Berlin 1983, Capitolo 5 Relativistic Spinning Particle
v. C. Altucci et al. loc. cit. pag. 99 Sez. 5.2.1
2.9 Moto dello spin
61
con A1 , A2 e A3 opportune costanti tali da garantire (a) il rispetto della condizione (2.42),
e (b) che nel sistema proprio la (2.43) si riduca all’equazione di precessione
d
s = s × ω c = γ g s × B,
dt
(2.44)
con ω c frequenza di ciclotrone (=2 frequenza di Larmor)
(2.45)
ω c = 2ω L = γ g B ,
e γ g = ge µB = ge 2me e rapporto giromagnetico
Soluzione: Dalla condizione (2.42) discende per l’equazione del moto BMT che
A1 A2
− 2
c2
c
uα Fαβ sβ + 1 +
A3
c2
sλ u̇λ = 0 .
Dovendo questa relazione valere per il moto più generale, deve risultare Ac21 − Ac22 = 1+ Ac23 =
0. Inoltre nel sistema proprio (2.43) deve ridare la (2.44). Pertanto si ha:
d α
ge e
s =
Fαβ sβ + sλ Fλµ uµ uα − sλ u̇λ uα .
dτ
2me
Tenuto conto che
u̇λ =
e λµ
F uµ
me
si ottiene:
d α
e ge αβ
ge
s =
F sβ +
− 1 sλ Fλµ uµ uα .
dτ
me 2
2
Infine, per ge ≃ 2 e γ ≃ 1 questa equazione si riduce a::
(2.46)
d α
e αβ
s =
F sβ .
dt
me
Esprimendo ora le componenti sαL (sL , s4L ) in KL in funzione di sαP (0, sP ) in KP
γ2 + 1
ββ · sP
γ+1
= γβ · sP
sL = sP +
s0L
13
e tenendo conto dei componenti di Fαβ
L si ottiene :
d
e
1
s=
s× B− β×E
dt
me
2c
.
(2.47)
A questo moto corrisponde il potenziale:
Vs campo = ge µB B −
13
1
β × E ·s ,
2c
(2.48)
v.p.e. J. D. Jackson, , “Classical Electrodynamics”, J. Wiley and Sons, New York, 1962, E1. 11.170
62
Richiami di teoria della relatività
2.10
Campi e.m. in sistemi rotanti
Esercizio 2.10.1 Si consideri un sistema inerziale KL ed un sistema KR , detto di Born14 ,
che ruota con velocità angolare costante Ω rispetto a KL . (a) Passare dalla metrica di
Minkowski in KL a quella in KR effettuando le trasformazioni15
t′ = t, x = ρ cos (Ωt + φ) , y = ρ sin (Ωt + φ) , z ′ = z
(a) Esprimere la distanza tra due eventi in KL utilizzando le coordinate cilindriche di KR
con asse z parallelo a Ω. Utilizzare questa espressione per definire la metrica di KR (b)
Scrivere le equazioni di Maxwell in KR
Soluzione: Si verifica facilmente che:
ds2 = −c2 dt2 + dx2 + dy 2 + dz 2
′
= − c2 − ρ2 Ω2 dt2 + dρ2 + ρ2 dφ2 + dz 2 + 2cρ2 Ωdtdφ
Pertanto in KR il tensore metrico è espresso da:

2
− 1 − ρ2 Ωc


0
gµν = 

0
ρ2 Ωc
0
1
0
0
0
0
1
0
ρ2 Ωc


0 

0 
ρ2
La presenza del termine non-diagonale gρφ segnala che il sistema di Born non è ortogonale.
Con riferimento allo spazio tangente delle derivate direzionali s ha
∂
∂s
2
1
= − 2
c
+
∂
∂t
∂
∂ρ
2
2
+
Ω∂ ∂
c2 − ρ2 Ω2
+2
+
c ∂t ∂φ
ρ2
∂
∂z ′
∂
∂φ
2
2
Esercizio 2.10.2 Le equazioni di Maxwell in assenza di sorgenti in KR sono date da16 :
∂
B
∂t
1 ∂
∇ × αB − 2 E
c ∂t
∇·E
∇·B
∇ × αE +
dove α =
1 − ρ2
Ω 2
.
c
= 0 ,
=
,
= 0,
= 0.
(2.49)
Ricavare l’equazione soddisfatta da B
14
Born, M. (1909). Die Theorie des starren Elektrons in der Kinematik des Relativitäts-Prinzipes Ann.
Phys. 30: 1. 1909 Wikisource translation: The Theory of the Rigid Electron in the Kinematics of the
Principle of Relativity
15
v.p.e. C. Mōller, The Theory of Relativity, Clarendon Press, Oxford, 1072
v.p.e. K. S. Thorne, R. H. Price and D. A. Macdonald, Black Holes: The Membrane Paradigm, Yale
University Press, New Haven, 1988
16
2.10 Campi e.m. in sistemi rotanti
63
Soluzione: Semplici calcoli portano a scrivere
1 ∂2
B + ∇ × (α∇ × αB) = 0
c2 ∂t2
Tenuto conto che
1
α∇ × αB = α2 ∇ × B + ∇α2 × B
2
ne segue che
1
∇ × (α∇ × αB) = ∇ × α2 ∇ × B + ∇ × ∇α2 × B
2
1
= α2 ∇ + ∇α2 × (∇ × B) + ∇ × ∇α2 × B
2
ovvero, tenendo conto che ∇ · B = 0,
∇ × (α∇ × αB)
= −α2 ∇2 + ∇α2 × ∇× B
1
1
− ∇α2 · ▽B + B · ▽∇α2
2
2
2 2
= −α ∇ B
1
+ −∇α2 · ▽ − ∇2 α2 + ▽∇α2 · B
2
1
1
1
= −α2 ∇2 − ∇α2 · ▽ −
∇2 α2 +
▽∇α2 · B
2
2
2
In definitiva tenuto conto che
Ω
∇ α =−
c
2
∂2
1 ∂
Ω
+
ρ2 = −4
2
∂ρ
ρ ∂ρ
c
2 2
2
si ha
1
1
1
−α2 ∇2 − ∇α2 · ▽ −
∇2 α2 +
▽∇α2 ·
2
2
2
Ω 2 ∂
Ω 2
Ω 2
= −α2 ∇2 −
ρ +2
−
ρ̂ρ̂·
c
∂ρ
c
c
ovvero
1 ∂2
− α2 ∇2 −
2
2
c ∂t
Ω
c
2
∂
Ω
ρ +2
∂ρ
c
2
−
Ω
c
2
ρ̂ρ̂· B = 0
Esercizio 2.10.3 In un famoso articolo del 1939 L. Schiff17 , utilizzando il formalismo della relatività generale, provò che le equazioni di Maxwell assumono in un sistema
rotante con velocità Ω la seguente forma:
∂
B = 0 ,
∂t
∂
∇ × B − ε0 µ0 E = µ0 (J + j) ,
∂t
1
∇·E =
(ρ + σ) ,
ε0
∇ · B = 0.
∇×E+
17
v. L.I. Schiff, Proc. Natl. Acad. Sci. U.S. 25, 391 (1939)
(2.50)
64
Richiami di teoria della relatività
con v =Ω×r,
σ = ε0 ∇ · (v × B)
j = ε0 {∇ × [v × (E − v × B)] + v × (∇ × E)}
(2.51)
Dimostrare che risulta soddisfatta la relazione di continuità:
∇ (J + j) +
∂
(ρ + σ) = 0
∂t
Esercizio 2.10.4 Con riferimento al precedente Eserc. 2.10.3 risolvere le equazioni perturbativamente al primo ordine in Ω
Soluzione: All’ordine 0 il sistema si riduce a:
∂ (0)
B
= 0 ,
∂t
∂
∇ × B(0) − ε0 µ0 E(0) = µ0 J(0) ,
∂t
1 (0)
∇ · E(0) =
ρ ,
ε0
∇ · B(0) = 0 .
∇ × E(0) +
Al primo ordine invece si ottiene
∂ (1)
B
= 0 ,
∂t
∂
∇ × B(1) − ε0 µ0 E(1) = µ0 j(1) ,
∂t
1 (1)
∇ · E(1) =
σ ,
ε0
∇ · B(1) = 0 .
∇ × E(1) +
con
σ (1) = ε0 ∇ · v × B(0)
j(1) = ε0 ∇ × v × E(0) + v × ∇ × E(0)
avendo trascurato termini quadratici in v
Esercizio 2.10.5 Si consideri una soluzione delle equazioni di Maxwell-Schiff dell’Eserc.
2.10.3 del tipo
E = E0 eiS(r,t)
B = B0 eiS(r,t)
discusso nell’Eserc. 1.6.1.Si ignorino le derivate di v rispetto a quelle di S (r,t)
2.10 Campi e.m. in sistemi rotanti
65
Soluzione: In via preliminare si ha
ε−1
0 σ = ∇ · (v × B) ≃ v · ▽ × B ≃ iv · k × B
ε−1
0 j = ∇ × [v × (E − v × B)] + v × (∇ × E)
≃ ∇ × (v × E) + v × (∇ × E)
≃ (E · ▽) v = Ω × E
Pertanto
k × E0 − ωB0
ω
k × B0 + 2 E0
c
k · E0
k · B0
= 0 ,
1
= − 2 Ω × E0 ,
c
= −v · k × B0 ,
= 0.
Degno di nota è il fatto che il campo elettrico non è più in generale perpendicolare a k
Infatti
k · E0 = −v · k × B0 = −ωv · k × (k × E0 )
ovvero
(1 + ωv · k) k · E0 = ωk 2 v · E0
Risolvendo rispetto a E0 si ottiene:
ω
c
k × (k × E0 ) +
2
E0 = −
ω
Ω × E0
c2
ovvero
ω
c
2
ω
Ω × E0 − k (k · E0 )
c2
ω
ωk 2
= − 2 Ω × E0 −
v · E0 k
c
1 + ωv · k
− k 2 E0 = −
In particolare per E0 parallelo a Ω si ha:
ω
c
2
− k 2 E0 = 0
ovvero la propagazione non risente della rotazione.
Esercizio 2.10.6 Mostrare che le equazioni di Maxwell-Shiff si possono ottenere con
considerazioni classiche partendo dalle seguenti regole di trasformazione18
t′ = t
ρ′ = ρ , J′ = J−ρv
B′ = B , E′ = E + v×B
con v =Ω×r
18
v G. E. Modesitt, Am. J. Phys. 38, 1487 (1970)
66
Richiami di teoria della relatività
Soluzione: Tenuto conto della conservazione delle cariche e dei volumi si ha che per
un osservatore che si muove con velocità v le sorgenti in KL e KR , indicate con un apice,
sono date da
ρ′ = ρ , J′ = J − ρv
D’altra parte passando da KL e KR la forza di Lorentz F agente sulle cariche non cambia,
per cui
F = ρE + J × B
= ρ′ E′ + J′ × B′
Discende così per i campi:
B′ = B , E′ = E + v×B
Per scrivere le equazioni di Maxwell si deve tener conto delle leggi di trasformazione
temporali nei due sistemi. un sistema rotante
v =Ω×r
Il tasso di variazione di un generico vettore A in KL è legato a quello in KR dalla relazione
di trasformazione:
′
d
d
A=
A +Ω×A
dt
dt
Per un punto fisso del sistema rotante risulta
d
A
dt
′
=
∂
A
∂t
D’altra parte
d
∂
∂
A = A + v · ∇A =
A
dt
∂t
∂t
ovvero
∂
∂
A=
A
∂t
∂t
′
′
′
+Ω×A
+ Ω × A − v · ∇A
Dal momento che ∇ · v = 0 quest’ultima equazione si può riscrivere nella forma
′
∂
∂
A + (∇ · A) v =
A
∂t
∂t
+ ∇ × (v × A)
In particolare per il vettore B (∇·B = 0) si ha
∂
B=
∂t
∂
B
∂t
′
+ ∇ × (v × B)
mentre tenuto conto che ∇·E = ε−1
0 ρ
∂
E + ε−1
0 ρv =
∂t
=
′
∂
E + ∇ × (v × E)
∂t
′
∂ ′
E + ∇ × (v × j′ )
∂t
2.10 Campi e.m. in sistemi rotanti
67
dove
′
j = −v ×
∂
B
∂t
′
+ ∇ × [v × (E′ − v × B)]
Inoltre tenuto conto che
E′ = E + v×B
si ha
∇′ · E′ = ∇′ · E + ∇′ · v × B =
1
(ρ + σ)
ε0
dove
σ = ∇′ · (v × B′ )
In definitiva si ha
∂ ′
B
∂t
∂
= ε0 E ′ + J + i
∂t
= 0
1
=
(ρ + σ)
ε0
∇′ × E′ = −
∇′ × H′
∇′ · B′
∇′ · E′
2.10.1
Effetto Sagnac
Esercizio 2.10.7 Si consideri un interferometro formato da 4 specchi (v. Fig. 2.3) ai
vertici di un quadrato e poggiati rigidamente su una piattaforma ruotante intorno ad un
asse perpendicolare con velocità angolare costante Ω Un laser inietta nell’interferometro
un fascio a frequenza ω che, diviso in due da un beam-splitter percorre il quadrato in
versi opposti. Calcolare la differenza di fase tra due raggi che percorrono in senso orario
ed antioarario i lati del quadrato. Per velocità degli specchi non relativistiche si può
assumere γ = 1. I due fasci controrotanti si possano trattare nel sistema rotante come
onde localmente piane caratterizzate dai 4-vettori di propagazione (ω ′ , ±k′ ) legati a quelli
(ω, ±k) del sistema di laboratorio dalle relazioni:
ω
v
c2
= ω−v·k
k′ = k −
ω′
Soluzione: Per velocità non relativistiche si ha per i vettori d’onda dei due fasci
controrotanti (±):
Ω
k(±)′ (r) = k(±) (r) ± 2 r × Ω
c
Pertanto la fase accumulata percorrendo l’intera traiettoria chiusa è data da
!
(±)
∆ϕ
=
k(±)′ (r) · dℓ
!
!
Ω
(±)
=
k · dℓ ± 2 (r × Ω) · dℓ
c
68
Richiami di teoria della relatività
Figura 2.3: Rappresentazione schematica di una cavità a 4 specchi con due modi che si
propagano in verso orario ed antiorario. Quando la cavità è posta su un piano che ruota
rigidamente attorno ad un asse, i vettori d’onda locali dipendono localmente dallo shift
Doppler, dando così luogo ad una differenza tra le fasi accumulate in un giro completo
dai due modi.
Pertanto si ha
(+)
∆ϕ
(−)
− ∆ϕ
!
!
ω
ω
= 2 (r × Ω) · dℓ = 2 2 (r × dℓ) ·Ω
c
c
ω
= 4 2 AΩ
c
ovvero la differenza dei ritardi di fase è proporzionale alla velocità di rotazione Ω. Questo
ritardo di fase si traduce in uno spostamento delle frange di interferenza, che si formano
all’uscita della cavità viene con la sovrapposizione dei due modi. Questo spostamento fornisce un segnale contenente un componente proporzionale a cos ∆ϕ+ − ∆ϕ− =
cos 4ωΩ
A . La presenza di questa componente è nota come effetto Sagnac, dal nome di
c2
chi la scoprì nel 1911.
2.11
Propagazione in spazi curvi
La discussione delle Sezz. 1.2 e 2.1 può essere facilmente estesa al caso di uno spazio
curvo rappresentato da un tensore metrico 4 × 4
gαβ =
g00 0T
0 [gij ]
in cui l’elemento −1 è stato sostituito da una funzione g00 delle 4-coordinate. Per uno
spazio-tempo reso curvo dalla presenza di una distribuzione di masse indipendente dal
tempo il 4-tensore gαβ è funzione delle sole coordinate spaziali. Si vede facilmente che
2.11 Propagazione in spazi curvi
69
come l’espressione (1.4) della divergenza resta inalterata. In particolare
g = det [gαβ ] = g00 det [gij ]
Questo implica che g = i det [gij ] risulta immaginario.
In uno spazio-tempo ad n linearmente indipendenti vettori infinitesimali dxiα (α =
1, 2, . . . , n) è associato un parallelepipedo di volume
dV = ǫij...n dxi1 dxj2 · · · dxrn
Il quadrato della variazione del tempo proprio che intercorre tra due eventi ravvicinati
coinvolgenti una massa di prova puntiforme è dato da
dτ 2 = g00 c2 dt2 −
gij dxi dxj
ij
Di particolare importanza per il moto di una massa di prova è l’introduzione della
lunghezza L di una curva dello spazio-tempo che connette due eventi
E2
L=
E1
g00 c2 ṫ2 −
gij ẋi ẋj dτ
ij
dove si è posto ẋi = dxi /dτ .
La curva di minima lunghezza tra due punti prende il nome di geodesica
Esercizio 2.11.1 Ricavare l’equazione per le geodesiche utilizzando le equazioni di EuleroLagrange
Soluzione: Dalle equazioni di Eulero -Lagrange
d ∂
dτ ∂ ẋα
gαβ ẋα ẋβ =
∂
∂xα
gαβ ẋα ẋβ
discende il sistema di equazioni
d α
ẋ + Γαβγ ẋα ẋβ = 0
dτ
(2.52)
con Γαβγ i coefficienti di Christoffel di seconda specie (v. Eq. (1.2)).
1
Γαβγ = Γαγβ = g αm (gmβ,γ + gmγ,β − gβγ,m )
2
(2.53)
Esercizio 2.11.2 Dimostrare che dalla (2.52) discende che se in un punto di una geodesica ẋα ẋα = 0 il 4-vettore ẋα risulta nullo lungo tutta la geodesica. A questa curva
vien dato il nome di geodesica nulla. Le geodesiche nulle rappresentano le traiettorie dei
fotoni. Nel caso in cui la tangente risulta nulla
70
Richiami di teoria della relatività
2.11.1
Metrica di Schwarzshild
Esercizio 2.11.3 Di particolare importanza è la metrica di Schwarzshild19 che caratterizza lo spazio intorno ad una massa puntiforme isolata. In tal caso il tensore metrico è
dato da


−c2 1 − rrg
0
0
0
1


0
0
0
r
1− rg

gαβ = 
(2.54)


0
0
r2
0
0
0
0 r2 sin2 θ
Calcolare i coefficienti di Christoffel
Soluzione: Dall’espressione (2.53) discende
 1
Γ00 Γ001 0
0
0
1
2
3

Γ
Γ
Γ
Γ
01
11
12
13
Γγαβ = 
 0 Γ212 Γ122 0
0 Γ313 0 Γ133
dove
Γ001 =
Γ100 =
Γ111 =
Γ122 =
Γ133 =
Γ212 =
Γ313 =




1 00
g g00,1
2
1
− g 11 g00,1
2
1 11
g g11,1
2
1
− g 11 g22,1
2
1
− g 11 g33,1
2
1 22
g g22,1
2
1 33
g g33,1
2
(2.55)
Esercizio 2.11.4 Verificare che sostituendo la coordinata r con
r =r 1+
rg
4r
2
la metrica (2.54) si trasforma in una metrica isotropa per cui
2
2
ds = c
19
1−
1+
rg
4r
rg
4r
2
dt2 − 1 +
rg
4r
4
dr2 + r2 dθ2 + sin2 θdφ2
v.p.e. Ya. B. Zeldovich and I. D. Novikov, Stars and Relativity, The Univ. of Chicago Press, 1971 Cap
3. R. C. Tolman, Relativity, Thermodynamics and Cosmology, DoverN. Y. 2011, R. C. Tolman, Rev.
Mod. Phys. 21, 374 (1949): E. F. Taylor and J. A. Wheeler, Exploring Black Holes, Introduction to
General Relativity, Addison Wesley Longman, 2000.
2.11 Propagazione in spazi curvi
71
Esercizio 2.11.5 Analizzare le equazioni del moto per un particella di massa M che si
muove in una metrica di Schwarzchild associata ad una massa di raggio
rg = 2
G
M
c3
dove G/c3 = 2.5 × 10−39 s/g . Integrare le equazioni del moto per una particella di prova
che descrive un’orbita equatoriale. (b) Analizzare in particolare la traiettoria di un fotone
Soluzione: (a) La metrica di Schwarzshild caratterizza lo spazio intorno ad una massa
puntiforme isolata. In tal caso l’equazione del moto di una particella di prova discende
dalla Lagrangiana
2
L = −M c2
e−λ ṫ2 − c−2 eλ ṙ2 + r2 θ̇ + sin2 θφ̇
2
(2.56)
dove · indica la derivata d/dτ rispetto al tempo proprio τ definito da:
dτ 2 = e−λ dt2 − c−2 eλ dr2 + r2 dθ2 + sin2 θdφ2
mentre
eλ =
1
1 − rrg
Poichè sia t che φ non appaiono esplicitamente nella (2.56) i momenti coniugati sono
costanti
∂L
= e−λ ṫ = A = const
∂ ṫ
∂L
=
= r2 φ̇ = h = const
∂ φ̇
pt =
pφ
avendo tenuto conto del fatto che il radicando e−λ ṫ2 − c−2 eλ ṙ2 + r2 φ̇
2
= 1. Pertanto
ṙ2 + e−λ r−2 h2 = c2 A2 − e−λ
Sostituendo
d
dτ
con
h d
r2 dφ
e e−λ = 1 −
1 dr
r2 dφ
2
rg
r
si ottiene
e−λ
A2 − e−λ − 2
r
"
#
2
1
rg
M
1
= 2 − 1−
+ 2
b
r
J
r
=
c
h
1
=
b2
Ac
h
dove
2
2
= 1−
rg
R
1
R2
con R distanza distanza di massimo avvicinamento alla massa (v. Fig. 2.5). J = M h/c
indica il momento della quantità di moto mentre b rappresenta il parametro di impatto
(v. Fig. 2.4).
72
Richiami di teoria della relatività
Ponendo M = 0 si ottiene l’equazione della traiettoria di un fotone:
2
1 dr
r2 dφ
1
r2
1
rg
− 1−
2
b
r
=
(2.57)
Sostituendo r con y = R/r si ottiene
dy
dφ
2
rg
+ y2 = 1 − −
R
2
M
J
R2 + rg R
"
M
J
2
#
1 2
+ 2y y
R
(b) Per un raggio luminoso M = 0 e quest’ultima equazione si riduce a:
2
dy
dφ
+ y2 = 1 +
Ne segue che
rg 3
y −1
R
dy
dφ =
1 − y2 +
rg
R
(y 3 − 1)
Ponendo ora
y = cos α
si ottiene
dα
dφ =
1−
rg 1−cos3 α
R sin2 α
Dal momento che
1 − cos3 α
1
= cos α +
2
1 + cos α
sin α
rg
ed in vista della piccolezza di R si può porre
dφ = 1 +
rg
2R
cos α +
1
1 + cos α
dα
Ne segue che
π/2
−π/2
dφ
2rg
dα = π +
dα
R
cos α +
1
1 + cos α
dα = π +
2rg
= π + ∆φ
R
ovvero l’angolo di deflessione ∆φ rispetto alla traiettoria rettilinea è dato da (v. Fig. 2.5)
∆φ =
2rg
R
Esercizio 2.11.6 La propagazione in uno spazio con metrica deformata per la presenza
di una massa M può essere descritta20 come quella in uno spazio con indice di rifrazione
n (r) =
1
1−
20
rg
r
v. J. L. E. Synge, Relativity: the general theory, North-Holland, Amsterdam 1971 Sez. 11.4 Eq. (104)
2.11 Propagazione in spazi curvi
73
Figura 2.4: Geometria della deflessione di un raggio luminoso in prossimità di un buco
nero
Figura 2.5: Angolo di deflessione di un raggio luminoso da parte di una black-hole
74
Richiami di teoria della relatività
e tensore metrico

gij = 
r
r−rg
0
0

0
0

0
r2
2
2
0 r sin θ
(2.58)
Analizzare la propagazione di raggi equatoriali utilizzando le equazioni di Eulero-Lagrange
seguendo l’approccio dell’Es. 1.6.3
Soluzione: Tenuto conto che per una traiettoria equatoriale
ds2 =
r
dr2 + r2 dφ2
r − rg
il cammino ottico L si può riscrivere nella forma
PB
L=
rB
1
n (r (s)) ds =
PA
rA
1−
rg
r
r
2
ṙ2 + r2 φ̇ ds
r − rg
dove . indica la derivata rispetto a s. Poichè la Lagrangiana risulta indipendente da φ si
ha
r2
φ̇ = h
1 − rrg
Pertanto si ha
rg 1 dr
r r2 dφ
rg 1
φ̇ = h 1 −
r r2
ṙ = h 1 −
Sostituendo queste espressioni di ṙ e φ̇ nella relazione
r
2
ṙ2 + r2 φ̇ = 1
r − rg
si ottiene l’Eq. (2.57).
Capitolo 3
Interazioni elettriche e magnetiche
3.1
Riepilogo grandezze elettriche e magnetiche
ρ=densità di carica elettrica per unità di volume, espresso in C/m3 (C = Coulomb).
E=campo elettrico, V /M —rappresenta il gradiente dell’energia o il momento che agisce
su un dipolo elettrico ℘. La risposta di un materiale ad un campo E è originato da una
polarizzazione P, espressa in C/m2 —momento di dipolo elettrico per unità di volume
= ℘/m3 . D = P+ε0 E =vettore induzione, espresso in C/m2 con ε0 = 8.854 × 10−12
F/m.(F = F araday). In un materiale dielettrico a risposta lineare P=ε0 χE, e D =ε0 ε̂E
con χ=suscettività dielettrica e ε̂=costante dielettrica relativa. P e D indicano entrambi
come un materiale risponde ad un campo E.
H= campo magnetico, A/m —rappresenta il gradiente della densità di energia magnetica o il momento che agisce su un dipolo magnetico. B= vettore induzione, espresso in
Tesla T o W eber/m2 —numero di linee di campo per unità di area. M= magnetizzazione,
A/m—momento magnetico per unità di volume. La risposta di un materiale ad un campo
i
H è originato da una corrente i = J (r)·n̂dσ che produce un campo tangenziale H = 2πr
o
da un materiale magnetico. B dipende da H nello spazio libero B = µ0 H con µ0 = 4π10−7
Henry/m mentre in un materiale B = µ0 (H + M) = µ0 µr H con µr = permeabilità relativa
o M = H(µr − 1) = χH con χ = µr − 1 =suscettività. M e B indicano entrambi come un
materiale risponde ad un campo H. Per l’assenza di cariche magnetiche le linee di forza di
B sono continue. Spesso si usano unità cgs: B (Oersted) = H (Gauss) + 4πM (emu/cc)
legate a quelle M KS dalle relazioni 1 Oe = (1000/4π) A/m = 79.6 A/m ;1 G = 10−4 T
; 1 emu/cc = 1 kA/m
Ua distribuzione di carica ρ (r, t) e di corrente J (r, t) , legati tra loro dalla relazione
di continuità
∂
∇ · J (r, t) + ρ (r, t) ,
∂
si possono associare in un volume finito ad una distribuzione di multipoli elettrici Qkm e
magnetici Mkm
Qkm
=
∗
rk Ykm
(θ, φ) ρ (r, t) d3 r ,
Mkm
=
∗
rk Ykm
(θ, φ) ∇ · (r × J (r, t)) d3 r .
Qkm ha le dimenioni di una carica [q] per [Ll ], mentre Mkm quelle di Qkm × c.
75
(3.1)
76
Interazioni elettriche e magnetiche
Nel caso in cui si debba tener conto della magnetizzazione M dovuta agli spin elettronici
e nucleari di un atomo si dimostra1 che bisogna aggiungere ai multipoli di ρ e ∇ · (r × J)
rispettivamente i contributi di ∇ · (r×M) e ∇ · M.
Una distribuzione di cariche indipendenti dal tempo contenute in una sfera di raggio
R genera un potenziale che può essere espanso per r > R in armoniche sferiche
1
V (r) =
ε0
l
∞
l=0
1
Ylq (θ, φ)
Qlq
,
2l + 1
rl+1
q=−l
(3.2)
I contributi proporzionali a Q11 , Q10 e Q1,−1 sono di dipolo, mentre quelli proporzionali a
Q22 , Q21 , Q20 , Q2,−1 e Q2,−2 sono associati a quadrupoli elettrici2 .
Ponendo
QNkq = e
Q′ekq = −e
rk Ykq (θ, φ) nN (r) d3 r
1
rk+1
Ykq (θ, φ) ne (r) d3 r.
ed assumendo con buona approssimazione r′ = r< , r = r> , si ha
ne (r) nN (r′ )
k
r<
Y ∗ (θ, φ) Ykq (θ′ , φ′ ) d3 rd3 r′ = QNk,−q Q′ekq
k+1 kq
r>
per cui
k
VC k
3.2
3.2.1
1
1
=
QNk,−q Q′ekq .
ε0 2k + 1 q=−k
Interazioni elettrostatiche
Multipoli
Esercizio 3.2.1 Con riferimento ad una carica puntiforme q in (r0 , θ0 , φ0 ) rapppresentarla come una sovrapposizione di multipoli Qkq
Soluzione: Una carica puntiforme q in (r0 , θ0 , φ0 ) crea un potenziale pari3
∞
k
k
1
r<
qe
Y ∗ (θ0 , φ0 ) Ykq (θ, φ) =
k+1 kq
2k + 1 r>
4πε0
k=0 q=−k
∞
k
r<
Pk (cos Θ) ,
rk+1
k=0 >
(3.3)
dove r< = min (r, r0 ) , r> = max (r, r0 ). L’ultimo termine, dipendente dal polinomio di
Legendre Pk e dall’angolo Θ l’angolo formato da r e r0 , discende dalla relazione
1
qe
qe
V (r) =
=
4πε0 |r − r0 |
4πε0
k
4π
Y ∗ (θ 0 , φ0 ) Ykq (θ, φ) = Pk (cos Θ)
2k + 1 q=−k kq
1
2
3
v.p.e. J. M. Blatt and V. F. Weisskopf, “Theoretical Nuclear Physics”, J. Wiley & Sons, New York
1952, App. B Eqq. (4.16),(4.19).
v.p.e. S. Flugge, loc. cit. pag. 29, Prob. 54.
v.p.e. A.Balzarotti, M. Cini & M. Fanfoni, Atomi, Molecole e Solidi, Springer, Milano 2004, App. A6.
3.2 Interazioni elettrostatiche
77
che esprime il cosiddetto teorema di addizione delle armoniche sferiche.
Confrontando (3.3) con (3.2) si vede che la carica spostata dall’origine equivale all’insieme di multipoli
Qkq = qe r0k Ykq∗ (θ0 , φ0 )
Esercizio 3.2.2 Calcolare il potenziale di interazione tra (a) una carica puntiforme di
coordinate (r0 , θ 0 , φ0 ) ed una sfera di costante dielettrica di raggio a; (b) come (a) con la
carica sostituita da un dipolo. (c) Discutere il caso limite in cui la distanza dipolo-sfera
tende all’infinito.
Soluzione: La carica associata al potenziale
qe
V (r) =
ε0
k
∞
k
r<
1
Ykq∗ (θ0 , φ0 ) Ykq (θ, φ) ,
k+1
2k + 1 r>
k=0 q=−k
induce un potenziale di scattering qe Vsc (r) all’esterno e qe Vtr (r) all’interno della sfera
rappresentati rispettivamente da
1
Vq sc (r) =
ε0
k
1
Ykq (θ, φ)
Rkq
2k + 1
rk+1
k=0 q=−k
1
ε0 ε̂
Vq tr (r) =
∞
∞
k
1
Tkq rk Ykq (θ, φ)
2k + 1
k=0 q=−k
Imponendo la continuità di V (r)+Vsc (r) e Vtr (r) sulla sfera di raggio a e della componente
radiale dell’induzione si ottiene il sistema
ak ∗
Rkq
1
Tkq ak
Y (θ 0 , φ0 ) + k+1 =
k+1 kq
a
ε̂
r0
ak−1
Rkq
k k+1 Ykq∗ (θ0 , φ0 ) − (k + 1) k+2 = kTkq ak−1
a
r0
che risolto fornisce
ε̂ − 1 a2k+1 ∗
= −q
Y (θ0 , φ0 )
k+1 kq
ε̂ + k+1
k r0
1
2k + 1
= qε̂ k+1
Ykq∗ (θ 0 , φ0 )
r0 kε̂ + k + 1
Rkq
Tkq
Pertanto si ha
1 a
Vq sc (r) = −
4πε0 r0
Vq tr (r) =
1
4πε0
∞
∞
k
4π
1 ε̂ − 1
k+1
2k + 1 r
ε̂ + k+1
k
k=0 q=−k
k
a
r0
k
Ykq (θ, φ) Ykq∗ (θ0 , φ0 )
2k + 1
rk
4π
Y (θ, φ) Ykq∗ (θ 0 , φ0 )
k+1 kq
2k
+
1
kε̂
+
k
+
1
r0
k=0 q=−k
78
Interazioni elettriche e magnetiche
Per ε̂ ≫ 1 si ha
1 ε̂ − 1 a
Vq sc (r) = −
4πε0 ε̂ + 1 Rq
+
∞
k
a2
Rq
4π
1
k+1
2k + 1 r
k=0 q=−k
k
Ykq (θ, φ) Ykq∗ (θ0 , φ0 )
1 ε̂ − 1 R0 1
4πε0 ε̂ + 1 Rq r
a
ovvero Vsc (r) coincide col potenziale di una carica −q ε̂−1
posta in a2 /Rq e di una
ε̂+1 Rq
a
seconda carica q ε̂−1
al centro della sfera.
ε̂+1 Rq
A questo punto il potenziale di interazione è dato da
1
Vq = qe Vq sc (r0 )
2
(b) Se si sostituisce la carica con un dipolo ℘ V℘ sc (r) è dato da
V℘ sc (r) =
q
℘q ·∇Vq sc (r)
Pertanto
V℘ =
=
1
1
℘ · ∇V℘ sc (r) = ℘ · ∇
2
2
℘q ·∇Vq sc (r)
q
1
℘℘ : ∇∇Vq sc (r)
2
(c) Quando r0 → ∞ si ha
1 1 ε̂ − 1 a3
Vq sc (r) = −
3ε0 r2 ε̂ + 2 Rq2
1
Y1q (θ, φ) Y1q∗ (θ0 , φ0 )
q=−1
In particolare per θ0 = 0 si ha
1
∗
Y1m (θ, φ) Y1m
(0, φ0 ) =
m=−1
per cui
Vq sc (r) = −
3
cos θ
4π
q 1 ε̂ − 1 a3
cos θ
4πε0 r2 ε̂ + 2 r02
ovvero Vq sc (r) coincide con quello di un dipolo elettrico
℘=
q ε̂ − 1 a3
3 ε̂ + 2 r02
℘ dà luogo ad una energia di interazione pari a
1
1 qe2 ε̂ − 1 a3
Vq = qe Vq sc (rq ) =
2
2 4πε0 ε̂ + 2 r04
3.2 Interazioni elettrostatiche
79
Esercizio 3.2.3 Calcolare l’energia di interazione di un dipolo elettrico℘1 posto a distanza R da: (a) un altro dipolo℘2 ; (b) da un atomo di polarizzabilità αp
Soluzione: (a) L’energia di interazione è data da V21 = −℘2 ·E21 dove E21 = −∇2 V21
è il campo elettrico generato dal dipolo 1 nel punto occupato da℘2 . Poichè V21 =
− 4πε10 R2 ℘1 · R̂21 con R̂21 = (R2 − R1 ) /R risulta4
E21 =
1 1
−℘1 + 3℘1 · R̂R̂
4πε0 R3
e pertanto
V = −℘2 · E21 =
1 1
℘1 ·℘2 − 3℘1 ℘2 :R̂R̂
4πε0 R3
(b) Nel caso in cui ℘2 = ε0 αp E12 si ha che
1
V21 = − ℘2 · E21
2
In tal caso infatti si deve tener presente che al crescere del momento del dipolo 1 da 0 al
valore finale ℘1 , quello del secondo dipolo cresce proporzionalmete:
dV21
dE21
= −℘2 ·
d℘1
d℘1
= −ε0 αp E12 ·
=
dE21
d℘1
1
d
ε0 α p
|E21 |2
2
d℘1
Pertanto V21 è pari a
1
1 ε0 α p 1
V = − ε0 αp |E21 |2 = −
℘1 − 3℘1 · R̂R̂
2
2 (4πε0 )2 R6
2
1 ε0 αp 1
2
= −
℘
+
3
℘
·
R̂
1
1
2 (4πε0 )2 R6
3.2.2
2
Interazione elettroni-nucleo
Esercizio 3.2.4 Esprimere VC k relativo ad uno stato atomico
|F jImF = Rnl (r)
mj mI
jImj mI |F mF |jmj |ImI
utilizzando le matrici ridotte di multipolo degli elettroni j !Q′e k ! j e dei protoni del
nucleo I !QN k ! I
4
per evitare confusoni con l’energia le componenti scalari del campo vengono indicate con Ex,y,z .
80
Interazioni elettriche e magnetiche
Soluzione: VC k è proporzionale alla somma su q degli elementi di matrice
$
%
F jImF QN k,−q Q′e kq F jImF
$
$
%
%
jImj mI |F mF jIm′j m′I |F mF ImI |QN k,−q | Im′I jmj Q′e kq jm′j
=
mj m′j mI m′I
j !Q′e k ! j I !QN k ! I
=
×
jImj mI |F mF
mj m′j mI m′I
%$
%
$
jIm′j m′I |F mF jkmj q|jm′j IkmI , −q|jm′I
In particolare si può dimostrare che
q
mj m′j mI m′I
= (−1)F +j+I
jImj mI |F mF
I, j, F
j, I, k
$
%$
%
jIm′j m′I |F mF jkmj q|jm′j IkmI , −q|Im′I
dove la quantità in parentesi graffe è il cosiddetto simbolo5 6j.
VC k =
1
4π
(−1)F +j+I
4πε0 2k + 1
I, j, F
j, I, k
j !Q′e k ! j I !QN k ! I
(3.4)
Esercizio 3.2.5 (a) Analizzare l’interazione di quadrupolo tra un nucleo e gli elettroni.
(b) Analizzare il caso del deuterone
Soluzione: (a) Per un insieme di cariche qn = e in rn associate allo stato |Ψ , le
componenti del tensore di quadrupolo elettrico sono date da:
Q2q = Ψ |Q2q | Ψ
dove
√
6
=
qn (xn ± iyn )2
4 n
√
6
= ∓
qn zn (xn ± iyn )
2 n
Q2,±2
Q2,±1
Q20 =
1
2
n
qn 3zn2 − rn2
(3.5)
Il momento di quadrupolo elettrico del nucleo è dato da
QN =
2
I, MI = I |QN2,0 | I, MI = I
e
dove QN2,0 è un operatore tensoriale del secondo ordine:
QN 2,0 =
5
v.p.e.
44.
e
2
p
(3zp2 − rp2 ) = e
p
4 2
π r Y20 (Ωp )
5 p
M. Weissbluth, Atoms and Molecules, Academic Press, N. Y. 1978 pp.
I, j, F
=SixJSymbol[{I,j,F},{j,I,k}]
j, I, k
38-
3.2 Interazioni elettrostatiche
81
Dal teorema di Wigner-Eckart discende che6
I, MI = I |QN2,0 | I, MI = I
=
I !QN2 ! I I2I0|II
√
I(2I − 1)
= (−1)4I 1 + 2I
3 + 11I + 12I 2 + 4I 3
Pertanto tenuto conto che
= ClebschGordan[{I,I},{2,0},{I,I}]
√
I(2I − 1)
= (−1)4I 1 + 2I
3 + 11I + 12I 2 + 4I 3
I2I0|II
si ha:
I !QN2 ! I
1
√
=
2
2I + 1
Analogamente si ha per7 Q′e 2
Q′e =
con
Q′e 2,0 =
e
2
n
(I + 1) (2I + 3)
eQ
I (2I − 1)
%
2$
j, mj = j Q′e 2,0 j, mj = j
e
3zn2 − rn2
=e
rn5
n
4 1
π Y20 (Ωn )
5 rn3
dove la somma è estesa agli elettroni. Pertanto
1
j !Q′e 2 ! j
√
=
2
2j + 1
(j + 1) (2j + 3) ′
eQe
j (2j − 1)
Sostituendo nella (3.4) si ha per k = 2
VC 2 =
1 4π
(−1)F +j+I
4πε0 3
I, j, F
j, I, 2
j !Q′e 2 ! j I !QN 2 ! I
dove8
I, j, F
j, I, 2
= SixJSymbol[{I,j,F},{j,I,k}]
= (−1)−F −I−j
2 [3X (X − 1) − 4I (I + 1) j (j + 1)]
(2I − 1) 2I (2I + 1) (2I + 3) (2j − 1) 2j (2j + 1) (2j + 3)
con
X = I (I + 1) + j (j + 1) − F (F + 1)
6
7
I2I0|II =ClebschGordan[{I,I},{2,0},{I,I}]
%
jm
Yl 1 j = l 12 jm risultano dalla composizione del momento orbitale l e lo spin s =
1
2
si possono
%
1
e
dello
spin
m
esprimere sovrapponendo prodotti di funzioni
d’onda
del
momento
orbitale
|lm
L
s
2
$
pesati
dai coefficienti
di Clebsch-Gordan l 12%mz ms |jmj |lml (v. Appendice D),
%
$
&
l 12 jm = mz ,ms l 12 mms |jmj |lml ⊗ 12 ms
8
v.p.e.M. Weissbluth, “Atoms and Molecules”, Academic Press, N.Y. 1978 Eq. (18.3-11)
2
82
Interazioni elettriche e magnetiche
Infine
$
%
F jImF QN k,−q Q′e kq F jImF
2
= e
3
X
′
4
Qe QN
(X − 1) − I (I + 1) − j (j + 1)
2I (2I − 1) j (2j − 1)
(3.6)
La quantità e2 Q′e QN è nota come costante di accoppiamento di quadrupolo.
(b) Il deuterone ha spin nucleare I = 1 con associato un momento magnetico nucleare
pari a 0.857 µN ed un momento di quadrupolo QD = e 0.0028 (misurato in barns 1
barn = 10−28 m2 ≃ sezione del nucleo di uranio). pari a
 1

−2 0 0
QD = e 0.0028  0 − 12 0
0
0 1
Pertanto (3.6) si riduce a
$
%
F jImF QN k,−q Q′e kq F jImF
e2 Q′e QN
3
=
X (X − 1) − 2j (j + 1)
2j (2j − 1) 4
Esercizio 3.2.6 Calcolare l’energia di interazione coulombiana tra la carica nucleare Ze
di un atomo idrogenoide supposta distribuita uniformemente in una sfera di raggio ”b”
ed un elettrone associato ad una funzione d’onda ve (r) ∝ exp(−Zr/a0 ). Utilizzare unità
MKS.
Soluzione: Si cominci con l’assumere una carica Z uniformemente distribuita in una
sfera di raggio assegnato R (raggio del nucleo). In tal caso il potenziale sarà dato da (v.
Fig. (3.1))
− Zr
r>R
2
V (r) =
(3.7)
Z
r
−3
r<R
2R R2
Pertanto, chiamando ∆V la differenza tra il potenziale 1/r e V (r)
∆V (r) =
0
Z
2R
r2
R2
+
2R
r
−3
r>R
r<R
e trattando ∆V come una perturbazione, si può facilmente calcolare la correzione del
livello n-esimo
2π 2
2 Z4 2
∆E ≃
R |ψ n00 (0)|2 =
R
(3.8)
5
5 n3
Si considerino ora due atomi idrogenoidi corrispondenti a due isotopi di uno stesso elemento. Dal momento che il raggio R varia col numero di massa A di un nucleo secondo
la relazione9
R = r0 A1/3
(3.9)
con
9
r0 ≃ 1.2 × 10−15 m = 1.22 F ermi
R. Hofstadter, Structure of nuclei and nucleons, Science, 136:1013, 1962, discorso tenuto in occasione
del Nobel
3.2 Interazioni elettrostatiche
83
e che A ≃ 2Z si avranno correzioni diverse
δ∆E ≃
1 A11/3 2
r δA
60 n3 0
(3.10)
Indicando con
Figura 3.1: Potenziale all’interno del nucleo per una carica nucleare distribuita
uniformemente.
exp −2 Zr
a0
ve2 (r)
4
ρe (r) = −Ze
= eZ
∞ 2
3
2
πa0
4π 0 ve (r) r dr
la densità di carica associata all’elettrone, l’energia di interazione risulta pari a
Ve−Z =
1
4πε0
ρZ (rp ) ρe (re )
drp dre =
re−Z
dove
VZ (re ) =
1
4πε0
ρe (r) VZ (r) dr
ρZ (r)
dr
|r − re |
rappresenta il potenziale VZ associato alla distribuzione di carica ρZ (rp ) del nucleo. Per
calcolare VZ (r) si nota che (i) esso dipende solo dalla distanza re , (ii) all’esterno della
sfera di raggio b coincide con quello di una carica puntiforme Ze:
VZ (r) =
Ze 1
4πε0 r
mentre (iii) all’interno della sfera il campo elettrico varia linearmente:
E (r) = c1 r ,
per cui
1
VZ (r) = c1 r2 + c2 .
2
84
Interazioni elettriche e magnetiche
Imponendo ora la continuità di VZ (r) e della derivata per r = b si ha:
Ze 1
4πε0 b2
Ze 1
=
4πε0 b
c1 b =
1 2
c1 b + c2
2
ovvero
VZ (r) =
Ze
4πε0
1
r
2b3
1
r
2
+
1
2b
Finalmente,
b
∞
1 2
Z 5 e2
Zr
1
Zr
2
rdr
r
exp
−2
r
+
dr
+
exp
−2
3
πε0 a0 0
a0
2b3
2b
a0
b
Z 5 e2 3 + c2 − (1 + c2 ) (3 + 6c + 4c2 ) exp (−2c)
=
+ (1 + 2c) exp (−2c)
4πε0 a0
2c3
Ve−Z =
con c = Zb/a0 . Poichè la carica nucleare è distribuita in volumi di raggio b dell’ordine di
1 F ermi = 10−5 Å c = 2 · 10−5 Z, Ve−Z si semplifica in
Ve−Z =
3.2.3
Z 5 e2 3a20
Z 3 e2 3a0
=
4πε0 a0 Z 2 b2
4πε0 b2
Interazione elettrone-elettrone in un atomo
Esercizio 3.2.7 Calcolare l’energia di interazione (a) diretta e (b) di scambio
'
(
1
J =
n1 n2 l1 l2 m1 m2 | |n1 n2 l1 l2 m1 m2
r12
'
(
1
K =
n1 n2 l1 l2 m1 m2 | |n2 n1 l2 l1 m2 m1
r12
tra due elettroni di un atomo associati alle funzioni d’onda Rn1 l1 (r1 ) Yl1 m1 (Ω1 ) e Rn2 l2 (r2 ) Yl2 m2 (Ω2 ) .
Soluzione: (a) Sviluppando in serie di polinomi di Legendre il potenziale di interazione
coulombiano 1/r12
L
∞
L
1
4π
r<
=
L+1
r12 L=0 2L + 1 r>
∗
YLM (Ω2 )YLM
(Ω1 )
M=−L
si ottiene per l’integrale coulombiano J
J =
∞
4π
2L + 1
L=0
∞
0
∞
dr1
dr2
0
L
r<
P 2 (r )Pn22 l2 (r2 )
L+1 n1 l1 1
r>
l1 l2 L
×Im
1 m2
con
l1 l2 L
=
Im
1 m2
∗
dΩ1 |Yl1 m1 (Ω1 )|2 YL0
(Ω1 )
dΩ2 |Yl2 m2 (Ω2 )|2 YL0 (Ω2 )
3.2 Interazioni elettrostatiche
85
Dalla formula di Gaunt discende:
∗
∗
dΩ |Yl1 m1 (Ω1 )|2 YL0
(Ω1 )Yl1 m1 (Ω) Yl∗2 m2 (Ω) YL,m
(Ω)
1 −m2
2l1 + 1
l1 l1 00 |L0 l1 l1 m1 , −m1 |L0
4π(2L + 1)
=
Pertanto
(2l1 + 1) (2l1 + 1)
l1 l1 00 |L0 l1 l1 m1 , −m1 |L0
4π(2L + 1)
l2 l2 00 |L0 l2 l2 m2 , −m2 |L0
l1 l2 L
=
Im
1 m2
l1 l2 L
= 0 solo per |l1 − l2 | ≤ L ≤ |l1 + l2 |, da cui
Ne segue che Im
1 m2
∞
J =
∞
dr1
0
|l1 +l2 |
L=|l1 −l2 |
0
dr2 Pn21 l1 (r1 )Pn22 l2 (r2 )
L
r<
4π
l1 l2 L
× Im
1 m2
L+1
2L + 1 r>
(b) Per quello di scambio si ha
∞
4π
K =
2L + 1
L=0
∞
∞
dr1
0
0
dr2
L
r<
P (r )Pn2 l2 (r1 )Pn1 l1 (r2 )Pn2 l2 (r2 )
L+1 n1 l1 1
r>
l1 l2 L
×Im
1 m2 ,m1 −m2
dove
l1 l2 L
Im
=
1 m2 ,m1 −m2
∗
dΩ1 Yl1 m1 (Ω1 ) Yl∗2 m2 (Ω1 ) YL,m
(Ω1 )
1 −m2
dΩ2 Yl2 m2 (Ω2 ) Yl∗1 m1 (Ω2 ) YL,m1 −m2 YLM (Ω2 )
×
Dalla formula di Gaunt discende:
∗
dΩYl1 m1 (Ω) Yl2∗m2 (Ω) YL,m
(Ω)
1 −m2
= (−1)m1 −m2
(2l1 + 1)(2l2 + 1)
l1 l2 00 |L0 l1 l2 m1 , −m2 |L, m1 − m2
4π(2L + 1)
Per cui
l1 l2 L
Im
=
1 m2 ,m1 −m2
(2l1 + 1)(2l2 + 1)
l1 l2 00 |L0
4π(2L + 1)
2
l1 l2 m1 , −m2 |L, m1 − m2
In definitiva si ha:
K =
∞
∞
dr1
0
|l1 +l2 |
L=|l1 −l2 |
dr2 Pn1 l1 (r1 )Pn2 l2 (r1 )Pn1 l1 (r2 )Pn2 l2 (r2 )
0
L
r<
4π
l1 l2 L
× Im
L+1
1 m2 ,m1 −m2
2L + 1 r>
2
86
Interazioni elettriche e magnetiche
Esercizio 3.2.8 Calcolare gli integrali coulombiano J0 e di scambio K0 per due elettroni
negli orbitali n1 s e n2 s.
Soluzione:
∞
1 2
Pn1 0 (r1 )Pn22 0 (r2 )
r>
0
0
∞
∞
1
1 r1
2
2
= 4π
dr1 Pn1 0 (r1 )
dr2 Pn2 0 (r2 ) +
dr2 Pn22 0 (r2 )
r1 0
r2
0
r1
∞
∞
1
= 4π
dr1
dr2 Pn1 0 (r1 )Pn2 0 (r1 )Pn1 0 (r2 )Pn2 0 (r2 )
r>
0
0
J0 = 4π
K0
∞
∞
= 4π
dr1
dr2
dr1 Pn1 0 (r1 )Pn2 0 (r1 )
0
×
3.2.4
1
r1
r1
dr2 Pn1 0 (r2 )Pn2 0 (r2 ) +
∞
dr2
r1
0
1
Pn 0 (r2 )Pn2 0 (r2 )
r2 1
Interazione elettrone-elettrone in una molecola
Esercizio 3.2.9 Calcolare l’energia di interazione coulombiana tra i due elettroni di una
molecola di idrogeno che occupano rispettivamente l’orbitale atomico u1s (rA ) centrato sul
nucleo A e u1s (rB ) centrato su B10
j=
u21s (r1A ) u21s (r2B ) 3 3
d r1 d r2
r12
u2 (r
)
1s 1A
Soluzione: Innanzitutto conviene notare che
d3 r1 rappresenta il potenziale in
r12
r2 creato dalla distribuzione di carica u21s (r1A ) , ovvero coincide con l’integrale J (r2A ) (v.
Eq. (??)). Pertanto, Utilizzando coordinate sferoidali (v. (1.5)) si ha
j =
=
=
=
=
=
u21s (r1A ) u21s (r2B ) 3 3
d r1 d r2 = u21s (r2B ) J (r2A ) d3 r2
r12
1
1
2π u21s (r2B )
− e−2r2A 1 +
d3 r2
r2A
r2A
1
2πJ (R) − 8π e−2(r1A +r1B ) 1 +
d3 r2
r2A
2
2πJ (R) − R3 e−2Rξ 1 +
ξ 2 − η 2 dξdηdφ
R (ξ − η)
∞
1
2
2πJ (R) − 2πR3
e−2Rξ
ξ 2 − η 2 + (ξ + η) dη dξ
R
1
−1
π −2R
2πJ (R) − e (9 + 2R(9 + 2R))
3
Esercizio 3.2.10 Calcolare l’integrale k analogo al precedente ma con gli elettroni associati alla somma degli orbitali u1s (rA ) + u1s (rB )
10
W. Heitler and F. London, Z.f.Phys. 44, 455 (1927); Y. Sugiura, Z.f. Phys. 45, 484 (1927); L. Pauling
and E. B. Wilson, Introduction to Quantum Mechanics, Dover Publications (1985) p. 343
3.2 Interazioni elettrostatiche
87
u21s (r1A ) u21s (r2B ) + u21s (r1B ) u21s (r2A ) 3 3
d r1 d r2
r12
u1s (r1A ) u1s (r1B ) u1s (r2A ) u1s (r2B ) 3 3
d r1 d r2
r12
k =
+
con
u1s (r1A ) u1s (r1B ) u1s (r2A ) u1s (r2B ) 3 3
A−B
d r1 d r2 =
r12
5
espressione calcolata da Sugiura che si ritrova sul libro di Pauling e Wilson, dove
6
S 2 (R) (γ + ln R) − S 2 (−R)E1 (−4R) + 2S(R)S(−R)E1 (−2R)
R
25 23
1
B =
− + R + 3R2 + R3 e−2R
8
4
3
A =
con S(R) = u1s (rA ) u1s (rB ) dr = e−R 1 + R + 13 R2 integrale di sovrapposizione, γ =
0.57722... la costante di Eulero-Mascheroni ed E1 (x) la funzione integrale esponenziale.
Esercizio 3.2.11 Calcolare l’energia di interazione tra due elettroni associati alle combinazioni di orbitali atomici v1s (r1A ) , v1s (r2A ) , v1s (r2B )
Soluzione: Seguendo la procedura illustrata nei precedenti esercizi si ottiene
ℓ =
=
1
2
2
(r1A ) v1s (r2A ) v1s (r2B ) 3 3
v1s
d r1 d r2
r12
1 5
1 5
2R + + R e−R −
+ R e−3R
4 8
4 8
Esercizio 3.2.12 Altri integrali di Coulomb e di scambio che intervengono quando si
utilizzano le funzioni d’onda di Heitler-London sono
J
2
2
(r1A )v1s
(r2B )
v1s
=
K =
1
1
1
−
−
r12 r1A r2B
v1s (r1A )v1s (r1B )v1s (r2A )v1s (r2B )
d3 r1 d3 r2 = j − 2j ′
1
1
1
−
−
r12 r2A r1B
d3 r1 d3 r2
= k − 2k ′ S
Calcolare j ′ e k ′ .
Soluzione: Per normalizzare gli orbitali molecolari che siano combinazioni degli orbitali
atomici v1s , bisogna introdurre l’integrale di sovrapposizione S(R)
S(R) =
v1s (rA ) v1s (rB ) dr =
1
π
exp (−rA − rB ) dr
Per calcolarlo conviene utilizzare le coordinate sferoidali (v. Eq. (1.5)), ottenendo così
S (R) =
1 R3
π 8
exp (−Rξ) ξ 2 − η 2 dξdηdφ =
1
1 + R + R2 exp (−R)
3
(3.11)
88
Interazioni elettriche e magnetiche
Inoltre, il funzionale dell’energia di molecole biatomiche descritte da orbitali molecolari
che siano combinazioni degli orbitali atomici u1s , può essere espresso come combinazione
di un integrale di Coulomb j’ k,
1 2
R2
v1s (rA ) dr =
exp (−R (ξ + η)) (ξ + η) dξdη
rB
2
1
1+R
=
−
exp (−2R) ,
R
R
j′ =
(3.12)
e di risonanza k’
1
R2
v1s (rB ) v1s (rA ) dr = −
rB
2
= (1 + R) exp (−R)
k′ =
exp (−R (ξ − η)) (ξ − η) dξdη
(3.13)
Per calcolare l’energia di interazione tra due elettroni che occupano l’orbitale molecolare 1sσ si utilizzano integrali del tipo11
j =
=
1
R
k =
ℓ =
=
m =
1
2
2
2
(r1A ) v1s
(r2B )
v1s
dr1 dr2
r12
1 2
11 3
1
−
+
+ R + R2 exp (−2R) ,
2 R
4
2
3
v1s (r1A ) v1s (r2A ) v1s (r1B ) v1s (r2B )
A−B
dr1 dr2 =
r12
5
2
v1s (r1A ) v1s (r2A ) v1s (r2B )
dr1 dr2
r12
1 5
1 5
2R + + R e−R −
+ R e−3R
4 8
4 8
2
2
5
v1s (r1A ) v1s (r2A )
dr1 dr2 =
r12
8
(3.14)
dove
6
S 2 (R) (γ + ln R) − S 2 (−R)E1 (4R) + 2S(R)S(−R)E1 (2R)
R
25
R
R3 −2R
B =
− + 23 + 3R2 +
e
8
4
3
A =
essendo γ = 0.57722... la costante di Eulero ed E1 (x) la funzione integrale esponenziale.
Altri integrali di Coulomb e di scambio che intervengono quando si utilizzano le
funzioni d’onda di Heitler-London sono
j =
K =
11
2
2
v1s
(r1A )v1s
(r2B )
1
1
1
−
−
r12 r1A r2B
v1s (r1A )v1s (r1B )v1s (r2A )v1s (r2B )
dr1 dr2 = j − 2j ′
1
1
1
−
−
r12 r2A r1B
dr1 dr2 = k − 2k ′ S
per una descrizione più completa di queste tecniche di integrazione v. J. C. Slater, Quantum Theory
of Molecules and Solids, McGraw-Hill (1963), S. P. McGlynn, L. G. Vanquickenborne, M. Kinoshita
and D.G. Carroll, Introduction to Applied Quantum Chemistry, Holt, Rinehart and Winston (1972)
3.2 Interazioni elettrostatiche
3.2.5
89
Elettroni di un metallo
Esercizio 3.2.13 Si considerino gli elettroni di conduzione di un metallo. Questi si
muovono in un potenziale periodico associato agli ioni nei vertici di un reticolo cristallino supposto cubico. Calcolare l’energia di interazione di questo potenziale con un elettrone assimilato ad una particella in una buca di potenziale, come assunto nel modello di
Sommerfeld
Soluzione: Il potenziale associato agli ioni del reticolo soddisfa l’equazione di Poisson
∇2 Vret = −
che trasformata diventa
Ṽret (k) =
1
ρ (r)
ε0 ioni
1 ρ̃ioni (k)
ε0 k 2
con k 2 = kx2 + ky2 + kz2 ovvero
Vret (r) =
1
ε0
ρ̃ioni (k) ik·r 3
e dk
k2
Ne segue che un elettrone confinato in una scatola cubica di lato L è descritto da una
funzione d’onda
1
v1 (r) =
sin k̄x x sin k̄y y sin k̄z z
(L/2)3/2
con kx,y,z L multipli pari di π. Ne segue che l’energia di interazione è data da
1
sin2 k̄x x sin2 k̄y y sin2 k̄z z Vret (r) d3 r
(L/2)3 V
1
1
ρ̃ioni (k) 3
d k sin2 k̄x x sin2 k̄y y sin2 k̄z z eik·r d3 r
=
3
ε0 (L/2)
k2
V
1
1
ρ̃ioni (k) 2k̄x sin (kx L/2) 2k̄y sin (ky L/2) 2k̄z sin (kz L/2) i(kx +ky +kz )L/2 3
= −
e
dk
3
ε0 (L/2)
k2
kx2 − 4k̄x2
ky2 − 4k̄y2
kz2 − 4k̄z2
V = −
dal momento che
L
0
sin2 k̄x x eikx x dx = −
1
4
1
≃ −
4
= −i
L
"0
kx2
ei2k̄x x + e−i2k̄x x − 1 eikx xdx
ei(2k̄x +kx )L − 1
ei(−2k̄x +kx )L − 1
−i
+i
2k̄x + kx
−2k̄x + kx
#
k̄x
2k̄x
eikx L − 1 = 2
sin (kx L/2) eikx L/2
2
− 4k̄x
kx − 4k̄x2
essendo ei2k̄x L = 1.Poichè ρioni (r) = ρioni (r + Rlmn ) per r all’interno del cristallo
ρ̃ioni (k) ≃
lmn
ρ̃{lmn} δ (3) k − K{lmn}
90
Interazioni elettriche e magnetiche
con K{lmn} vettore del reticolo reciproco. Pertanto
V = −
1
1
ε0 (L/2)3
lmn
ρ̃ioni{lmn} 2k̄x sin K{lmn}x L/2
2
2
K{lmn}
K{lmn}x
− 4k̄x2
2k̄y sin K{lmn}y L/2 2k̄z sin K{lmn}z L/2 i(K{lmn}x +K{lmn}y +K{lmn}z )L/2
e
2
2
K{lmn}y
− 4k̄y2
K{lmn}z
− 4k̄z2
Essendo i contributi più importanti alla sommatoria quelli per cui K{l̄m̄n̄} ≃ 2k̄ si può
porre
V ≃ −
1 1
ε0 4k̄ 2
sin
ρ̃ioni{lmn}
lmn
K{lmn}x − 2k̄x L/2 sin K{lmn}y − 2k̄y L/2 sin K{lmn}z − 2k̄z L/2
(3.15)
K{lmn}x − 2k̄x L/2
K{lmn}y − 2k̄y L/2
K{lmn}z − 2k̄z L/2
In conclusione la distribuzione di carica associata alla funzione d’onda sin k̄x x sin k̄y y sin k̄z z
interagisce con la componente di vettore d’onda 2k̄ della distribuzione ionica
Esercizio 3.2.14 Considerare due elettroni di un metallo descritti da funzioni d’onda
tipiche di una scatola cubica che si annullano sulle pareti
Soluzione: Partendo dall’esercizio precedente si ha che l’elettrone 1 si muove nel
potenziale periodico dell’elettrone 2 di funzione d’onda
v2 (r) =
1
(L/2)3/2
sin k̄2x x sin k̄2y y sin k̄2z z
Prtanto ρ̃ioni{lmn} dovuta al secondo elettrone è data da
1
sin2 k̄2x x sin2 k̄2y y sin2 k̄2z z eiK{lmn} ·r d3 r
(L/2)3 V
1 sin K{lmn}x − 2k̄2x L/2 sin K{lmn}y − 2k̄2y L/2 sin K{lmn}z − 2k̄2z L/2
=
8
K{lmn}x − 2k̄2x L/2
K{lmn}y − 2k̄2y L/2
K{lmn}z − 2k̄2z L/2
ρ̃ioni{lmn} =
per cui applicando la (3.15) si ha
V ≃ −
1 1
ε0 4k̄12
sin
sin
sin
lmn
K{lmn}x − 2k̄2x L/2 sin K{lmn}x − 2k̄1x L/2
K{lmn}x − 2k̄2x L/2
K{lmn}x − 2k̄1x L/2
K{lmn}y − 2k̄2y L/2 sin K{lmn}y − 2k̄1y L/2
K{lmn}y − 2k̄2y L/2
K{lmn}y − 2k̄1y L/2
K{lmn}z − 2k̄2z L/2 sin K{lmn}z − 2k̄1z L/2
K{lmn}z − 2k̄2z L/2
K{lmn}z − 2k̄1z L/2
Se ne evince che i due elettroni interagiscono solo se k̄1 ≃ k̄2 .
3.3 Interazioni nei cristalli
91
Esercizio 3.2.15 Si considerino due elettroni confinati su un segmento di lunghezza L e
descritti dalla funzione d’onda
Ψ (x1 , x2 ) =
2
det
L
sin (k1 x1 ) sin (k2 x1 )
sin (k1 x2 ) sin (k2 x2 )
con k1 e k2 tali che k1,2 L.sono multipli entrambi pari di π. Calcolare l’energia di interazione
assumendo un potenziale del tipo
V (x1 , x2 ) =
e2
4πε0
∞
eik(x1 −x2 )
dk
2
2
−∞ k + a
Soluzione: L’energia di interazione sarà data da
V=
e2
4πε0
L
∞
−∞
0
eik(x1 −x2 ) 2
Ψ (x1 , x2 ) dx1 dx2 dk
k 2 + a2
Sviluppando gli integrali si ottiene
L
eik(x1 −x2 ) Ψ2 (x1 , x2 ) dx1 dx2
0
= 32
sin2 (kL/2) k22 k12 (k12 − k22 )
2
2
6k 4 + (k12 − k22 ) − 4k 2 (k12 + k22 )
2
(kL/2)2 (k 2 − 4k12 ) (k 2 − 4k22 ) k 4 + (k12 − k22 ) − 2k 2 (k12 + k22 )
2
= f (k, k1 , k2 )
Si può verificare che f (k) è una funzione continua di k . Pertanto applicando il teorema
dei residui si ottiene
∞
f (k, k1 , k2 )
e2
V =
dk
4πε0 −∞ k 2 + a2
f (ia, k1 , k2 )
e2
2πi
=
4πε0
2ia
2
2
4 sinh2 (aL/2) k12 k22 (k12 − k22 ) 6a4 + 4a2 (k12 + k22 ) + (k12 − k22 )
e2
=
2π
4πε0
(aL/2)2 a (a2 + 4k12 ) (a2 + 4k22 ) a4 + 2a2 (k12 + k22 ) + (k12 − k22 )2
3.3
3.3.1
2
Interazioni nei cristalli
Campo locale
Esercizio 3.3.1 Il campo locale in un dieletrico nel punto di raggio vettore r si ottiene
isolando una sfera con centro in r e raggio abbastanza piccolo. Il campo in r è dato dalla
somma del campo medio E + quello creato dalla distribuzione di cariche sulla superficie sferica Esup + quello prodotto dai dipoli elettrici contenuti nella sfera Ep . Mentre il
campo Esup può essere calcolato trattando il dielettrico come un mezzo continuo, Ep va
calcolato tenendo esplicitamente conto della distribuzione dei dipoli ai vertici di un reticolo cristallino. Nel caso in cui questo è del tipo cubico semplice, verificare che Ep è
trascurabile.
92
Interazioni elettriche e magnetiche
Soluzione: Si immagini che il cristallo sia polarizzato lungo l’asse z. In tal caso la
componente Ez (0) creata dai dipoli atomici ℘ = a3 P, posti all’interno di una sfera di
raggio N a, con “a” passo reticolare, e localizzati ai vertici a (lx̂ + mŷ + nẑ) del reticolo
cristallino sarà dato da
℘
Ez (r) = −
4πε0 a3
l2 +m2 +n2 ≤NN 2
l2 + m2 − 2n2
(l2 + m2 + n2 )5/2
lmn
Questa somma si può facilmente calcolare utilizzando Mathematica: Clear[N N, a, b, c]; a =
IntegerP art[NN 2 − m2 − n2 ]; b = IntegerP art[NN 2 − n2 ]; c = IntegerP art[NN 2 −
m2 ]; Sum[{n, 1, NN }, Sum[{m, −b, b}, Sum[{l = −a, a}, (l2 +m2 −2n2 )/(l2 +m2 +n2 )5/2 ]]]
+Sum[{m, 1, N N}, Sum[{l, −c, c}1/((l2 + m2 )3/2 + Sum[{l, 1, N N}, 1/l3 ]]]. Si vede così
che già per N N = 3 la sommatoria risulta minore di 10−16 .
Esercizio 3.3.2 Il campo locale in un dielettrico nel punto di raggio vettore r si ottiene
isolando una sfera con centro in r e raggio abbastanza piccolo. Il campo in r è dato
dalla somma del campo medio E più quello creato dalla distribuzione di cariche sulla
superficie sferica Esup + quello prodotto dai dipoli elettrici contenuti nella sfera Ep . Mentre
il campo Esup può essere calcolato trattando il dielettrico come un mezzo continuo, Ep va
calcolato tenendo esplicitamente conto della distribuzione dei dipoli ai vertici di un reticolo
cristallino. Nel caso in cui questo è del tipo cubico semplice, verificare che Ep è nullo.
Soluzione: Il campo creato da un dipolo℘ in r0 è dato da
E (r) =
1
4πε0
3
1
1
RR − 3 ∇R ·℘ =
3R̂R̂ − 1 ·℘
5
R
R
4πε0 R3
con R = r − r0 . Pertanto, il campo nel vertice del reticolo cubico R{000} = 0 è dato da
E (0) =
1
4πε0 a3
1
3R̂{m} R̂{m} − 1 ·℘
3
R{m}
{m}∈V
con la somma estesa ai vertici R{lmn} ∈ V contenuti in una sfera di raggio N a con ′′ a′′
passo reticolare. Ne segue che il campo dipende dalla diade
1
Γ=
{m}∈V
(l2 + m2 + n2 )3/2
3R̂{m} R̂{m} − 1
con
lx̂ + mŷ + nẑ
R̂{m} = √
l2 + m2 + n2
Per quanto riguarda i termini fuori diagonale di Γ si ha
lm
Γxy = 3
{m}∈V
(l2 + m2 + n2 )3/2
)
*
Dal momento che ad un generico vertice {lmn} ∈ V corrisponde un altro vertice ¯l, mn ∈
V Γxy risulta nullo e così per gli altri elementi fuori diagonale. D’altra parte
l2
Γxx = 3
{m}∈V
(l2 + m2 +
n2 )5/2
−
1
{m}∈V
(l2 + m2 + n2 )3/2
3.3 Interazioni nei cristalli
93
Figura 3.2: Geometria relativa al calcolo della costante di Madelung per un cristallo di
NaCl.
Per ragioni di simmetria
l2
{m}∈V
(l2 + m2 + n2 )5/2
m2
=
{m}∈V
=
1
3
(l2 + m2 + n2 )5/2
{m}∈V
n2
=
{m}∈V
l2 + m2 + n2
(l2 + m2 + n2 )5/2
(l2 + m2 + n2 )5/2
per cui Γxx = 0. Se ne evince che Γ = 0
Esercizio 3.3.3 Calcolare l’energia di interazione elettrostatica tra gli ioni del cristallo
di NaCl
Soluzione: Gli ioni del cristallo di NaCl sono disposti alternativamente ai vertici di un
reticolo cubico di costante reticolare. Ogni ione è circondato da 6 primi vicini
di carica
√
opposta a distanza a, da 12 secondi vicini aventi la stessa carica e distanti 2a e così via.
Pertanto l’energia di un singolo ione è data da
V=
e2
12
8
6
−6 + √ − √ + + · · ·
4πε0 a
2
3 2
=
e2
M
4πε0 a
con M costante di Madelung (formula di Benson)12
M=
∞′
i,j,k=−∞
(−1)i+j+k
i2 + j 2 + k 2
= −12π
∞
m,n=1,3,...
sec h2
π√ 2
m + n2 = −1.74756
2
Esercizio 3.3.4 Calcolare per un cristallo di NaCl il potenziale locale in cui sono immersi
i singoli ioni.
12
http://mathworld.wolfram.com/BensonsFormula.html
Finch, S. R. Madelung’s Constant. §1.10 in Mathematical Constants, pp. 76-81, Cambridge University
Press, Cambridge, 2003.
94
Interazioni elettriche e magnetiche
Soluzione: Gli ioni del cristallo di NaCl sono disposti alternativamente ai vertici di un
reticolo cubico di costante reticolare a. Ogni ione è circondato da 6 primi √
vicini di carica
opposta a distanza a, da 12 secondi vicini aventi la stessa carica e distanti 2a e così via.
In regioni prive di cariche il potenziale soddisfa l’equazione di Laplace ∇2 V = 0 e può
essere espanso in multipoli
Alm rl + Blm
V (r) =
lm
1
Ylm (θ, φ)
rl+1
Se si sceglie come origine la posizione dello ione in prossimità del quale si vuole calcolare
il potenziale creato dagli altri ioni ed assi coincidenti con quelli del reticolo cristallino si
1
svaniscono e
ha che i termini in rn+1
Alm rl Ylm (θ, φ)
V (r) =
lm
Questo potenziale deve risultare invariato imprimendo una rotazione di π/2 attorno a z
ed una riflessione rispetto al piano xy,
Alm rl Ylm (θ, φ) =
lm
Alm rl Ylm θ, φ +
lm
l
Alm r Ylm (θ, φ) =
lm
lm
π
2
Alm rl Ylm (π − θ, φ)
Poichè queste uguaglianze debbono valere per ogni terna di valori di r, θ, φ deve risultare13
Alm Ylm (θ, φ) =
Alm Ylm θ, φ +
m
m
Alm Ylm (θ, φ) =
m
m
π
2
π
Alm Ylm (θ, φ) eim 2
=
Alm Ylm (π − θ, φ) =
m
Alm Ylm (π, φ) (−1)l+m
m
avendo tenuto conto dell’espressione di Ylm (θ, φ) e Plm (x)
Ylm (θ, φ) = (−1)m
(2l + 1) (l − m)! m
Pl (cos θ) eimφ
4π (l + m)!
(−1)m
1 − x2
2l l!
Plm (x) =
m/2
dl+m
x2 − 1
dxl+m
l
Ne segue che m deve risultare uguale a 0, ±4p ed l = 2q. Si avrà quindi Alm = 0 per
l = 1, 2, 3 mentre per l = 4 V (r) si riduce a
V (r) = V0 + r4
(A40 Y40 (θ, φ) + A44 Y44 (θ, φ) + A∗44 Y44∗ (θ, φ))
m
= V0 + r4
dal momento che
13
9
4π
2
A40 P40 (cos θ) + √ A44 Pl4 (cos θ) cos 4φ
8!
∗
(θ, φ)
Ylm̄ (θ, φ) = (−1)m Ylm
v. http://mathworld.wolfram.com/LegendrePolynomial.html
3.3 Interazioni nei cristalli
95
e
1
35x4 − 30x2 + 3
8
9
9
P44 (x) =
105 1 − x2
4π8!
4π8!
Y40 (θ, φ) = P40 (x) =
Y44 (θ, φ) =
2
con x = cos θ. In conclusione espandendo il potenziale fino 4 ordine in r si ha
V (r) = V0 + r4
9
4π
1
210
A40 35x4 − 30x2 + 3 + √ A44 1 − x2
8
8!
2
cos (4φ)
Limitandosi a considerare il potenziale lungo l’asse z si ha
9
q
A40 =
4π
4πε0 a
1
q
≃
6 + z4
4πε0 a
2
V (x = y = 0, z) = V0 + z 4
da cui
A40
4
1
1
√
+
+
1 + z2 1 + z 1 − z
√
π q
=
3 4πε0 a
Lungo l’asse x
210
3
A40 + √ A44
8
8!
q
1
≃ V0 +
6 + x4
4πε0 a
2
V (r) = V0 + x4
9
4π
=
q
4πε0 a
1
4
1
√
+
+
2
1+x 1−x
1+x
per cui
210
3
1 q
A40 + √ A44 =
8
2 4πε0 a
8!
√
√
10 8! √
q
π q
=
π
≃
3 × 16 × 210
4πε0 a
5 4πε0 a
3
√
2 π
da cui
A44
Esercizio 3.3.5 Calcolare la variazione di energia elettrostatica di un cristallo di NaCl
dovuta allo spostamento delle cariche dai vertici del reticolo cristallino
Soluzione: Per piccoli spostamenti dj una generica coppia di ioni in Rlmn e Rl′ m′ n′ il
relativo potenziale si modifica in potenziale crea un momento di dipolo ℘lmn = qlmn δRlmn .
Limitandosi a considerare la sola interazione con i primi vicini si ha
V (Rlmn − Rl′ m′ n′ + δRlmn − δRl′ m′ n′ )
qlmn ql′ m′ n′
1
=
4πε0
(X + x)2 + (Y + y)2 + (Z + z)2
dove
X = Xlmn − Xl′ m′ n′
x = δXlmn − δXl′ m′ n′
96
Interazioni elettriche e magnetiche
ed analogamente per le altre quantità. D’altra parte
1
(X + x)2 + (Y + y)2 + (Z + z)2
1
X
Y
Z
− 3x − 3y − 3z
R R
R
R
2X 2 − Y 2 − Z 2 2 −X 2 + 2Y 2 − Z 2 2 −X 2 − Y 2 + 2Z 2 2
+
x +
y +
z
2R5
2R5
2R5
3XZ
3XY
3Y Z
+ 5 xz +
xy
+
yz
R
R5
R5
≃
√
dove R = X 2 + Y 2 + Z 2 . La somma dei contributi lineari estesi a tutti glio ioni del
cristallo deve annallarsi perchè la configurazione di equilibrio corrisponde ad un minimo di
W. Limitandosi inoltre a considerare l’interazione con i primi vicini si ha qlmn ql′ m′ n′ = −e2
per cui
δV (1) = −
1 e2
2 4πε0 a3
{lmn}
ε
(δxlmn − δxl+ε,mn )2 + (δylmn − δyl,m+ε,n )2 + (δzlmn − δzlm,n+ε )2
con ε = ±1. Per i secondi vicini del piano xy
2X 2 − Y 2 2 −X 2 + 2Y 2 2 −X 2 − Y 2 2
x +
y +
z
2R5
2R5
2R5
3XY
1
+ 5 xy = 3 x2 + y 2 + z 2 + 3xy
R
2a
ed analogamente per gli altri piani. Pertanto
δV (2) =
1 e2
4 4πε0 a3
{lmn} εε′
(δxlmn − δxl+ε,m+ε′ ,n )2 + (δxlmn − δxl+ε,m,n+ε′ )2
+ (δylmn − δyl+ε′ ,m+ε,n )2 + (δylmn − δyl,m+ε,n+ε′ )2
+ (δzlmn − δzl+ε′ ,m,n+ε )2 + (δzlmn − δzl,m+ε′ ,n+ε )2
3.3.2
Interazioni in dielettrici non omogenei
Esercizio 3.3.6 Calcolare l’energia elettrostatica associata (a) ad una carica puntiforme
q in un mezzo di costante dielettrica relativa ε1 distante d da un semispazio z < 0 di
costante ε2 , (b) a due cariche poste rispettivamente nei semispazi z > 0 e z < 0, (c)
ad una generica distribuzione di cariche in z>0 e (d) ad una distribuzione che occupa
entrambi i semispazi.
Soluzione: (a) Il potenziale V (r) può essere associato per z > 0 alla carica q e ad una
carica q’ speculare di q rispetto al piano z = 0
1
V (r) =
4πε0 ε1
q
q′
+
R1
R2
ε1 q ′′
z
ε2 R1
z>0
<0
3.3 Interazioni nei cristalli
97
Imponendo le condizioni al contorno sul piano z = 0
E⊥ (r⊥ , z = 0+ ) = E⊥ (r⊥ , z = 0− )
Dz (r⊥ , z = 0+ ) = Dz (r⊥ , z = 0− )
si ottiene
q + q′ =
ε1 ′′
q
ε2
Ne segue che
ε2 − ε1
q
ε2 + ε1
2ε2
=
q
ε2 + ε1
q′ = −
q ′′
e
V (r) =
q
4πε0 ε1
1
R1
ε2 −ε1 1
ε2 +ε1 R2
2ε1 1
z<
ε2 +ε1 R1
−
z>0
0
Pertanto su q agisce un campo elettrico
E=−
q ε2 − ε1 1
ẑ
4πε0 ε1 ε2 + ε1 (2d)2
Se si sposta q del tratto δz il campo E compie un lavoro δL pari a
δL =
q 2 ε2 − ε1 1
δz = δV
4πε0 ε1 ε2 + ε1 (2d)2
Ne segue che all’elettrone va associata un’energia potenziale
V (r) = −
q 2 ε2 − ε1 1
4πε0 ε1 ε2 + ε1 2d
(b) Nel caso di due cariche q1 , q2 poste rispettivamente in r1 , r2 l’energia di interazione
è data per z1 , z2 > 0 da
1 ε2 − ε1 q12
q22
V (r) = −
+
4πε0 ε1 ε2 + ε1 2d1 2d2
q1 q2
2ε1
1
2ε2
−
+
4πε0 ε1 ε2 + ε1 |r1 − r2 | ε2 + ε1
|r′1
1
1
+
− r2 | |r1 − r′2 |
mentre per z1 , −z2 > 0
V (r) = −
1 ε2 − ε1 q12
1 ε2 − ε1 q22
q1 q2
2
1
+
−
4πε0 ε1 ε2 + ε1 2d1 4πε0 ε2 ε2 + ε1 2d2 4πε0 ε2 + ε1 |r1 − r2 |
(c) Per una generica distribuzione di cariche contenute nello spazio z > 0 si ha
V=−
1
4πε0 ε1
1 ε2 − ε1
ρ (r) ρ (r′ ) 3 3 ′
d rd r +
′
|r − r |
4πε0 ε1 ε2 + ε1
ρ (r) ρ (r′ ) 3 3 ′
d rd r
|r − r′′ |
98
Interazioni elettriche e magnetiche
Figura 3.3:
con r′′ = σr′ l’immagine speculare (σ) di r′ rispetto al piano z = 0
(d) mentre per una distribuzione estesa sia a z > 0 che a z < 0
ρ1 (r) ρ1 (r′ ) 3 3 ′
1
1
d rd r −
′
4πε0 ε1
|r − r |
4πε0 ε2
r,r′ ∈z>0
′
1 ε2 − ε1
ρ1 (r) ρ1 (r ) 3 3 ′
+
d rd r
4πε0 ε1 ε2 + ε1
|r − σr′ |
r,r′ ∈z>0
1 ε2 − ε1
ρ2 (r) ρ2 (r′ ) 3 3 ′
d rd r
−
4πε0 ε2 ε2 + ε1
|r − σr′ |
r,r′ ∈z<0
1
2
ρ1 (r) ρ2 (r′ ) 3 3 ′
−
d rd r
4πε0 ε2 + ε1
|r − r′ |
r∈z>0,r′ ∈z<0
V = −
ovvero,
1
V = −
4πε0
1
+
4πε0
r,r′
r,r′
r,r′ ∈z<0
ρ2 (r) ρ2 (r′ ) 3 3 ′
d rd r
|r − r′ |
ρ (r) ρ (r′ ) 3 3 ′
2
d rd r
ε (r) + ε (r′ ) |r − r′ |
ε (r) − ε (σr′ ) ρ (r) ρ (r′ ) 3 3 ′
d rd r
ε (r) (ε (r) + ε (r′ )) |r − σr′ |
Esercizio 3.3.7 Una sfera dielettrica di raggio a e costante dielettrica ε1 è posta in un
liquido di costante dielettrica ε2 in cui preesisteva un campo elettrico E. Calcolare il campo
elettrico risultante all’interno ed all’esterno della sfera
Soluzione: Con riferimento alla Fig. (3.3) si vede facilmente che il potenziale V (r)
è invariante per rotazione intorno all’asse passante pe il centro della sfera e parallelo ad
E. Pertanto, utilizzando coordinate sferiche con centro nella sfera ed orientate secondo E
e tendendo conto che V (r) è soluzione dell’equazione di Laplace, conviene espanderlo in
armoniche sfere Yl0 (θ, φ) = Pl (cos θ) ottenendo così
"
#
(1)
ql
(1) l
V1 (r, θ) =
Ql r + l+1 Pl (cos θ)
r
l=0
"
#
(2)
ql
(2) l
V2 (r, θ) =
Ql r + l+1 Pl (cos θ)
r
l=0
3.3 Interazioni nei cristalli
(1)
(2)
(1)
(2)
con Ql , Ql , ql , ql
sulla sfera
99
costanti da determinare imponendo: (a) le condizioni al contorno
V1 (a, θ) = V2 (a, θ)
∂
∂
ε1
V1 (r, θ)
= ε2
V2 (r, θ)
∂r
∂r
r=a
r=a
(b) la condizione che V1 (r, θ) sia finito per r = 0 e (c) che per r → ∞ ∇V2 → − E . Ne
segue che
3ε2
Er cos θ
ε1 + 2ε2
ε2 − ε1 a
V2 (r, θ) = − 1 +
ε1 + 2ε2 r
V1 (r, θ) = −
3
Er cos θ
ovvero
3ε2
E
ε1 + 2ε2
ε2 − ε1 a
= −∇V2 = E+
ε1 + 2ε2 r
E1 = −∇V1 =
E2
3
(1 − 3r̂r̂) · E
Pertanto il campo all’esterno si arricchisce del contributo di un dipolo elettrico
℘ = 4πε0 ε2
ε2 − ε1 3
aE
ε1 + 2ε2
corrispondente ad una polarizzazione
P=
℘
Vsf era
= 3ε0 ε2
ε2 − ε1
ε2 − ε1
E = αp 3
D
ε1 + 2ε2
ε1 + 2ε2
ovvero la sfera dielettrica si polarizza rispondendo ad un campo locale
Eloc = 3
ε2 − ε1
1
E=
E
ε1 + 2ε2
1 − nα3 p
Questo risultato conferma indirettamente la formula di Clausius-Mossotti14 .
Esercizio 3.3.8 Calcolare (a) il potenziale15 creato da una carica puntiforme posta a
distanza r0 da una sfera dielettrica di raggio ′′ a′′ , (b) la forza agente sulla carica
Soluzione: Conviene dividire lo spazio in tre regioni : I r < a, II a < r < r0 , III r0 <
r. Dal momento che il sistema è invariante per rotazione intorno all’asse passante per
l’origine e la carica conviene scegliere questo come asse z. Inoltre in un guscio sferico
privo di cariche e compreso tra r = a e r = b il potenziale è soluzione dell’equazione di
Laplace e può essere espresso nella forma
An rn + Bn
n
14
15
1
rn+1
Pn (cos θ)
v.p.e. C. Altucci et al. Eq. (10.44)
W. K. H. Panofsky and M. Phillips, Classical Electricity and Magnetism, Dover Publications Inc., N.
Y. 2005
100
Interazioni elettriche e magnetiche
dove An e Bn sono dei generici coefficienti.
Pertanto il potenziale nelle 3 regioni è espresso da:
An rn Pn (cos θ)
VI =
n
VII
q
=
4πε0 r0
VIII =
q
4πε0 r
n
r
r0
n
r0
r
n
Pn (cos θ) +
Bn
n
n
Pn (cos θ) +
1
rn+1
1
Bn
rn+1
n
Pn (cos θ)
Pn (cos θ)
Imponendo le condizioni al contorno sulla sfera
VI (a, θ) = VII (a, θ)
∂
∂
ε
=
VI (r, θ)
VII (r, θ)
∂r
∂r
r=a
r=a
si ha
An an Pn (cos θ) =
n
ε
n
q
4πε0 r0
q
nAn an−1 Pn (cos θ) =
4πε0 r0 a
a
r0
n
n
n
Pn (cos θ) +
Bn
n
a
n
r0
1
an+1
n
Pn (cos θ) −
Bn
n
Pn (cos θ)
n+1
Pn (cos θ)
an+2
Ne discende il sistema di equazioni
n
q
a
1
+ Bn n+1
4πε0 r0 r0
a
n
q
a
n+1
=
n
− Bn n+2
4πε0 r0 a
r0
a
An an =
εnAn an−1
da cui
n
(n + 1)2
q
a
(1 + ε) n + 1 4πε0 r0 r0
"
#
qan+1
(n + 1)2 an
a
=
−1
4πε0 r0 (1 + ε) n + 1
r0
An =
Bn
n
(b) Una volta trovato il potenziale si ha che l’energia potenziale è data da
V = −qVII (r0 , 0) = −
3.4
q2
4πε0 r0
1 + Bn
n
1
r0n+1
Pn (1)
Forze di van der Waals
Esercizio 3.4.1 Due atomi di idrogeno sufficientemente distanti interagiscono tra di essi
come due dipoli (forze di Van der Waals ). Considerando come sistema imperturbato i due
atomi di idrogeno non interagenti nello stato fondamentale, calcolare perturbativamente al
secondo ordine la correzione all’energia dovuta all’interazione dipolo-dipolo (si trascurino
gli spin elettronici).
3.4 Forze di van der Waals
101
Soluzione: Sia R la distanza tra i due atomi, r1 la coordinata dell’elettrone legato
all’atomo 1 con origine nel nucleo, r2 l’analogo per l’elettrone dell’atomo 2. I momenti
di dipolo sono ℘1 = −er1 e ℘2 = −e r2 . Si usi il risultato dell’esercizio precedente. La
correzione al primo ordine è nulla mentre al secondo ordine si ricorre alla relazione di
chiusura stabilita nella teoria perturbativa. Si trova
∆E (2) ≃
C
R6
dove C è una costante.
Esercizio 3.4.2 Si ripeta l’esercizio precedente considerando come stato imperturbato
quello costruito con un atomo nello stato fondamentale e l’altro nel primo stato eccitato
(si trascurino gli spin elettronici).
Soluzione: Dalla teoria delle perturbazioni per stati degeneri discende
∆E ≃ ±
C
R3
Esercizio 3.4.3 Si consideri un atomo di idrogeno posto ad una distanza R da una
parete metallica perfettamente conduttrice. Usando il metodo delle immagini dell’elettrostatica, (a) calcolare la correzione all’energia dello stato fondamentale al primo ordine
perturbativo; (b) discutere di cosa accade al primo stato eccitato.
Soluzione: L’interazione tra atomo e parete è quella tra il dipolo atomico ℘ = er e
quello immagine ℘i = eri (essendo ri ≡ (x, y, −z)). Quindi dalla Eq. (??) discende
Vad = −
x2 + y 2 + 2z 2
r2
=
−
(1 + cos2 (θ)) (u.a.)
16R3
16R3
dove x, y e z sono le coordinate dell’elettrone rispetto al protone. Per lo stato fondamentale
al primo ordine si ha
1
∆E = − 3
8R
L’atomo viene attirato dalla superficie dando così luogo al fenomeno dell’adsorbimento.
(b) Per il livello degenere n = 2 conviene quantizzare la proiezione del momento angolare
lungo l’asse z. Notiamo subito che
2p1 |Vad | 2p−1 = 2s |Vad | 2p−1 = 2s |Vad | 2p0 = 2p0 |Vad | 2p±1 = 0
mentre per i termini diagonali si ha
2p1 |Vad | 2p1
=
2p0 |Vad | 2p0
= −
2s |Vad | 2s
2p−1 |Vad | 2p−1 = −
3
R3
3.5
= − 3
R
2.25
R3
In conclusione, il livello 4-volte degenere n = 2 si separa in tre livelli di cui quello più
elevato risulta 2-volte degenere.
102
Interazioni elettriche e magnetiche
Esercizio 3.4.4 Le forze di interazione di van der Waals dipendono dalla fluttuazione
quadratica media del momento di dipolo dell’atomo allo stato fondamentale. Calcolare
℘2 = e2 r2 per l’atomo di idrogeno.
Soluzione: Dalle espressioni degli elementi di matrice nlm|r2 |nlm risulta 100|e2 r2 |100 =
3e2 a20
Esercizio 3.4.5 Due atomi di idrogeno sufficientemente distanti interagiscono tra loro
come due dipoli (forze di Van der Waals ). Considerando come sistema imperturbato i due
atomi di idrogeno non interagenti nello stato fondamentale, calcolare perturbativamente al
secondo ordine la correzione all’energia dovuta all’interazione dipolo-dipolo (si trascurino
gli spin elettronici).
Soluzione: Sia R la distanza tra i due atomi, r1 la coordinata dell’elettrone legato
all’atomo 1 con origine nel nucleo, r2 l’analogo per l’elettrone dell’atomo 2. I momenti
di dipolo sono ℘1 = −er1 e ℘2 = −e r2 . Si usi il risultato dell’esercizio precedente. La
correzione al primo ordine è nulla mentre al secondo ordine si ricorre alla relazione di
chiusura stabilita nella teoria perturbativa. Si trova
∆E (2) ≃
C
R6
dove C è una costante.
Esercizio 3.4.6 Si ripeta l’esercizio precedente considerando come stato imperturbato
quello costruito con un atomo nello stato fondamentale e l’altro nel primo stato eccitato
(si trascurino gli spin elettronici).
Soluzione: Dalla teoria delle perturbazioni per stati degeneri discende
∆E ≃ ±
C
r3
Esercizio 3.4.7 Si consideri un atomo di idrogeno posto ad una distanza R da una
parete metallica perfettamente conduttrice. Usando il metodo delle immagini dell’elettrostatica, (a) calcolare la correzione all’energia dello stato fondamentale al primo ordine
perturbativo; (b) discutere cosa accade al primo stato eccitato.
Soluzione: L’interazione tra atomo e parete è quella tra dipolo atomico ℘ = er e
immagine ℘i = eri (essendo ri ≡ (x, y, −z)). Quindi, dalla Eq. (??) discende
Vad = −
x2 + y 2 + 2z 2
r2
=
−
(1 + cos2 (θ)) (u.a.)
16R3
16R3
dove x, y e z sono le coordinate dell’elettrone rispetto al protone. Per lo stato fondamentale
al primo ordine si ha
1
∆E = − 3
8R
L’atomo viene attirato dalla superficie dando così luogo al fenomeno dell’adsorbimento.
(b) Per il livello degenere n = 2 conviene quantizzare la proiezione del momento angolare
lungo l’asse z. Notiamo subito che
2p1 |Vad | 2p−1 = 2s |Vad | 2p−1 = 2s |Vad | 2p0 = 2p0 |Vad | 2p±1 = 0
3.5 Interazioni magnetiche
103
mentre per i termini diagonali si ha
2p1 |Vad | 2p1
=
2p−1 |Vad | 2p−1 = −
2p0 |Vad | 2p0
= −
2.25
R3
3
R3
3.5
2s |Vad | 2s = − 3
R
In conclusione, il livello 4-volte degenere n = 2 si separa in tre livelli di cui quello più
elevato risulta 2-volte degenere.
3.5
3.5.1
Interazioni magnetiche
Bobine di Helmholtz
Esercizio 3.5.1 (a) Calcolare il potenziale vettore creato da una corrente I a forma di
anello circolare di raggio a; (b) calcolare il campo a piccola distanza ρ dall’asse z per una
coppia di spire coassiali con baricentri in a Z e −Z ed attraversate da correnti I uguali
circolanti in versi opposti; (c) come in b in possimità di z=0.
Soluzione: (a) Utilizzando un sistema di coordinate cilindriche coassiali alla spira
avente origine nel centro della stessa e normalizzando le lunghezze ad R (= 1) , A e B a
µ0 I si ha16
!
µ0 I
ds
Aφ (ρ, z) =
4π
r
π
1
cos φ
= µ0 Ia
dφ
2
2
2π 0
z + a + ρ2 − 2aρ cos φ
=
µ0 I 2
π ξ
µ0 I
π
µ0 I
≃
32
=
1−
ξ
2
π/2
π/2
dθ
2
1 − ξ sin θ
0
−
0
1 − ξ sin2 θdθ
a
ξ
1−
K [ξ] − E [ξ]
ρξ
2
a 3/2
3
75 2
ξ
ξ + ···
1+ ξ+
ρ
4
128
dove
4ρa
+ (a + ρ)2
e K [ξ] , E [ξ] integrali ellittici (v. (1.28) e Fig. 1.4).
(b) Sviluppando al primo ordine rispetto a ρ si ha per la coppia di bobine (v. Fig.
3.5):
ξ=
z2
Aφ (ρ, z + Z) − Aφ (ρ, z − Z)
"
#
a2
1
1
≃ −µ0 I
−
ρ
4 (a2 + z 2 + 2zZ + Z 2 )3/2 (a2 + z 2 − 2zZ + Z 2 )3/2
= µ0 If (z, Z) ρ
16
v.p.e. W. R. Smythe, Static and Dynamic Electricity, McGraw-Hill Co., N. Y. 1950 Sez. 7.10
(3.16)
104
Interazioni elettriche e magnetiche
Figura 3.4: Coppia di bobine di Helmholtz
Pertanto, posto
1 ∂
1
d
(ρAφ ) = Aφ + Aφ = 2µ0 If (z, Z)
ρ ∂ρ
ρ
dρ
∂
I ∂
= − Aφ ≃ −µ0 2 f (z, Z) ρ
∂z
a ∂z
Bz =
Bρ
(c) In prossimità del baricentro f (z, Z) si riduce a
f (z, Z) ≃ −
3
Za2
z
2 (a2 + Z 2 )5/2
Pertanto
1 ∂
1
∂
Za2
Bz =
(ρAφ ) = Aφ + Aφ ≃ −µ0 I3
z
ρ ∂ρ
ρ
∂ρ
(a2 + Z 2 )5/2
Bρ = −
3.5.2
∂
3
Zρa2
Aφ ≃ −µ0 I
∂z
2 (a2 + Z 2 )5/2
(3.17)
Magnetismo nei solidi
In alcuni materiali ciascun atomo possiede un momento di dipolo permanente a seguito
dell’incompleta cancellazione degli spin elettronici e/o del momento magnetico orbitale.
In assenza di un campo esterno questi momenti risultano orientati in modo disordinato
lungo tutte le direzioni. Inoltre questi dipoli non interagiscono tra loro e sono liberi
di ruotare. L’applicazione di un campo esterno produce un parziale orientamento di
questi dipoli dando così luogo al fenomeno del paramagnetismo. Allineandosi in parte col
campo esterno contribuiscono ad aumentare il campo esterno applicato dando così lugo ad
una permeabilità relativa µr maggiore di 1. Le suscettività sono comprese nell’intervallo
10−5 ÷ 10−2
3.5 Interazioni magnetiche
105
0.2
0.1
-6
-4
2
-2
4
6
-0.1
-0.2
Figura 3.5: Andamento di 4a∆Aφ / (µ0 Iρ) lungo l’asse z/a di un quadrupolo costituito da
due spire di raggio a e distanti tra loro 2Z, per Z/a = 1.2, 0.8, 0.6, 0.4, 0.2.
3.5.3
Materiali diamagnetici
Esercizio 3.5.2 Un atomo diamagnetico non ha un momento magnetico proprio, ma un
campo esterno induce un piccolo momento diretto in verso opposto. In questi casi la
suscettività è negativa. La suscettività magnetica del Si è pari a −0.4 · 10−4 . Calcolare la
densità di flusso e la magnetizzazione in un campo H di 106 A/m
Esercizio 3.5.3 Stimare la suscettività diamagnetica del Si assumendo che gli elettroni
atomici descrivono orbite circolari di un Å
3.5.4
Materiali ferromagnetici
I materiali ferromagnetici posseggono un momento magnetico in assenza di campo esterno.
Queste proprietà sono tipiche dei metalli di transizione Fe (nella forma BCC α ferrite), Co
Ni ed alcuni metalli delle terre rare come il gadolinio (Gd). Essi presentano suscettività
magnetiche molto alte 106 I momenti magnetici permanenti nei materiali ferromagnetici
sono quelli degl termine fondamentale. Inoltre, nei ferromagneti l’accoppiamento tra spin
di atomi adiacenti produce il mutuo allineamento degli stessi. L’origine di queste forze di
accoppiamento è legata alla struttura elettronica del metallo. Si definisce magnetizzazione
di saturazione Ms il valore massimo raggiunto quando tutti i momenti atomici si allineano
col campo esterno.
Esercizio 3.5.4 Calcolare la magnetizzazione di saturazione per singolo atomo di Fe,Ni
e Co sulla base dei termini fondamentali previsti dalle regole di Hund. Discutere perchè
questi valori differiscono da quelli sperimentali Fe, Co e Ni si ha 2.22 µB (Fe) , 1.72 µB
(Co) e 0.60 µB (Ni)
Esercizio 3.5.5 Calcolare la magnetizzazione di saturazione del Ni che ha una densità
di 8.90 g/cm3
106
3.5.5
Interazioni elettriche e magnetiche
Equazioni di Bloch
Se indichiamo con mN il momento magnetico di un atomo, di qualunque natura esso sia,
l’energia di interazione con un campo magnetico B sarà:
Vm = −mN · B
L’interazione con il campo farà sì che l’atomo sia sottoposto ad una forza F = −∇Vm di
trascinamento ed ad un momento T = m × B che provocherà un moto di precessione del
momento magnetico intorno alla direzione del campo magnetico B.
Lo spin I di un nucleo sottoposto ad un campo magnetico costante B0 diretto lungo
l’asse z e ad un campo oscillante 2B1 cos(ωt) diretto lungo x, effettua un movimento di
precessione descritto dall’equazione
d
I = γ N I×[B0 ẑ + 2B1 cos(ωt)x̂]
dt
dove
γN =
(3.18)
gI µN
sta per il rapporto giromagnetico del nucleo, µN è il magnetone nucleare e gI il fattore di
Landè.
In un sistema x′ , y ′ , z ′ = z
x = x′ cos(ωt) − y ′ sin(ωt)
y = y ′ cos(ωt) + x′ sin(ωt)
rotante attorno all’asse z con velocità angolare ω la Eq. (3.18) si riduce a
d
I = I×[(ω L − ω) ẑ + γB1 x̂′ + γB1 (cos(2ωt)x̂′ + sin(2ωt)ŷ ′ )]
dt
(3.19)
(ω L = γB0 ) ovvero, lo spin risulta sottoposto ad un campo efficace
Bef f =
1
(ω L − ω) ẑ + B1 x̂′
γN
costante nel tempo e ad un campo ruotante a frequenza 2ω di ampiezza B1 .
Per integrare l’equazione del moto di I si dovrebbe tener conto delle relazioni di commutazione delle tre componenti del momento nucleare. Per semplicità si immaginerà di
trattare I come una grandezza classica. Nel caso in cui ω ≫ |ω L − ω| si può ignorare il
campo rotante, per cui I si muove effettuando un movimento di precessione
I = Ix′ cos(ΩR t + ϕx′ )x̂′ + Iy′ cos(ΩR t + ϕy′ )ŷ ′ + Iz cos(ΩR t + ϕz )ẑ
attorno al vettore di Rabi
ΩR = γ N Bef f
(3.20)
e scambiando col campo oscillante la potenza
P = − γ N B1 ·
d
I = − γ 2N B1 · ( I×B0 ) = −2 γ 2N B0 B1 Iy (t) cos(ωt)
dt
(3.21)
3.5 Interazioni magnetiche
107
D’altra parte
Iy (t) cos(ωt) = Ix′ cos(ΩR t + ϕx′ ) sin(ωt) cos(ωt) + Iy′ cos(ΩR t + ϕy′ ) cos2 (ωt)
Per una frequenza di Rabi molto più piccola di ω si può dividere P in un termine oscillante
a frequenza 2ω ed in un altro lentamente variabile
P (t) = − γ 2N B0 B1 Iy′ cos(ΩR t + ϕy′ )
che descrive uno scambio periodico di energia tra lo spin ed il campo.
Che lo spin possa restituire periodicamente al campo l’energia acquisita è conseguenza
del fatto che abbiamo trascurato i fenomeni dissipativi legati all’interazione con altri spin
e col reticolo cristallino. Questi fenomeni dipendono dalla temperatura. Infatti il moto
di precessione degli spin nucleari sotto l’effetto dei campi applicati perturba lo stato di
equilibrio del sistema macroscopico. Pertanto, l’equazione del moto dello spin (3.18) andrà
modificata aggiungendovi un termine che descriva la deviazione dall’equilibrio,
d
I = γI×B+Lter (I)
dt
Lter (I) avrà l’effetto di ricondurre I al valore di equilibrio Ieq una volta cessata la perturbazione. Per campi non molto intensi Lter (I) può essere approssimato, come mostrato da
Felix Bloch, con termini lineari in I,
Lter (I) = −
Iz − Ieq
I⊥
ẑ −
τ long
τ tr
In altri termini, una volta cessata la perturbazione la componente longitudinale Iz (parallela al campo applicato) e trasversa I⊥ tendono esponenzialmente ai valori di equilibrio
Ieq e 0 con costanti di tempo generalmente diverse tra loro. Sia τ long che τ tr dipendono
dalla temperatura.
Pertanto, il moto dello spin è descritto dalla cosiddetta equazione di Bloch
d
Iz − Ieq
I⊥
I=−
ẑ −
+ γ N I× (B0 ẑ + 2B1 cos(ωt)x̂)
dt
τ
τ⊥
(3.22)
dove Ieq rappresenta il valore di equilibrio di I, I⊥ sta per la componente trasversa del
vettore, mentre τ e τ ⊥ indicano rispettivamente i tempi di rilassamento spin-reticolo
e spin-spin. Questa equazione può essere utilizzata anche per analizzare l’interazione
risonante di un fascio laser con un sistema atomico a due livelli.
3.5.6
Approssimazione di onda rotante
Quando la fequenza ω è prossima alla frequenza di Larmor, si possono ignorare nella Eq.
(3.19) i termini oscillanti a frequenza 2ω, riducendo così l’equazione di Bloch (v. (3.22))
nel sistema rotante a frequenza ω ad una equazione a coefficienti costanti,
d
Iz − Ieq
I⊥
I = I × ΩR −
ẑ −
dt
τ
τ⊥
(3.23)
con ΩR = (ω L − ω) ẑ + γB1 x̂′ . Pertanto nel sistema rotante I effettuerà un movimento
di precessione smorzato attorno al vettore di Rabi ΩR , stabilizzandosi esponenzialmente
108
Interazioni elettriche e magnetiche
dopo un tempo dell’ordine di τ long e τ tr sul vettore I∞ che rende nulla la derivata
(v. Eq. (3.22)), Ovvero (ω L − ω) ẑ + γB1 x̂′ ]
I∞ ×ΩR −
d
I ,
dt ∞
Iz∞ − Ieq
I⊥∞
ẑ −
=0
τ
τ⊥
da cui
γτ ⊥ Ieq
B1
1 + γ 2N τ ⊥ τ B12 + τ 2⊥ (ω L − ω)2
= τ ⊥ (ω L − ω) Iy′
Iy′ ∞ =
Ix′ ∞
(3.24)
Ne segue che nel sistema di laboratorio il vettore I∞ ruoterà a velocità ω. In particolare
Ix∞ (t) può essere espressa nella forma
Ix∞ (t) = Ieq τ ⊥ γ N B1 (χ′ cos(ωt) − χ′′ sin(ωt))
′′
Utilizzando l’Eq. (3.24) si può facilmente dimostrare che χ′ e χ sono rispettivamente la
parte reale ed immaginaria della suscettività complessa
′′
χ(ω) = χ′ (ω) − iχ (ω) = −i
1
1 + γ 2N B21 τ ⊥ τ + i(ω − ω L )τ ⊥
(3.25)
Da queste considerazioni discende che la componente Ix (t) del momento nucleare
lungo l’asse della bobina di eccitazione di un apparato NMR è proporzionale al campo
magnetico oscillante B1 attraverso la suscettività generalizzata χ.
L’oscillazione a frequenza ω di Ix (t) indurrà a sua volta un potenziale
VNMR (t) = ℜ [VNMR (ω) exp (iωt)]
ai capi della bobina di eccitazione del campione pari a
′′
VNMR (ω) = iLNMR ω χ′ (ω) − iχ (ω) I (ω)
con LNMR un coefficiente di proporzionalità dipendente dal numero di momenti nucleari
coinvolti nella risonanza e con
I(t) = ℜ [I (ω) exp(iωt)]
l’ampiezza della corrente che circola nella bobina. Pertanto, ai morsetti A e B del circuito
di eccitazione (vedi Fig. (??)) si vedrà un’impedenza pari a
′′
Z (ω) = iω Lbobina + LNMR χ′ (ω) − iχ (ω)
′′
Lorentziana
dove Lbobina indica l’induttanza in assenza di risonanza nucleare. Notiamo che χ risulta
massima alla risonanza, che presenta una larghezza pari a Γ = 1/τ ⊥ (v. Fig. (3.6).
Questo tipo di risonanza, tipico di molti fenomeni di interazione tra radiazione e materia,
è caratterizzata da una suscettività complessa della forma
′′
χ′ (ω) − iχ (ω) ∝ −i
1
1 + i(ω − ω r )τ
3.5 Interazioni magnetiche
109
Figura 3.6: Parte reale ed immaginaria della suscettività di un campione in prossimità di
una risonanza nucleare. In basso è riportato il circuito equivalente visto ai morsetti A e
B della bobina di eccitazione indicata nel precedente schema dell’apparato.
Nota 3.5.6 La relativa riga di assorbimento è chiamata Lorentziana.
Per variare la larghezza Γ della risonanza si deve modificare il tempo di rilassamento
spin-spin. A tal fine si usa sciogliere nell’acqua dei sali di ferro. I momenti magnetici
dispersi nell’acqua si termalizzano rapidamente e, interagendo coi protoni, finiscono per
termalizzare rapidamente quest’ultimi.
Utilizzando questa espressione della suscettività si ottiene facilmente la potenza assorbita
1
P (ω) = γ N τ ⊥ ωB21 Ieq χ′′ (ω)
2
Si vede dunque che per effetto dei rilassamenti la potenza media scambiata non oscilla
′′
più alla frequenza di Rabi. Essa presenterà una dipendenza da ω simile a quella di χ (ω).
Esercizio 3.5.7 Discutere il moto di una grandezza vettoriale L descritta dall’Hamiltoniana
H=L·N
in cui N è un vettore costante
Esercizio 3.5.8 Partendo dall’esercizio precedente analizzare il caso in cui l’hamiltoniana della grandezza vettoriale L sia della forma
H = L· (N + M cos (Ωt))
dove M è un vettore perpendicolare a N e di ampiezza molto piccola rispetto a quest’ultimo. Considerare il caso in cui Ω sia prossimo alla frequenza di precessione di L attorno
ad N .
Esercizio 3.5.9 Si consideri un elettrone che descrive un’orbita circolare di raggio “a”
con velocità ω attorno all’origine di un sistema K di assi cartesiani X, Y, Z. Il momento
angolare L (costante) della particella forma un angolo α con ẑ. Calcolare il momento
agolare nel sistema di laboratorio KL (x,y,z) che ruota con velocità angolare ω L (≪ ω)
attorno a Ẑ.
110
Interazioni elettriche e magnetiche
Soluzione: In un sistema cartesiano K ′ (ξ, η, ς) ottenuto ruotando K attorno all’origine
facendo coincidere l’asse ς con L̂ l’orbita dell’elettrone è descritta dalle equazioni
ξ = a cos (ωt)
η = a sin (ωt)
ζ = 0
Pertanto passando da K ′ ad K
X = ξ = a cos (ωt)
Y = cos α η + sin α ς = a cos α sin (ωt)
Z = − sin α η + cos α ζ = −a sin α sin (ωt)
e da K a KL
x = cos (ω L t) X + sin (ω L t) Y = a cos (ω L t) cos (ωt) + a cos α sin (ω L t) sin (ωt)
1
1
=
(1 − cos α) a cos [(ω + ω L ) t] + (1 − cos α) a cos [(ω − ω L ) t]
2
2
1
1
= a sin2 α cos [(ω + ω L ) t] + a cos2 α cos [(ω − ω L ) t]
2
2
y = − sin (ω L t) X + cos (ω L t) Y = −a sin (ω L t) cos (ωt) + a cos α cos (ω L t) sin (ωt)
1
1
= −a sin2 α sin [(ω + ω L ) t] + a cos2 α sin [(ω − ω L ) t]
2
2
z = Z = −a sin α sin (ωt)
Pertanto,
Lx
y ż − z ẏ
1
1
=
= − − sin2 α sin [(ω + ω L ) t] + cos2 α sin [(ω − ω L ) t] ω sin α cos (ωt)
2
2
ma
a
2
2
1
1
+ sin α sin (ωt) − sin2 α (ω + ω L ) cos [(ω + ω L ) t] + cos2 α (ω − ω L ) cos [(ω − ω L ) t]
2
2
1
1
= ω sin2 α sin α cos (ωt) sin [(ω + ω L ) t] − ω cos2 α sin α cos (ωt) sin [(ω − ω L ) t]
2
2
1
1
− (ω + ω L ) sin2 α sin α sin (ωt) cos [(ω + ω L ) t] + (ω − ω L ) cos2 α sin α sin (ωt) cos [(ω − ω L ) t
2
2
= Lx,ωL sin (ω L t) + Lx,2ω+ωL sin ((2ω + ω L ) t) + Lx,2ω−ωL sin ((2ω − ω L ) t)
dove
Lx,ωL = (ω − ω L cos α) sin α
1
1
Lx,2ω+ωL =
ω − ω L sin2 α sin α
2
2
1
1
ω L sin2 α sin α
Lx,2ω−ωL =
2
2
3.5 Interazioni magnetiche
111
Espressioni analoghe valgono per Ly mentre
=
=
=
=
=
Lz
ma2
xẏ − y ẋ
a2
1
1
1
1
sin2 α cos [(ω + ω L ) t] + cos2 α cos [(ω − ω L ) t]
− sin2 α (ω + ω L ) cos [(ω + ω L ) t] + cos2 α
2
2
2
2
1
1
1
1
+ − sin2 α sin [(ω + ω L ) t] + cos2 α sin [(ω − ω L ) t]
sin2 α (ω + ω L ) sin [(ω + ω L ) t] + cos2
2
2
2
2
1
1
1
− sin4 α (ω + ω L ) cos2 [(ω + ω L ) t] + cos4 α (ω − ω L ) cos2 [(ω − ω L ) t] − sin2 αω L cos [(ω + ω L )
2
2
2
1
1
1
− sin4 α (ω + ω L ) sin2 [(ω + ω L ) t] + cos4 α (ω − ω L ) sin2 [(ω − ω L ) t] + sin2 αω L sin [(ω + ω L )
2
2
2
1
1
1
− sin4 α (ω + ω L ) + cos4 α (ω − ω L ) − sin2 αω L cos (2ωt) cos (2ω L t)
2
2
2
Lz,0 + Lz,2ω+2ωL cos (2 (ω + ω L ) t) + Lz,2ω−2ωL cos (2 (ω − ω L ) t)
In conclusione il momento angolare dell’elettrone in KL può essere decomposto in componenti oscillanti a varie frequenze
LL = LLz,0 +LL⊥,ωL +LL⊥,2ω+ωL + LL⊥,2ω−ωL + LLz,2ω+2ωL + LLz,2ω−2ωL
In particolare, ignorando le componenti oscillanti ad alte frequenze si vede che LL ≃ Lz,0 +L⊥,ωL
effettua una precessione intorno all’asse z a frequenza ω L . Inoltre la componente lungo z
presenta due componenti
LLz,0 = ma2 cos α ω −
1 + cos2 α
ωL
2
di cui la prima è legata al moto in K mentre la seconda rappresenta il contributo della rotazione di KL rispetto ad K. In conclusione il passaggio ad un riferimento rotante implica
2α
la precessione di L e l’aggiunta di una compnente costante −ma2 1+cos
ω L dipendente sia
2
dalla velocità di rotazione ω L che dall’inclinazione di L rispetto a ω L , ovvero l’elettrone
acquista un momento angolare pari a
∆LLz = −I⊥ ω L ẑ
dove
$
%
1 + cos2 α
= m x2 + y 2
2
rappresenta il momento di inerzia medio dell’elettrone rispetto all’asse z.
I⊥ = ma2
Esercizio 3.5.10 Si consideri una insieme di elettroni che muovendosi in un campo centrale descrivono orbite circolari di raggio a, con velocità angolari ω e comunque orientate
attorno ai rispettivi nuclei. Applicando ora un campo magnetico costante ed uniforme
B diretto lungo z, si induce una precessione delle orbite. Si calcoli il momento angolare
medio. Si utilizzino le conclusioni dell’esercizio precedente ed il teorema di Larmor.
112
Interazioni elettriche e magnetiche
Soluzione: Dal teorema di Larmor discende che il moto degli elettroni in presenza di
B coincide con quello senza campo in un sistema O che ruoti con velocità ω L rispetto
al laboratorio OL . In O abbiamo quindi un insieme di orbite di raggio a, descritte con
velocità angolare ω con momento angolare costante e comunque diretto diretto rispetto a
B. Pertanto il momento angolare medio risulta nullo, L = 0. Passando da O a OL LL
differisce da L del termine −ma2 12 (1 + cos2 α) ω L B̂ e si ha
∆LL = −ma2
3.5.7
$
%
1
2
1 + cos2 α ω L B̂ = −ma2 ω L B̂
2
3
Accoppiamento spin nell’NMR
Esercizio 3.5.11 Si calcoli la suscettività magnetica χ di un insieme di elettroni che si
muovono attorno ai rispettivi nuclei.
Per calcolare χ si deve calcolare la magnetizzazione M indotta da un campo magnetico
B. Essa è proporzionale alla variazione ∆LL indotta da B (v. esercizio precedente)
e
attraverso il fattore giromagnetico 2m
M=
N
N e
N e
2
m=−
Z ∆LL = −
Zma2 ω L B̂ = −χB
V
V 2m
V 2m
3
dove Z sta per il numero di elettroni per atomo. Pertanto la suscettività diamagnetica
sarà data da
N e2 $ 2 % 2
χ=−
Z a
V 6m
3
Esercizio 3.5.12 Calcolare l’effetto sulla risonanza di un nucleo di spin 2 posto in r1 , la
vicinanza di un altro nucleo di spin 2 in r2 , e distante r12 = |r2 − r1 |. Assumere il vettore
r12 = r2 − r1 perpendicolare al campo magnetico B0 (asse z) e supporre che i due nuclei
presentino spostamenti chimici diversi. Dall’entità dello sdoppiamento della risonanza
dedurre la distanza r12 .
Soluzione: Il modo più semplice per studiare questo sistema è quello di partire dalla
hamiltoniana:
H = γ (1 − σ 1 )B0 · I1 + γ N (1 − σ 2 )B0 · I2 +
µ0 1 2
γN
3
4π r12
2
[I1 · I2 − 3 (I1 · r̂12 ) (I2 · r̂12 )]
≡ H0 + ξ (r12 ) V
essendo µ0 la permeabilità del vuoto,
ξ (r12 ) = −
µ0 1 2
γN
3
4π r12
2
e
V = I1 · I2 − 3 (I1 · r̂12 ) (I2 · r̂12 )
1
3
= I1z I2z + (I1+ I2− + I1− I2+ ) + (I1+ I2+ + I1− I2− )
4
4
3.5 Interazioni magnetiche
113
avendo introdotto le componenti I± dello spin nucleare17 . Ponendo ω 1 = 12 γ N (1 − σ 1 )B0
e ω 2 = 12 γ N (1 − σ 2 )B0 , e, nell’ipotesi
che ω 1 > %ω 2 si introducono
%
% gli autostati
% di H0 di
1
1
1 1
1
1
1 1
energia crescente |1 = − 2 , − 2 , |2 = − 2 , 2 , |3 = 2 , − 2 , |4 = 2 , 2 . Pertanto,
(H0 + V) / avrà la forma


−ω 1 − ω 2 + A
0
0
B


0
−ω 1 + ω 2 − A
C
0




0
C
ω1 − ω2 − A
0
B
0
0
ω1 + ω2 + A
Per V sufficientemente debole gli autovalori si possono calcolare perturbativamente18 .
17
18
v.p.e. C. Altucci et al. loc. cit. pag. 99 Appendice F, Eq. (F.3)
per r̂12 orientato arbitrariamente rispetto a B si può consultare [?], vol. II , “Interazione dipolo-dipolo
tra due spin 1/2”, p. 1110
Capitolo 4
Onde elettromagnetiche
4.1
Teoremi di reciprocità
Esercizio 4.1.1 Si considerino due distribuzioni di corrente J1 (r, t) e J2 (r, t) che generino due campi E1 (r, t) e E2 (r, t) tali che E1 (r, t1 ) = E2 (r, t2 ) = 0. Analizzare la relazione
t
t
che corre tra t12 dt V J1 (r, t)·E2 (r, t) d3 r e t12 dt V J2 (r, t)·E1 (r, t) d3 r con V un volume
che contiene tutte le sorgenti.
Soluzione: Utilizzando l’espressione della divergenza
∂
∂
∂
B1 · B2 − B2 · B1 − D1 · E2
∂t
∂t
∂t
∂
− D2 · E1 − J2 · E1 + J1 · E2 ,
∂t
∇ · (E1 × H2 + E2 × H1 ) = −
ed integrando su un intervallo di tempo (t1 , t2 ), si ottiene il seguente teorema di reciprocità
t2
dt
t1
t2
3
V
J1 · E2 d r = −
dt
t1
Esercizio 4.1.2 Stabilire la relazione che intercorre
sinusoidali.
V
J2 · E1 d3 r .
J̃1 · Ẽ2 d3 r e
(4.1)
J̃2 · Ẽ1 d3 r per campi
Soluzione: Dalle equazioni di Maxwell discende che nel dominio della frequenza
∇ · Ẽ1 × H̃2 − Ẽ2 × H̃1 = J̃1 · Ẽ2 − J̃2 · Ẽ1 − iω Ẽ1 · ε̃·Ẽ2 − Ẽ2 · ε̃·Ẽ1 .
Se ε̃ è un tensore simmetrico, l’ultimo termine a destra svanisce ed integrando ambo
i membri si ottiene l’equivalente del teorema di reciprocità nel dominio della frequenza,
controparte di quello nel dominio del tempo espresso dalla (4.1):
J̃1 · Ẽ2 − J̃2 · Ẽ1 d3 r = 0 .
(4.2)
N.B. Nei mezzi giroscopici il tensore dielettrico (5.10) non è simmetrico, per cui
quest’ultima equazione non è valida. Si vede quindi che esiste un legame stretto tra
le proprietà analitiche delle relazioni costitutive di un mezzo ed il comportamento dei
campi che vi si propagano.
115
116
4.2
Onde elettromagnetiche
Teorema dell’energia
Esercizio 4.2.1 Estendere il teorema di Poynting
∇ · S̃ =
1
1
∗
H̃∗ · ∇ × Ẽ − Ẽ · ∇ × H̃∗ = − J̃∗ · Ẽ − 2iω (w̃e − w̃m
),
2
2
dove
1
w̃m = µ0 µ̃ H̃
4
2
1
, w̃e = ε0 ε̃ Ẽ
4
2
(4.3)
ad un pacchetto d’onda.
Soluzione: Combinando l’identità vettoriale
∇·
∂
∂
Ẽ × H̃∗ + Ẽ∗ ×
H̃
∂ω
∂ω
= H̃∗ · ∇ ×
∂
∂
Ẽ −
Ẽ · ∇ × H̃∗
∂ω
∂ω
∂
∂
+
H̃ · ∇ × Ẽ∗ − Ẽ∗ · ∇ ×
H̃ ,
∂ω
∂ω
con le equazioni di Maxwell per mezzi dispersivi in frequenza (ε → ε̃ (r,ω) , µ → µ̃ (r,ω)):
∂
∂
∂
Ẽ =
∇ × Ẽ = iµ0 ωµ̃H̃
∂ω
∂ω
∂ω
∂
∂
= iµ0 µ̃H̃ + iωµ0
µ̃ H̃ + iωµ0 µ̃ H̃ ,
∂ω
∂ω
∂
∂
∂
∇×
H̃ =
∇ × H̃ =
J̃ − iωε0 ε̃Ẽ
∂ω
∂ω
∂ω
∂
∂
∂
=
J̃ − iε0 ε̃Ẽ − iωε0
ε̃ Ẽ − iωε0 ε̃ Ẽ ,
∂ω
∂ω
∂ω
∇×
si ottiene il teorema dell’energia per mezzi dispersivi 1 :
∇·
∂
∂
Ẽ × H̃∗ + Ẽ∗ ×
H̃
∂ω
∂ω
(disp)
−
= i4 w̃e + w̃e(disp) + w̃m + w̃m
(disp)
dove w̃e e w̃m sono le densità definite in (4.3) mentre w̃e
1
∂ε̃
w̃e(disp) = ε0 ω
Ẽ
4
∂ω
2
∂
Ẽ · J̃∗ −
∂ω
(disp)
e w̃m
1
∂ µ̃
(disp)
, w̃m
= µ0 ω
H̃
4
∂ω
2
rappresentano contributi dipendenti dalla dispersione di ε̃ e µ̃.
Esercizio 4.2.2 Ricavare la densità di energia e.m. in un plasma
1
C.H. Papas, “Theory of
electromagnetic wave propagation”, McGraw-Hill (1965), Sez. 6.3.
,
:
∂
J̃ · Ẽ∗ ,
∂ω
4.3 Onde piane
117
Soluzione: In un plasma si ha
ω 2p (r)
ε̃ (r,ω) = 1 −
,
ω2
per cui applicando il teorema dell’energia si ottiene
1 ω 2p
w̃e(disp) = ε0 2 Ẽ
4 ω
2
.
(disp)
rappresenta l’energia cinetica degli elettroni del plasma
Si vede in tal caso che w̃e
oscillanti sotto l’azione del campo elettrico.
4.3
Onde piane
Esercizio 4.3.1 In molte situazioni i campi e le correnti sono rappresentati da onde
piane
Aα (xν ) = Ãα exp (ikµ xµ )
J α (xν ) = J˜α exp (ikµ xµ )
Fαβ (xν ) = F̃αβ exp (ikµ xµ )
Gαβ (xν ) = G̃αβ exp (ikµ xµ )
dove kµ = (−c−1 ω, k) sta per il 4-vettore d’onda. In particolare k si può esprimere nella
forma
ω
k = ñ ω, k̂
c
con ñ ω, k̂ l’indice di rifrazione e k̂ direzione di propagazione dell’onda. Pertanto si ha:
kµ k µ = ñ2 ω, k̂ − 1
ω2
.
c2
Analizzare la dipendenza di k µ dal sistema di riferimento inerziale dell’osservatore
Soluzione2 : Applicando a k µ = (c−1 ω, k) la trasformazione di Lorentz 2.1.1 k µ si
modifica in (c−1 ω ′ , k′ ) , ovvero:
ω′
v + (γ − 1) v̂v̂ · k′
2
c
ω = γ (ω ′ + v · k′ ) ≡ ω + δω .
k = k′ + γ
(4.4)
In particolare
k ′2 + 2γ 2
k=
ω′ ′
ω ′2 2
2 − 1) (k′ · v̂)2 + γ 2
k
·
v+
(γ
β
c2
c2
Ne segue che
ck
n=
=
ω
2
v. C. H. Papas loc. cit.
n′2 + 2γ 2 n′ β cos θ′ + (γ 2 − 1) n′2 cos2 θ ′ + γ 2 β 2
Sez. 7.5
γ (1 + n′ β cos θ ′ )
(4.5)
118
Onde elettromagnetiche
D’altra parte cos θ′ è legato a cos θ dalle relazioni:
γ (n cos θ − β)
cos θ′ =
n2 sin2 θ + γ 2 (n cos θ − β)2
n cos θ =
,
n′ cos θ′ + β
,
1 + n′ cos θ′ β
che combinate tra loro danno
1 − n′2 − 1 γ 2 β 2 cos2 θ n2 + 2 n′2 − 1 γ 2 β cos θ n − γ 2 n′2 − β 2 = 0
Risolvendo si ottiene
n=
4.3.1
1 + γ 2 (n′2 − 1) 1 − β 2 cos2 θ − γ 2 β (n′2 − 1) cos θ
1 − (n′2 − 1) γ 2 β 2 cos2 θ
Funzione di Green
I campi creati da cariche e correnti ooscillanti ad assegnate frequenza soddisfano l’equazione di Helmholtz
∇2 +
ω2 2
ñ ũ (r, ω) = −s̃ (r, ω) ,
c2
(4.6)
con s̃ (r, ω) funzione sorgente che può rappresentare la densità di carica ρ̃ (r, ω) o di
corrente J̃ (r, ω) . ũ (r, ω) è legato a s̃ (r, ω) dalla relazione integrale
ũ (r, ω) =
G r − r′ ,
ω
ñ d3 r′
c
dove G r − r′ , ωc ñ è nota come funzione di Green scalare.
Esercizio 4.3.2 Calcolare la funzione di Green scalare G (r, k) relativa all’equazione di
Helmholtz (4.6)
Soluzione: La funzione di Green scalare G (r, k)è soluzione dell’equazione:
∇2 + k 2 G (r, k) = −δ (3) (r) ,
(4.7)
equivalente alla coppia di condizioni
∇2 + k 2 G (r, k) = 0
per r = 0 e
∇2 + k 2 G (r, k) d3 r = −1 .
Essendo la (4.7) invariante per rotazione intorno all’origine, si può scegliere G (r, k)
funzione solo di r = |r| che soddisfa la coppia di condizioni
1 d 2d
r
+ k 2 G (r, k) = 0
r2 dr dr
(4.8)
4.3 Onde piane
e
119
r
d
r ′ G (r′ , k ′ )
dr
2
+k
2
r′2 G (r′ , k) dr′ = −
0
r′ =r
1
4π
(4.9)
per un generico r = 0. Si vede facilmente che la funzione
G (r, k) =
eikr
4πr
(4.10)
soddisfa sia (4.8) che (4.9). Infatti
∇2
eikr
1 1
1
= eikr ∇2 + ∇2 eikr + 2∇eikr · ∇
r
r r
r
1 1 d 2 d ikr 2ik
=
r
e − 2
r r2 dr dr
r
1 1 d 2 ikr 2ik
eikr
= ik 2 r e − 2 = −k 2
r r dr
r
r
D’altra parte
r2
d eikr
dr′ r′
′
r
′
r′ eikr dr′
+ k2
0
r′ =r
r
∂
′
= ikre − e − ik
eikr dr′
∂k 0
∂ eikr − 1
= (ikr − 1) eikr − k 2
∂k
k
= (ikr − 1) eikr + eikr − ikreikr − 1 = −1
ikr
ikr
2
In alternativa si può procedere passando attraverso la trasformata di Fourier dell ’Eq.
(4.7):
−K 2 + k 2 G (K, k) = −1 ,
da cui discende
G (K, k) =
Pertanto
G (r, k) =
=
=
=
=
K2
1
− k2
eiK·r 3
1
dK
K 2 − k2
(2π)3
∞
1
K2
eiKr cos θ
dφ sin θdθ
(2π)3 0 K 2 − k 2
∞
1
K2
1
eiKru du
(2π)2 0 K 2 − k 2 −1
∞
K eiKr − e−iKr
i
−
dK
K 2 − k2
(2π)2 r 0
∞
i
K
−
eiKr dK
2
2
(2π) r −∞ K − k 2
Si assuma a questo punto che
k = lim
ε→0+
ω
+ iε
c
120
Onde elettromagnetiche
si avrà quindi
∞
−∞
K
eiKr dK = lim
2
2
ε→0+
K −k
Γ
ω
c
K−
KeiKr
− iε K +
ω
c
+ iε
dK
avendo indicato con Γ il cammino chiuso che va da −∞ a +∞ e che si chiuda con la
semicirconferenza con Im K ≥ 0 di raggio infinito con centro in K = 0 . Si avrà così
∞
K
eiKr dK =
2
2
−∞ K − k
lim
ε→0+
Γ
K−
KeiKr
− iε K +
ω
c
= 2πi lim residuo
ε→0+
K= ωc +iε
= iπeiKr
Pertanto
G (r, k) =
dK
ω
c
K−
ω
c
KeiKr
− iε K +
ω
c
eikr
4πr
Esercizio 4.3.3 Verificare che G (k, |r − r′ |) per k |r| → ∞ tende a
G (|r − r′ | , k) ≃
eikr −ikn̂·r′
.
e
4πr
(4.11)
dove n̂ = r̂
Esercizio 4.3.4 Calcolare la funzione di Green G(2) (ρ, k⊥ ) relativa ad una corrente sinusoidale e rettilinea immersa in un mezzo di indice di rifrazione ñ (ω)
k0
J̃ (r, ω) = −eδ (2) (ρ) ei β z
k 2 − β 2 è soluzione dell’equazione:
Soluzione: La funzione di Green scalare G(2) ρ,
2
∇2⊥ + k⊥
G(2) (ρ, k⊥ ) = −δ (2) (ρ) ,
dove
2
k⊥
= k02 ñ2 (ω) −
2
Per k⊥
> 0 si ha
1
β2
i
G(2) (ρ, k⊥ ) = H0(1) (k⊥ ρ)
4
(1)
2
dove H0 è la funzione di Hankel di prima specie di ordine 0, mentre per k⊥
<0
i
G(2) (ρ, k⊥ ) = K0 (|k⊥ | ρ)
4
dove K0 sta per la funzione di Bessel modificata di ordine 0
4.4 Focalizzazione onde piane
121
Esercizio 4.3.5 Legare il campo elettrico Ẽ (r, ω) alla densità di corrente J̃ (r, ω) attraverso l’integrale di convoluzione
Ẽ (r, ω) = iωµ
Γ (r − r′ , k) · J̃ (r′ ) d3 r′
(a) Esprimere la funzione di Green tensoriale Γ (r − r′ , k) mediante quella scalare G (r, k) .
(b) Ottenere delle espressioni di Γ (r − r′ , k) per k |r − r′ | ≪ 1 e ≫ 1
Soluzione: (a) Dal momento che Ẽ (r, ω) è legato al potenziale vettore dalla relazione
Ẽ (r, ω) = iω Ã (r, ω) − ∇Ṽ (r, ω)
e che nella gauge di Lorentz
∇ · Ã (r, ω) = iωεµṼ (r, ω)
si ha
Ẽ (r, ω) = −iω 1 + k −2 ∇∇ · Ã (r, ω)
Esprimendo à (r, ω) in funzione di J̃ (r′ ) si ha
Ẽ (r, ω) = −iωµ 1 + k −2 ∇∇ ·
G (r − r′ , k) J̃ (r′ ) d3 r′
da cui
Γ (r − r′ , k) =
=
1 + k −2 ∇∇ G (R, k)
3
1
−1 +
2 − 3i
kR
(kR)
n̂n̂ +
−
1
1
+i
+1 1 G (R, k) ,
kR
kR
(4.12)
con R = |r − r′ | e n̂ = (r − r′ ) / |r − r′ | .
(b) Γ si riduce a piccola distanza a
Γ (r − r′ , k) ≃
1
′
2 (3n̂n̂ − 1) G (R, k) k |r − r | ≪ 1 ,
(kR)
e in vista dell’espansione (4.11) di G (k, |r − r′ |) a grande distanza a
Γ (r − r′ , k) ≃
4.4
eikr −ikn̂·r′
e
(1 − n̂n̂) k |r − r′ | ≫ 1 .
4πr
Focalizzazione onde piane
Per ottenere campi intensi si utilizzano onde piane oppotunamente focalizzate. Si possono
così indurre nei materiali risposte non lineari, che a loro volta danno orgine a campi
oscillanti a frequenze multiple di quella del campo incidente. Molti dispositivi atti a
generare 2a e 3a armoniche di un’onda utilizzano delle cavità ottiche del tipo Fabry-Perot
illuminate attraverso uno degli specchi. Al centro della cavità la sezione del modo di
oscillazione può risultare confrontabile con la lunghezza d’onda.
122
Onde elettromagnetiche
Per esaminare in dettaglio oggetti molto piccoli si utilizzaano spesso microscopi ottici
costituiti essenzialmente di un oculare ed un obiettivo. Quest’ultimo è esssenzialmente
un dispositivo che converte un’onda piana in una con fronte d’onda sferico.
I casi succitati evidenziano l’importanza di calcolare il campo in prossimità di un fuoco.
Per fissare le idee si consideri la distribuzione del campo in una regione prossima ad un
piano in cui l’illuminazione sia ristretta ad una zona piccola rispetto a λ, generato da
un’onda sferica ideale definita su una calotta sferica con centro in z = y = x = 0 e raggio
R ≫ λ. In prossimità del fuoco Ẽ (r) può essere espresso, con riferimento a coordinate
sferiche riferite all’asse z e centro nel fuoco, mediante l’integrale di Luneburg-Debye 3 :
2π θmax
Ẽ (θ, φ) exp iv
Ẽ (u, v̄, ψ) =
0
0
sin θ cos (φ − ψ)
cos θ
sin θdθdφ,
− iū
NA
NA2
(4.13)
con Ẽ (θ, φ) tangente al fronte d’onda sferico, mentre u e v̄ stanno per le coordinate ottiche:
v = k0
x2 + y 2 N A , ū = k0 zNA2 ,
e N A = sin θmax rappresenta l’apertura numerica nel vuoto.
Esercizio 4.4.1 Con riferimento all’integrale di Luneburg-Debye (4.13) si assuma che
Ẽ (θ, φ) sia ottenuto focalizzando con un sistema ottico privo di aberrazioni un’onda polarizzata lungo x̂. Si calcoli Ẽ (θ, φ) e si ottenga un’espressione semplificata di Ẽ (u, v̄, ψ)
Soluzione: Assumendo che l’onda quasi-piana incidente sia polarizzata linearmente
lungo x̂ e tenendo conto che Ẽ (θ, φ) risulta tangente ad una sfera con centro in (x = y =
z = 0) semplici considerazioni geometriche portano a scrivere con buona approssimazione:
(1 − n̂n̂) · x̂
sin φφ̂ + cos θ cos φθ̂
f (θ, φ) =
f (θ, φ) .
|(1 − n̂n̂) · x̂|
1 − sin2 θ cos2 φ
con n̂ il versore che congiunge il punto di coordinate θ, φ della calotta col fuoco geometrico
(x = y = z = 0).
In particolare si ha
Ẽ (θ, φ) =
2π θmax
sin θ cos (φ − ψ)
cos θ sin φφ̂ + cos θ cos φθ̂
− iū
sin θf (θ, φ) dθdφ .
NA
NA2
1 − sin2 θ cos2 φ
0
0
(4.14)
Moltiplicando per la diade unità 1 = ẑ ẑ + v̂v̂ + ψ̂ ψ̂ si ha
Ẽ (u, v̄, ψ) =
exp iv
φ̂ = φ̂ · v̂v̂ + φ̂ · ψ̂ ψ̂ = sin (φ − ψ) v̂ + cos (φ − ψ) ψ̂
θ̂ = θ̂ · ẑ ẑ + θ̂ · v̂v̂ + θ̂ · ψ̂ ψ̂ = sin θẑ + cos θ cos (φ − ψ) v̂ + cos θ sin (φ − ψ) ψ̂
Pertanto
sin φφ̂ + cos θ cos φθ̂
= sin φ sin (φ − ψ) v̂ + cos (φ − ψ) ψ̂
+ cos θ cos φ sin θẑ + cos θ cos (φ − ψ) v̂ + cos θ sin (φ − ψ) ψ̂
= cos θ cos φ sin θẑ + sin φ sin (φ − ψ) + cos2 θ cos φ cos (φ − ψ) v̂
+ sin φ cos (φ − ψ) + cos2 θ cos φ sin (φ − ψ) ψ̂
3
v.p.e. A. B. Shafer, J. Opt. Soc. Am. 57, 630 (1967)
4.5 Tecnica SNOM
123
Sostituendo questa espressione nella (4.14) si ottengono le componentiẼz,v,ψ del campo
Ẽ (u, v̄, ψ) = Ẽz (u, v̄, ψ) ẑ + Ẽv (u, v̄, ψ) v̂ + Ẽψ (u, v̄, ψ) ψ̂. In particolare si nota che per
NA non trascurabile (ovvero θmax non rascurabile) Ẽ (r) presenta in prossimità del fuoco
una componente Ẽz (r) non trascurabile.
4.5
Tecnica SNOM
Per esaminare oggetti molto piccoli si fa tradizionalmente uso di microscopi ottici4 . Questi
strumenti riproducono oggetti bidimensionali, posti su un piano oggetto, su un piano immagine, contenendo al massimo gli effetti delle aberrazioni e della diffrazione. Gli effetti
dovuti al valore finito di λ, descritti dalla teoria della diffrazione, portano ad assegnare
a questi sistemi un limite dmin alla distanza minima risolvibile tra punti adiacenti fornito
dal criterio di Rayleigh dmin = 0.6 λ/N A, con NA apertura numerica, che nei moderni
microscopi con obiettivo immerso in olio non supera 1.45. Con simili apparecchi la massima risoluzione non scende per radiazione visibile al di sotto di 200 nm. Viene quindi
preclusa ogni possibilità di esaminare nanostrutture.
Per esaminare oggetti nanometrici non resta che lavorare sul campo in prossimità dell’oggetto5 . Da qui è nata la tecnica di microscopia a scansione del campo vicino (SN OM )
(Scanning Near Field Optical Microscopy). Non disponendo di rivelatori con dimensioni
dell’ordine di ∼ 10 nm, si può procedere in vari modi:
(a) riprodurre il campo in un’altra regione dove sia più agevole ispezionarlo: per far
questo si utilizza uno schema ottico confocale in cui il piano oggetto e quello immagine si
trovano nei due fuochi di un sistema ottico afocale; il campo immagine viene poi analizzato
con un rivelatore munito di diaframma di diametro ≃ 10 λ (S − SNOM );
(b) far scorrere in prossimità dell’oggetto una punta, che si polarizza generando un
momento di dipolo ℘˜ ∝ Ẽ (r): quest’ultimo diventa a sua volta sorgente di un campo
che si irradia nelle varie direzioni fino a colpire un rivelatore (macroscopico) posto nelle
vicinanze;
(c) utilizzare una fibra ottica che termina con una apertura Σ di dimensioni ≪ λ,
attraverso cui si fa viaggiare sia l’onda che illumina il campione, che l’onda riflessa da
quest’ultimo e catturata attraverso Σ;
(d) schema analogo al (c) in cui però si misura il campo trasmesso dal campione
(A − SN OM ).
Con gli apparati S − SNOM si possono raggiungere risoluzioni6 dell’ordine di 5 − 10
nm.
Esercizio 4.5.1 Si analizzi il campo in prossimità di una striscia, di larghezza molto
minore di λ, illuminata da una sovrapposizione di onde piane con polarizzazione TM
(Bŷ) ed ampiezze distribuite con leggge gaussiana
4
5
6
a cura del Dr. A. D’Ambrosio
M. Allegrini, N. Garcia, O. Marti, “Nanometer Scale Science and Technology”, Proc. E. Fermi School,
Course CXLIV, IOP Press, Amsterdam, 2001; L. Novotny and B. Hecht, “Principles of Nano-optics”,
Cambridge U. P., Cambridge, 2006 Sez. 4.3
R. Hillenbrand, T. Taubner, F. Keilmann, Nature 418, 159 (2002); K. Wang, D. M. Mittleman, N. C.
J. van der Valk, P. O. M. Planken, Appl. Phys. Lett., 85, 2715 (2004); A. Ambrosio and P. Maddalena,
Appl. Phys. Lett., 98, 091108 (2011)
124
Onde elettromagnetiche
Figura 4.1: Schema di principio di un apparato SNOM. Il cilindro che termina a punta
rappresenta una fibra ottica cava utilizzata per illuminare il campione. Il campo riflesso
dal campione può essere raccolto dalla stessa fibra. Il sistema può anche lavorare in
trsmissione misurando l’intensità trasmessa dal campione.
Figura 4.2: Campo vicino |Ex | ( sinistra ) e |Ez | (destra). Curve tratteggiate: z = 0.04 λ,.1
λ (sinistra) e z = 0.1 λ,.5 λ (destra). Le curve a tratto continuo più interne rappresentano
i campi a z = 0, mentre quelle più esterne si riferiscono√alle distribuzioni misurate con
microscopi ideali che lasciano passare valori di |kx | < k0 / 2.
4.6 Campi trasversi e longitudinali
125
Soluzione: Per fornire un’idea del campo in prossimità di una regione piana illuminata
di dimensione molto minore di λ, sono stati riportati in Fig. (??) le componenti Ẽx (sinistra) e Ẽz (destra) a diverse distanze dal piano z = 0 di un campo ottenuto sovrapponendo
onde piane con polarizzazione TM (Bŷ) ed ampiezze distribuite con leggge gaussiana,
∞
eikz z−
Ẽ (r) =
2
w2 kx
2
x̂ cos (kx x)
0
con kz =
kz
kx
− iẑ sin (kx x)
k
k
k02 − kx2 , k = k0 per k0 > kx e kz = i kx2 − k02 , k =
dkx ,
2kx2 − k02 per kx > k0 =
ω/c. Ẽx ed Ẽz corrispondono a strisce illuminate parallele a ŷ e di spessore dell’ordine
di λ/20. Mentre la larghezza del picco centrale di Ẽx appare raddoppiato già ad un
distanza z = λ/20, la distribuzione di Ẽz devia da quella a z = 0 più lentamente. A
tratto spesso è stato anche riportato l’andamento riprodotto da un microscopio ideale:
√
k0 / 2
e−
Ẽmicro (r) =
2
w2 kx
2
x̂ cos (kx x)
0
kz
kx
− iẑ sin (kx x)
k0
k0
dkx .
Risulta evidente che mentre il microscopio restituisce un’immagine molto diversa dall’orginale, per osservare quest’ultimo bisogna esaminare il campo vicino a distanze z
dell’ordine delle dimensioni dell’oggetto.
4.6
Campi trasversi e longitudinali
Esercizio 4.6.1 Si consideri un materiale in cui ε̂ dipenda solo dalle coordinate trasverse
(x, y) = r⊥ . Riscrivere le equazioni di Maxwell in assenza di correnti decomponendo i campi nelle componenti longitudinali Ẽz , B̃z e trasverse Ẽ⊥ , B̃⊥ per un sistema di
coordinate cartesiane.
Soluzione: Dalle equazioni di Maxwell
∇ × B̃
∇ × Ẽ
∇ · B̃
∇ · ε̂Ẽ
=
=
=
=
−iωε0 µ0 ε̂Ẽ
iω B̃
0
0
moltiplicando vettorialmente per ẑ si ha
k2
ẑ × Ẽ
ω
ẑ × ∇ × Ẽ = iωẑ × B̃
ẑ × ∇ × B̃ = −i
Tenendo conto che
ẑ × ∇ × B̃ = ∇⊥ B̃z −
∂
B̃⊥
∂z
126
Onde elettromagnetiche
si ottiene:
∂
k2
B̃⊥ = −i ẑ × Ẽ⊥
∂z
ω
∂
∇⊥ Ẽz − Ẽ⊥ = iωẑ × B̃⊥
∂z
Moltiplicando ancora vettorialmente per ẑ si perviene al sistema
∇⊥ B̃z −
∂
k2
ẑ × B̃⊥ + i Ẽ⊥
∂z
ω
∂
=
Ẽ⊥ + iωẑ × B̃⊥
∂z
ẑ × ∇⊥ B̃z =
∇⊥ Ẽz
da cui
k2 +
k2 +
∂2
∂z 2
∂2
∂z 2
Ẽ⊥ = −iωẑ × ∇⊥ B̃z +
ẑ × B̃⊥ = −i
∂
∇⊥ Ẽz
∂z
k2
∂
∇⊥ Ẽz + ẑ × ∇⊥ B̃z
ω
∂z
(4.15)
Se ne evince che i campi si possono raggruppare in due classi E (TM) ed H (TE)
secondo che B̃z = 0 e Ẽz = 0.
Esercizio 4.6.2 Analizzare i modi di propagazione di onde E ed H, dipendenti da z
secondo il fattore eiβz , per mezzi dieletttrici in cui ε̂ dipenda solo dalle coordinate trasverse
x, y.
Soluzione: Per modi E il sistema (4.15) si riduce a
2
k⊥
Ẽ⊥ = iβ∇⊥ Ẽz
i 2
2
k ∇⊥ Ẽz
k⊥
ẑ × B̃⊥ =
ω
2
dove k⊥
= k2 − β 2 .
Moltiplicando un’onda E scalarmente per ∇⊥ si ha
∇⊥ ·
Poichè
∇⊥
si ha
∇2⊥ −
Analogamente per un’onda H
k2
∇⊥ Ẽz = 0
2
k⊥
2 2
k2
k⊥
k − k4
=
∇⊥ ln ε̂
2
4
k⊥
k⊥
β2
2
∇⊥ ln ε̂ · ∇⊥ + k⊥
Ẽz = 0
2
k⊥
(4.16)
2
k⊥
Ẽ⊥ = −iωẑ × ∇⊥ B̃z
2
k⊥ B̃⊥ = iβ∇⊥ B̃z
Moltiplicando scalarmente per ∇⊥ si ha
∇2⊥ −
k2
2
∇⊥ ln ε̂ · ∇⊥ + k⊥
B̃z = 0
2
k⊥
I modi E ed H saranno quindi autofunzioni rispettivamente di (4.16) e (4.17).
(4.17)
4.7 Fibre ottiche
4.7
127
Fibre ottiche
Esercizio 4.7.1 Le fibre ottiche sono dei dei mezzi dielettici in cui ε̂ (ρ) dipenda solo
dalla coordinata radiale ρ. L’esempio più semplice è fornito da una fibra a salto d’indice
con ε̂ (ρ) = n21 per ρ < a e ε̂ (ρ) = n22 per ρ > a. (a) Calcolare i modi E ed H e (b)
discutere le relazioni di dispersione β = β (ω)
Soluzione: (a) Per ρ < a (4.16) si riduce a
∇2⊥ + k02 n21 − β 2 Ẽz = 0 ρ < a
∇2⊥ + k12 k02 n22 − β 2 Ẽz = 0 ρ > a
(4.18)
In particolare si ha
Ẽz =

 AJl
 BH (1)
l
k02 n21 − β 2 ρ eimφ ρ < a
k02 n22 − β 2 ρ eimφ ρ < a
(b) Imponendo la continuità di Ẽz e B̃φ =
si ottiene il sistema di equazioni:
i k2 ∂
2 ∂r Ẽz
ω k⊥
sulla superficie di discontinuità ρ = a
AJl
k02 n21 − β 2 a
= BHl
(1)
k02 n22 − β 2 a
n1 AJl′
k02 n21 − β 2 a
= n2 BJl′
k02 n22 − β 2 a
Da cui discende
n2
Jl′
k02 n22 − β 2 a
Jl
k02 n22 − β 2 a
= n1
Jl′
k02 n21 − β 2 a
Jl
k02 n21 − β 2 a
(4.19)
Questa equazione fissa β in funzione di ω.
Esercizio 4.7.2 Analizzare la relazione di dispersione β (ω) del precedente esercizio per
n1 ≃ n2
4.8
Guide d’onda
Esercizio 4.8.1 Le guide d’onda sono sono mezzi delimitati da cilindri metallici di sezione
generalmente rettangolare o cilindica. Rifacendosi all’analisi precedente relative alle fibre
ottiche trovare i modi E ed H per guide rettangolari di lati Lx , Ly .
Soluzione: I modi E ed H debbono sooddisfare sulle pareti della guida le condizioni
Ẽz = 0 modi E
∂
B̃z = 0 modi H
∂n
128
Onde elettromagnetiche
I modi sono dati da
lπ
mπ
x sin
y
Lx
Ly
lπ
mπ
= cos
x cos
y
Lx
Ly
Ẽz lm = sin
B̃z lm
Pertanto
2
2
mπ
nπ
=k −
−
Lx
Ly
Ad una assegnata frequenza si possono solo propagare quei modi tali che
2
k⊥
mn
2
2
k >
4.9
lπ
Lx
2
+
mπ
Ly
2
Oscillazioni di una cavità
Esercizio 4.9.1 Si discutano i modi di oscillazione di una cavità prismatica. (a) Calcolare le frequenze di risonanza e (b) la densità dei modi
Soluzione: Il concetto di autofunzioni della MQ trova il suo corrispettivo nella rappresentazione di campi e.m. come prodotti di funzioni del tempo per funzioni dello
spazio
En (t)en (r) ,
E(r, t) =
n
B(r, t) =
Bn (t)bn (r) ,
n
D(r, t) =
Dn (t)dn (r) ,
n
H(r, t) =
Hn (t)hn (r) ,
(4.20)
n
con en (r) e bn (r)/kn modi (spaziali) a divergenza nulla, normalizzati, mutuamente ortogonali, e legati tra loro dalle relazioni:
∇ × en = bn ,
∇ × bn = kn2 en ,
con kn = ω n /c un autovalore della equazione d’onda,
∇2 en + kn2 en = 0 ,
(4.21)
associata ad opportune condizioni al contorno sulle pareti del volume che contiene il
campo.
Inserendo nelle equazioni di Maxwell (2.13) i campi espansi nei modi suddefiniti e
tenendo conto di (4.20) e (4.21) si ottiene per le ampiezze
En
2
kn Bn
Ën + ω 2n En
= −Ḃn ,
= µ0 ε0 Ėn − Jn ,
1
= − J˙n .
ε0
4.9 Oscillazioni di una cavità
129
(a) Imponendo che la componente di en tangente alle pareti si annulli, per gran parte
dei modi di una cavità prismatica risulta
en =
√
πlx
V −1 ǫ̂n sin
Lx
πmy
Ly
sin
sin
πnz
Lz
.
en è associabile a un vettore d’onda
πl
πm
πn
x̂ +
ŷ +
ẑ
Lx
Ly
Lz
kn =
a componenti positivi e multipli interi di π/Lx,y,z .
Le frequenze di risonanza saranno definite dalla condizione k 2 = kn2
ωn
c
2
=
πl
Lx
2
+
2
πm
Ly
+
πn
Lz
2
(b) Al variare di l, m ed n kn definisce un reticolo con cella elementare di volume
π 3 /V . Il numero N (ω) di modi di frequenza angolare inferiore ad un assegnato valore
ω sarà quindi doppio7 del numero di vertici di questo reticolo compresi in un ottante di
raggio k = ω/c
1 ω 3
N(ω) = 2
V.
3π
c
A parità di volume il numero dei modi cresce col cubo della frequenza angolare, una dipendenza questa che, come vedremo nell’ultimo capitolo, condiziona fortemente lo spettro di
corpo nero. La densità g(ω) = V −1 dN(ω)/dω del numero di modi per unità di volume e
di intervallo di frequenza angolare, risulta uguale a
g(ω) =
1
c3
ω
π
2
.
(4.22)
Anche se questa formula per g(ω) è stata ricavata per una cavità prismatica, resta valida per cavità di forma generica quando la lunghezza d’onda è piccola rispetto alle sue
dimensioni caratteristiche.
Spesso si incontrano campi che si propagano lungo direzioni particolari, p.e. asse ẑ.
In tal caso la componente monocromatica del campo Ẽ (ed analogamente per gli altri
vettori) assume la forma
E(r, t) =
n
e−iωt+iβ n z Ẽn en (r⊥ ) ,
dove β n rappresenta la costante di propagazione lungo l’asse di propagazione mentre
en (r⊥ ) soddisfa un’equazione di Helmholtz simile a (4.21)
∇2⊥ en (r⊥ ) + k 2 − β 2n en (r⊥ ) = 0 ,
ed è sottoposta ad opportune condizioni al contorno sul piano r⊥ . Gli autovalori β n di
quest’ultima per assegnato k = ω/c stabiliscono una relazione di dispersioneω β = ω (β n ) .
Esercizio 4.9.2 Trovare i modi di oscillazione di una cavità prismatica.
7
ad ogni kn corrispondono due polarizzazioni ǫ̂.
130
Onde elettromagnetiche
Soluzione: Per fissare le idee si consideri una cavità a forma di prisma retto di lati e
scegliamo un sistema di coordinate cartesiane x, y, z con gli assi paralleli agli spigoli a, b,
c e con centro in un vertice. Se la cavità è limitata da pareti metalliche, i modi possono
essere raggruppati in due classi secondo che il campo magnetico (modi T M o modi E) o
il campo elettrico (modi T E o modi H) siano rispettivamente perpendicolari all’asse z
nπ
z
c
nπ
nπ
(H)
(H)
blmn (r) = btlm (r⊥ ) cos
z + bzlm (r⊥ ) sin
z ẑ
c
c
nπ
nπ
(E)
(E)
elmn (r) = etlm (r⊥ ) sin
z + ezlm (r⊥ ) sin
z ẑ
c
c
nπ
(E)
blmn (r) = btlm (r⊥ ) cos
z
c
(H)
elmn (r) = etlm (r⊥ ) sin
dove r⊥ = xx̂ + y ŷ. In particolare,
(H)
nπ/c lπ
lπx
mπy
sin
B
cos
lm
2
ktlm
a
a
b
nπ/c mπ
lπx
mπy
−i 2
Blm cos
sin
ktlm b
a
b
lπx
mπy
Blm cos
cos
a
b
ω lmn µ0 (H)
h
nπ/c ylm
ω lmn µ0 (H)
−
h
nπ/c xlm
hxlm = −i
(H)
hylm =
(H)
hzlm =
(H)
exlm =
(H)
eylm =
2
ktlm
lπ
a
=
Blm =
2
π
ω lmn = c
2
mπ
b
+
2
2
ktlm
l2 ab + m2 ab
lπ
a
2
+
mπ
b
2
+
nπ
c
2
Campi relativi ai vari modi TE di una cavità rettangolare di lati a, b e c ed indici l, m
ed n
Esercizio 4.9.3 Tenuto conto che l’energia elettromagnetica è espressa da
1
Uem = ε0
2
ω 2n B2n + Ḃ2n .
(4.23)
n
se ne deduce che i singoli modi vi contribuiscono con termini del tutto simili a quelli
di oscillatori meccanici unidimensionali di frequenza angolare ω n in cui 12 ε0 ω 2n B2n misura
l’energia “potenziale” e 12 ε0 Ḃ2n quella “cinetica”. Bn si comporta come la posizione x di
4.9 Oscillazioni di una cavità
131
un oscillatore unidimensionale di massa M = ε0 , frequenza angolare ω n = K/M con K
costante elastica, mentre8 −ε0 En = ε0 Ḃn si comporta come il momento coniugato,
x
p
M
K
←→
←→
←→
←→
Bn ,
−ε0 En = ε0 Ḃn ,
ε0 ,
ε0 ω 2n .
Uem di (4.23) può essere trattata come l’hamiltoniana del campo ed il termine n-esimo
come l’hamiltoniana del rispettivo modo9 ,
1
Hem n = ε0 ω 2n B2n + Ḃ2n
2
.
(4.24)
Alla luce di questa corrispondenza quantizzare le ampiezze Bn dei rispettivi modi
Soluzione: Analogamente a quanto si fa per la posizione fluttuante di un oscillatore meccanico, si può introdurre una funzione d’onda normalizzata ψ (Bn ; t) tale che
|ψ (Bn ; t)|2 misura la densità di probabilità che l’induzione abbia il valore Bn . Per quantizzare si utilizzerà come hamiltoniana l’espressione (4.24) dell’energia sostituendo ε0 Ḃn
con −i ∂B∂ n , ottenendo così per ψ (Bn ; t)
i
∂
ψ (Bn ; t) =
∂t
2
−
1
∂2
+ ε0 ω 2n B2n ψ (Bn ; t) .
2
2ε0 ∂Bn 2
Per la coppia Bn , En deve valere il principio di indeterminazione di Heisenberg ovvero
il prodotto delle relative fluttuazioni risulta maggiore di
ε0 ∆Bn ∆En ≥
,
(4.25)
ed analogamente per l’energia Uem n = ε0 E2n e la fase10 ϕn
∆ϕn ∆Uem n ≥ ω n .
Anche in assenza di sorgenti il campo all’interno di una cavità fluttua. Ciò diventa
particolarmente evidente quando vi si immette un atomo. Se ad esempio questo possiede
due livelli di energia Ea e Eb tali che Eb − Ea = ω n , risentirà delle fluttuazioni del
campo a questa frequenza. Se per esempio è caratterizzato dalle funzione d’onda va , vb ,
interagendo col campo fluttuante si porterà in uno stato αva e−iEa t/ +βvb e−iEb t/ , creando
un momento di dipolo oscillante alla frequenza angolare (Ea − Eb ) / che accoppiandosi
con le fluttuazioni del campo e.m. irradierà facendo decadere l’atomo da Eb a Ea .
ωn
Le ampiezze Bn = X εo ωn , En = P
si comportano come variabili coniεo
ugate di un oscillatore armonico di frequenza angolare ω n , associato agli operatori di
distruzione e creazione.
In particolare nello stato |0 la densità di probabilità f (E) che il campo elettrico abbia
ampiezza E è descritta dalla Gaussiana
8
9
10
En ha le dimensioni di un campo elettrico per la radice quadrata di un volume.
per una introduzione semplice alla quantizzazione del campo e.m., agli stati del campo quantizzato ed
alla interazione con gli atomi v.p.e. R. Loudon, The Quantum Theory of Light, Oxford Univ. Press,
Oxford 2000, Capp. 6-8; D. Marcuse, Engineering Quantum Electrodynamics, Harcourt, Brace &
World, New York 1970, Cap. 2; E. Landi Degl’Innocenti, loc. cit. pag. 57,
√ Cap. 4.
En (t) = En 0 cos (ω n t + ϕn ) ha le dimensioni di un campo elettrico per V
132
4.10
Onde elettromagnetiche
Campi trasversi e radiali
Esercizio 4.10.1 Si consideri un materiale in cui ε̂ (r) dipenda solo dalla coordinata
radiale. Riscrivere le equazioni di Maxwell decomponendo i campi nelle componenti radiali
Er , Br e trasverse Ẽ⊥ , B̃⊥ per un sistema di coordinate sferiche:
Soluzione: Dalle equazioni di Maxwell discende che:
∇⊥ Br −
∇⊥ Er −
∂
1
+
B̃⊥ = −iωε0 ε̂r̂ × Ẽ⊥ + µ0 r̂ × J̃⊥
∂r r
1
∂
+
Ẽ⊥ = iωr̂ × B̃⊥
∂r r
Dal momento che
r̂ · ∇ × B̃ = −∇⊥ · r̂ × B̃
si ha
−∇⊥ · r̂ × B̃ = −iωε0 ε̂Ẽr + µ0 Jr
−∇⊥ · r̂ × Ẽ = iω B̃r
Combinando queste equazioni si ottiene
∂
1
+
∂r r
B̃⊥ = iωε0 ε̂ 1⊥ +
1 2
∇ r̂ × Ẽ⊥ − µ0 r̂ × J̃⊥
k2 ⊥
∇⊥ Br = iωε0 ε̂r̂ × Ẽ⊥
∂
1
1
ζ
+
Ẽ⊥ = −iω 1⊥ + 2 ∇2⊥ r̂ × B̃⊥ − i ∇⊥ Jr
∂r r
k
kc
∇⊥ Er = −iωr̂ × B̃⊥
(4.26)
Esercizio 4.10.2 Si consideri un materiale in cui ε̂ (r) dipenda solo dalla coordinata
radiale r. Si ponga
Ẽ⊥ = −∇⊥ V (E) − ∇⊥ × r̂V (H)
B̃⊥ = ∇⊥ × r̂I (E) − ∇⊥ I (H)
r̂ × J̃ = ∇⊥ × r̂J (E) − ∇⊥ J (H)
(4.27)
Si dimostri utilizzando il sistema (4.26) e la relazione
∂
∂
1
∇⊥ = ∇⊥ − ∇⊥
∂r
∂r r
che
∂ (E)
1
ζ
V
= ikζ 1 + 2 ∇2⊥ I (E) + i Jr
∂r
k
kc
∂ (E)
k
I
= i V (E) − µ0 J (E)
∂r
ζ
i
ζ
Er = − 2 ∇2⊥ I (E) − i Jr
k
kc
(4.28)
4.10 Campi trasversi e radiali
133
e
∂ (H)
k
1
1
= i
I
1 + 2 ∇2⊥ V (H) − J (H)
∂r
ζ
k
kc
∂ (H)
V
= ikζI (H)
∂r
i
Br = − ∇2⊥ V (H)
k
con k =
sferiche
ω
c
√
ε̂µ̂ e ζ =
ε0 ε̂
µ0 µ̂
= ζ0
ε̂
µ̂
(4.29)
. Espandendo V (E) , I (E) e V (H) , I (H) in armoniche
(E)
V (E) (r) =
Vlm (r) Ylm (θ, φ)
lm
e operando similmente per le altra grandezze ottenere l’equivalente dei sistemi (4.28) e
(E) (E)
(H) (E)
(4.29) per Vlm , Ilm , Vlm , Ilm .
Soluzione: (b) Tenuto conto che∇2⊥ Ylm = l (l + 1) Ylm /r2 si ottengono facilmente i due
sistemi di equazioni del moto:
l (l + 1)
ζ
d (E)
(E)
Vlm = iζ 1 −
Ilm + i 2 Jr lm
2
dξ
k c
ξ
d (E)
1 (E) µ (E)
Ilm = i Vlm − 0 Jlm
dξ
ζ
k
l (l + 1) (E)
ζ
Er lm = i
Ilm − i Jr lm
2
kc
ξ
(4.30)
ē
1
d (H)
l (l + 1)
Ilm = i
1−
dξ
ζ
ξ2
d (H)
(H)
V
= iζIlm
dξ lm
l (l + 1) (H)
Br lm = i
Vlm
ξ2
(H)
Vlm −
1 (H)
J
k 2 c lm
(4.31)
con ξ = kr.
(E,H)
Esercizio 4.10.3 Si analizzino le funzioni Vlm
genti
(E,H)
(r) e Ilm
(r) in regioni prive di sor-
Soluzione: Il sistema (4.30) si riduce a:
d (E)
l (l + 1)
Vlm = iζ 1 −
dξ
ξ2
d (E)
1 (E)
Ilm = i Vlm
dξ
ζ
Ne discende che
(E)
Ilm
l (l + 1) (E)
d2
Ilm = 0
2 +1−
dξ
ξ2
(4.32)
134
Onde elettromagnetiche
per cui
(E)
Ilm (ξ) = Ajl (ξ) + Bhl (ξ)
d (E)
(E)
Vlm = −iζ Ilm
dξ
con jl = ξjl , hl = ξhl e jl , hl funzioni di Bessel sferiche.
Analogamente per le onde H
l (l + 1)
d (H)
1
Ilm = i
1−
dξ
ζ
ξ2
d (H)
(H)
V
= iζIlm
dξ lm
(H)
(H)
Vlm
(4.33)
(H)
per cui anche Vlm , Ilm sono rapresentabili come combinazioni di funzioni di Bessel
sferiche.
(E) (E)
(H) (H)
Pertanto sia Vlm , Ilm che Vlm , Ilm si possono interpretare come coppie di tensionecorrente di linee di trasmissione sferiche caratterizzate dalle equazioni del moto (4.32) e
(4.33).
4.11
Scattering di Mie
Esercizio 4.11.1 Si consideri una sfera di raggio a ed indice di rifrazione n2 , immersa
in un mezzo di indice n2 e privo di sorgenti. (a) Ricavare le ampiezze dei campi sui due
lati della discontinuità. (b) Si analizzi il coefficiente di riflessione dei vari modi sferici.
(E)
(c) Si consideri il caso limite in cui Vlm si annulla sulla superficie della sfera.
Soluzione: (a) Le onde E soddisfano il sistema di equazioni
d (E)
l (l + 1)
Vlm = iζ 1 −
dξ
ξ2
d (E)
1 (E)
Ilm = i Vlm
dξ
ζ
(E)
Ilm
Ne discende che
(E)
= A(E) jl + B (E) hl
(E)
= −iζ 0 nIlm
(H)
= A(H) jl + B (H) hl
i (H)′
= −
V
.
ζ 0 n lm
Ilm
Vlm
(E)′
mentre per le H si ha:
Vlm
(H)
Ilm
Imponendo la continuità di I e V per le E si ottiene sulla superficie della sfera:
(E)
ξ̄ 1
= Ilm
(E)′
ξ̄ 1
= n2 Ilm
Ilm
n1 Ilm
(E)
ξ̄ 2
(E)′
ξ̄ 2
4.11 Scattering di Mie
135
ovvero
(E)
(E)
= A2 jl ξ̄ 2 + B2 hl ξ̄ 2
(E)
(E)
(E)
= n A2 j′l ξ̄ 2 + B2 h′l ξ̄ 2
A1 jl ξ̄ 1 + B1 hl ξ̄ 1
(E)
(E)
A1 j′l ξ̄ 1 + B1 h′l ξ̄ 1
(E)
Espressioni analoghe si hanno per le H.
(b) Ponendo
(E)
(E)
(E)
(E)
(E)
A1 = 1 , B1 = −Rl , A2 = Tl
si ha
(E)
(E)
Rl hl ξ̄ 1 + Tl
(E)
Rl h′l
jl ξ̄ 2
(E)
nTl j′l
ξ̄ 1 +
ξ̄ 2
= jl ξ̄ 1
= j′l ξ̄ 1
Ne discende che
(E)
Rl
=
Tl(E) =
njl ξ̄ 1 j′l ξ̄ 2 − j′l ξ̄ 1 jl ξ̄ 2
nhl ξ̄ 1 j′l ξ̄ 2 − h′l ξ̄ 1 jl ξ̄ 1
j′l ξ̄ 1 hl ξ̄ 1 − h′l ξ̄ 1 jl ξ̄ 1
nhl ξ̄ 1 j′l ξ̄ 2 − h′l ξ̄ 1 jl ξ̄ 1
(4.34)
Per le onde H valgono le stesse espressioni con n sostituito da n−1 .
(c) Se V (E) = 0 sulla superficie della sfera si avrà
j′l ξ̄ 2 = 0
e
(E)
Rl
=
j′l ξ̄ 1
h′l ξ̄ 1
Se si pone
j′l ξ̄ 1
(E)
= tan δ l
′
nl ξ̄ 1
ovvero
(E)
Rl
=
(E)
j′l ξ̄ 1
(E)
= sin δ l eiδl
′
hl ξ̄ 1
si ha
√
4π
I (E) (r) =
k1
∞
l=1
il
(E)
2l + 1
(E)
jl (ξ 1 ) − hl (ξ 1 ) sin δ l eiδl
Yl1 (Ω)
l (l + 1)
Esercizio 4.11.2 Si consideri un’onda piana polarizzata circolarmente
Ẽ (r) = (x̂ + iŷ) eikz
a cui corrisponde11 l’onda E di corrente I (E)
√
∞
4π
2l + 1
(E)
I (r) =
il
jl (ξ 1 ) Yl1 (Ω)
k1 l=1
l (l + 1)
11
v. J. Schwinger, loc. cit. pag. 208 Sez. 46.4
(4.35)
136
Onde elettromagnetiche
e tensione
V (H) (r) = −iI (E) (r)
(a) calcolare il campo scatterato da una sfera di raggio a ed indice di rifrazione n ≫ 1;
(b) Calcolare la potenza totale scatterata e la sezione d’urto . (c) Discutere il caso in cui
k2 a ≪ 1 (scattering di Rayleigh).
Soluzione: (a) Il campo scatterato è associato alla tensione (v- (4.35))
√
∞
(E)
4π
2l + 1
(E)
(E)
I (r) = −
il
hl (ξ 1 ) sin δ l eiδl Yl1 (Ω)
k1 l=1
l (l + 1)
√
∞
(E)
4π
2l + 1 ′
(E)
(E)
V
(r) = iζ 0
il
hl (ξ 1 ) sin δ l eiδl Yl1 (Ω)
k0 l=1
l (l + 1)
A grande distanza si può porre
hl (ξ) ≃ (−i)l+1 eiξ
per cui
I
(E)
V
(E)
√
4π iξ1
(r) = −i
e
k1
(r) = ζI
(E)
∞
(E)
2l + 1
(E)
sin δ l eiδl Yl1 (Ω)
l (l + 1)
l=1
(r)
Si vede pertanto che il campo scatterato è caratterizzato dalle fasi δ l .
(b) La potenza totale scatterata si ottiene integrando il vettore di Poynting relativo
al campo scatterato su una sfera di raggio molto gande. A grande distanza isulta
2
1
1
S̃ = Ẽ × H̃ =
Ẽ⊥ (Ω) n̂
2
2ζ
dove (v. (4.27))
Ẽ⊥ = −∇⊥ V (E)
si ha
Psc(E) =
1 2
r
2ζ
2
∇⊥ V (E) dΩ
D’altra parte
√ eikr
∇⊥ V (E) = −iζ 4π
k
∞
l=1
per cui
Psc(E) =
(E)
2l + 1
(E)
sin δ l eiδl ∇⊥ Yl1
l (l + 1)
il
2π 2
r
k2ζ
∞
(E)
(2l + 1) sin2 δ l
l=1
Tenuto conto che la potenza dell’onda incidente è stata posta uguale a Pinc = 1ζ , la sfera
presenta una sezione d’urto pari a
σ
(E)
2π
=
(2π)4 k 2
∞
l=1
(E)
(2l + 1) sin2 δ l
4.12 Potenziali di Debye
137
(c) Quando ka ≪ 1 si ha che il termine dominante è quello con l = 1. Inoltre si ha
ξ2
3
1
ξ4
−i
h1 (ξ) ≃
3
ξ
j1 (ξ) ≃
per cui
j′l (ξ)
2
= ξ3
′
hl (x)
3
In particolare ne discende che la sezione d’urto nello scattering di Rayleigh da parte di
una sfera metallica o dielettrica con alto n investita da un’onda E risulta pari a:
σ
(E)
2π
2
= 23
(k1 a)3
k
3
2
4 (2π)5 a6
=
3 λ41
Ripetendo i cacoli per la componente H si può dimostrare12 che ad essa corisponde una
sezione d’urto σ (H) = σ (E) /4.
4.12
Potenziali di Debye
Esercizio 4.12.1 Dimostrare che utilizzando i potenziali di Debye i campi Ẽ (r) , B̃ (r)
sono esprimibili nella forma13
Ẽ (r) =
∞
l
l=0 m=−l
B̃ (r) =
∞
l
l=0 m=−l
i
fl (kr) Xlm (Ω) + ∇×gl (kr) Xlm (Ω) ,
k
i
gl (kr) Xlm (Ω) − ∇×fl (kr) Xlm (Ω) .
k
Il vettore
Xlm (Ω) =
(4.36)
1
LYlm (Ω)
l (l + 1)
sta per l’armonica sferica vettoriale mentre
L = − ir × ∇
rappresenta l’operatore momento angolare. fl (kr) e gl (kr) sono integrali della parte
radiale dell’equazione di Helmholtz
1 d 2d
l (l + 1)
r
−
+ k2
2
2
r dr dr
r
fl (kr)
=0
gl (kr)
esprimibili cme combinazioni lineari di hl (kr) e jl (kr):
fl (kr) = Afhl (kr) + Bfjl (kr)
gl (kr) = Ag hl (kr) + Bgjl (kr)
12
13
v. J. Schwinger, loc. cit. pag. 208 Eq. (46.102)
J. Schwinger, loc. cit. pag. 208 Sez. 50.4 Prob. 4
Capitolo 5
Relazioni costitutive
5.1
Modello di Drude
Esercizio 5.1.1 Analizzare il campo e.m. prodotto da un’onda piana incidente normalmente su un metallo di assegnata conducibilità elettrica e frequenza di plasma che occupa
il semispazio z>0. Calcolare in funzione della frequenza (a) il coefficiente di riflessione,
(b) il campo all’interno del metallo, (c) il campo di temperatura all’interno dello stesso
Soluzione: La costante dielettrica di un metallo è rappresentata nel modello di Drude
da
ε̃ = 1 −
iωγ + ω 2 − ω 2p
ω 2p
=
= (ñr + iκ̃)2
ω(iγ + ω)
ω(iγ + ω)
(5.1)
con ω p frequenza di plasma
ne e2
mε0
ωp =
Per ω = 0 si riduce a
ε̃ (0) = i
con
ω 2p
σ0
=i
ωγ
ε0 ω
ω 2p
ne e2
σ0 =
= ε0
me γ
γ
conducibilità elettrica in continua. ñr e κ̃ (> 0) stanno per la parte reale dell’indice di
rifrazione e per il coefficiente di estinzione. Per ω ≪ ω p
ω2
ε̃ ≃
− ωp2 γ ≪ ω ≪ ω p
= (ñr + iκ̃)2
ω2p
i ωγ ω ≪ γ ≪ ω p
da cui segue per ñ
ñr + iκ̃ ≃
ωp
ω
i γ ≪ ω ≪ ωp
e
ω ≪ γ ≪ ωp
iπ/4
Per incidenza normale il metallo è caratterizzato da un coefficiente di riflessione pari a
r=
ñ − 1
ñ + 1
139
140
Relazioni costitutive
Figura 5.1: Modulo del coefficiente di riflessione per incidenza normale per un metallo
per diversi valori di γ/ω p = 0.05, 0, 1, 02, 0, 4
che per ω ≪ ω p si riduce ad 1.
Esercizio 5.1.2 Il coefficiente di riflessione di un materiale dipende dall’angolo di incidenza e dalla polarizzazione (onda p/s=polarizzazione parallela/perpendicolare al piano
di incidenza) secondo le formule di Fresnel
rp = −
rs = −
tan (θ′ − θ)
ñ2 cos θ − ñ2 − sin2 θ
=
tan (θ ′ + θ)
ñ2 cosθ + ñ2 − sin2 θ
−cosθ +
sin (θ ′ − θ)
=
′
sin (θ + θ)
cosθ +
1
ñ
ñ2 − sin2 θ
ñ2 − sin2 θ
con θ e θ′ angoli di incidenza e rifrazione ed ñ indice di rifrazione. In particolare rp si
annulla per un particolare valore di θ = θB detto angolo di Brewster
ñ2 cosθB
ñ4 − ñ4 sin2 θB
ñ2
ñ2 + 1
ñ
=
ñ2 − sin2 θB
= ñ2 − sin2 θB
= sin2 θB
= tan θB
ñ
θB = arcsin √ 2
ñ + 1
Dalla misura di θB si può risalire a ñ. Nel caso di ñ complesso anche θB risulta tale, per
cui è necessario illuminare il campione con un’onda evanescente. Analizzare l’andamento
della parte reale e immaginaria di θB per un metallo
5.1 Modello di Drude
141
Figura 5.2: Parte reale (tendente a π/4 per ω → ∞) ed immaginaria dell’angolo di
Brewster per un metallo descritto dal modello di Drude con ω p =frequenza di plasma e
γ = ω p /10
Per un metallo si ha
ñ = ñr + iκ̃ =
iωγ + ω 2 − ω 2p
ω(iγ + ω)
da cui
θB = arcsin
iωγ + ω 2 − ω 2p
2iωγ + 2ω 2 − ω 2p
In particolare per ω ≫ ω p θ B → π/4 mentre per ω = ω p θB ≃ i γ/ω p
Esercizio 5.1.3 La funzione dielettrica degli elettroni di un metallo è legata alla frequenza
dalla relazione
ω 2p
ε̃ = 1 −
ω(iγ + ω)
dove ω p sta per la frequenza di plasma, legata alla densità elettronica dalla relazione
ωp =
ne e2
me ε0
(5.2)
Per i metalli alcalini Li.Na,K,Rb,Cs le frequenze di plasma (espresse in lunghezze d’onda
λp = 2πc/ω p = c/fp ) sono riportate nella tabella1


λp Å
Li
Na
K
Rb
Cs


 calcolata 1550 2090 2870 3220 3620
misurata 1550 2100 3150 3400
Calcolare le rispettive (a) densità elettroniche e (b) costanti reticolari a tenendo presente
che questi metalli cristallizzano nel sistema bcc
1
v. C. Kittel, Introduzione alla Fisica dello Stato Solido, Casa Editrice Ambrosiana, Mi 2008, p. 399 e
Tabb. 3 e 4
142
Relazioni costitutive
Soluzione: (a) Tenendo presente che e = 1.6022 × 10−19 C, me = 9.1095 × 10−31 Kg,
ε0 = 8.8542 × 10−12 F/m, l’Eq. (5.2) si può riscrivere nella forma
√
fp ≈ 9 ne Hz
fp =
ne e2
=
me ε0
1
2π
ne
√
√
1.60222 × 10−38
= 8.97873 ne ≈ 9 ne Hz
−31
−12
9.1095 × 10 8.8542 × 10
con ne in m−3 . Pertanto
c
9λp
ne =
2
=
3 × 108
9 × 10−10
2
1
λ2p
=
Å
1036
1
9 λ2 Å
p
elettroni/m3 =
1030
1
9 λ2 Å
p
elettroni/cm3
da cui


elettroni/cm3
Li
Na
K
Rb
Cs
 calcolata
4.62 × 1022 2.54 × 1022 1.35 × 1022 1.07 × 1022 8.48 × 1021 
misurata
4.62 × 1022 2.52 × 1022 1.12 × 1022 9.61 × 1021
(b) per un reticolo bcc la costante reticolare a è data da
a=
per cui
5.2
2
ne
1/3


a Å
Li
Na
K
Rb Cs


 calcolata 3.51 4.28 5.30 5.71 6.18
misurata 3.51 4.29 5.63 5.92
Polarizzazione molecolare
Nel 1917 L. S. L. Silberstein2 di trattare una molecola investita da un campo Ẽ ext ∝
ǫ̂e−i(ωt−k·r) come un insieme di dipoli ℘˜m , localizzati sui nuclei costituenti, su ognuno dei
quali agisce un campo locale (v. 3.3.1)
Ẽloc
n
= Ẽext n +
m=n
Γdip mn · ℘˜m ,
con Ẽ n =Ẽ ext (Rn , ω) e Rn la posizione dell’atomo n-esimo, mentre Γnm = Γmn =
Γ (Rm − Rn , k) sta per la funzione di Green diadica (v. (4.12))
Γdip mn = Γdip (Rn − Rm ) = −
2
1
1
3R̂R̂ − 1 = −
2R̂R̂ − 1⊥
3
4πε0 R
4πε0 R3
L. S. L. Silberstein, Philos. Mag. 33, 92, 215 e 521 (1917). R. L. Rowell and R. S. J. Stein, J. Chem.
Phys. 47, 2985 (1967); E. M. J. Mortensen, J. Chem. Phys. 49, 3732 (1968); H. J. Devoe, J. Chem.
Phys. 43, 3199 (1965); R. R. J. Birge, J. Chem. Phys. 72, 5312 (1980); B. T. Thole, J. Chem. Phys.
59, 141 (1982); K. J. Miller, J. Am. Chem. Soc., 112, 8543 (1990)
5.2 Polarizzazione molecolare
143
calcolata nel limite statico in vista dela piccolezza di k |Rn − Rm | . Risolvendo rispetto a
℘˜n = ε0 α̃n · Ẽloc n con α̃n la polarizzabilità α̃℘n dell’atomo di indice n, si ottiene la relazione
tensoriale:



···
···
···
···
···
···
 ···


0
Γdip n−1,n Γdip n−1,n+1 · · · 

  ℘˜n−1 
 · · · Γdip n,n−1


0
Γdip n,n+1 · · ·  · ℘˜n 


 · · · Γdip n+1,n−1 Γdip n+1,n
0
· · ·   ℘˜n+1 
···
···
···
···
···
···


···
 Ẽloc n−1 − Ẽext n−1 



= 
 Ẽloc n − Ẽext n  .
 Ẽloc n+1 − Ẽext n+1 
···
D’altra parte

per cui






···
···
℘˜n−1
℘˜n
℘˜n+1
···






 = ε0




···
0
0
0
0
0 α̃n−1 0
0
0
0
0
α̃n
0
0
0
0
0 α̃n+1 0
0
0
0
0
···
 
···
  Ẽloc n−1
 
 ·  Ẽloc n
 
  Ẽloc n+1
···







 Ẽext n−1 


 Ẽext n 


 Ẽext n+1 
···

···
···
···
···
 Γdip n−1,n−2 α̃n−1
1
Γ
α̃
Γ
dip n−1,n n
dip n−1,n+1 α̃n+1


···
Γdip n,n−1 α̃n−1
1
Γdip n,n+1 α̃n+1
= 

···
Γdip n+1,n−1 α̃n−1 Γdip n+1,n α̃n
1
···
···
···
···
···
···
···
···
···
 
···
  Ẽloc n−1
 
 ·  Ẽloc n
 
  Ẽloc n+1
···
che invertita fornisce Ẽloc n in funzione di del campo esterno Ẽext n′ agente sui singoli atomi

 


···
···
···
···
···
···
···
 Ẽloc n−1   · · · Λ̃n−1,n−1 Λ̃n−1,n Λ̃n−1,n+1 · · ·   Ẽext n−1 

 


 Ẽloc n  =  · · · Λ̃n,n−1


(5.3)
Λ̃n,n
Λ̃n,n+1 · · · 

 
 · Ẽext n  .
 Ẽloc n+1   · · · Λ̃n+1,n−1 Λ̃n+1,n Λ̃n+1,n+1 · · ·   Ẽext n+1 
···
···
···
···
···
···
···
Poichè Γdip mn è funzione di |Rm − Rn | ed i momenti di dipolo indotti sui singoli atomi
dipendono dalla disposizione dei vari atomi rispetto alla direzione di propagazione n̂ di
Ẽ ext . Il momento di dipolo indotto in Rm non è in generale parallelo a Ẽ n , ovvero Λ̃mn è
una matrice 3×3 con autovalori generalmente diversi tra loro. Se le distanze interatomiche






144
Relazioni costitutive
sono piccole rispetto a 1/k ha senso introdurre un momento di dipolo totale associandolo
ad una polarizzabilità complessiva della molecola α̃℘mol riferita al baricentro R0 :
℘˜tot =
℘˜n =
n
mn
α̃n Λ̃nm · Ẽext m (Rm , ω) = α̃℘mol · Ẽext (R0 , ω) .
Queste considerazioni diventano significative nel caso di molecole con molti atomi.
In linea di principio si potrebbero utilizzare anche per lo studio dei solidi, anche se in
quest’ultimo caso si preferisce sfruttare sin dall’inizio la regolarità con cui gli atomi sono
distribuiti nel reticolo.
Esercizio 5.2.1 Si considerino due atomi in r1 e r2 , entrambi di polarizzabilità αp e
sottoposti ad un campo esterno uniforme Eext (a) calcolare i momenti di dipolo indotti,
(b) l’energia elettrostatica del sistema
Soluzione: I dipoli indotti ℘1,2 sono legati tra loro ed a Eext dal sistema di relazioni
lineari
℘1 − αp Γdip · ℘2 = ε0 αp Eext
℘2 − αp Γdip · ℘1 = ε0 αp Eext
dove R = r2 − r1 e
Γdip (R) =
1
1
3R̂R̂ − 1 =
2R̂R̂ − 1⊥
3
4πε0 R
4πε0 R3
Pertanto
℘1 −
α2p Γdip
℘2 = αp Γdip · ℘1 + ε0 αp Eext
· (Γdip · ℘1 + ε0 Eext ) = ε0 αp Eext
1 − α2p Γdip · Γdip · ℘1 = ε0 αp (1 + αp Γdip ) · Eext
Poichè
Γ2dip
1−
α2p Γ2dip
1 − α2p Γ2dip
−1
=
=
=
1 + αp Γdip =
1 − α2p Γdip · Γdip
−1
"
1
4πR3
1−
"
1−
1+
2
4R̂R̂ + 1⊥
#
"
#
2
2
αp
2αp
R̂R̂ + 1 −
1⊥
4πε0 R3
4πε0 R3
#
"
#
2 −1
2 −1
2αp
αp
R̂R̂ + 1 −
1⊥
4πε0 R3
4πε0 R3
2αp
4πε0 R3
R̂R̂ + 1 −
(1 − αp Γdip ) = AR̂R̂ + B1⊥
si ha
℘1 = ε0 αp AR̂R̂ + B1⊥ · Eext
αp
4πε0 R3
1⊥
5.3 Funzione dielettrica per elettroni delocalizzati
145
avendo posto
"
A =
1−
2αp
4πR3
αp
1−
4πR3
B =
2
2
#−1
−1
1+
1−
2αp
4πR3
αp
4πR3
(b) L’energia elettrostatica sarà pertanto uguale a
1
1
℘1 · Eext + ℘2 · Eext
2
2
= ε0 αp AR̂R̂ + B1⊥ : Eext Eext
V =
=
dove
α
1
ε0 α E 2 + α⊥ E⊥2
2
"
= 2αp 1 −
"
α⊥ = 2αp 1 −
2αp
4πε0 R3
2
αp
4πε0 R3
2
#−1
#−1
1+
2αp
4πε0 R3
1−
αp
4πε0 R3
V dipende dall’orientamento della coppia di atomi rispetto al campo e la coppia è rappresentata da due suscettività efficaci α , α⊥ secondo che il campo sia parallelo o perpendicolare alla congiungente dei due atomi.
5.3
Funzione dielettrica per elettroni delocalizzati
Nei solidi gli elettroni sono descritti da funzioni d’onda nonlocalizzate per cui l’interpretazione locale di P cade in difetto quando la si applica alle bande di valenza e di
conduzione.
Limitandoci al caso statico, assumiamo che la polarizzazione del mezzo sia prodotta
dall’introduzione di una distribuzione di carica esterna qρext (r) . Esprimendo il vettore
spostamento D = ε0 ε̂Ẽ in funzione del campo elettrico introducendo l’operatore ε̂ costante
dielettrica (v. Sez. 10.1.2) definito da (v. Eq. (10.6))
D̃ (0, k) = ε0 ε̃ (0, k) Ẽ (0, k)
e trasformando le equazioni di Poisson per D̃ ed Ẽ
enind (r) − qρnext (r)
ε0
1
1
qnext (r)
− ∇ · D̃ = − ∇ · ε̂Ẽ = ∇2 Vext = −
ε0
ε0
ε0
nello spazio k otteniamo
−∇ · Ẽ = ∇2 V =
eñind (0, k) − qñext (0, k)
eñind (0, k)
=
+ k 2 Ṽext (0, k)
ε0
ε0
qñext (0, k)
= k 2 ε̃ (0, k) Ṽ (0, k)
k 2 Ṽext (0, k) = −
ε0
k 2 Ṽ (0, k) =
146
Relazioni costitutive
possiamo definire ε̃ (0, k) come rapporto tra i due potenziali
ε̃−1 (0, k) =
Ṽ (0, k)
Ṽext (0, k)
Nel modello di Fermi-Thomas relativo al gas di elettroni di un metallo si ha
e
nind (r) = −κ2 V (r)
ε0
(5.4)
per cui
κ2
k2
Il modello di Fermi-Thomas può essere migliorato sostituendo la (5.4) con l’espressione
più accurata
ñind (0, k) = χ (k) Ṽ (0, k)
ε̃ (0, k) = 1 +
ovvero sostituendo la costante di schermaggio κ con una funzione χ (k) di k,
χ (k) =
e2
4π 3
fq− 1 k − fq+ 1 k
2
2
2
k·q
m
d3 q
in cui fp sta per la funzione di distribuzione di Fermi-Dirac di elettroni liberi
2 2
fp = fF
p kεFT
,e b ,T
2m
1
=
exp
2 p2 /2m−ε
kb T
F
+1
Ne segue che ε̃ (0, k).è legata a χ (k) dalla relazione
ε̃ (0, k) = 1 − 4π
χ (k)
k2
(5.5)
In particolare, Lindhard3 ha mostrato che a T = 0 χ (k) è espressa da
χ (k) = −e2
mkF
2 π2
F
k
2kF
con F (x) la funzione di correlazione delle coppie di elettroni discussa nel Cap. 9 Eq.
(9.35) .
5.4
Oscillatori di Lorentz
5.5
Relazioni di dispersione di Kramers-Kronig
La polarizzabilità α̃p = α̃′p − iα̃′′p è una funzione analitica di ω con poli nel semipiano
Im (ω) < 0. Infatti un’eccitazione alla frequenza di un polo ̟ induce una polarizzazione
∝e−i̟t . Poichè e−i̟t non può esplodere per t → ∞ la parte immaginaria di ̟ deve
3
J. Lindhard, Kgl. Danske Videnskab. Selskab Mat.-Fys. Medd 28, No. 8 (1954); G. Giuliani and G.
Vignale, Quantum Theory of the Electron Liquid, Cambridge Univ. Press 2005
5.5 Relazioni di dispersione di Kramers-Kronig
147
risultare negativa. Pertanto, possiamo esprimere α̃p (ω) in funzione di α̃p (z) relativo ad
un cammino chiuso contenuto nel semipiano Im (ω) > 0 utilizzando il teorema di Cauchy4 ,
!
α̃p (z)
1
α̃p (ω) =
dz .
2πi
ω−z
Se α̃p (z) → 0 per |z| → ∞ nel semipiano superiore, quest’ultimo integrale si riduce alla
coppia di relazioni per α̃′p (ω) ed α̃′′p (ω)
1
V.P
π
1
α̃′′p (ω) =
V.P.
π
α̃′p (ω) =
α̃′′p (ω ′ ) ′
dω
′
−∞ ω − ω
∞ α̃′ (ω ′ )
p
dω ′
′
−∞ ω − ω
∞
(5.6)
dove V.P. rappresenta il valor principale dell’integrale 5 , note come relazioni di dispersione di Kramers-Kronig.
Esercizio 5.5.1 Mostrare che la costante dielettrica ε̃ (ω) si può esprimere nella forma
1
= 1−i
ε̃ (ω)
con ǫ → 0 e
∞
−∞
f˜ (ω ′ )
dω ′
ω + iǫ − ω ′
f˜∗ (ω ′ ) = f˜ (−ω ′ )
N.B. Si tenga conto dell’identità
lim Im
ǫ→0
1
= δ (x)
x − iǫ
∞
Per semplicità conviene assumere che f (t) = −∞ f˜ (ω) e−iωt dω = f (−t) il che equivale
a porre
f˜ (ω) = iω Ṽ (ω)
con Ṽ (ω) = Ṽ (−ω) e reale. Ne segue
∞
ω ′2 Ṽ (ω ′ )
′
2 dω
′2
−∞ ω − (ω + iǫ)
1
=1−
ε̃ (ω)
Imponendo la condizione
Im (ε̃) ≥ 0 ω ≥ 0
segue che
D’altra parte per ω ≫ ω atom si ha
∞
Ṽ (ω) ≥ 0
ω ′2 Ṽ (ω ′ )
1
dω ′ ≈ − 2
2
′2
ω
−∞ ω − (ω + iǫ)
4
5
∞
−∞
ω ′2 Ṽ (ω ′ ) dω ′ ≈ −
ω 2p
4πne e2
=
−
me ω 2
ω2
v. p. e. D.B.Melrose and R.C.McPhedran, Electromagnetic Processes in Dispersive Media, Cambridge
Univ. Pres, Cambridge 1991
b f (x)
ω−ǫ
b
f (x)
V.P . a x−ω
dx = lim a + ω+ǫ x−ω
dx
ǫ→0
148
Relazioni costitutive
ovvero
∞
ω ′2 Ṽ (ω ′ ) dω ′ =
−∞
Pertanto ridefinendo Ṽ
ω ′2 Ṽ (ω ′ ) =
con
ne e2
me
ne e2
q (ω ′ )
me
q (ω ′ ) ≥ 0 , ω ′ > 0
e
∞
q (ω ′ ) dω ′ = 1
0
si ha
1
= 1 − ω 2p
ε̃ (ω)
In particolare
1
Im
ε̃
5.6
=
−ω 2p
∞
0
∞
0
q (ω ′ )
′
2 dω
′2
ω − (ω + iǫ)
q (ω ′ )
π ω 2p
′
dω =
q (ω)
2 ω
ω ′2 − (ω + iǫ)2
Mezzi anisotropi
Esercizio 5.6.1 Calcolare l’indice di rifrazione ñ ω, k̂
eare:
di un materiale a risposta lin-
D̃ (kµ ) = ε0 ε̃ ω, k̂ · Ẽ (kµ ) ,
H̃ (kµ ) = µ−1
0 B (kµ ) .
(5.7)
Soluzione: Dalla trasformata k µ del campo Ẽ (2.27-b) applicando l’identità k × k× =
kk − k 2 si ottiene per M = 0 l’equazione d’onda di Helmholtz
ñ2 k̂ k̂ + ε̃ − ñ2 · Ẽ = −i
c
µ J̃ .
kv 0
(5.8)
In particolare in assenza di correnti questa equazione fissa la dipendenza di ñ k̂,ω
da ω e k̂. Infatti, per k̂·Ẽ= 0 essa implica che
k̂ ·
1
ñ2 ω, k̂ − ε̃ ω, k̂
· k̂ =
1
ñ2 ω, k̂
.
(5.9)
A questa relazione vien dato il nome di equazione di Fresnel. Una volta assegnata la
direzione di propagazione k̂ e la frequenza ω l’indice di rifrazione ñ ω, k̂ si ottiene
risolvendo quest’ultima equazione di dispersione.
Esercizio 5.6.2 Analizzare i modi di propagazione di un mezzo dielettrico anisotropo
caratterizzato da un tensore dielettrico ε̂ e da una permeabilità magnetica scalare µ̂.
5.6 Mezzi anisotropi
149
Soluzione: Per un tale mezzo i potenziali A,V del campo soddisfano le equazioni
−∇∇ · A + ∇2 A −
µ̂ ∂ 2
µ̂
∂
ε̂ · A − 2 ε̂·∇ V
2
2
c ∂t
c
∂t
∂
∇ · ε̂·
A + ∇V
∂t
= 0
= 0
Trasformando A e V nel dominio k,ω
A (r, t) = Ã (k, ω) e−i(ωt−k·r)
V (r, t) = Ṽ (k, ω) e−i(ωt−k·r)
le precedenti equazioni diventano
µ̃
ε̂ · kṼ
c2
k · ε̃· −iω Ã + ikṼ
kk − k 2 + k02 µ̃ε̃ ·Ã − ω
= 0
= 0
da cui
Ṽ = ω
Pertanto,
k · ε̃ · Ã
k · ε̃ · k
Ẽ = iω Ã − ∇Ṽ = iω Ã − ikṼ = i
e
D̃ = iω ε̃ −
Ne segue che
−
i
ω
ε̃ −
ω
((k · ε̃ · k) − kk · ε̃) ·Ã
k · ε̃ · k
ε̃ · kk · ε̃
k · ε̃ · k
ε̃ · kk · ε̃
k · ε̃ · k
−1
·Ã
· D̃ = Ã
ovvero la relazione D̃ ↔ Ã dipende dalla diade
D̃Ã = ε̃ −
D’altra parte
D̃Ã · k = ε̃ · k −
ε̃ · kk · ε̃
k · ε̃ · k
ε̃ · kk · ε̃
k · ε̃ · k
·k = ε̃ · k 1 −
k · ε̃ · k
k · ε̃ · k
=0
ovvero se si decompone à parallelamente e perpendicolarmente a k à = à + Ã⊥ si
ha
D̃ = iω D̃÷ à + Ã⊥ = iω D̃à · Ã⊥
Esercizio 5.6.3 Analizzare i modi di propagazione di un mezzo anisotropo caratterizzato
da un tensore dielettrico ε̂ e da uno magnetico µ̂.
150
Relazioni costitutive
Soluzione: Per un materiale magnetico anisotropo si ha
−∇ × µ̂−1 ·∇×A − µ0 ε0
∂2
∂
ε̂ · A − µ0 ε0 ε̂·∇V
2
∂t
∂t
∂
∇ · ε̂· A + ∇ · ε̂·∇V
∂t
= 0
= 0
Quando ε̂ e µ̂ si riducono a degli scalari tenendo conto della identità vettoriale ∇ ×
(∇ × a) = ∇ (∇ · a) − ∇2 a il precedente sistema si riduce a
µ̂ ∂ 2
µ̂
∂
−∇∇ · A + ∇ A − 2 2 ε̂ · A − 2 ε̂·∇ V
c ∂t
c
∂t
∂
∇ · ε̂·
A + ∇V
∂t
2
5.7
= −µ0 µ̂J
= −
ρ
ε0
Isolatore di Faraday
Esercizio 5.7.1 Analizzare i modi di propagazione di un mezzo girotropico descritto dal
tensore dielettrico


a −ig 0
ε̃ =  ig a 0  ,
(5.10)
0
0 b
con
ω 2pe
ω 2pi
ω 2pe ω 2pi
a = 1− 2
−
, b=1− 2 − 2 ,
ω − ω 2ge ω 2 − ω 2gi
ω
ω
g =
ω 2pe ω ge
ω 2pi ω gi
+
,
ω ω 2 − ω 2ge
ω ω 2 − ω 2gi
Soluzione: Il tensore dielettrico ε̃ del plasma magnetizzato ha 3 autovalori b, a−g, a+g
corrispondenti rispettivamente agli autovettori ẑ, ix̂ + ŷ, −ix̂ + ŷ. Quindi, lungo l’asse di
magnetizzazione (z) si propagano con velocità diverse onde polarizzate circolarmente in
senso orario ed antiorario6
√
√
ε0 µ0 a−gz)
E (z) = E+ (ix̂ + ŷ) e−i(ωt−
√
√
ε0 µ0 a+gz)
+ E− (−ix̂ + ŷ) e−i(ωt−
,
Se l’onda è polarizzata linearmente lungo ŷ all’ascissa z = 0 (E+ = E− ) dopo aver percorso
un tratto l risulterà ancora polarizzata linearmente e ruotata rispetto ad ŷ di un angolo
θ (l) =
√
√
1√
ε0 µ0 a + g − a − g l ≃ V Bl .
2
Simili fenomeni, oltre che nei plasmi, si presentano in molti dielettrici ed in tal caso V
prende il nome di costante di Verdet.
Se la stessa onda vien fatta propagare nel verso opposto ritornerà al punto di partenza
formando con ŷ un angolo 2θ (l) . Ne discende che la propagazione in questi mezzi non
è invertibile. Ciò è connesso alla mancanza di simmetria per inversione temporale delle
6
v.p.e. A. Yariv and P. Yeh, Optical Waves in Crystals: Propagation and Control of Laser Radiation,
J. Wiley, N.Y. 2002, Sez. 4.10.
5.8 Mezzi periodici
151
equazioni di Maxwell. Poichè il tensore di Eq. (5.10) non è simmetrico, l’Eq. (4.2)
non è valida. Questa proprietà viene sfruttata negli isolatori di Faraday. Questi dispositivi, costituiti da opportuni materiali a cui viene applicato un forte campo magnetico
B0 costante, sono progettati in modo che un fascio laser, che si propaghi parallelamente
a B0 , polarizzato linearmente all’ingesso A si presenti all’uscita B con la polarizzazione
ruotata di 450 . Iniettando in B il fascio trasmesso questo si presenterà in A polarizzato
a 900 e sarà quindi bloccato dal polarizzatore posto all’ingresso7 . Per queste applicazioni
si utilizzano dei materiali, quali i granati di terbio e gallio (TGG) che presentano una V
particolarmente elevata (≈ 40 rad T −1 m−1 ).
5.8
Mezzi periodici
Esercizio 5.8.1 Analizzare i modi di propagazione per un mezzo con funzione dielettrica
ε̃ (r) scalare e periodica
Soluzione: Tenuto conto che il campo Ẽ (r,ω) soddisfa l’equazione d’onda
∇2 + ∇ ln ε̃ · ∇ + ∇∇ ln ε̃ · +ω 2 µ0 ε̃ Ẽ (r,ω) = 0 .
mentre per un materiale magnetico anisotropo si ha
−∇ × µ̂−1 · ∇ × A − µ0 ε0
∂2
∂
ε̂
·
A
−
µ
ε
ε̂·∇V
0
0
∂t2
∂t
∂
∇ · ε̂· A + ∇ · ε̂·∇V
∂t
= −µ0 µ̂J
= −
ρ
ε0
Quando ε̂ e µ̂ si riducono a degli scalari tenendo conto della identità vettoriale ∇ ×
(∇ × a) = ∇ (∇ · a) − ∇2 a il precedente sistema si riduce a
−∇∇ · A + ∇2 A −
5.9
µ̂ ∂ 2
µ̂
∂
ε̂ · A − 2 ε̂·∇ V
2
2
c ∂t
c
∂t
∂
∇ · ε̂·
A + ∇V
∂t
= −µ0 µ̂J
= −
ρ
ε0
Pacchetti d’onda in mezzi dispersivi.
Per mezzi che presentano sia dispersione nel tempo che nello spazio il campo elettrico può
essere posto nella forma
E (r, t) =
1
(2π)3
e−i(ωk t−k·r) Ẽ (k, ω k ) d3 k ,
dove ω k è soluzione di un’equazione di dispersione del tipo della (5.9). Spesso si utilizzano
impulsi e.m. i cui pacchetti d’onda presentino uno spettro Ẽ (k, ω k ) diverso da 0 in un
intorno di k0 sufficientemente stretto da giustificare le approssimazioni:
1
ω k = ω 0 + vg · ∆k + D : ∆k∆k ,
2
7
Questo effetto fu scoperto da M. Faraday nel 1845 e fornì la prima evidenza sperimentale della connessione tra luce ed effetti magnetici. v.p.e. D. Budker, D. F. Kimball and D. P. DeMille, Atomic
Physics Oxford Univ. Press, Oxford, 2008, Prob. 4.1.
152
Relazioni costitutive
con ∆k = k − k0 , vg la velocità di gruppo e D la matrice di diffusione rappresentata
dall’hessiano della relazione di dispersione ω k ,
ω 0 = ω k0 , vg = ∇k ω k |k=k0 , D = ∇k ∇k ω k |k=k0 .
Il simbolo “:” indica il doppio prodotto. Il campo E (r, t) ha la forma di un pacchetto
d’onda
E (r, t) = e−i(ω0 t−k0 ·r) E0 (r, t) ,
con E0 (r, t) che varia lentamente nello spazio rispetto alla lunghezza d’onda e nel tempo
rispetto al periodo 2π/ω 0 , traslando con la velocità di gruppo vg ed allargandosi in accordo
con Eq. (5.12) e con la matrice di diffusione D.
1
(2π)3
1
≃
(2π)3
eik·r−i(ωk+k0 −ωk0 )t Ẽ0 (k) d3 k
E (r, t) =
i
eik·(r−vg t) e− 2 tD:kk Ẽ0 (k) (k) d3 k ,
Si verifica facilmente utilizzando quest’ultima rappresentazione di E0 (r, t) che
∂
i
+ vg · ∇ + D :∇∇ E0 (r, t) = 0 .
∂t
2
Cambiando riferimento da r a r′ = r − vg t, l’ultima equazione diviene
∂
i
+ D :∇′ ∇′ E0 (r′ , t) = 0 .
∂t 2
Trasformando r′ in X = D−1/2 · r′ si vede facilmente che E0 (X; t) = E0 (r′ , t) è soluzione
di un’equazione simile a quella di diffusione (??) salvo per la presenza del coefficiente i,
i
∂
1
E0 (X, t) = ∇2 E0 (X, t) .
∂t
2
Pertanto, Φ (X; t) è legata a Φ (X; 0) dalla trasformazione integrale (5.11) con Gdif f (r, Dt)
(v. Eq. (??)) sostituito da √ 1 Gdif f |X| , 2i t
det D
E0 D−1/2 · r, t =
1
det !D!
Gdif f
con
Gdif f
i
r, t
2
i
D−1/2 · (r − r′ ) , t E0 D−1/2 · r′ , 0 (X; 0) d3 r′ ,
2
(5.11)
=
1
t3/2
exp i
r2
2t
.
. Ne segue che:
E0 (r, t) =
1
det !D!
Gdif f
i
D−1/2 · (r − vg t − r′ ) , t E0 (r′ , 0) d3 r′ .
2
(5.12)
Nell’introdurre la trasformazione X = D−1/2 · r′ si è implicitamente assunto che gli
autovalori di D fossero positivi. E’ facile verificare che quest’ultima relazione non perde
di significato quando questi risultano negativi.
5.10 Mezzi non-lineari: Effetto Kerr
153
Nel caso di un inviluppo gaussiano 1D A (z ′ , 0) = exp (−z ′2 /2σ 20 ) si trasforma al tempo
t in
√
2π
z ′2
′
exp −
E0 (z , t) =
t
2 (σ 20 + iDt)
− Di
σ2
0
= exp −
con
σ (t) =
5.10
σ 20 +
D 2 t2
, R (t) =
σ 20
σ 40
+ Dt .
Dt
Mezzi non-lineari: Effetto Kerr
8
5.10.0.1
8
compressione impulsi. Impulsi ulttracorti
v.p.e. D. Budker et al. loc. cit. pag. 151, Prob. 4.2.
z ′2
z ′2
+
i
2σ 2 (t)
2R2 (t)
,
Capitolo 6
Elettrodinamica
6.1
Teoria delle orbite
Un elettrone in un campo e.m. è descritto dall’equazione del moto
d
e
(γv) = −
(E + v×B)
dt
me
con
γ=
(6.1)
1
1 − v2 /c2
il fattore relativistico. Dalla (6.1) discende l’equazione dell’energia
me c2
d
γ = −eE · v
dt
(6.2)
Esprimendo i campi mediante i potenziali vettore A (r, t) e scalare V (r, t) il moto è
retto dalla Lagrangiana
1
L = − me c2 − ev · A + eV
(6.3)
γ
o equivalentemente dall’Hamiltoniana
H = v · p − L = γme c2 − eV
dove
pi =
∂
L = γme vi − eAi
∂vi
(6.4)
Pertanto
H = γme c2 − eV
(6.5)
Esercizio 6.1.1 Si consideri un elettrone relativistico in un campo magnetico statico. (a)
Dimostrare che il fattore γ è un integrale del moto e (b) esprimere γ = γ (r, p) in funzione
delle variabili coniugate r, p
Soluzione: (a) In vista di (6.5) in assenza di un campo elettrico H è data da
H = γme c2
155
156
Elettrodinamica
dove
γ=
1
1 − β2
Poichè γ è proporzionale a H, questi commutano tra loro
{γ, H} = 0
Ne segue che γ è un integrale del moto.
(b) In vista della (6.4)
p + eA = me γv
si ha
1
γ=
1−
|p+eA|
me γc
2
Pertanto
γ=
1+
|p + eA|
me c
2
(6.6)
Esercizio 6.1.2 Il moto dell’elettrone che orbita attorno ad un nucleo di carica Z si può
descrivere o nel sistema di riferimento proprio KP (t) , o in quello di laboratorio KL . Al
tempo τ /c che vede scorrere un osservatore solidale con KP (t) si dà il nome di tempo
proprio. L’elettrone, che si muove nel potenziale Coulombiano
VN = −
con
α=
e2 Z
α cZ
=−
4πεo r
r
e2
1
=
,
4πε0 c
137.035989
(6.7)
costante di struttura fine, è descritto dalla Lagrangiana relativistica
L=−
1 2
1
2
ṫ − 2 ṙ2 + r2 φ̇
2
c
+
VN
ṫ ,
me c2
dove si è indicato con un punto le derivate rispetto al tempo proprio τ (v. 2.1) e con
r, φ le coordinate polari nel piano dell’orbita descritta. Ricavare l’orbita descritta da un
elettrone utilizzando questa Lagrangiana
Soluzione: Dalle equazioni del moto
∂
d ∂
L− L = 0,
dτ ∂ ṙ
∂r
d ∂
∂
L−
L = 0,
dτ ∂ φ̇
∂φ
d ∂
∂
L− L = 0,
dτ ∂ ṫ
∂t
discende che:
6.1 Teoria delle orbite
157
Figura 6.1: Moto a rosetta di un elettrone in un’orbita ellittica di Bohr. Dopo una
rivoluzione completa attorno al nucleo l’asse maggiore risulta ruotato di un angolo ∆φ =
π (αZ/k)2 , con α costante di struttura fine.
158
Elettrodinamica
d
VN
2
ṙ − rφ̇ +
ṫ = 0 ,
dτ
me r
d 2
r φ̇ = 0 ,
dτ
VN
d
−
+ ṫ
= 0,
dτ
mec2
Questo sistema è caratterizato dai due integrali del moto:
me r2 φ̇ =
−
k,
VN
+ ṫ = A ,
me c2
(6.8)
Pertanto utilizzando la trasformazione
k 1 d
d
=
dτ
me r2 dφ
e sostituendo r con u = 1/r si ottiene
"
Zα
d2
1−
2u +
k
dφ
con
Integrando si ottiene
2
#
1
me
=
a (1 − ǫ2 )
k
1
=
r
1 + ǫ cos φ
u=
1
,
a (1 − ǫ2 )
VN
A.
me u
2
1−
Zα 2
k
a (1 − ǫ2 )
,
(6.9)
Si vede quindi che l’elettrone descrive un moto a rosetta (v. Fig. ??), con orbite ellittiche
di eccentricità ǫ ed asse maggiore a ruotante. Per una rotazione completa dell’elettrone
attorno al nucleo, corrispondente ad un incremento di φ pari a 2π, l’asse maggiore ruota
dell’angolo


2
2
Zα 
Zα

δφ = 2π 1 − 1 −
≃π
k
k
6.1.1
Lenti elettrostatiche e magnetostatiche
Il potere risolutivo di un microscopio è inversamente proporzionale alla lunghezza d´onda
della radiazione utilizzata. Nel 1931 i tedeschi Ernst Ruska e Max Knoll pensarono
di sostituire le onde e.m. con fasci di elettroni1 . Questi si comportano come onde di
lunghezza d’onda pari a quella di de Broglie:
1
=
λe
1
2 (γ − 1)
me c
h
v.p.e. O. Klemperer and H. E. Barnett, ”Electron Optics”, Cambridge Univ. Press, 1971; P. W.
Hawkes, ”Magnetic Electron Lenses”, Springer Verlag, N.Y. 1982
6.1 Teoria delle orbite
159
Figura 6.2: Microscopio elettronico a scansione. Un fascio di elettroni vien messo a fuoco
sul campione. Dalla regione di impatto del fascio focalizzato vengono sia emessi elettroni
secondari che riflessi parte di quelli incidenti. I rivelatori posti a lato raccolgono questi
elettroni e ne misurano le energie. da www.astarmathsandphysics.com
Utilizzando fasci elettronici (v. Fig. (6.2)) con energia γ sufficientemente elevata si
raggiungono risoluzioni di parecchi ordini di grandezza superiore a quella dei dispositivi
ottici2 .
Esercizio 6.1.3 Si scriva l’equazione del moto di un elettrone nel campo creato da una
lente elettrostatica (v. Figg. 6.3 e 6.4). Questa è caratterizzata da un’asse z e da un
potenziale V (ρ, z) indipendente dalla coordinata φ. (a) Scrivere le equazioni del moto di
un elettrone utilizzando le equazioni di Lagrange. (b) Discutere ed integrare le equazioni
del moto per traiettorie che non si allontanano molto dall’asse z.
2
v.p.e. L. Artsimovitch et S. Loukianov, Mouvement des particules chargées dans des champs électriques
et magnétique, Ed. MIR Moscou 1975
160
Elettrodinamica
Figura 6.3: Rappresentazione schematica di una lente elettrostatica
Figura 6.4: Schemi di lenti elettrostatiche costituite (in alto) da una coppia di diaframmi
a potenziali diversi e (in baso) da cilindri coessiali da encyclopedia2.thefreedictionary.com
6.1 Teoria delle orbite
161
Figura 6.5: Focalizzazione degli elettroni emessi da da un catado mediante una lente
elettronica formata da due diaframmi circolari. La famiglia di curve rappresenta le sezioni
delle superfici equipotenziali da encyclopedia2.thefreedictionary.com
Soluzione: Dalle equazioni di Eulero-Lagrange
∂
d ∂
L−e V = 0
dt ∂ ż
∂z
d ∂
∂
L−e V = 0
dt ∂ ρ̇
∂ρ
d ∂
L = 0
dt ∂ φ̇
e dalla Lagrangiana di Eq. (6.3)
∂
L = −me γ ż
∂ ż
∂
L = me γ ρ̇
∂ ρ̇
∂
L = me γρ2 φ̇
∂ φ̇
discende
d
e ∂
(γ ż) = −
V
dt
me ∂z
d
e ∂
(γ ρ̇) =
V
dt
me ∂ρ
d
γρ2 φ̇
dt
= 0
Dall’ultima equazione discende che l’obita ruota attorno all’asse z con velocità angolare
inversamente proporzionale a ρ2
162
Elettrodinamica
Esprimendo ρ in funzione di z si ottiene la seguente equazione della traiettoria:
d2
2V 2 ρ =
dz
d ∂
∂
V − ρ V
∂ρ
dz ∂z
1+
d
ρ
dz
2
(6.10)
(b) Il potenziale soddisfa l’equazione di Laplace
∂
1 ∂
ρ
ρ ∂ρ
∂ρ
+
∂2
V =0
∂z 2
(6.11)
In prossimità dell’asse z V può essere convenientemente espresso nella forma
Vn (z) ρ2n
V (ρ, z) =
n
In vista della (6.11) si ha che i coefficienti Vn (z) sono legati tra loro dalla relazione di
ricorrenza
1
d2
Vn+1 (z) = −
Vn (z)
(2n + 2)2 dz 2
Pertanto V (ρ, z) si può esprimere nella forma
(−1)n (2n)
ρ
(z)
2 V0
2
(n!)
V (ρ, z) =
n
Sostituendo nella (6.10)
∂
V
∂z
con V0′ =
d
V
dz 0
2n
(6.12)
e tenendo conto della relazione
1
∂
V ≃ − V0′′ ρ
∂ρ
2
l’Eq. (6.10) si riduce a
d2
1 V0′ d
1 V0′′
+
+
dz 2 2 V0 dz 4 V0
che può essere riscritta nella forma
3
d2
+
2
dz
16
d
ln V0
dz
ρ=0
(6.13)
1/4
(6.14)
2
ρV0
=0.
Esercizio 6.1.4 Si consideri una lente elettrostatica costituita da due cilindri coassiali di
raggio a che si estendono rispettivamente da (−∞, 0) e (0, ∞), separati da una piccola
gap (v. Fig. 6.4-b). In tal caso si dimostra che il potenziale3 lungo l’asse è espresso da
V0 (z) = V1 tanh 1.32
z
a
Analizzare l’equazione delle traiettorie parassiali. (b) Calcolare la lunghezza focale.
Soluzione: La (6.14) si specializza in:
d2
3
1
2 +
16 sinh2 ζ
dζ
dove ζ =
2.64
z
a
1/4
ρV0
=0.
6.1 Teoria delle orbite
163
Figura 6.6: Rappresentazione schematica di un quadrupolo elettrico costituito da 4
cililindri posti a potenziali alternativamente pari a V e -V
Figura 6.7: Superfici equipotenziali per un quadrupolo elettrico
164
Elettrodinamica
Esercizio 6.1.5 Si consideri una lente elettrostatica costituita da quattro elettrodi formanti un quadrupolo (v. Fig. 6.6). (a) Calcolare il potenziale in prossimità dell’asse z
(v. Fig. 6.7) e (b) Analizzare le traiettorie parassiali
Soluzione: (a) Il potenziale
Vm ρ2 , z ei2mφ
V =
m
soddisfa l’equazione di Laplace. Pertanto
∂
∂ρ2
ρ2
∂
∂ρ2
−
m2 1 ∂ 2
Vm = 0
+
ρ2
4 ∂z 2
(6.15)
In prossimità dell’asse z Vm può essere convenientemente espresso nella forma
Vm ρ2 , z =
Vnm (z) ρ2n
n
doveIn vista della (6.11) si ha che i coefficienti Vnm (z) sono legati tra loro dalla equazione
n
1 ′′
Vnm n2 − m2 + Vn−1,m
ρ2n−2 = 0
4
da cui discende la relazione di ricorrenza

n
′′
2
Vn−1,m

Vnm =
=
4 (m2 − n2 )
q=|m|+1
Pertanto
V (ρ, φ, z) =
m
n


n
2
q=|m|+1

1
 V (2(n−|m|))
|m|,m
− q2)
4 (m2

1
 V (2(n−|m|)) ρ2n ei2mφ
|m|,m
4 (m2 − q 2 )
Scegliendo opportunamente gli assi trasversi si ha
V (ρ, φ, z) =
V11 ρ2 + V21 ρ4 + · · · cos 2φ
+ V22 ρ4 + V23 ρ6 + · · · cos 4φ + · · ·
(b) Dalle equazioni di Eulero-Lagrange
d ∂
∂
L−e V
dt ∂ ż
∂z
d ∂
∂
L−e V
dt ∂ ρ̇
∂ρ
d ∂
∂
L−e V
dt ∂ φ̇
∂φ
3
= 0
= 0
= 0
v.p.e. S. Bertram, Proc. I.R.E. 28, 418 (1940); J. Appl. Phys. 13, 496 (1942)
6.1 Teoria delle orbite
165
e dalla Lagrangiana di Eq. (6.3)
∂
L = −me γ ż
∂ ż
∂
L = me γ ρ̇
∂ ρ̇
∂
L = me γρ2 φ̇
∂ φ̇
discende
d
e ∂
(γ ż) = −
V
dt
me ∂z
e ∂
d
(γ ρ̇) =
V
dt
me ∂ρ
d
e ∂
γρ2 φ̇ =
V
dt
me ∂φ
Approssimando V (ρ, φ, z) con
V (ρ, φ, z) = V11 ρ2 cos 2φ
si ha
d
e
(γ ż) = − V11′ ρ2 cos 2φ
dt
me
d
e
(γ ρ̇) = 2 V11 ρ cos 2φ
dt
me
d
e
γρ2 φ̇ = 2 V11 ρ2 sin 2φ
dt
me
Per velocità non relativistiche si ha:
e
z̈ = − V11′ ρ2 cos 2φ
me
e
ρ̈ = 2 V11 ρ cos 2φ
me
d
e
ρ2 φ̇ = 2 V11 ρ2 sin 2φ
dt
me
Esercizio 6.1.6 Si scriva l’equazione del moto di un elettrone in una lente magnetostatica
(v. Figg. 6.8 e 6.9). (a) Derivare le equazioni del moto (b) Esaminare il caso in cui
A = (0, 0, Aφ ).
Soluzione: (a) In vista delle equazioni di Hamilton
∂
∂
H = −ṗi ,
H = q̇i
∂qi
∂pi
e tenendo conto dell’espressione (6.6) si ha:
∂
∂qi
1+
|p + eA|
me c
2
me c2
∂
∂pi
1+
|p + eA|
me c
2
me c2
= −ṗi
= q̇i
166
Elettrodinamica
Figura 6.8: Rappresentazione schematica di una lente magnetica
Figura 6.9: Rappresentazione schematica della forza agente su un elettrone mentre
attraversa una lente magnetica
6.1 Teoria delle orbite
167
ovvero:
e
(p + 2eA) · ∇A = −ṗ
γme
1
(2p + eA) = v
γme
(b) Per A = (0, 0, Aφ ) si ha:
∂Aφ
∂Aφ
φ̂ρ̂ +
φ̂ẑ
∂ρ
∂z
∂Aφ
∂Aφ
p · ∇A = pφ
ρ̂ +
ẑ
∂ρ
∂z
∂Aφ
∂Aφ
A · ∇A = Aφ
ρ̂ +
ẑ
∂ρ
∂z
∇A =
Pertanto
e
(pφ + 2Aφ )
γme
∂Aφ
∂Aφ
ρ̂ +
ẑ
∂ρ
∂z
1
2p + eAφ φ̂
γme
= −ṗ
= q̇
In particolare
e
∂Aφ
(pφ (0) + 2eAφ )
γme
∂z
e
∂Aφ
= −
(pφ (0) + 2eAφ )
γme
∂ρ
= 0
ṗz = −
ṗρ
ṗφ
2
pz = ż
γme
2
pρ = ρ̇
γme
1
(2pφ (0) + eAφ ) = ρφ̇
γme
Combinando la prima con la quarta equazione e la seconda con la quinta si ottiene
2
2e ∂Aφ
ṗz = − 2 2
(pφ (0) + 2eAφ )
γme
γ me ∂z
2
2e ∂Aφ
ρ̈ =
ṗρ = − 2 2
(pφ (0) + 2eAφ )
γme
γ me ∂ρ
z̈ =
(c) Dall’identità vettoriale
∇ × ∇× = ∇∇ · −∇2
segue che
∇ × ∇ × A = µ0 J = ∇∇ · A − ∇2 A = −∇2 A
∇ × ∇ × B = µ0 ∇ × J = ∇∇ · B − ∇2 B = −∇2 B
(6.16)
168
Elettrodinamica
ovvero in assenza di J A e B soddisfano l’equazione di Laplace:
∇2 A = ∇2 B = 0
Per campi indipendenti da φ si ha per le componenti Aφ e Bz :
1 ∂ ∂
1
∂2
ρ − 2 + 2 Aφ = 0
ρ ∂ρ ∂ρ ρ
∂z
∂2
1 ∂ ∂
ρ +
Bz = 0
ρ ∂ρ ∂ρ ∂z 2
Pertanto, in analogia col precedente esercizio si può espandere Bz e Aφ in serie di potenze
di ρ:
Bz (ρ, z) =
∞
Bn (z) ρ2n
n=0
∞
ρ
(−1)n (2n)
(z)
2 B0
2
(n!)
n=0
ρ 2
= B0 (z) − B0′′ (z)
2
=
2n
dove B0 (z) = Bz (0, z).
In vista della relazione
∇·B=0
o equivalentemente
1 ∂
∂
(ρBρ ) + Bz = 0
ρ ∂ρ
∂z
si ha
ρ
ρ3
Bρ = − B0′ (z) + B0′′′ (z) + · · ·
2
16
Ragionando in termini di Aφ (ρ, z) si ottiene
∞
(−1)n
ρ
B0(2n) (z)
2
2
(n!) (n + 1)
n=0
ρ 1
ρ 3
= B0 (z) − B0′′ (z)
2 2
2
Aφ (ρ, z) =
2n+1
Sostituendo queste espansioni in (6.16) si perviene all’equazione del moto:
e
2
γ m2e
e
ρ̈ = − 2 2
γ me
z̈ = −
B0′ ρ − B0′′′
3
B0 − B0′′ ρ2
8
ρ
2
3
ρ
2
pφ (0) + e B0 ρ − B0′′
pφ (0) + e B0 ρ − B0′′
Infine, trascurando termini di ordine superiore a ρ si ha
e
z̈ = − 2 2 pφ (0) B0′ ρ
γ me
e
ρ̈ = − 2 2 B0 (pφ (0) + eB0 ρ)
γ me
ρ
2
3
3
6.1 Teoria delle orbite
169
Per pφ (0) = 0 si ha
z̈ = 0
ρ̈ = −
ovvero
Per γ ∼ 1
2
eB0
γme
ρ
d2
1 1
ρ
+
dz 2
c2 γ 2 − 1
eB0
γme
2
d2
1
1
ρ
+
dz 2
2c2 γ − 1
eB0
me
2
ρ=0
ρ=0
Esprimendo l’energia me c2 (γ − 1) in eV si ottiene
d2
e B02
ρ
+
ρ=0
dz 2
2me V
(6.17)
ovvero
2
d2
10 B0
ρ=0
ρ
+
2.20
×
10
dz 2
V
Esercizio 6.1.7 Con riferimento all’esercizio precedente calcolare (a) la lunghezza focale
di una lente magnetica costituita da una spira circolare perpendicolare all’asse z e centro
in z e (b) l’angolo di rotazione φ.
Soluzione: In prossimità dell’asse z normale ad una lente magnetica formata da una
spira di raggio a attraversata dalla corrente I, il potenziale vettore Aφ (z, ρ) è dato da (v.
Eq. (3.16))
Aφ (z, ρ) = µ0 If (z) ρ
B (z, 0) = B0 (z) = 2µ0 If (z)
con
f (z) =
1
a2
4 (a2 + z 2 )3/2
Integrando l’equazione parassiale (6.17) della traiettoria per un elettrone proveveniente
da z = −∞ parallelamente all’asse si ha:
d
ρ
dz
z=∞
∞
e
=−
8meV
−∞
B02 ρdz
D’altra parte se si trascura la variazione di ρ (z) nel tratto in cui B02 (z) = 0 si ottiene:
d
ρ
dz
z=∞
=−
e
ρ (−∞)
8me V
∞
−∞
B02 dz ≡ −
1
ρ (−∞)
f1
dove f1 rappresenta la distanza tra le intersezioni della traiettoria rettilinea d’uscita con
l’asse z e con la traiettoria di ingresso; ovvero f1 rappresenta la lunghezza focale. In
particolare per la spira si ha
∞
1
e
eµ20 I 2 1
2
−
=
B dz =
f1
8me V −∞ 0
2me V 16a
2 2
I2
eµ0 I 1 3
π=
=
2me V 16a 8
97.9V a
∞
−∞
1
dz
(1 + z 2 )3
(6.18)
170
Elettrodinamica
Tenuto conto che
φ̇ =
si ottiene
1
φ= √
2 V
∞
e
2me
e
B0
2γme
B0 dz =
−∞
1.480 × 105
√
V
∞
B0 dz
−∞
Esercizio 6.1.8 Si consideri un elettrone che viaggi attraverso un quadrupolo magnetico
costituito da due spire parallele di raggio a, distanti 2Z tra loro ed attraversate da correnti
I uguali ed opposte (v. Sez. ). Analizzare la lunghezza focale della lente al variare di Z.
Soluzione: In prossimità dell’asse z del quadrupolo, costituito da due bobine di Helmholtz
coassiali di raggio a, con centri in ±Z dell’asse z di un sistema di coordinate cilindriche,
ed attraversate dalla corrente I, il potenziale vettore Aφ (z, ρ) è dato da (v. Eq. (3.16))
Aφ (z, ρ) = µ0 If (z) ρ
B0 (z) = 2µ0 If (z)
con
1
f (z) = a2
4
"
1
(a2 + z 2 − 2zZ + Z 2 )3/2
−
1
(a2 + z 2 + 2zZ + Z 2 )3/2
#
Pertanto la (6.18) relativa ad una singola sira viene sostituita da
−
eµ2 I 2 1
1
= 0
g
f1
2me V 16a
Z
a
dove
g
Z
a
=
∞
−∞



1
1+
z2
−
2z Za
+
3/2
Z 2
a
−
1
1+
z2
+
2z Za
+
è riportata in Fig. (6.10)
2

3/2 
Z 2
a
dz
Esercizio 6.1.9 Si consideri un quadrupolo magnetico del tipo indicato in figg. 6.11 e
6.12. Calcolare l’azione dello stesso su un fascio di particelle
Soluzione: Si scelga un sistema di riferimento con l’asse z perpendicolare al piano di
figura. In prossimità di z si ha il campo magnetico è dato da:
Bx = by
By = −bx
Esercizio 6.1.10 Si consideri la lente magnetica di Glazer4 in cui
B0 (z) =
B1
1+
z 2
a
Verificare che per questo profilo di campo si può calcolare analiticamente la traiettoria di
un generico elettrone parassiale
4
W. Glazer, Zeit. für Physik 111, 285 (1941); L. Marton and R. G. E. Hutter, Optical constants of a
magnetic type electron microsope, Proc. I.R.E. 32, 546 (1944)
6.1 Teoria delle orbite
171
Figura 6.10: Andamento dell’inversa lunghezza focale di un quadrupolo magnetico al
variare della distanza tra le due spire
Figura 6.11: Sezione di un quadrupolo magnetico inserito su una linea di trasporto di
fasci di particelle cariche
Figura 6.12: Lente magnetica a quadrupolo trasverso da www.helmholtz-berlin.de
172
Elettrodinamica
Soluzione: Una traiettoria parassiale è definita dall’equazione
"
#
1
d2
eB12 a2
+
ρ=0
dζ 2 8me V 1 + ζ 2 2
(6.19)
dove ρ/a = y, ζ = z/a. Ponendo
v (ϕ)
sin ϕ
ζ = cot ϕ , ρ =
e tenendo conto della relazione
d
d
= − sin2 ϕ
,
dζ
dϕ
sostituendo nella (6.19) si ottiene
d
eB12 a2
d v
sin2 ϕ
+
sin ϕv = 0
dϕ
dϕ sin ϕ 8me V
Sviluppando si arriva facilmente all’equazione:
d2
eB12 a2
+
1
+
dϕ2
8me V
Pertanto si ha:
ρ=C

v=0
eB12 a2

a
sin  1 +
ϕ + ψ
sin ϕ
8me V
Esercizio 6.1.11 Si consideri un elettrone non relativistico che si muove (a) in un campo
magnetostatico costante del tipo B = B (x, y) ẑ, (b) in un campo magnetostatico associato
in un sistema di coordinate cilindriche (ρ, φ, z) ad un potenziale vettore A = (0, Aφ (ρ) , 0),
(c) come per (b) con Aφ (ρ) = const
Soluzione: In tutti i tre casi sia v che v⊥ = vx2 + vy2 sono costanti. (a) La proiezione
della traiettoria (v. Fig. 6.13) sul piano x, y perpendicolare al campo B presenta un
raggio di curvatura pari a:
v⊥
ρ (x, y) =
ω c (x, y)
dove
eB (x, y)
me
rapresenta la girofrequenza locale. D’altra parte si ha
ω c (x, y) = −
dvx
dvy
= ω c (x, y) vy ,
= −ω c (x, y) vx
dy
dy
ovvero
dy
=
dx
vy
2
v⊥
− vy2
(6.20)
6.1 Teoria delle orbite
173
Figura 6.13: Traiettorie spiraliformi di ioni ed elettroni in un campo magnetico uniforme
(b) Se in coordinate cilindriche (ρ, φ, z) il potenziale vettore è dato da A = (0, Aφ (ρ) , 0)
risulta
1 d
Bz (ρ) =
(ρAφ (ρ))
ρ dρ
per cui
eBz (ρ)
ω c (ρ) =
me
ovvero la forza di Lorentz è data da
FL = Bz (ρ) ρφ̇ρ̂ + ρ̇φ̂
me
=
ω c (ρ) ρφ̇ρ̂ + ρ̇φ̂
e
D’altra parte, muovendosi l’elettrone in un campo magnetostatico v 2 risulta constante,
ovvero
2
ρ̇2 + ρ2 φ̇ = const
Pertanto dalla (6.20) discende per l’accelerazione tangenziale:
1d
ω c (ρ)
ρ2 φ̇ = −
ρ̇
ρ dt
e
ovvero
vφ (ρ) = ρφ̇ = −
(c) Per Aφ (ρ) = const si ha
1
e
ρ
ω c (ρ′ ) ρ̇dt = −
1
B (ρ) = Aφ
ρ
e
vφ (ρ) = −
eAφ
me
1−
ρ0
ρ
1
e
ρ
ω c (ρ′ ) dρ′
174
Elettrodinamica
Figura 6.14: Traiettorie dell’elettrone relative ai casi a, b e c.
D’altra parte
2
vφ2 (ρ) + vρ2 (ρ) = v⊥
Pertanto deve in genere risultare
2
v⊥
≥
eAφ
me
2
ρ
1− 0
ρ
2
Si potranno quindi distinguere 3 casi a,b, e c . a e b corrispondono alla condizione
eA
2
eA
2
2
2
v⊥
≥ meφ
mentre c corrisponde a meφ
≥ v⊥
. A sua volta a e b corrispondono
rispettivamente a ρ0 > 0 e ρ0 < 0. Nel casi a e b ρmin ≤ ρ < ∞ con ρmin definito
eA
rispettivamente da v⊥ = meφ 1 ∓ ρρ0 . Nel caso c la traiettoria si sviluppa lungo il
min
centro guida ρ = ρ0 (v. Fig. 6.14)
Esercizio 6.1.12 Calcolare la traiettoria di un elettrone nel campo B di un monopolo
magnetico
Soluzione: In questo caso
B = B0 r02
In tal caso
v̇ =r02 ω c0
r
r3
v×r
r3
Pertanto
d
v×r
(v × r) = v̇ × r =r02 ω c0 3 × r
dt
r
v
ṙ
d
− 3 r = r02 ω c0 Ω
= r02 ω c0
r
r
dt
Da cui segue
v × r =r02 ω c0 (Ω − k)
6.1 Teoria delle orbite
175
Figura 6.15: Rappresentazione schematica della traiettoria di un elettrone nel campo di
un ondulatore
con k un vettore costante. In coordinate sferiche (r, θ, φ) con l’asse polare diretto lungo
k si ha
(v × r)r = r02 ω c0 (1 − k cos θ) = 0
(v × r)θ = r2 φ̇ sin θ = −r0 v
Tenuto conto che
2
ṙ2 + r2 φ̇ sin2 θ = v2
si ha
2
ṙ2 = v 2 − r2 φ̇ sin2 θ = v 2 1 −
r02
r2
Scegliendo l’origine del tempo tale che r (0) = r0 si ottiene
r2 = r02 + v 2 t2
Esercizio 6.1.13 Calcolare l’orbita di una particella di carica q e massa m che, partendo da condizioni di riposo si muove in un campo magnetico uniforme B con sovrapposto un campo elettrico costante ed uniforme E perpendicolare al primo. Si suggerisce di
decomporre il movimento in una rotazione attorno a B + un movimento di deriva
6.1.2
Ondulatore
Un ondulatore è un dispositivo usato per ottenere radiazione monocromatica in fasci
estremamente collimati. Un tipico ondulatore è schematicamente composto da una serie
di dipoli magnetici disposti in modo tale da generare un campo magnetico trasversale
alternato lungo tutto l’asse del dispositivo. Attraverso l’ondulatore vengono fatti passare
fasci di particelle cariche (il più delle volte elettroni) provenienti da un sincrotrone, un
ciclotrone o un anello di accumulazione. Le particelle, attraversando questa struttura
magnetica periodica, sono forzate a oscillare (v. Fig. 6.15) e quindi a emettere radiazione.
Ogni ondulatore è caratterizzato da un parametro adimensionale K definito come
176
Elettrodinamica
K=
eBλw
2πmβγc
(6.21)
dove λw /2 è la distanza minima tra due dipoli magnetici il cui campo magnetico è orientato
in verso opposto, e è la carica dell’elettrone, B il campo magnetico, la massa a riposo
dell’elettrone e c la velocità della luce nel vuoto.
Ogni elettrone che attraversi l’ondulatore subisce una forza di Lorentz trasversale a
cui è associata un’accelerazione. L’elettrone oscilla trasversalmente con una frequenza
ω0 =
2π
v
λw
(6.22)
mentre procede lungo l’asse con velocità media v con sovrapposte oscillazioni a frequenza
2ω 0 . Il fascio di elettroni che passa nell’ondulatore emette radiazione diretta lungo l’asse
dell’ondulatore ad una lunghezza d’onda dell’ordine di
λs ∼
λw
2γ 2
(6.23)
Scegliendo opportunamente λw e γ, si può ottenere radiazione in un range molto ampio
che va dal lontano IR fino ai raggi X ( ∼ 10 pm).
Esercizio 6.1.14 Calcolare la traiettoria di un elettrone di energia γ che si muove sul
piano y-z attraverso un campo magnetico statico
B (r) =
B cos ζ x̂ z ∈ (0, L = Nλw )
0z∈
/ (0, L)
dove ζ = λ2πw z, λw rappresenta il periodo spaziale di un ondulatore (wiggler) che può
variare tra alcuni mm e cm. Si assuma che l’elettrone venga iniettato nell’ondulatore con
velocità v (z = 0) = v ẑ. (b) Analizzare il caso in cui B sia sufficientemente piccolo da
poter approssimare la legge del moto con espressioni lineari nel parametro K (v. (6.21)).
(c) Analizzare il movimento dell’elettrone nel sistema di riferimento che si muove con la
stessa velocità di iniezione nell’ondulatore
Soluzione: (a) In via preliminare si ricorda che:
1
L = − me c2 − ev · A + eV
γ
dove
A (r, t) = B
λw
sin ζ ŷ
2π
V (r, t) = 0
Pertanto
avendo posto
1
λw
L = − mec2 − me Ω sin ζ ẏ
γ
2π
eB
=Ω
me
6.1 Teoria delle orbite
177
Figura 6.16:
Dalle equazioni di Lagrange
d ∂
∂
L=
L
dt ∂ q̇j
∂qj
discende che
d ∂
dt ∂ q̇j
"
c2
v
1−
c
2
λw
+ Ω sin ζ ẏ
2π
#
d
λw
−γ q̇j + Ω
sin ζ δ jy
dt
2π
= Ω
λw ∂
sin ζ ẏ
2π ∂qj
= Ω cos ζ ẏδ jz
ovvero
d
(γ ẋ) = 0
dt
d
(γ ẏ) = Ω cos ζ ż
dt
d
(γ ż) = −Ω cos ζ ẏ
dt
Dal momento che il campo elettrico è nullo γ risulta costante. Pertanto si ha
d
Ω
ẏ =
cos ζ ż
dt
γ
Ω
d
ż = − cos ζ ẏ
dt
γ
178
Elettrodinamica
ovvero
Ω
d
ẏ = cos ζ
dz
γ
Ne segue che
(6.24)
ẏ (z) = Kv sin ζ
dove K è definito in Eq. (6.21):
K=
Ωλw
Ω
=
γ2πv
γω 0
con ω 0 definito in (6.22). Poichè v 2 ≥ ẏ 2 deve risultare
|sin ζ| ≤
Possiamo quindi individuare due regimi,
regime I
:
1
K
1 ≤ K ⇒ z ∈ (0, zmax ) con zmax =
λw
arcsin
2π
regime II 1 ≥ K ⇒ z ∈ (0, L)
1
K
Nel regime I l’elettrone rimane intrappolato in un periodo dell’ondulatore ed oscilla avanti
ed indietro. Nel regime II l’elettrone viaggia attraverso l’ondulatore con una legge del moto
rappresentata da
ż (z) = v
ovvero
1
t=
v
dζ ′
ζ
0
1 − K2 sin2 ζ
1−
K2
2
sin ζ
′
2
= ω −1
0 K ζ, K
(6.25)
con K (ζ, K2 ) integrale ellittico di prima specie (v. Eq. (1.28)).
D’altra parte dalla (6.24) discende che
d
y = ż −1 Kv sin ζ =
dz
Pertanto
y (z) =
K
1 − K2 sin2 ζ
sin ζ
λw
1+K
ln
2π K cos ζ + 1 − K2 sin2 ζ
(6.26)
(b) Sviluppando (6.26) rispetto a K si ha:
y (z) ∼
λw
1
(1 − cos ζ)K +
2 − 2 cos3 ζ − 3 cos ζ sin2 ζ K3 + O[K]4
2π
6
D’altra parte sviluppando K (ζ, K2 ) in serie rispetto a K2 (v. Eq. (??)) e troncando al
primo ordine in K2 si ottiene
t (z) ≃ ω −1
ζ+
0
ζ sin 2ζ
−
4
8
K2
6.1 Teoria delle orbite
179
Pertanto al primo ordine in K si ha:
λw
K(1 − cos (ω 0 t))
2π
λw
z (t) ∼
ω0 t
(6.27)
2π
(c) Nel sistema K ′ che si muove con la velocità di iniezione v la trasformazione di
Lorentz (2.1.1) si specializza in
y (t) ∼
Pνµ =
ovvero:
t
x
y
z
γ
−γv
=
=
=
=
−c−1 γvT
1 + (γ − 1)ẑ ẑ
(6.28)
γt′ + c−1 γv z ′
x′
y′
γ v t′ + z ′
In K ′ si ha z ′ = 0 per cui sostituendo nella (6.27) si ottiene
x′ = x0
λw
y ′ (t′ ) ∼
K(1 − cos (ω 0 γt′ ))
2π
z ′ (t) = 0
D’altra parte i campi si modificano come segue (v. (2.27)):
E′
B′
γ + (1 − γ) ẑ ẑ
γv × x̂
γ + (1 − γ) ẑẑ
− cγ2 v ×


−γBv cos λ2πw γv t′ ẑ

= 
γB cos λ2πw γv t′ x̂
=
·
0
B cos ζ x̂
(6.29)
ovvero in K ′ risulta investito da un’onda e.m. oscillante a frequenza
ω ′0 = γω 0
Pertanto l’elettrone oscillante a frequenza ω ′0 sotto l’azione di questo campo, emette un’onda scatterata che nel sistema di laborario presenta una frequenza ω s legata a ω ′0 dalla
relazione
ω ′0 = γ ω s − ks · v
= γω s 1 − β cos θ
dove θ rappresenta l’angolo formato dall’onda scatterata lungo ks con v . Osservando il
campo irradiato in prossimità dell’asse dell’ondulatore (θ ≃ 0) si ha
1
ω ′0 ≃ γω s 1 − β + β θ2
2
1 1
1
1
= γω s
+
1− 2
2
2 γ
2
2γ
ωs
1 + γ 2 θ2
=
2γ
θ2
180
Elettrodinamica
Figura 6.17: Rappresentazione schematica di un ciclotrone. Al centro è riportata la
camera a vuoto a forma di una doppia D metallica, compresa tra due elettromagneti. Alle
due D è applicata una tensione oscillante che crea un campo localizzato tra i due bordi
rettilinei affacciati. Attraversando ciclicamente questa regione gli ioni vengono accelerati.
ovvero
1 1
1
≃ 2
1 + γ 2 θ2
ωs
2γ ω 0
(6.30)
corrispondente alla lunghezza d’onda (v. (6.23))
λs ∼
6.1.3
λw
1 + γ 2 θ2 .
2
2γ
Ciclotrone
Un ciclotrone è una macchina usata per accelerare fasci di particelle elettricamente cariche
(normalmente ioni leggeri) utilizzando una corrente alternata ad alta frequenza ed alta
tensione, in associazione con un campo magnetico perpendicolare. La traiettoria percorsa
dalle particelle è a spirale a partire dal centro. Raggiunto il bordo esterno della macchina
il fascio fuoriesce ad alta velocità, prossima alla velocità della luce.
Il principio sfruttato è la risonanza ionica ciclotronica. All’interno di una camera a
vuoto circolare (v. Fig. 6.17) sono presenti due elettrodi semicircolari cavi a forma di
D. La camera è posta tra le espansioni polari di un potente magnete. Quando una particella viene introdotta tangenzialmente alla camera, ortogonalmente al campo magnetico,
essa viene deviata e mantenuta su un’orbita circolare per effetto della forza di Lorentz.
Applicando una opportuna differenza di potenziale alternata ad alta frequenza tra i due
elettrodi, le particelle subiscono un’accelerazione ogni volta che passano nello spazio tra
essi. Accelerando, il diametro dell’orbita aumenta, fino a quando il fascio non fuoriesce
tangenzialmente dal bordo del dispositivo (v. Fig. 6.18).
Esercizio 6.1.15 Analizzare il moto a spirale descritto dalle particelle di un ciclotrone
sottoposte ad un campo magnetico uniforme B diretto verticalmente e ad potenziale V(t)=V0 cos ωt
localizzato nello spazio compreso tra i bordi delle due D
6.1 Teoria delle orbite
181
Figura 6.18: Traiettorie spiraliformi degli ioni all’interno di un ciclotrone. Gli ioni, iniettati al centro sono sottoposti ad un campo magnetico uniforme verticali che li obbliga
a descrivere orbite circolari. Attraversando il campo elettrico oscillante localizzato tra le
due D gli ioni vengono accelerati finendo cosè per descrivere orbite circolari spiralizzanti
verso la periferia.
6.1.4
Invarianti adiabatici
Esercizio 6.1.16 Si consideri un elettrone non relativistico che si muove in un campo
magnetico statico e spazialmente variabile. Si supponga inoltre che localmente B (r) vari di
(r)
e v⊥ (r)
poco in un intorno di r avente le dimensioni del raggio di curvatura ρp (r) = meB(r)
= vω⊥c (r)
dell’orbita spiraliforme. Espandere la traiettoria in una serie asintotica5 nel parametro di
piccolezza ε = mee :
Soluzione: Si ponga
r (t) = r0 (t) + s (t) + s∗ (t)
dove
s (t) =
∞
εn einθ sn (t)
(6.31)
(6.32)
n=1
con
t
θ=
ω c (r0 (t′ )) dt′ .
Derivando (6.32) si ottiene
ṡ (t) =
∞
inBεn−1 sn + εn ṡn einϕ
n=1
5
P. C. Clemmow and J. P. Dougherty, Electrodynamica of Particles and Plasma, Addison-Wesley Publ.
Co., Reading 1969, Sez. 4.4
182
Elettrodinamica
Derivando successivamente si ottiene
s̈ (t) =
∞
−n2 B 2 εn−2 sn + inεn−1 Ḃsn + B ṡn + εn s̈n einϕ
n=1
Inserendo questi sviluppi nell’equazione del moto non relativistica
εv̇ = −v × B
e limitandosi ai temini al primo ordine in ε si ha:
r = r0 + ε s1 eiϕ + c.c. + O ε2
v = u + i (Bs1 − iεṡ1 ) eiϕ + c.c. + O ε2
εv̇ = εu̇ +
−B 2 s1 + iεḂs1 + iεB ṡ1 + εB ṡ1 eiϕ + c.c. + O ε2
B (r) = B (r0 ) + ε eiϕ s1 + c.c. · ▽B + O ε2
−v × B = − u + i (Bs1 − iεṡ1 ) eiϕ + c.c.
iϕ
−ε u + iBs1 e + c.c.
iϕ
× B (r0 )
× e s1 + c.c. · ▽B + O ε2
avendo posto u = ṙ0 . Isolando i termini che non contengono einϕ si ottiene
εu̇ = −u × B (r0 ) − i2εB (s1 × s∗1 ) · ▽B
(6.33)
mentre salvando quelli proporzionali a eiϕ si ha:
−B 2 s1 + iεḂs1 + iεB ṡ1 + εB ṡ1
= −i (Bs1 − iεṡ1 ) × B (r0 ) − εu × s1 · ▽B
ovvero
−B (Bs1 − iεṡ1 ) + i (Bs1 − iεṡ1 ) × B (r0 )
= iε −Ḃs1 − B ṡ1 + iu × s1 · ▽B
(6.34)
In particolare dall’ultima equazione segue all’ordine 0 in ε:
s1 = is1 × B̂
(6.35)
Ne discende che s1 è combinazione di due vettori reale s′1 , s′′2 di uguale ampiezza, mutuamente ortogonali e perpendicolari a B:
s1 = s′1 + is′′2
con
s′1 = −s′′2 × B̂ , s′′2 = s1 × B̂
Pertanto al primo ordine in ε la (6.31) diventa
r (t) = r0 (t) + 2ε (s′1 cos ϕ − s′′2 sin ϕ) + O ε2
per cui
(6.36)
6.1 Teoria delle orbite
Ne segue che
183
v = u − 2εB (s′1 sin ϕ + s′′2 cos ϕ) + O (ε)
u
|v⊥ − u⊥ |
≃ sin ϑ
(6.37)
B
B
con ϑ angolo formato da u con B, ovvero ρ = |s′1 | = |s′′1 | rappresenta il raggio di curvatura
della traiettoria spiraliforme
Moltiplicando scalarmente (6.34) per s∗1 si ottiene
ρ=
s∗1 · [−B (Bs1 − iεṡ1 ) + i (Bs1 − iεṡ1 ) × B (r0 )]
= iε −Ḃs∗1 · s1 − Bs∗1 · ṡ1 + is∗1 · u × (s1 · ▽) B
ovvero
D’altra parte
Ḃs∗1 · s1 + Bs∗1 · ṡ1 − iu · s∗1 × (s1 · ▽) B = 0
−iu · s∗1 × (s1 · ▽) B
= [s′1 × (s′′2 · ▽) − s′′2 × (s′1 · ▽)] B
= −ρ2 ▽B
per cui
Ḃs∗1 · s1 + B (s∗1 · ṡ1 + ṡ1 · s1 ) = 0
6.1.5
(6.38)
Specchi e bottiglie magnetiche
Esercizio 6.1.17 Si consideri un campo magnetico lentamente variabile nello spazio. (a)
Mostrare che il momento magnetico m associato alla componente dell’orbita perpendicolare
a B è un invariante adiabatico del moto; (b) ricavare l’equazione del moto del centro guida
dell’elettrone rappresentato da r0 (t)
Soluzione: (a) All’orbita circolare ε (eiϕ s1 + e−iϕ s∗1 ) è associato il momento magnetico
m definito come la corrente efficace eω c / (2π) moltiplicata per l’area 4πε2 ρ2 della singola
spira:
m = 2eω c ε2 ρ2 = 2Bme ρ2 = B me s1 · s∗1
2
2 sin ϑ
= 2me u
B
avendo fatto uso della (6.37) ed indicato con ϑ l’angolo formato da u con B. . Dalla (6.38)
discende che
d
ṁ = (Bs∗1 · ṡ1 ) = 0
dt
ovvero il momento magnetico si mantiene costante durante il moto del singolo elettrone
lungo la traiettoria di guida r0 (t) . Inoltre dal momento che u è costante segue anche che
sin2 ϑ
= const
B
(6.39)
184
Elettrodinamica
(b) Dall’Eq. (6.33) discende
u × B = ε u̇ +
m
▽B
me
Moltiplicando vettorialmente per B si ottiene per la componente di u perpendicolare a B:
ε
m
B × u̇ +
▽B
2
B
me
" 2
#
u
m
ε
B × ▽B
=
+
B2 B
me
u⊥ =
ovvero che il centro guida è caratterizzato da un moto di deriva perpendicolare a B e
proporzionale al gradiente di B (gradient drift)), mentre la componente u è sottoposta
ad una accelerazione:
m
u̇ = − ▽B + O (ε)
me
Dalla costanza dell’energia discende inoltre
m
2u2 cos2 ϑ +
B = const + O (ε)
(6.40)
me
Esercizio 6.1.18 (a) Utilizzare l’Hamiltoniana della particella per dimostrare che l’integrale di azione Ji di un elettrone che si muove lungo la traiettoria guida in un campo
magnetostatico è un invariante adiabatico.
Soluzione: (a) Se qi sono generiche coordinate canoniche e pi i rispettivi momenti che
descrivono delle curve periodiche nello spazio delle fasi, si definisce integrale di azione Ji
la quantità
!
Ji =
con
!
pi dqi
esteso ad un ciclo completo della coordinata qi . Tenendo conto dell’Hamiltoniana
H=
si ha per l’azione J:
J =
=
!
!
(p + eA) · dℓ =
me
!
e
S
!
!
p · dℓ + e
!
∇ × A·n̂da
p · dℓ + e
= me
D’altra parte
m2e c4 + (p + eA)2
S
v⊥ · dℓ + e
B·n̂da
S
v⊥ · dℓ = 2πme ω c ρ2
!
1
B·n̂da = − me v⊥ · dℓ
2
A·dℓ
6.1 Teoria delle orbite
185
Figura 6.19:
per cui
J = πme ω c ρ2 = eBπρ2
Esercizio 6.1.19 Si consideri un campo magnetico creato da una bobina, che si intensifichi nella direzione positiva dell’asse z come mostrato in figura formando così uno specchio
magneticoUtilizzando l’Eq. (??) analizzare l’andamento di u lungo z
Soluzione: Tenendo conto della relazioneDall’Eq. (6.39) segue che la particella avanza
lungo z fin dove ϑ (zR ) = π/2 dove viene riflessa per poi procedere nella direzione opposta.
Se si indica con zmin la posizione di minimo del campo (B (zmin ) = Bmin ) si avrà che le
particelle con angolo di lancio (pitch angle) ϑ (zmin ) > ϑmin con
sin2 ϑmin =
Bmin
Bmax
risulteranno riflesse dallo specchio.
Esercizio 6.1.20 Si consideri una bottiglia magnetica (v. Fig. (6.20)) caratterizzata
da un rapporto Bmax /Bmin . Per quali valori dell’angolo di lancio ϑ le particelle vengono
trattenute dalla bottiglia?
Soluzione: L’angolo di lancio per le particelle al centro della bottiglia deve soddisfare
la disuguaglianza:
Bmin
sin2 ϑ ≥
(6.41)
Bmax
6.1.6
Intrappolamento nel campo magnetico terrestre
Il campo geomagnetico è un fenomeno naturale presente sul pianeta Terra. Esso è assimilabile al campo magnetico generato da un dipolo magnetico con poli magnetici non
coincidenti con quelli geografici e non statici, e con asse inclinato di 11,5◦ rispetto all’asse
di rotazione terrestre. D’altra parte non può esistere un vero dipolo in quanto il centro
della Terra si trova a temperature ben superiori ai 1043 K (circa 770◦ C), valore della
temperatura di Curie al di sopra della quale qualunque minerale ferromagnetico perde le
186
Elettrodinamica
Figura 6.20: Particelle cariche imbottigliate in una bottiglia magnetica. La forza di
Lorentz trattiene tra i due punti di massimo di B le particelle con angoli di lancio ϑ tali
che sin2 ϑ > Bmin /Bmax da physics.miami.edu
Figura 6.21: Rappresentazione schematica di un Tokamak in grado di confinare un plasma
da fusione in una regione toroidale
6.1 Teoria delle orbite
187
Figura 6.22: Andamento schematico delle linee del campo magnetico terrestre
sue proprietà magnetiche, divenendo paramagnetico. Oggi le teorie sono orientate verso
un modello analogo a quello di una dinamo ad autoeccitazione.
Esso si estende per svariate decine di migliaia di chilometri nello spazio formando una
zona chiamata magnetosfera la cui presenza genera una sorta di scudo elettromagnetico
che devia i raggi cosmici e tutte le particelle cariche riducendo la quantità che raggiunge
il suolo dando origine alle fasce di Van Allen.
Dall’equatore ai poli, sulla superficie terrestre, il valore del campo varia da circa poco
più di 20.000 nT all’equatore ai circa 70.000 nT delle zone polari.
6.1.7
Elettroni in un’onda piana
Esercizio 6.1.21 Analizzare la traiettoria di un elettrone in un’onda piana monocromatica di frequenza ω che viaggia lungo la direzione n̂. Analizzare i casi (a) onda polarizzata linearmente, (b) onda polarizzata circolarmente con sovrapposto un campo magnetico
statico B0 = B0 n̂ diretto lungo n̂.e (c) come in (b) per B0 = 0 6
Soluzione: In via preliminare si ricorda che:
1
L = − me c2 − ev · A + eV
γ
dove
1
A (r, t) = A0 exp −iω t − n̂·r
c
V (r, t) = 0
Pertanto
6
1
1
L = − me c2 − ev · A0 exp −iω t − n̂·r
γ
c
v.p.e. P. C. Clemmow et al. loc. cit. pag. 181 Sez. 4.5.2
188
Elettrodinamica
Dalle equazioni di Lagrange
d ∂
∂
L=
L
dt ∂ q̇j
∂qj
discende che
v
d
e
v
1
1 − · n̂ + n̂ · A0 exp −iω t − n̂ · r
(γv) = iω
dt
me
c
c
c
Sostiuendo iωA0 exp −iω t − 1c n̂ · r
con −E (r, t) l’equazione del moto assume la forma
d
e
v
v
(γv) = −
1 − · n̂ + n̂ · E .
dt
me
c
c
(6.42)
Moltiplicando scalarmente per n̂ e tenendo conto del fatto che E · n̂ = 0 si ottiene:
e v
d
(γv · n̂) = −
·E
dt
me c
Tenendo ora conto dell’equazione dell’energia
me c2
d
γ = −eE · v
dt
ne discende che la quantità
1
λ = γ 1 − n̂ · v
c
è un invariante del moto. Pertanto la (6.42) si può riscrivere nella forma
d
e
(γv) = −
dt
me
λ
v
+ n̂ · E .
γ
c
(6.43)
Passando dalla variabile t alla fase ϕ:
1
ϕ = ω t − n̂ · r
c
si ha
d
1 d
= λω
dt
γ dϕ
In particolare
γv = λω
e
d
r
dϕ
d
1 d2
(γv) = λ2 ω 2
r
dt
γ dϕ2
Sostituendo quest’ultima espressione nella (6.43) si ottiene
e
ω
d2
r=−
E+
2
2
dϕ
me ω λ
c
E·
d
r n̂
dϕ
(6.44)
6.1 Teoria delle orbite
189
(a) Per un’onda polarizzata linearmente si ha
n̂ = ẑ , E = E cos ϕx̂ ,
e l’Eq. (6.44) si riduce a
eE
d2
x
=
−
cos ϕ
dϕ2
me ω 2 λ
d2
y = 0
dϕ2
d2
eE
d
z
=
−
x cos ϕ
dϕ2
cme ωλ dϕ
Da cui integrando si ottiene:
eE
x = α1 + α2 ϕ +
cos ϕ
me ω 2 λ
y = α3 + α4 ϕ
eE
e2 E 2
z = α5 + α6 ϕ +
α2 cos ϕ −
sin 2ϕ
cme ωλ
8cm2e ω 3 λ2
(6.45)
con αi costanti dipendenti dalle condizioni iniziali.
(b) Per un’onda polarizzata circolarmente
n̂ = ẑ , E = E (cos ϕx̂ − sin ϕŷ) ,
con sovrapposto un campo magnetico B0 = B0 n̂ si ottiene
d2
eE
Ω d
cos ϕ +
y
x = −
2
2
dϕ
me ω λ
ω dϕ
d2
eE
Ω d
y =
sin ϕ −
x
2
2
dϕ
me ω λ
ω dϕ
eE
d2
d
z = −
x cos ϕ −
2
dϕ
cmeωλ
dϕ
d
y sin ϕ
dϕ
con
eB0
me λ
la girofrequenza relativistica. Integrando si ha:
Ω=−
E
ϕ sin ϕ
B0 ω
E
y = α4 + α3 cos ϕ + α2 sin ϕ +
ϕ cos ϕ
B0 ω
E 2
E2 3
z = α5 + α6 ϕ + α3
ϕ +
ϕ
2cB0
6cB02
x = α1 + α2 cos ϕ − α3 sin ϕ +
(c) Per Ω = 0 il sistema (6.46) si riduce a:
d2
eE
x = −
cos ϕ
2
dϕ
me ω 2 λ
d2
eE
y =
sin ϕ
2
dϕ
me ω 2 λ
eE
d
d2
z = −
x cos ϕ −
2
dϕ
cmeωλ
dϕ
d
y sin ϕ
dϕ
(6.46)
190
Elettrodinamica
Ne discende che:
eE
cos ϕ
me ω 2 λ
eE
sin ϕ
y = α4 +
me ω 2 λ
z = α5 + α6 ϕ
x = α1 −
(6.47)
Esercizio 6.1.22 Pe i casi (a) e (c) dell’Eser. 6.1.21 analizzare la dipendenza della fase
ϕ da t in funzione del campo incidente, assumendo che i coefficienti α1···6 di Eq. (6.45) e
α1 , α4 , α5 , α6 di (6.47) siano nulli.
Soluzione: (a) Dal momento che la fase è data da
ϕ=ω t−
z
c
dalla (6.45) si ottiene
ϕ − ε sin 2ϕ = ωt
dove
ε=
e2 E 2
8c2 m2e ω 2 λ2
Ponendo
ϕ = ωt + ψ
si ha
ψ = ε sin (2ωt + 2ψ)
Per ε ≪ 1 si può sviluppare ψ in serie
ψ = εψ (1) + ε2 ψ (2) + ε3 ψ (3) + · · ·
che sostituita nella (6.48)
ψ (1) + εψ (2) + · · · = sin 2ωt + 2εψ (1) + 2ε2 ψ (2) + · · · ψ
fornisce
ψ (1) = sin (2ωt)
ψ (2) = 2 cos (2ωt) sin (2ωt) = sin (4ωt)
ψ (3) = sin (2ωt) cos 2εψ (1) + cos (2ωt) sin 2ε2 ψ (2)
= −2 sin3 (2ωt) + 2 cos (2ωt) sin (4ωt)
(b) Per il caso (c) si ha
ϕ = ωt
(6.48)
6.1 Teoria delle orbite
6.1.8
191
Reazione di radiazione
Esercizio 6.1.23 Si integri l’equazione del moto di un elettrone investito da un’onda
polarizzata circolarmente tenendo conto della formula di Abraham-Lorentz7 per la reazione
di radiazione:
2 q2
Frr (t) = −
ȧ (t) .
3 4πε0 c3
Soluzione: Tenuto conto che in tal caso t a ϕ = ω t − rc ·n̂ = ωt si ha
2 q2
2 q2ω d
ȧ
(t)
=
−
a (ϕ)
3 4πε0 c3
3 4πε0 c3 dϕ
2 e2 ω
eE d
= −
(cos ϕx̂ − sin ϕŷ)
3
3 4πε0 c me ω 2 λ dϕ
2 e2 ω
eE
=
(sin ϕx̂ + cos ϕŷ)
3
3 4πε0 c me ω 2 λ
Frr (t) = −
Pertanto
eE
eE
d2
2 e2 ω
x =
cos ϕ +
sin ϕ =
cos (ϕ − ϕrr )
2
2
3
2
dϕ
me ω λ
3 4πε0 c
me ω λ cos ϕrr
d2
eE
2 e2 ω
eE
sin
ϕ
−
y
=
−
cos ϕ = −
sin (ϕ − ϕrr )
2
2
3
2
dϕ
me ω λ
3 4πε0 c
me ω λ cos ϕrr
d2
eE
d
d
x cos ϕ −
y sin ϕ
z =
2
dϕ
cme ωλ
dϕ
dϕ
dove
2 e2 ω
tan ϕrr =
3 4πε0 c3
per cui
eE
cos (ϕ − ϕrr )
me ω 2 λ cos ϕrr
eE
y = α4 +
sin (ϕ − ϕrr )
2
me ω λ cos ϕrr
x = α1 −
Ne discende che
d2
eE
eE
z
=
[sin (ϕ − ϕrr ) cos ϕ − cos (ϕ − ϕrr ) sin ϕ]
dϕ2
cmeωλ me ω 2 λ cos ϕrr
1
e4 E 2
e2 E 2
2
=
tan
ϕ
=
rr
2
2t
2
3
4
2
6 4πε0 c me λ
cme ω λ
7
una prima eq. fu proposta da P. A. M. Dirac, Proc. Roy. Soc. (London) A 167, 148 (1938); il problema
fu riesaminato da L. D. Landau e E. M. Lifshitz,”Fisica teorica: Elettrodinamica dei mezzi continui
VIII”, Ed. Riuniti, Roma 1986 Sezz. 75 e 76; un esame critico delle implicazioni legate alla presenza
di soluzioni ”in” e ”out” fu pubblicato da J. A. Wheeler and R. P. Feynman, Rev. Mod. Phys. 17,
157 (1945); una nuova eq. fu proposta da T. C. Mo e C. H. Papas, Phys. Rev. D 4, 3566 (1971);
queste eqq. sono state riesaminate da G. W. Ford and R. F. O’Connell, Phys. Lett. A 174 (1993);
F. Rohrlich, Phys. Rev. D 60, 084017 (1997); –— Phys. Lett. A 283, 276 (2001); R. Rivera and D.
Villaroel, Phys. Rev. E 66, 46618 (2002).
192
Elettrodinamica
ovvero
z = α5 + α6 ϕ +
e4 E 2
1
2
2t
4
2
3 4πε0 c me λ
In conclusione sull’elettrone agisce una forza pari a
F(rad) (ω) =
I˜ (ω)
σ T n̂ = w̃ (ω) σ T n̂ ,
c
(6.49)
dove w̃ (ω) rappresenta la densità spettrale di energia dell’onda incidente di frequenza ω.
Esercizio 6.1.24 Si utilizzi l’espressione (6.54) di F(rad) per calcolare la pressione di
radiazione esercitata sull’elettrone legato di un atomo.
Soluzione: Le considerazioni precedenti si estendono al caso di un elettrone legato
sostituendo α̃ e lib con quella dell’elettrone legato α̃℘ (ω). Ne segue che la sezione d’urto
dell’elettrone libero σ T va sostituita da una σ̃ (ω) dipendente dalla frequenza secondo la
legge (v. Fig. ??):
(6.50)
σ̃ (ω) = σ T ω 4 α2℘ (ω) .
Per ω prossimo a quella di risonanza ω f i relativa ad una transizione di dipolo elettrico
da uno stato iniziale |i ad uno finale |f la sezione d’urto σ̃ (ω) assume la forma
σ̃ f i (ω) =
γf i
σ f i max
.
π (ω − |ω f i |)2 + γ 2f i
(6.51)
con σ f i max ∝ |℘f i |2 essendo ℘f i = f |℘ · ǫ̂| i l’elemento di matrice della transizione
atomica interessata. Pertanto l’espressione (6.49) si modifica in
F(rad) (ω) = w̃ (ω)
6.1.9
γf i
σ f i max
n̂ .
π (ω − |ω f i |)2 + γ 2f i
(6.52)
Forza ponderomotrice
Esercizio 6.1.25 Si consideri una carica q di massa m in un campo elettrico oscillante
mr̈ = qE (r) cos (ωt)
Analogamente a quanto si è fatto nell’esercizio 6.1.16 si ponga (v. Eq. (6.31))
r (t) = r0 (t) + s1 (t)
dove s1 (t) rappresenta il contributo oscillante alla frequena ω e r0 (t) un contributo di
deriva lentamente variabile. Si derivino le equazioni del moto di r0 (t) e s1 (t)
Soluzione: Ponendo
l’eq. del moto diventa
E (r) = E (r0 ) + ∇E (r0 ) · s1 (t)
m (r̈0 (t) + s̈1 (t))
= q [E (r0 ) + ∇E (r0 ) · s1 (t)] cos (ωt)
(6.53)
6.1 Teoria delle orbite
193
Trascurando il contributo di r̈0 (t) si ottiene
ms̈1 (t) = qE (r0 ) cos (ωt)
ovvero
q
E (r0 ) cos (ωt)
mω 2
Sostituendo in (6.53) e rimuovendo i contributi oscillanti si ottiene8
s1 (t) = −
mr̈0 (t) = q∇E (r0 ) · s1 (t) cos (ωt) − ms̈1 (t)
q2
q2
= −
∇E
(r
)
·
E
(r
)
=
−
∇ |E (r0 )|2
0
0
2
2
mω
4mω
Se ne evince che la carica viene spinta verso le regioni in cui il campo risulta minimo.
Esercizio 6.1.26 Si consideri una particella di polarizzabilità α℘ (ω) di massa m in un
campo elettrico oscillante
E (r,t) = Re Ẽ (r) e−iω0 t = Re Ẽ0 (r) eiϕ(r) e−iω0 t+ikz .
Si calcoli la forza F(rad) esercitata sulla particella.
Soluzione: Se si indica con ρ̃℘ (r) la densità di carica indotta nella sfera polarizzata,
su quest’ultima si esercita una forza
1
Re
ρ̃℘ (r) · Ẽ∗ (r) d3 r
2
sf era
1
≃
Re
ρ̃℘ (r) · Ẽ∗ (r0 ) + (r − r0 ) · ∇Ẽ∗ (r)
2
sf era
1
Re ℘˜ · ∇Ẽ∗ (r)
=
2
r=r0
1
=
Re α℘ (ω) Ẽ∗ (r0 ) · ∇Ẽ∗ (r)
.
2
r=r0
F(rad) =
6.1.10
r=r0
d3 r
(6.54)
Pinzette ottiche
Esercizio 6.1.27 La descrizione delle forze associate all’interazione di campi ottici con
particelle immerse in fluido assume una forma particolarmente semplice quando si considerano nanoparticelle di raggio a < λ/20, essendo λ la lunghezza d’onda della radiazione nel
mezzo. In tal caso una nanoparticella dielettrica illuminata da un’onda e.m. si comporta
come un dipolo oscillante alla frequenza del campo incidente. Il moto della particella non
è in grado di seguire le variazioni istantanee di E(r,t), ma è influenzato dal suo inviluppo
Ẽ0 (r).
8
P. Luchini, C. H. Papas, S. Solimeno, Appl. Phys. B28, 15 (1982); A. Ashkin, J. M. Dziedzic, J. E.
Bjorkholm, S. Chu, Opt. Lett. 11 288 (1986). Forze analoghe, utilizzate da Townes per realizzare il
maser ad ammoniaca, si sono incontrate parlando dell’NH3 in Sez. 8.7.4 con F proporzionale a ∇ Ẽ0
attraverso il momento di dipolo permanente ℘ della molecola.
2
194
Elettrodinamica
Soluzione9 : Si consideri una particella sferica di raggio a (< λ/20) , con centro in r0
e indice di rifrazione ñ2 + iκ̃2 , immersa in un fluido di indice ñ1 + iκ̃1 ed investita da
un’onda quasipiana
E (r,t) = Re Ẽ (r) e−iω0 t = Re Ẽ0 (r) eiϕ(r) e−iω0 t+ikz ,
in cui Ẽ0 (r) varia lentamente sia lungo z che trasversalmente, esprimibile p.e. con
l’integrale di Luneburg-Debye (v. Eqq. (4.13,??) . In tal caso α̃℘ che descrive la risposta dell’intera sfera, calcolabile facendo riferimento al caso elettrostatico in vista della
disuguaglianza a ≪ λ, è data da
α̃℘ = α̃0 − i
dove
α̃0 = 4πa3
|α̃0 |2 k 3
= α̃′℘ − iα̃′′℘ ,
6πñ21
(ñ2 + iκ̃2 )2 − (ñ1 + iκ̃1 )2
′
′′
2
2 = α̃0 − iα̃0 ,
(ñ2 + iκ̃2 ) + (ñ1 + iκ̃1 )
dipende dal salto di indice di rifrazione tra la sfera ed il fluido circostante. Il contributo alla
polarizzabilità proporzionale a k 3 (k = ωñ1 /c) tiene conto del fatto che la sfera reirradia
parte della potenza e.m. incidente dando luogo alla diffusione di Rayleigh, come si è già
visto per l’elettrone.
Tenendo conto che ∇Ẽ = eiϕ(r) ∇Ẽ0 + iẼ0 ∇ϕ dalla Eq. (6.54) discende che
F(rad) =
ε0
ε0
Re α̃℘ Ẽ · ∇Ẽ∗ =
2
2
2
2
1 ′
α̃℘ ∇ Ẽ0 + α̃′′℘ Ẽ0 ∇ϕ
2
,
ovvero F(rad) è costituita da un contributo dovuto al gradiente della densità di energia
e.m.
α̃′℘ ∇ Ẽ0
2
e da un termine proporzionale al gradiente della fase
"
|α̃0 |2 k 3
′′
α̃0 +
6πñ21
#
2
Ẽ0 ∇ϕ .
La forza proporzionale al gradiente della densità di energia e.m. tende ad intrappolare
la particella nella regione di massima densità. Nel 1986 Ashkin et al pensarono di utilizzare
questo tipo di forze per catturare nel fuoco di un fascio laser ben focalizzato particelle
micrometriche. Da allora l’uso di queste pinzette ottiche (OT) è stato esteso a diversi
sistemi di micro e nanoparticelle.
6.2
Trappole
Per studiare con un alto grado di precisione le proprietà di atomi isolati occorre mantenerli
fermi in una piccola regione di spazio per un certo intervallo di tempo ricorrendo al
raffreddamento mediante laser. In una camera a vuoto un fascio ben collimato di atomi
di sodio 23 Na ottenuti per evaporazione di un campione a una temperatura di 1000 K
viene investito frontalmente da un fascio di luce laser di elevata intensità la cui frequenza è
9
a cura della D.ssa G. Rusciano
6.2 Trappole
195
in risonanza con gli atomi che si muovono con velocità v0 . Quando la frequenza del campo
incidente coincide con una risonanza dell’atomo, α̃℘ (ω) può diventare molto grande e con
essa la pressione di radiazione. Tra i primi a notare questo fatto fu all’inizio del 1900 il
fisico russo P.N. Lebedev.
Assorbendo un fotone, l’atomo passa al primo stato eccitato di energia E = 2.097 eV
e larghezza Γ = 10.566 M Hz mentre la sua velocità varia della quantità ∆v1 = v1 −v0 .
In seguito l’atomo ritorna al proprio stato fondamentale emettendo di nuovo un fotone,
subendo una variazione di velocità ∆v1′ = v′1 −v1 e modificando la direzione del moto di un
angolo φ. Questa serie di processi di assorbimento-emissione si ripete molte volte finché la
velocità degli atomi si riduce di una quantità ∆v per cui non è più possibile l’assorbimento
di fotoni alla frequenza ν di risonanza. Questo effetto è ampiamente sfruttato oggi in
spettroscopia per esercitare azioni meccaniche su atomi mediante fasci laser.
La tecnica di raffreddamento laser non funziona per le molecole a causa della complessa
struttura dei livelli elettronici, vibrazionali e rotazionali
La tecnica si fonda sulla pressione di radiazione di un fascio laser a frequenza ω ℓ
esercitata su un atomo risonante a frequenza ω f i e che si muove con velocità v. Su di esso
si eserciterà una forza radiativa F(rad) pari a:
F(rad) ≃
σ f i max
πcγ
Iℓ
1+
ωℓ −|ωf i |
γ
2 n̂ ,
in cui n̂ indica la direzione di volo dei fotoni, ω = ω ℓ (1 − v·n̂/c) è la frequenza vista
dall’atomo e γ sta per la larghezza della transizione. Se l’atomo è investito da due fasci
laser di frequenza ω ℓ provenienti rispettivamente da sinistra (n̂) e da destra (−n̂) risulta
sottoposto ad una forza risultante (v. Fig. (6.24)).
Il tasso R± con cui gli atomi diffondono fotoni è dato da
R± =
κγ
1+ 1±
2ωℓ
β
γ
2
(6.55)
dove κ sta per il parametro di saturazione
κγ ω ℓ
σ f i max
=
c
πcγ
σ f i max
κγ =
πγ ω ℓ
κ=
I˜in (ω) dω
I˜in (ω) dω
tasso di eccitazione
tasso di rilassamento
D’altra parte ogni evento di assorbimento o emissione impartisce all’atomo un impulso
∆p± = ±
ω
c
Pertanto la forza media sull’atomo dovuta ai fasci che si propagano in direzioni opposte
196
Elettrodinamica
è data da
F (rad) = ∆p+ R+ − ∆p− R−

=
κγ ω ℓ 

c
= −8
κ ω 2ℓ
c
1
1+ 1+
2ωℓ
β
γ
β
4+
2ωℓ
β
γ
4,
2
−
1
1+ 1−
2ω ℓ
β
γ


2
(6.56)
Essendo F(rad) opposta a v·n̂n̂ l’atomo viene sottoposto ad una forza viscosa che tende
a ridurre la componente v·n̂ della velocità. Colpendo l’insieme di atomi con più fasci laser,
provenienti da direzioni distribuite sull’intero angolo solido, si indurrà un frenamento
collettivo delle velocità atomiche nelle varie direzioni.
6.2.1
MOT
Per raffreddare un insieme di atomi e tenerli confinati in una piccola regione si utilizzano
trappole magnetoottiche10 (MOT) che combinano l’azione di gradienti di campi magnetici,
generati p.e. da coppie di bobine di Helmholtz, e di fasci laser contropropaganti nelle 3
direzioni x,y e z, per intrappolare e raffreddare atomi. Le MOT riducono per effetto della
pressione di radiazione le agitazioni termiche. Tale tecnica va sotto il nome di raffredamento laser 11 (laser cooling) e fu proposta da Wineland, Dehmelt, Hänsch e Schawlow12
nel ’75 e dimostrata sperimentalmente da Wineland et al. nel ’78.
Una trappola magneto-ottica (MOT) (v. Fig. (6.25) utilizza sia il raffreddamento
laser che l’intrappolamento magneto-ottico per produrre campioni di atomi intrappolati a
temperature dell’ordine dei µK. Sfruttando un gran numero di processi di assorbimentodecadimento spontaneo con sezioni d’urto dipendenti dalla posizione atomi con velocità
inziali di centinaia di m/s si possono raffreddare fino decine di cm/s.
L’intrappolamento magnetico è ottenuto aggiungendo un campo magnetico di quadrupolo ad un campo laser utilizzato per il raffreddamento laser. Ciò determina uno spostamento Zeeman nei livelli, che aumenta col crescere della distanza dal centro. Pertanto la
(6.55) si modifica in
κγ
R± =
2
1 + 1 ± 2ωγ ℓ β ± 2gµγ0 B
10
11
12
D. E. Pritchard, E. L. Raab, V. Bagnato, C. E. Wieman and R. N. Watts, Phys. Rev. Lett. 57,
310313 (1986); E. Raab, M. Prentiss, A. Cable, S. Chu and D. Pritchard, Phys. Rev. Lett. 59, 2631
(1987)
Il premio Nobel per la fisica fu assegnato nel 1997 a Chu, Cohen Tannoudji e Phillips per aver sviluppato
le tecniche di raffreddamento di molasse di atomi sfruttando la pressione di radiazione indotta da vari
fasci laser confluenti verso il bersaglio da varie direzioni.
v. H. J. Metcalf and P. van der Straten, J. Opt. Soc. Am. B 20, 887 (2003);––, “Laser Cooling
and Trapping”, Springer-Verlag, N. Y. 1999; E. Arimondo, W. D. Phillips, and F. Strumia, ”Laser
Manipulation of Atoms and Ions”, Proc. International School of Physics ”Enrico Fermi” Course
CXVIII, North-Holland, Amsterdam, 1992.
6.2 Trappole
197
Figura 6.23: Andamento schematico della sezione d’urto di un elettrone atomico in funzione della frequenza nell’approssimazione di una risonanza singola.. Si possono individuare tre regioni: bassa frequenza (o regione di Rayleigh) in cui la polarizzabilità è costante
e si osserva una variazione con la quarta potenza di ω; una regione di risonanza, in cui
l’elettrone viene eccitato ad un livello risonante, ed infine una regione in alta frequenza
in cui l’elettrone risponde solo al campo incidente e si comporta quindi come l’elettrone
libero della diffusione alla Thomson.
Figura 6.24: Pressione di radiazione agente su un atomo investito da due fasci
contropropaganti in funzione della velocità assiale
198
Elettrodinamica
Figura 6.25: Rappresentazione schematica di una trappola MOT. Gli atomi da intrappolare e raffreddare vengono investiti da tre coppie di fasci conrpropaganti diretti lungo gli
assi x,y e z. Il campo magnetico quadrupolare è utilizzato per modulare spazialmente per
effetto Zeeman la frequenza di transizione, determinando così l’intrappolamento.
Figura 6.26: Schema di una trappola MOT 1D. Gli atomi sono sottoposti ad una pressione
di radiazione dipendente dalla posizione lungo l’asse, ottenuta variando spazialmente il
detuning per effetto Zeeman. In basso sono indicati gli spostamenti dei livelli accoppiati
con i due fasci con polarizzazioni circolari σ ± .
6.2 Trappole
199
Conseguentemente la forza (6.56) si modifica in
F
(rad)
κ ω 2ℓ
= −8
c
β+
4+
2ωℓ
β
γ
gµ0 B
ωℓ
+
2 gµ0γB
4,
La novità sta nel fatto che F (rad) dipende oltre che dalla velocità β dalla posizione
lungo l’asse attraverso la dipendenza di B da z. Quando gli atomi sono diventati freddi al
punto da poter trascurare β nella espressione di F (rad) per un campo di quadrupolo con
B = αz F (rad) si riduce a
αz
κω ℓ
gµ0
F (rad) = −8
4,
c
4 + 2 gµ0γαz
Pertanto un atomo che si allontana dal centro della trappola F (rad) agisce come una forza
di richiamo verso il centro.
Dal momento che un atomo normale possiede un momento pari a migliaia di volte
quello di un fotone, il raffreddamento di un atomo richiede molti cicli di assorbimentoemissione spontanea, con l’atomo che perde un momento k per ciclo. Pertanto perchè
un atomo possa venir raffreddato interagendo con un laser, deve possedere una specifica
struttura di livelli, in cui sia presente un ciclo ottico chiuso in cui alla fine di un evento
eccitazione-emissione spontanea, l’atomo ritorna sempre al suo stato iniziale. Per esempio
il 85 Rb presenta un ciclo ottico chiuso tra lo stato 5 S1/2 F = 3 ed il 5 P3/2 F = 4. Una
volta nello stato eccitato l’atomo non può decadere in un 5 P1/2 , che non conserverebbe la
parità, ed in 5 S1/2 F = 2, che richiederebbe un cambio di −2 del momento angolare.
Molti atomi non contengono cicli ottici chiusi. In tal caso vengono utilizzati dei laser
per rieccitare la popolazione nel ciclo ottico dopo che questi atomi sono finiti fuori dal
ciclo. L’intrappolamento magneto-ottico del 85 Rb comporta il ciclo 5 S1/2 F = 3 →5 P3/2
F = 4. Per l’eccitazione il detuning necessario per il raffreddamento si accompagna ad
una leggera sovrapposizione con lo stato 5 P3/2 F = 3. Se l’atomo viene eccitato a questo
stato, può decadere sia sull’F = 3, debolmente accoppiato allo stato iperfine superiore,
o a quello più basso F = 2. Se ricade nello stato dark, l’atomo viene escluso dal ciclo
fondamentale⇔stato eccitato, interrompendo così il raffreddamento e l’intrappolamento.
Per evitare tutto ciò viene utilizzato un laser di ripompaggio, risonante con la transizione
5
S1/2 F = 2 →5 P3/2 F = 3.
6.2.2
Trappola di Paul
Gli ioni possono essere intrappolati utilizzando opportune configurazioni di campi elettrici
statici ed oscillanti13 . L’invenzione di trappole ioniche 3D a quadrupolo è dovuta14 a W.
Paul a cui fu assegnato il premio Nobel per la fisica nel 1989.
Con riferimento ad una terna cartesiana x,y,z la trappola 3D consiste di due elettrodi
metallici a forma di iperboli di rivoluzione con fuochi sull’asse z e di un elettrodo ad anello
iperbolico posto a metà strada ta i primi due. Gli ioni vengono intrappolati nello spazio
tra questi tre elettrodi applicando contemporamente un campo oscillante ed uno continuo.
13
14
per una presentazione generale v.p.e. P. K. Ghosh, Ions Traps, Oxford Univ. Press, Oxford 1995.
W. Paul, H. P. Reinhard and U. von Zahn, Zeit. fur Physik, 152 143 (1958); W. Paul, Rev. Mod.
Phys., 62 531 (1990)
200
Elettrodinamica
Figura 6.27: Struttura iperfine delle transizioni del Rb utilizzate nel raffreddamento laser
Esercizio 6.2.1 Dimostrare che utilizzando come elettrodi una coppia di iperboli di rivoluzione attorno a z di equazione X 2 + Y 2 − γ Z Z 2 = CZ ed un terzo elettrodo ottenuto
per rotazione di una sezione iperbolica di equazione R2 − γ ρ Z 2 = Cρ attorno all’asse z si
può ottenere un potenziale di quadrupolo
Soluzione: Dall’equazione di Laplace
∆2 V (r) = 0
discende che il più generale potenziale di quadrupolo di rotazione attorno a z è del tipo
V (r) =
x2 + y 2 − 2z 2
r02
Dovendo V (r) risultare costante sulla prima coppia di elettrodi e sull’elettrodo a forma
di anello si deve avere rispettivamente
X 2 + Y 2 − γ Z Z 2 = CZ
X 2 + Y 2 − 2Z 2
= VZ
r02
e
R2 − γ ρ Z 2 = Cρ
R2 − 2Z 2
= Vρ
r02
Pertanto deve risultare γ Z = γ ρ = 2. Inoltre si deve avere
∆V = VZ − Vρ = U + V cos (Ωt) =
CZ − Cρ
r02
La tensione AC è applicata all’ellettrodo a forma d’anello. Gli ioni sono prima sbattuti
assialmente su e giù mentre sono spinti radialmente. Gli ioni sono estratti radialmente
6.2 Trappole
201
Figura 6.28: Rappresentazione schematica del campo in una trappola di Paul. La trappola
è circondata da una bobina che crea un campo magnetico che obbliga le particelle cariche
a spiralizzare attorno a B. A loro volta gli elettrodi creano un potenziale di quadrupolo
oscillante proporzionale a U + V cos (Ωt) .
e spinti dentro assialmente. Così facendo la nuvola ionica oscilla tra una configurazione
lunga e stretta e corta e larga.
In definitiva la trappola a quadrupolo crea un potenziale elettrico a forma di punto
sella del tipo
1
V = 2 (U + V cos (Ωt)) x2 + y 2 − 2z 2
r0
con Ω la frequenza del potenziale applicato all’elettrodo anulare.
Ne discende che il moto del singolo ione è descritto dal sistema di equazioni di Mathieu
(v. Eq. (1.24))
4 d2 x
4 2e
+ 2
(U + V cos (Ωt)) x = 0
2
2
Ω dt
Ω mr02
4 d2 y
4 2e
+ 2
(U + V cos (Ωt)) y = 0
2
2
Ω dt
Ω mr02
4 d2 z 4
4e
−
(U + V cos (Ωt)) z = 0
2
2
2
Ω dt Ω
mr02
2e
2 2e
Se i parametri a = ± Ω42 mr
2 U, q = ± Ω2 mr 2 V sono tali da cadere nelle regioni tratteggiate di
0
0
Fig. ?? la carica oscilla in una regione molto piccola attorno al baricentro della trappola.
In particolare debbono essere sodisfatte le condizioni
bn+1 (q) ≥ a ≥ an (q) stabilità piano x,y
bn+1 (2q) ≥ 2a ≥ an (2q) stabilità asse z
Il fatto di dover soddisfare entrambe le condizioni rende critico la scelta dei parametri.
Per migliorarne la stabilità si può aggiungere un campo magnetico (v. Fig. (6.28)) assiale
che contribuisce alla stabilizzazione delle traiettorie sul piano x-y.
202
Elettrodinamica
6.2.3
Trappola di Penning
Le trappole di Penning sono utilizzate per intrappolare particelle cariche utilizzando un
campo magnetic uniforme ed uno elettrico non omogeneo. Questo tipo di trappola è
utilizzata per misure di precisione delle proprietà di ioni e di particelle subatomiche stabili.
Esse utilizzano un campo magnetico uniforme per confinare le particelle radialmente ed
un campo elettrico di quadrupolo per confinarle assialmente. Il campo elettrico è generato
utilizzando tre elettrodi: un anello e due endcaps. L’anello ed i due elettrodi hanno la
forma di iperboloidi di rivoluzione. Le particelle si muovono nel piano radiale composto
di due modi a frequenza di magnetrone e di ciclotrone modificata. La somma di queste
due frequenze è la frequenza di ciclotrone.
6.3
Campi di cariche in movimento
6.3.1
Potenziali di Liénard-Wiechert
Esercizio 6.3.1 Si consideri un elettrone che descrive una traiettoria rettilinea lungo
l’asse z con velocità costante v in un mezzo di indice di rifrazione n indipendente dalla
frequenza. Si calcolino i potenziali di Liénard-Wiechert elativi al punto r = (ρ, φ, 0),
V (r, t) = −
A(r, t) =
e
1
4πε0 n |r − r (te )| 1 − nn̂ · β (te )
n
β (te ) V (r, t) .
c
dove il tempo di emissione te da parte dell’elettrone e quello di rivelazione t del campo
sono legati dalla relazione:
t = te + nc−1 |r − r (te )|
(6.57)
Soluzione: L’elettrone descrive una traiettoria rettilinea z (te ) = vte . Pertanto si ha
|r − r (te )| =
(vte)2 + ρ2 . Risolvendo (6.57) rispetto a te si ottiene:
te =
1−
Pertanto
c
|r − r (te )| =
n
e
1−n2 β 2 2
ρ
c2
2
n2 β
t − n β 2 t2 +
β 2 t2 +
1−n2 β 2 2
ρ
c2
2 2
1−n β
1
1
=
2
1 − nn̂ · β (te )
1 + (nβ) 1 −
− n2 β 2 t
ct
n|r−r(te )|
,
.
Perchè la (6.58) abbia significato deve risultare
β 2 c2 t2 + 1 − n2 β 2 ρ2 > 0
Per nβ > 1 al tempo t di misura del campo corrisponde un te reale solo se
β
ct
>
ρ
n2 β 2 − 1
(6.58)
6.3 Campi di cariche in movimento
203
Per quanto iguarda i potenziali si ha
V (r, t) = −
= −
1
1
en
4πε0 c |r − r (te )| 1 + (nβ)2 1 −
en2 1 − (nβ)2
4πε0 c3 1 + (nβ)2
ct
n|r−r(te )|
1
β 2 t2 +
en2 1 − (nβ)2
n
A(r, t) = − β (te )
c
4πε0 c3 1 + (nβ)2
1−(nβ)2 2
ρ
c2
−
2(nβ)2
t
1+n2 β 2
1
β 2 t2 +
1−(nβ)2 2
ρ
c2
−
2(nβ)2
t
1+n2 β 2
Esercizio 6.3.2 Calcolare i potenziali vettori e scalari del campo irradiato da un elettrone
che descrive un’assegnata orbita,
r = r (t)
Questo problema fu affrontato inizialmente da Schott nel 1912 [?].
Soluzione: Per risalire ai campi si deve associare alla traiettoria la densità di corrente
J(r,t) = δ (3) (r − r (t)) v (t)
La trasformata di Fourier della corrente sarà quindi data da
1
2π
1
= −e
2π
J̃ (r,ω) = −e
δ(3) (r − r (t)) v (t) eiωt dt
δ(3) (r − r (s)) v̂ (s)
eiωtk (s) ds
k
dove s è l’ascissa curvilinea lungo la traiettoria e tk (s) il k-esimo istante di tempo corrispondente ad una assegnata ascissa s. Una volta ottenuta la densità di corrente J̃ (r,ω)
il potenziale à (r,ω) ed il campo Ẽ (r,ω) si otterranno utilizzando le rappresentazioni
integrali
à (r,ω) = µ0
e
2π
e
Ẽ (r,ω) = −
2π
G (|r − r′ (s)| , k) v̂ (s) δ (2) (r′⊥ (s))
Γ (|r − r′ | , k) · v̂ (s) δ (2) (r′⊥ (s))
eiωtk (s) ds
k
eiωtk (s) ds
k
con G (|r − r′ | , k) e Γ (|r − r′ | , k) rispettivamente funzioni i Green scalare e tensoriale.
Esercizio 6.3.3 Calcolare J̃ (r,ω) e J (k,t) per un elettrone che descrive una traiettoria
rettilinea lungo l’asse z con (a) una generica legge del moto s = s (t), (b) velocità costante
vo ẑ; (c) velocità oscillante v (t) = vo cos (Ωt) ẑ; e (d)
s = vo t + sa arctan
t
τa
204
Elettrodinamica
Soluzione: (a)
J̃ (r,ω) = −
e (2)
δ (ρ) ẑ
2π
eiωtk (z)
k
con tk (z) radice dell’Eq. z = z (t) e
J (k,t) = −
e ikz z(t)
v̂ (t)
e
2π
(b) tenuto conto che s = vo t si ha
J̃ (r,ω) = −
ω
e (2)
δ (ρ) v̂o ei vo z
2π
e
J (k,t) = −
(c) in tal caso s =
vo
Ω
sin (Ωt) e tk (s) =
J̃ (r,ω) = −
1
Ω
e ikz vo t
v̂o
e
2π
arcsin
Ω
z
vo
+
ω
Ω
e
vo δ (2) (ρ) ei Ω arcsin( vo z)
2π
e
J (k,t) = −
2π
k.
Ω
Pertanto
2π
ei Ω ωk
k
e ikz vo sin(Ωt)
e Ω
v̂ (t)
2π
(d)
J̃ (r,ω) = −
e (2)
δ (ρ) ẑeiωt(z)
2π
con t (z) radice dell’Eq. z = vo t + sa arctan τta e
J (k,t) = −
e ikz (vo t+sa arctan τt )
a ẑ
e
2π
Esercizio 6.3.4 Si completi l’esercizio precedente calcolando J̃ (r,ω) per un elettrone
che descrive un’orbita (a) circolare di raggio R0 con velocità angolare costante Ω e (b)
spiraliforme con x = R0 cos Ωt, y = R0 sin Ωt, z = vz t.
Soluzione: (a)
J̃ (r,ω) = −
e (1)
δ (z) δ (1) (ρ − R0 )
2π
ω
ei Ω φk
k
(b)
J̃ (r,ω) = −
e iω vz (1)
e o δ (ρ − R0 ) R0 δ (1)
2π
φ−Ω
z
vo
2π
6.3 Campi di cariche in movimento
6.3.2
205
Effetto Stewart-Tolman
Si tratta di un effetto di tipo elettrodinamico in virtù del quale in un conduttore in
moto non uniforme si genera una forza elettromotrice non nulla, anche nelle condizioni
in cui l’applicazione della legge di induzione di Faraday-Neumann prevederebbe risultato
nullo. Storicamente gli esperimenti sul moto non uniforme dei conduttori consentirono
di dimostrare l’esistenza di elettroni liberi nei conduttori e di determinarne il rapporto
carica-massa. Sia v la velocità del sistema di riferimento Kcond , solidale col conduttore,
rispetto al sistema di laboratorio KL . Giacché si suppone che l’accelerazione v̇ sia non
nulla, gli elettroni di conduzione risentiranno di una forza inerziale pari a −me v̇. Questa
esercita sull’elettrone lo stesso effetto che creerebbe un campo elettrico di intensità me v̇/e.
Nell’esperimento di Stewart-Tolman (vedi figura ??) si imprimeva ad un anello metallico immerso in un campo magnetico uniforme e costante, un moto rotatorio a velocità
angolare sinusoidale. Dalla misura delle correnti circolanti si risaliva al rapporto The
Stewart—Tolman effect is a phenomenon in electrodynamics caused by the finite mass of
electrons in conducting metal, or, more generally, the finite mass of charge carriers in an
electrical conductor.
It is named after T. Dale Stewart and Richard C. Tolman, two American physicists
who carried out their experimental work in the 1910s.[1] This eponym appears to be first
used by Lev Landau.[2]
In a conducting body undergoing accelerating motion, inertia causes the electrons
in the body to lag behind the overall motion. In the case of linear acceleration, negative charge accumulates at the end of the body; while for rotation the negative charge
accumulates at the outer rim. The accumulation of charges can be measured by a
galvanometer.
This effect is proportional to the mass of the charge carriers. It is much more significant
in electrolyte conductors than metals, because ions in the former are 103-104 times more
massive than electrons in the latter.[1]
Esercizio 6.3.5 Con riferimento all’esperimento di Stewart-Tolman (a) ricavare le equazioni
di Maxwell relative a Kcond . (b) Analizzare la forza e.m. indotta dalle variazioni temporali
della velocità angolare Ω (t) in un anello metallico ruotante attorno al suo asse.
Soluzione: Il campo elettrico efficace agente sugli elettroni di conduzione è dato da
me
Econd = EL +
v̇
(6.59)
e
′
Se si esprime E in funzione di E e si sostituisce nella equazione di Maxwell
∇ × E = −µ0
∂H
∂t
si ottiene
∂HL me
+
∇ × v̇
(6.60)
∂t
e
Decomponendo il movimento in traslatorio (velocità u) e rotatorio (velocità angolare Ω):
∇ × Econd = −µ0
v =u+Ω×r
e derivando rispetto al tempo, si ha per l’accelerazione:
v̇ = u̇ + Ω × v + Ω̇ × r = u̇ + Ω × u + Ω × (Ω × r) + Ω̇ × r .
206
Elettrodinamica
I primi due termini sono indipendenti da r per cui si annullano per derivazione rispetto alle
1
coordinate; il terzo termine può essere scritto nella forma: Ω× (Ω × r) = − ∇ |Ω × r|2 ,
2
il cui rotore è nullo. Resta, infine, ∇× Ω̇ × r = 2Ω̇, sicché la (6.60) si riscrive
∇ × Econd = −µ0
ovvero
dove si è introdotto il campo
∂HL 2me
+
Ω̇
∂t
e
∇ × Econd = −µ0
∂Hcond
∂t
(6.61)
2me
Ω.
eµ0
Si consideri ora la quarta equazione di Maxwell15
Hcond = HL −
∇×H=J
dove J è la densità di corrente. Questa conserva la sua forma se si esprime H in funzione
′
di H , essendo Ω indipendente dalle coordinate
′
∇ × Hcond = σEcond
(6.62)
Eliminando E dalle (6.61) e (6.62) e tenendo conto che ∇ · HL = ∇ · Hcond = 0 si ottiene
per Hcond :
∂Hcond
∇2 Hcond = µ0 σ
(6.63)
∂t
coincidente con quella soddisfatta da HL in un conduttore immobile.
All’esterno del conduttore HL verifica l’equazione ∇2 HL = 0; lo stesso vale per Hcond .
Sulla superficie del conduttore sia Hcond che HL sono continui, mentre diversa è la
2me
condizione all’infinito: HL tende a zero, Hcond a un limite finito: −
Ω.
eµ0
Riassumendo un corpo in rotazione con velocità angolare dipendente dal tempo visto
nel sistema del conduttore Kcond si comporta come se fosse fermo e sottoposto ad un
campo magnetico esterno Hext omogeneo:
2me
Hext = −
Ω (t) .
(6.64)
eµ0
Quando il campo magnetico Hext varia nel tempo induce nel conduttore una forza elettromotrice non nulla16 che genera correnti elettriche.
(b) Un anello conduttore in rotazione non costante con velocità angolare Ω (t) (??),
si ha in Kcond
!
∂H
2me
′
′
′
∇ × E · ds =
E · dℓ = f = −µ0
· ds +
Ω̇ · ds
S
S ∂t
S eµ0
dV
2me
2me
= −
+
Ω̇ · S =
Ω̇ · S
dt
eµ0
eµ0
ovvero la f.e.m. è proporzionale alla superficie dell’anello e alla derivata della velocità
angolare.
15
16
∂E
Si sta implicitamente assumendo che il campo sia quasi-stazionario, per cui σ/ω ≫ 1 e il termine
∂t
sia trascurabile.
Si consulti il Landau, Elettrodinamica dei mezzi continui, par. 64
6.3 Campi di cariche in movimento
6.3.3
207
Interazioni tra cariche in movimento
Esercizio 6.3.6 Due elettroni, distanti R tra loro lungo l’asse y, viaggiano parallelamente
ed a velocità costante v lungo l’asse x. Calcolare la forza di interazione
Soluzione: Nel sistema proprio KP gli elettroni si respingono con una forza
FP =
e2
ŷ
4πε0 R2
parallela ad ŷ.
D’altra parte la forza FL nel sistema di laboratorio KK è espressa da:
FL = −e (EL + v × BL )
dove
EL
BL
=
=
=
γ + (1 − γ) β̂ β̂
γv×
γ
− c2 v×
γ + (1 − γ) β̂ β̂
·
EP
0
γEP + (1 − γ) β̂ β̂ · EP
− cγ2 v × EP
γEP
− cγ2 v × EP
Pertanto
FL = −e γEP −
6.3.4
γ
v × (v × EP ) = −e 1 − β 2 EP =
c2
1 − β 2 FP
Densità spettrale energia e potenza irradiata
Esercizio 6.3.7 Esprimere la potenza irradiata a grande distanza da una distribuzione
di correnti
Soluzione: A grande distanza da una distribuzione di correnti contenute in un volume
finito il campo si riduce ad un’onda piana diretta lungo n̂ = k̂. Pertanto
Ẽ (r, ω) = i
µ0 ω
8π 2
J̃⊥ (k, ω)
eik·r
d3 k
k 2 − ñ2 k02
∞
π
k2
eikr cos θ sin θdθ dk
2 − ñ2 k 2
r→∞
k
0
0
0
∞
µ0 ω
k2
sin kr
= i
J̃⊥ (ñk0 n̂, ω)
dk
4π
k 2 − ñ2 k02 kr
0
1 ∞ x sin x
µ ω
= i 0 2 J̃⊥ (ñk0 n̂, ω)
dx
4π
r 0 x2 − ñ2 k02 r2
eikr
µ0 ω
J̃⊥ (ñk0 n̂, ω)
= i
2
4πr
→i
µ0 ω
J̃⊥ (ñk0 n̂, ω)
4π
208
Elettrodinamica
con J̃⊥ (ñk0 n̂, ω) = (1 − n̂n̂) · J̃ (ñk0 n̂, ω) . Pertanto la densità spettrale d2 E/dωd2 Ω di
energia irradiata17 lungo n̂ per unità di angolo solido è espressa da:
d2 E (n̂, ω)
=
dωd2 Ω
2
4π 2
R Ẽ (R, ω)
R→∞ ζ
4π µ20 ω 2
J̃⊥ (ñk0 n̂, ω)
=
ζ (8π)2
lim
2
(6.65)
.
In alternativa si può calcolare la densità spettrale di energia irradiata utilizzando la
√
trasformata di Fourier spaziale J⊥
ε̃k0 n̂, t della distribuzione di corrente e ponendo
′
′
T = (t + t ) /2 e τ = t − t :
∞
√
d2 E (n̂, ω)
4πµ0 2
∗
J
ε̃k0 n̂, t e−iωt dt ·
=
ω
⊥
2
dωd Ω
c
−∞
2
d P (T )
=
dT
dωdΩ
∞
−∞
J⊥
√
′
ε̃k0 n̂, t′ eiωt dt′
d2 P (T ) /dωdΩ rappresenta la densità spettrale della potenza irradiata al tempo T :
∞
d2 P (n̂, ω, T )
4πµ0 2
=
ω
dωdΩ
c
−∞
√
√
ε̃k0 n̂, T + τ /2 · J⊥
ε̃k0 n̂, T − τ /2 dτ
e−iωτ J∗⊥
(6.66)
Inoltre risulta:
∞
2
J̃⊥ (ñk0 n̂,ω)
=
−∞
∞
=
′
e−iωt J∗⊥ (kn̂,t′ )dt′ ·
dt
−∞
Esercizio 6.3.8 Si calcoli
terzultimo esercizio.
d2 E(n̂,ω)
dωd2 Ω
∞
∞
−∞
′′
eiωt J⊥ (kn̂,t′′ )dt′′
1
e−iωτ J∗⊥ kn̂,t + τ
2
−∞
e
d2 P (n̂,ω,t)
dωd2 Ω
1
· J⊥ kn̂,t − τ
2
dτ
per i casi (a), (b) e (d) considerati nel
Soluzione: (a)
d2 P (n̂, ω, t)
1
e
=
2
dωd Ω
2π 2π
∞
2
µ0
4π
2
ζ −1 (ω) ω 2 sin2 θ
1
exp −iωτ − ikz z t + τ
2
−∞
1
1
×v̂ t + τ · v̂ t − τ dτ
2
2
×
1
+ ikz z t − τ
2
(b)
d2 P (n̂, ω, t)
1
e
=
2
dωd Ω
2π 2π
17
2
µ0
4π
2
ζ −1 (ω) ω 2 sin2 θδ ω −
ω
ñ (ω) v cos θ
c
J. Schwinger, ”Classical Electrodynamics”, a cura di L.L. de Raad, K. A. Milton, Wy-yangTsai Adv.
Book Program, Boulder, 1998, Sez. 35.1 Eq. (35.32).
6.3 Campi di cariche in movimento
ovvero
d2 P (n̂,ω,t)
dωd2 Ω
209
è indipendente da t e diverso da 0 per
ñ (ω) v cos θ = c
ovvero l’elettrone irradia lungo un cono di apertura θ (cf 6.5).
(c)
d2 P (n̂, ω, t)
1
e
=
2
dωd Ω
2π 2π
∞
−∞
=
0
µ0
4π
2
ζ −1 (ω) ω 2 v02 sin2 θ
exp −i (ω − 2kz vo ) τ − i2kz sa arctan
e
1
2π 2π
4π
Ω
2
2
µ0
4π
2
t + 12 τ
t − 12 τ
− arctan
τa
τa
dτ
ζ −1 (ω) ω 2 v02 sin2 θ
exp −iωτ − i2kz
1
vo
cos Ωt sin
Ωτ
Ω
2
Esercizio 6.3.9 Utilizzando la formula di Larmor per la potenza irradiata da un elettrone
accelerato,
2 e2 a2
P =
(f ormula di Larmor)
(6.67)
3 4πε0 c3
essendo ”a” l’accelerazione, descrivere l’orbita di un elettrone in un campo magnetico B
Soluzione: Si mmagini che l’elettrone descriva un’orbita piana sotto l’azione di un
campo B0 uniforme diretto lungo B0 = B0 ẑ. Ignorando le perdite per irragiamento esso
ruoterà alla frequenza di ciclotrone
ω cicl = −
e
B0
me
dove il segno − tiene conto del fatto che per B0 > 0 l’elettrone descrive un’orbita circolare
in senso antiorario rispetto a ẑ. D’altra parte
1
A = B0 × r
2
per cui
1
1
H=
(p+eA)2 =
2me
2me
D’altra parte
per cui
ovvero
con
1
−me ω cicl ẑ×r + eB0 ẑ×r
2
d
2 e2 a2
2 e2 ω 4cicl 2
H=−
=
−
r
dt
3 4πε0 c3
3 4πε0 c3
d 2
16 e2 ω 2cicl 2
r =−
r
dt
3 4πε0 me c3
r (t) = r (0) e−t/τ
32 e2 ω 2cicl
τ=
3 4πε0 me c3
2
1
= me ω 2cicl r2
8
210
Elettrodinamica
6.4
Interazione atomi-sciami di particelle cariche
6.4.1
Esperimento di Franck ed Hertz
Esercizio 6.4.1 (a) Calcolare il campo generato da un elettrone che viaggi con velocità costante v in un mezzo di costante dielettrica ε̃ (ω); (b) calcolare il campo a grande
distanza in funzione della frequenza ω
Soluzione: (a) Utilizzando l’espressione (6.69) della trasformata Ẽ(k,ω) del campo
elettrico generato dall’elettrone
∞
∞+iǫ
1
3
Ẽ(r,t) =
dk
dωe−i(ωt−k·r) Ẽ(k,ω)
4
(2π) −∞
−∞+iǫ
∞+iǫ
∞
1 eµ0
1
−iωt
= −i
dωe
ω
d3 keik·r 2
2
4
k − ε̃ ωc2
(2π) 4π −∞+iǫ
−∞
1
2π
=
∞+iǫ
v−
c2
kk · v δ (ω − k · v)
ε̃ω 2
Ẽ(r,ω)e−iωt dω
−∞+iǫ
col cammino di integrazione in ω spostato di iǫ con ǫ > 0 e ǫ → 0 al fine di garantire
Ẽ(r,t) → 0 , si ha
t→−∞
∞
eik·r
c2
Ẽ(r,ω) = C
dk
v − 2 kk · v δ (ω − k · v)
2
ε̃ω
k 2 − ε̃ ωc2
−∞
∞
ik⊥ ·r
e
c2
= C
d2 k⊥ 2
k +k⊥
v
−
2
ε̃ω
k⊥ + ωv2 1 − ε̃β 2
−∞
∞
= C
0
3
2π
k⊥ dk⊥
2
2
k⊥
+ ωv2 1 − ε̃β 2
avendo posto
C = −i
0
eik⊥ ρ cos φ Ẽ(k⊥ ,ω, φ)dφ
1 eµ0 i ω z
ωe v
(2π)4 2
e posto
Ẽ(k⊥ ,ω, φ) =
1−
1
ε̃β 2
v−
c2
k⊥ ρ̂ cos φ
ε̃ω
Integrando rispetto a φ
1
π
1
π
π/2
−π/2
dφeik⊥ ρ cos φ Ẽ(k⊥ ,ω, φ) ≡ Ẽa (k⊥ ρ,ω)
1
= (J0 + iH0 ) 1 − 2
ε̃β
c2
v − (iJ1 + H−1 ) k⊥ ρ̂
ε̃ω
3π/2
π/2
dφeik⊥ ρ cos φ Ẽ(k⊥ ,ω, φ) ≡ Ẽb (k⊥ ρ,ω)
= (J0 − iH0 ) 1 −
1
ε̃β 2
v − (iJ1 − H−1 )
c2
k⊥ ρ̂
ε̃ω
6.4 Interazione atomi-sciami di particelle cariche
211
con J0,1 = J0,1 (k⊥ ρ) funzioni di Bessel di ordine 0 e 1, e H0,−1 = H0,−1 (k⊥ ρ) quelle di
Struve (v. Eq. 1.19), si ottiene
∞
Ẽ(r,ω) = πC
0
k⊥ dk⊥
2
2
k⊥
+ ωv2 1 − ε̃β 2
Ẽa (k⊥ ρ,ω) + Ẽb (k⊥ ρ,ω)
= πC Ẽa (r,ω) + Ẽb (r,ω)
Per calcolare questi ultimi integrali conviene modificare il cammino di integrazione nel
piano complesso di k⊥ = |k⊥ | eiϕk tenendo conto che eik⊥ ρ cos φ tende a 0 per |k⊥ | → ∞
quando sono soddisfatte le coppie di condizioni (a) cos φ > 0, π2 ≥ ϕk ≥ 0 e (b) cos φ <
0, 0 ≥ ϕk ≥ − π2 . D’altra parte l’integrando presenta un polo in
k̄⊥ =
ω + iǫ
v
ε̃β 2 − 1
k̄⊥ giace nel semipiano superiore per ε̃′ β 2 > 1 (I) ed a sinistra dell’asse immaginario con
Re k̄⊥ > 0 per ε̃′ β 2 < 1 (II).
∞
i∞
Per Ẽa il cammino di integrazione 0 · · · dk⊥ può essere modificato in 0 aggiungen∞
−i∞
dovi il contributo del residuo in k̄⊥ mentre per Ẽb 0 · · · dk⊥ va modificato in 0
senza
aggiungervi il residuo,
i∞
Ẽa (r,ω) =
0
2
k⊥
+
ω2
v2
−i∞
Ẽb (r,ω) =
0
2
k⊥
+
1
Ẽa (k⊥ ρ,ω)k⊥ dk⊥ + iπ Ẽa (k̄⊥ ρ,ω)
1 − ε̃β 2
ω2
v2
1
Ẽb (k⊥ ρ,ω)k⊥ dk⊥
1 − ε̃β 2
(b) Per calcolare il campo a grande distanza conviene utilizzare le espressioni asintotiche
delle funzioni di Struve
1
Hα (x) = Yα (x) + √
πΓ α +
1
2
x
2
α−1
+O
x
2
α−3
da cui discende
2
π
exp ix − i
πx
4
J0 (x) + iH0 (x) ≈
iJ1 (x) + H−1 (x) ≈ i (J1 (x) + iY1 (x)) ≈ i
2
π
π
exp ix − i − i
πx
2
4
Ne segue che
c2
π
xρ̂ exp ix − i
ε̃ωρ
4
Ẽa (x,ω) ≈
2π
x
1−
1
ε̃β 2
v−
Ẽa (ix,ω) ≈
2π
x
1−
1
ε̃β 2
v−i
c2
xρ̂ exp (−x)
ε̃ωρ
212
Elettrodinamica
Pertanto
Ẽ(r,ω) = iCπ 2 Ẽa (k̄⊥ ρ,ω)
eµ0 2πωv
1
=
2
32π
ρ
ε̃β 2 − 1
× exp i
ω
v
z+
1
ε̃β 2
1−
1/4
v−
c2
ε̃v
ε̃β 2 − 1ρ̂
π
4
ε̃β 2 − 1ρ − i
1/4
2πωv ε̃β 2 − 1
ε̃β 2 − 1β − βρ̂
2
ρ
ε̃β
π
ω
× exp i
z + ε̃β 2 − 1ρ − i
v
4
eµ0 c
=
32π 2
Se ne evince che l’elettrone irradia lungo le direzioni di un cono di apertura
1
tan θ =
ε̃β 2 − 1
Appare chiaro che i campi così creati sono ben lontani dalle onde piane. Le ampiezze
possono variare significativamente su distanze molto minori delle lunghezze d’onda. Non
c’è quindi da sorprendersi che essi diano luogo in un atomo a transizioni solitamente
proibite nell’approssimazione di dipolo. Questi campi, caratterizzati da un ampio spettro
di frequenze, sono responsabili delle eccitazioni dei vari livelli elettronici per impatto elettronico. A differenza delle eccitazioni con onde piane, in quest’ultimo caso il campo magnetico non risulta più trascurabile rispetto alla componente elettrica e può quindi indurre
transizioni altrimenti proibite. Un caso interessante si ha ad esempio nella eccitazione di
stati di tripletto nell’elio.
(b) Per calcolare la potenza persa dall’elettrone si può utilizzare il teorema dell’energia
nella forma
1
Re J̃∗ · Ẽ d3 r
2
2
1
= − ω
µ0 Im (µ̃) H̃ + ε0 Im (ε̃) Ẽ
2
−
2
3
d r+
!
Re S̃ ·n̂d2 r
Dal momento che l’integrale di Re S̃ si annulla estendendo l’integrazione a tutto lo
spazio e per Im (µ̃) = 0 si ha
∗
∞
3
Re J̃ · Ẽ d r = 2πωε0 Im (ε̃)
2
ρ Ẽ dρ
0
D’altra parte
∞
0
ω
ρ Ẽ dρ = e2 µ20
ε̃
2
c
= e2 µ20
ε̃
2
2
∞
0
∞
0
ρ
"
k⊥
k0
2
k⊥
k0
2
"
k⊥
k0
2
#
K02 (k⊥ ρ) + β −2 K0′2 (k⊥ ρ) dρ
#
K02 (x) + β −2 K0′2 (x) dx
6.4 Interazione atomi-sciami di particelle cariche
213
Figura 6.29: Rappresentazione schematica dell’interazione tra un atomo ed un elettrone
sufficientemente veloce. La nuvola di elettroni atomici viene eccitata dalle componenti
spettrali del campo prodotto dall’elettrone “proiettile” a frequenze “f” coincidenti con le
risonanze dell’atomo. Perché ciò avvenga la regione in cui si estende il campo a frequenza
f deve comprendere la nuvola elettronica che circonda l’atomo.
k0
l’integrale si riduce
con K0 funzione di Bessel modificata di seconda specie. Per k⊥ ≃ βγ
a
∞
∞
2
e2 µ 1
C
ρ Ẽ dρ = 2 0 2
γ −2 K02 (x) + K0′2 (x) dx = 2
β ε0 |ε̃| 0
|ε̃|
0
con
∞
e2 µ
γ −2 K02 (x) + K0′2 (x) dx
C= 2 0
β ε0 0
Ne segue che
∗
3
Re J̃ · Ẽ d rdω = 2πε0
= 2πε0 C
∞
ω Im (ε̃)
2
ρ Ẽ dρdω
0
ω Im (ε̃)
dω
|ε̃|2
Esercizio 6.4.2 Un elettrone viaggia con velocità v passando in prossimità di un atomo.
Calcolare il campo che agisce sull’atomo visto nel sistema proprio dell’elettrone.
Soluzione: Tenendo conto delle trasformazioni di Lorentz del campo (v. Es. ??) si ha
per la componente di E perpendicolare a v
E⊥ (t) = γ
e
b
2
2
4πε0 (b + γ v 2 t2 )3/2
Questo campo avrà la forma di un impulso tanto più breve quanto più veloce è l’elettrone.
Poiché l’atomo si comporta classicamente come un oscillatore che risuona alla frequenza
214
Elettrodinamica
Figura 6.30: Sezione d’urto di eccitazione dell’idrogeno in funzione dell’energia degli elettroni. Le curve continue sono teoriche (cf. H.S.W. Massey and E.H.S. Burhop, “Electronic and Ionic Impact Phenomena” , Clarendon Press, Oxford (1952)) mentre quella
tratteggiata è stata misurata da W.L. Fite et al., Phys. Rev. vol. 116, pg. 356 (1959).
ω 0 , l’effetto di E⊥ (t) dipenderà dalla componente spettrale18 (v. Problemi precedenti)
Ẽ⊥ (ω 0 ) =
1
2π
∞
−∞
E⊥ (t) exp (iω 0 t) dt =
e 1 1
ξK0 (ξ)
4πǫ0 γvb π
(6.68)
dove K0 sta per la funzione di Bessel modificata di ordine 0 mentre ξ = ω 0 b/ (γv). Questo
parametro ξ misura il rapporto tra la durata della collisione τ ∼ b/γv ed il periodo d’oscillazione dell’elettrone legato all’atomo. L’energia ∆E trasferita all’atomo è proporzionale
a
1, ξ ≪ 1
2
1
1
∆E ∝ Ẽ⊥ (ω 0 ) ∝ 2 ξ 2 K02 (ξ) ≈ 2
1 + γ12 π2 ξ exp(−2ξ), ξ ≫ 1
b
b
Esercizio 6.4.3 Si consideri un atomo bombardato da uno sciame di particelle cariche.
Quando una di queste si avvicina e supera l’atomo, quest’ultimo viene sottoposto ad un
campo elettrico impulsivo che, a sua volta, può produrre una eccitazione o, al limite, la
ionizzazione del bersaglio. Tale fenomeno fu osservato nel 1906 da Lenard, che notò una
notevole perdita di energia da parte di una corrente di elettroni in una ampolla contenente
gas. Successivamente, Franck ed Hertz utilizzarono questo apparato per misurare i livelli
di energia di varie specie atomiche. Calcolare (a) lo spettro del campo prodotto ed (b) il
tasso di perdita di energia da parte di una particella carica che attraversa un gas.
Soluzione: (a) Per studiare l’interazione tra un elettrone che si muove di moto uniforme
con velocità v = vẑ ed un atomo, conviene rappresentare il primo come una corrente di
18
V. p.e. J.D. Jackson, loc. cit. pag. 61 Cap. 13.
6.4 Interazione atomi-sciami di particelle cariche
215
densità
J(r, t) = −evδ (3) (r − vt)
Trasformando rispetto al tempo, si vede facilmente che l’elettrone in moto equivale ad
una sovrapposizione di correnti sinusoidali J pari a
∞
J(k, ω) =
J(r, t)ei(ωt−k·r) d3 rdt
−∞
= −ev
= −ev
∞
∞
−∞
∞
−∞
δ (3) (r − vt) ei(ωt−k·r) d3 r dt
ei(ω−k·v)t dt
−∞
= −2πeδ (ω − k · v) v
che genera il potenziale vettore Ã(k,ω)
Ã(k,ω) = −µ0
ed il potenziale scalare
Ṽ (k,ω) =
associati al campo elettrico
Ne segue che
J̃(k, ω)
2πδ (ω − k · v)
v
2 = −eµ0
2
ω
k 2 − ε̃ c2
k 2 − ε̃ ωc2
c2
ec2 2πδ (ω − k · v)
k · Ã = −
k·v
2
ε̃ω
ε̃ω
k 2 − ε̃ ωc2
Ẽ(k,ω) = iω Ã(k,ω) − ikṼ (k,ω)
µ
1
c2
= −ie 0 2
ω
v
−
kk · v δ (ω − k · v)
2
4π k − ε̃ ωc2
ε̃ω 2
Ẽ(k,ω) · v = −i
1
e 2πδ (ω − k · v)
2
ω
β
−
2
ω
ε0
ε̃
k 2 − ε̃ c2
Pertanto la perdita di energia dell’elettrone per unità di tempo è data da
dE
−
= −ev·E(vt,t)
dt
e2
2πδ (ω − k · v)
1
=
Im
ω β2 −
d3 kdω
2
ω
2
ε0
ε̃
k − ε̃ c2
Integrando rispetto alla direzione di k rispetto a v
1
−1
δ (ω − k · v) d (cos θ) =
1
η (kv − |ω|)
kv
con η (x) la funzione gradino unirario. Pertanto
dE
dt
e2
=
Re
4πε0 v
−
∞
K2
−iω
ω2
2
−∞ ω2 /v 2 k − ε̃ c2
∞
1
e2
Re
(−iω) β 2 −
=
4πε0 v
ε̃
−∞
β2 −
ln
1
dk 2 dω
ε̃
K 2 v2
dω
ω 2 1 − ε̃β 2
(6.69)
216
Elettrodinamica
2
con K 2 ≫ |ε̃| ωc2 . Sostituendo dt con dz/v si ha
−
∞
dE
e2
Im
=
dz
4πε0 c2
0
ω 1−
1
ε̃β 2
2
=
e
(I + II)
4πε0 c2
ln
K 2v2
dω
ω 2 1 − ε̃β 2
dove
∞
I = Im
0
∞
II = Im
0
Intanto
I =−
ω 1−
1
ω 1− 2
ε̃β
∞
1
β2
ω Im
0
Dal momento che
Im
1
ε̃
ln
K 2 v2
dω
ω2 1 − β 2
1 − β2
ln 2
dω
ω 1 − ε̃β 2
1
K 2v2
ln 2
dω
ε̃
ω 1 − β2
q (ω ′ )
π ω 2p
′
dω
=
q (ω)
2 ω
ω ′2 − (ω + iǫ)2
∞
= −ω 2p
1
ε̃β 2
0
con
q (ω) ≥ 0
∞
q (ω) dω = 1
0
per cui
K 2v2
1 π 2 ∞
q
(ω)
ln
dω
ω
β2 2 p 0
ω2 1 − β 2
∞
1 π 2
K 2 v2
1
=
q (ω) ln 2 dω
2 ω p ln
2 +
ω
β 2
1−β
0
2 2
1 π 2
K v
=
2 ω p ln
β 2
1 − β 2 ω 2e
I =
dove ω e è definita da
∞
1
1
dω = ln 2
(6.70)
2
ω
ωe
0
ω e rappresenta una energia efficace di eccitazione degli atomi costituenti il dielettrico.
Il cammino di integrazione ω ∈ (0, ∞) del secondo integrale II può essere sostituito
da Γ1 ∪ Γ2 con Γ1 = i (0, ∞) e Γ2 l’arco di circonferenza compreso tra i∞, ∞.
II = Im
Γ1 +Γ2
q (ω) ln
ω 1−
1
ε̃β 2
Dal momento che ε̃ risulta reale lungo Γ1
ε̃ (iω) = 1 + ω 2p
∞
0
ln
1 − β2
dω = IIΓ1 + IIΓ2
1 − ε̃β 2
q (ω ′ )
dω ′ < ε̃ (0) , q (ω ′ ) ≥ 0
ω ′2 + ω 2
6.4 Interazione atomi-sciami di particelle cariche
si ha
i∞
IIΓ1 = Im
ω 1−
0
217
1
ε̃β 2
Fintantochè la velocità è così bassa per cui
ln
1 − β2
dω
1 − ε̃β 2
ε̃ (0) β 2 < 1
risulta IIΓ1 = 0. Quando β supera il valore di soglia 1/ ε̃ (0) per 0 < iω < iω v con ω v ,
radice dell’equazione
β2 =
1
= 1 − ω 2p
ε̃ (iω v )
∞
0
q (ω ′ )
dω ′
ω ′2 + ω 2v
risulta ε̃ (0) β 2 > 1 e EΓ′′1 è espresso da
1 − β2
1
Im
dω
ε̃β 2
1 − ε̃β 2
0
iωv
1
π
ω 1 − 2 dω
ε̃β
0
iωv
∞
q (ω ′ )
π
2
2
ω
β
−
1
+
ω
dω ′ dω
p
′2 + ω 2
ω
β2 0
0
∞
π
ω ′2 + ω 2v ′
1 π
2
2
2
′
−
1
ω
−
ω
q
(ω
)
ln
dω
β
v
2 β2
ω ′2
β2 p 0
2
1 π 2
ω 2e
2 ωv
− 2 ωp 1 − β
+ ln 2
2β
ω 2p
ω ve
iωv
IIΓ1 =
=
=
=
=
ω 1−
dove la frequenza efficace ω ve è definita in modo analogo a (6.70)
∞
q (ω ′ ) ln
0
ω ′2
1
1
dω ′ = ln 2
2
+ ωv
ω ve
Poichè per |ω| ≫ ω atom
ε̃ ≈ 1 −
si ha
ln
per cui
IIΓ2 =
1−
1
β2
ω 2p
ω2
ω 2p β 2
1 − β2
≈
−
ω2 1 − β 2
1 − ε̃β 2
ω Im ln
Γ2
Sommando i vari contributi si ottiene
−
(6.71)
1 − β2
1 − ε̃ (ω) β 2
π
dω = − ω 2p
2
2
e2 π 2
dE
K 2 γ 2 β 2 c2
2
2 ωv
=
ω
ln
−
β
+
1
−
β
dz
4πε0 v 2 2 p
ω 2ve
ω 2p
(6.72)
218
Elettrodinamica
Esercizio 6.4.4 Confontare l’espressione di
spressione di Bethe-Bloch
dE
dz
ricavata nel precedente esercizio con l’e-
me c2 Zz 2
2γ 2 β 2 me c2
dE
ε δ (β)
= 2C 2
ρ ln
− β2 − −
dz
A
EI
2
2
β
con
C = 2πNA
e2
4πε0 mec2
2
= 0.03006 kg −1 m2
Z ed A numero atomico e massa del mezzo attraversato, z=carica in unità di e della
particella proiettille, ρ= densità del mezzo ed EI energia media di ionizzazione; ε sta per
la correzione deguscio e δ (β) per l’effetto densità
Soluzione: Se si confronta la (6.72) con quella di Bethe si deduce che energia media di
ionizzazione EI che compare in quest’ultima risulta legata alla frequenza efficace ω ve ed
a K dalla relazione:
ω2
EI = 2me ve2
K
Felix Bloch ha mostrato nel 1933 che EI è approssimativamente rappresentato da
EI = 10 · Z eV .
6.5
Radiazione Cerenkov
Esercizio 6.5.1 Calcolare la densità spettrale della potenza irrradiata da una particella
carica che si muove con velocità costante
Soluzione: Per una particella che si muove con velocità uniforme v = cβ diretta lungo
l’asse z si ha
k0
J̃(r,ω) = −ecβδ (2) (ρ) exp i z
β
Pertanto, utilizzando la funzione di Green scalare 2D
i (1)
G(2) (ρ, k⊥ ) = H0 (k⊥ ρ)
4
si ottiene per il potenziale vettore nel caso di un mezzo omogeneo di indice di rifrazione
ñ (ω)
i k0 (1) k0
Ã(r,ω) = −ecβµ0 ei β z H0
ñ (ω)2 β 2 − 1ρ
(6.73)
4
β
(1)
Se ne evince che l’argomento della funzione di Hankel risulta reale, ed H0 si comporta
√
come una funzione oscillante che decade come 1/ ρ, solo se la velocità risulta maggiore
(1)
di c/ñ (ω) . Nel caso opposto l’argomento di H0 risulta immaginario ed Ã(r,ω) decade
esponenzialmete con la distanza ρ. Si vede quindi che nel caso di moto uniforme un
ruolo critico viene giocato dall’indice di rifrazione del mezzo ñ (ω) . Quando ñ (ω) > v/c
l’elettrone emette radiazione Cerenkov di frequenza ω.
Per calcolare la potenza irradiata si inserisce nell’Eq. (??) la trasformata di Fourier
J (kn̂, t) = −ecβ exp (ickn̂ · β)
6.6 Radiazione di frenamento (Bremsstrhalung)
219
Figura 6.31: Radiazione di bremsstrhalung emessa nella collisione tra due particelle (da
www4.nau.edu)
della corrente ottenendo così per la densità spettrale della potenza irradiata:
d2 P (n̂, ω, t)
ω 2 e2
=
|n̂ × βñ (ω)|2 2πδ (ω [1 − ñ (ω) n̂ · β])
dωd2 Ω
4π 2 c
(6.74)
Si vede quindi che l’elettrone irradia potenza lungo le direzioni n̂ che formano con v un
angolo θ tale che
1 1
cos θ =
,
β ñ (ω)
ovvero il diagramma di radiazione Cerenkov19 ha la forma di un cono. Integrando rispetto
a d2 Ω si ottiene per la densità spettrale potenza irradiata
dP (ω, t)
ωe2 v
= 2
dω
c
6.6
1−
ñ2
1
(ω) β 2
Radiazione di frenamento (Bremsstrhalung)
Esercizio 6.6.1 Calcolare il campo irradiato da una particella carica soggetta ad una
variazione brusca della velocità (Radiazione di frenamento (Bremsstrahlung))
Soluzione: Per una particella che subisce una variazione brusca di velocità passando
da v2 per t < 0 a v1 per t > 0 si muove con velocità uniforme si ha per la trasformata
della densità di corrente
1
v1
v2
J (kn̂, ω) = i e
−
ω
1 − n̂ · β 1 1 − n̂ · β 2
19
v.p.e. J. Schwinger, loc. cit. pag. 208, Cap. 36; J. D. Jackson, loc. cit. pag. 61, Sez. 13.4; D.
Marcuse, loc. cit. pag. 131, Sez. 4.6.
220
Elettrodinamica
Figura 6.32: From: Elements of X-ray Diffraction, B.D.Cullity, Addison-Wesley
Publishing, Third Edition, 1967
6.7 Radiazione di sincrotrone
221
che sostiuita nella Eq. (??) dà la distribuzione spettrale20 dell’energia irradiata lungo la
direzione n̂,
2
e 2
d2 E (n̂, ω)
β1
β2
=
−
(6.75)
dωd2 Ω
2πc
1 − n̂ · β 1 1 − n̂ · β 2
L’espressione ricavata dell’energia irradiata di bremsstrahlung poggia sull’ipotesi di variazione brusca della velocità nel corso di una collisione di un elettrone con un atomo od
un nucleo. Espressioni accurate di queste perdite che tengono conto della dinamica delle
collisioni sono state sviluppate da molti autori, a partire dalla teoria di Bethe-Heitler21
Esercizio 6.6.2 Si consideri un elettrone diffuso da un bersaglio. Descrivendo la traiettoria come costituita da due semirette percorse rispettivamente convelocità costanti v1 e
v2 . calcolare: (a) la trasformata k,ω della densità di corrente, (b) la densità spettrale
Soluzione: La corrente è data da
J (r, t) = −e
Pertanto
v1 δ (3) (r − v1 t) t < 0
v2 δ (3) (r − v2 t) t > 0
0
J̃ (k, ω) = lim −ev1
ǫ→0
d3 r
∞
−∞
dtei(ω+iǫ)t−ik·r δ (3) (r − v1 t)
−ev2 d3 r
dtei(ω+iǫ)t−ik·r δ (3) (r − v2 t)
0
"
#
e
v2
v1
= i
−
ω 1 − k̂ · β 2 1 − k̂ · β 1
La densità spettrale per angolo solido è data da
2
d2 E
ω2
=
k̂
×
J̃
(k,
ω)
dωdΩ
4π 2 c3
#
"
ee
v1
v2
=
k̂ ×
−
4π 2 c3
1 − k̂ · β 2 1 − k̂ · β 1
6.7
2
Radiazione di sincrotrone
La radiazione di sincrotrone è generata da particelle cariche, solitamente elettroni o
positroni, che viaggiano a velocità prossime alla velocità della luce e vengono costrette
da un campo magnetico a muoversi lungo una traiettoria curva. Tanto più elevata è
la velocità della particella, tanto minore è la lunghezza d’onda della radiazione emessa.
Generalmente il picco dell’emissione avviene alle lunghezze d’onda dei raggi X. Questa
radiazione viene utilizzata per diversi scopi: in litografia per la produzione di chip per
computer, negli studi di assorbimento e scattering, nella cristallografia di proteine e molecole complesse, nella spettroscopia per l’analisi dei materiali, in medicina per la diagnosi
per immagini e la terapia tumorale, nell’ambito dei Beni culturali ( datazioni, attribuzioni,
tecniche pittoriche etc.).
20
21
v.p.e. J. D. Jackson, loc. cit. pag. 61, Eq. (15.2) e Cap. 15; J. Schwinger, loc. cit. pag. 208, Sez.
37.2 Eq. 37.31.
H. A. Bethe and W. Heitler, Proc. Phys. Soc. London 146, 83 (1934); v. anche D. Marcuse, loc. cit.
pag. 131, Sez. 4.3 per la discussione della rad. di bremsstrahlung stimolata.
222
Elettrodinamica
Figura 6.33: Geometria relativa ad una carica che descrive un’orbita circolare. L’osservatore in P sarà investito da un impulso di radiazione emesso mentre l’elettrone percorre
l’arco di traiettoria indicato in figura. Per utilizzare questa radiazione si utilizzano anelli
di accumulazione in cui, come schematizzato in (b), gli elettroni passano attraverso dei
magneti curvanti, emettendo impulsi di radiazione.
Esercizio 6.7.1 La radiazione di sincrotrone è prodotta da particelle che sottoposte ad
un campo magnetico B descrivono orbite spiraliformi con velocità di modulo costante v.
Mentre la componente vcos α parallela a B è costante, il moto sul piano perpendicolare a
B avviene lungo una circonferenza di raggio assegnato
rB =
v sin α
ωs
con frequenza di sincrotrone
ωs =
1 |q| sin αB
.
γ
m
con α l’angolo formato da B col piano dell’orbita circolare descritta da
r (t) = rB (cos (ω s t) x̂ + sin (ω s t) ŷ) .
Per elettroni relativistici di energia E = me c2 γ si ha
ωs =
1 e
1
sin αB = ω c
γ me
γ
con ω c frequenza di ciclotrone. Calcolare (a) il potenziale vettore à (r,ω) a grande distanza; i campi elettrici (b) Ẽm (Rn̂, ω) e (c) E (Rx̂, t); la densità spettrale della (d) potenza
istantanea, (e) potenza media nel periodo di rivoluzione, (f) energia e (g) potenza totale
irradiate
6.7 Radiazione di sincrotrone
223
Soluzione:(a) A r (t) corrisponde una densità di corrente distribuita trasversalmente
all’orbita circolare sun una sezione infinitesima,
J (r, t) = −erB ω s (− sin (ω s t) x̂ + cos (ω s t) ŷ) δ (3) (r − r (t))
= −eω s δ (2) (r⊥ ) (− sin φ x̂ + cos φ ŷ)
∞
m=−∞
δ (φ − ω s t − 2πm)
dove δ (2) (r⊥ ) = δ (ρ − rB ) δ (z). Essendo J (r, t) una funzione periodica del tempo, la relativa trasformata di Fourier si compone di una sequenza infinita di armoniche di ampiezzesi
ha
J̃ (r,ω) = −eδ
(2)
∞
=
m=−∞
con
∞
ω
(r⊥ ) (− sin φx̂ + cos φŷ) exp i φ
ωs
m=−∞
δ (ω − mω s )
J̃m (r) δ (ω − mω s ) ,
J̃m (r) = −eδ (2) (r⊥ ) (− sin φ x̂ + cos φ ŷ) eimφ
densità di corrente che agisce come sorgente dell’armonica m-esima. Il campo presenta
uno spettro discreto con armoniche distanziate della frequenza di rivoluzione ω s (v. Fig.
??).
(a) Per calcolare il potenziale vettore nel punto r =rx̂ posto a distanza sufficientemente
grande si può utilizzare l’integrale
à (r,ω) =
µ0
4πr
(2)
eik|r−R| J̃ (ω, s) dsdr⊥ ≃ µ0 G
(2)
e−iks J̃ (s, ω) dsdr⊥
con G = exp (ikR) / (4πR). Pertanto
à (rx̂, ω) ≃ −eµ0 rB G
= −eµ0 rB G
∞
2π
m=−∞
∞
m=−∞
δ (ω − mω s )
0
2π
δ (ω − mω s )
ω
e−ikrB cos φ+i ωs φ (− sin φx̂ + cos φŷ) dφ
ω
e−ikrB cos φ+i ωs φ (− sin φx̂ + cos φŷ) dφ
0
D’altra parte ricorrendo all’identità di Jacobi (v. Eq. (1.21))
e−ix cos φ =
∞
(−i)n Jn (x) einφ
n=−∞
con Jn (x) la funzione di Bessel di prima specie di ordine n, si ha
à (rx̂, ω) = −eµ0 GrB
∞
m=−∞
δ (ω − mω s )
2π
×
ei(m+n+1)φ
0
= eµ0 2πrB G
m
×
∞
(−i)n Jn (krB )
n=−∞
ix̂ + ŷ
−ix̂ + ŷ
+ ei(m+n−1)φ
dφ
2
2
im δ (ω − mω s )
Jm+1 (|m| β ⊥ ) + Jm−1 (|m| β ⊥ )
Jm+1 (|m| β ⊥ ) − Jm−1 (|m| β ⊥ )
ŷ + i
x̂
2
2
224
Elettrodinamica
dove
ω s rB
c
(b) Anche il campo elettrico nel generico punto di osservazione E(rn̂, t) è una funzione
periodica di frequenza ω s , che si compone di armoniche Ẽm (rn̂, ω) associate alle correnti
J̃m (r, ω) . Facendo uso dell’identità di Jacobi e tenendo conto delle relazioni (??) si ottiene
per l’armonica22 Ẽm (rn̂, ω) corrispondente a J̃m lungo la direzione n̂ = sin θx̂ + cos θẑ (v.
Eq. (??) per ω = mω s ):
β⊥ =
2π
Ẽm (rn̂, ω) = imω s rB µ0 G (r, k) 1⊥ ·
e−ikm n̂·r J̃m (r,ω) dr2⊥ dφ
0
2π
= −iecmβ ⊥ µ0 G (r, k) 1⊥ ·
e−imβ ⊥ sin θ cos φ+imφ
0
(− sin φ x̂ + cos φ ŷ) eimφ dφ
= ecmβ ⊥ µ0 G (r, k) X cos2 θx̂ − iY ŷ
con β ⊥ =
ωs rB
c
la velocità dell’elettrone normalizzata a quella della luce e
X=
Jm (mβ ⊥ sin θ)
′
, Y = Jm
(mβ ⊥ sin θ) .
β ⊥ sin θ
In particolare per m molto grande (v. Es. 6.7 Eq. (1.22))
Jm
exp − 13 mα3
m
exp (m (tanh α − α))
√
√
≈
≈
cosh α
2πmα
2πm tanh α
(6.76)
per cui
Jm (mβ ⊥ ) =
exp − 3γm3
2πm/γ
con γ = 1/ 1 − β 2⊥ il fattore relativistico. Quest’ultima espressione mostra che l’ordine
massimo delle armoniche è dell’ordine di γ 3 .
Lo spettro quindi si estende fino alla frequenza critica
ω crit = γ 3 ω s = γ 2 ω c
(6.77)
Dall’espressione asintotica (6.76) discende che
exp − 13 mδ 3
Ẽm (rn̂, ω) ∝ m √
2πmδ
con cosh δ = (β ⊥ sin θ)−1 . Pertanto Ẽm (rn̂, ω) risulta massima per m =
discende che lo spettro si estende fino alla frequenza angolare critica
3
ω crit = γ 3 ω s .
2
22
J. Schwinger, loc. cit. pag. 208, Sez. 38.2.
3 3
γ .
4
Ne
(6.78)
6.7 Radiazione di sincrotrone
225
Im
1.5
1.0
0.5
armonica m
50
100
150
200
250
300
Figura 6.34: Distribuzione delle intensità delle armoniche in funzione dell’ordine m.
(c) Il campo elettrico E (rx̂, t) presenta uno spettro discreto con armoniche distanziate
della frequenza di rivoluzione ω s
E (rx̂, t) = ieζ 0 β ⊥
1
r
m
m
Jm+1 (|m| β ⊥ ) − Jm−1 (|m| β ⊥ ) −imωs (t−r/c)
e
ŷ
2
(6.79)
Un osservatore viene colpito da una sequenza di impulsi di durata dell’ordine di 1/γ 3 volte il periodo di rivoluzione.
(d) La trasformata J (kn̂, t) della densità di corrente rispetto a r è data da
J (kn̂, t) = −ecβ ⊥ (− sin (ω s t) x̂ + cos (ω s t) ŷ) e−ikrB n̂·(cos(ωs t) x̂+sin(ωs t) ŷ)
Ne discende che
n̂ × J (kn̂, t)
= −eβ ⊥ c [cos (ω s t) (− cos θx̂ + sin θ ẑ) − sin (ω s t) cos θ ŷ]
×e−iβ ⊥ n̂·(cos(ωs t) x̂+sin(ωs t) ŷ) ,
per cui
2
1
1
∗
kn̂,t + τ
· n̂ × J kn̂,t − τ
2 n̂ × J
2
2
(eβ ⊥ c)
= [A cos (2ω s t) + B cos (ω s τ )] e−i2β ⊥ sin θ sin(ωs τ /2) sin ωs t .
dove A = cos2 θ, B = 1 + cos2 θ
Dall’espressione della densità spettrale della potenza irradiata lungo n̂ al tempo t nello
spazio libero (ζ̃ = 1)
1 µ0
d2 P (n̂, ω, t)
=
2
dωd Ω
2π 4π
2
ω
2
∞
1
e−iωτ n̂ × J∗ kn̂,t + τ
2
−∞
1
· n̂ × J kn̂,t − τ
2
dτ
226
Elettrodinamica
Figura 6.35: (a) Spettro della radiazione di sincrotrone emessa lungo il piano dell’orbita;
(b) andamento temporale della radiazione vista da un osservatore. Si nota che questa è
costituita da una sequenza periodica di impulsi con periodo pari al tempo T di circolazione
dell’elettrone. Il rapporto tra T e la durata del singolo impulso è circa uguale al numero
di armoniche presenti nello spettro.
6.7 Radiazione di sincrotrone
227
discende che
∞
d2 P (n̂, ω, t)
1
=
Cω 2
2
dωd Ω
4π
e−iωτ −ia sin ωs τ /2 sin ωs t [A cos (2ω s t) + B cos (ω s τ )] dτ
−∞
2
µ0
con C = 4π
eβ ⊥ c .
Utilizzando l’identità di Jacobi (1.21) si ha
−iωτ −ia sin
e
ωs τ
2
sin ωs t
=
∞
Jn (a sin ω s t) e−i(2 ωs +n)
ω
ωs
τ
2
.
n=−∞
Per cui
∞
=
=
e−iωτ −ia sin
−∞
∞
ωs τ
2
sin ωs t
∞
Jn (a sin ω s t)
n=−∞
∞
[A cos (2ω s t) + B cos (ω s τ )] dτ
e−i(2 ωs +n)ωs τ /2 [A cos (2ω s t) + B cos (ω s τ )] dτ
ω
−∞
Jn (a sin ω s t) A cos (2ω s t) δ 2
n=−∞
ω
+n
ωs
B
B
ω
ω
+ δ 2 +n+2 + δ 2 +n−2
2
ωs
2
ωs
2
P (n̂,ω,t)
Se ne evince che d dωd
oscilla nel tempo con frequenza ω s .
2Ω
(e) Mediando su un periodo T = 2π/ω s l’espressione
T
eia sin
ωs τ
2
sin ω s t
(A cos (2ω s t) + B cos (ω s τ )) dt
0
= (B cos (ω s τ ) − A) J0 (a sin ω s τ /2) + 2AJ1 (a sin ω s τ /2)
si ottiene per la densità spettrale media
' 2
' 2
(
(
d P (n̂, ω)
d Pm (n̂, ω)
=
δ (ω − mω s )
dωd2 Ω
d2 Ω
m
dove23 (v. Fig. ??)
'
d2 Pm (n̂, ω)
d2 Ω
(
β 2⊥ 2
2
2
Jm+1 + Jm−1
− Jm
2
∝ ω 2s m2
(6.80)
è espresso con funzioni di Bessel di argomento mβ ⊥ sin θ. Per m sufficientemente grande
queste ultime sono rappresentabili come in Eq. (6.76).
(f) Per ottenere la densità spettrale dell’energia irradiata conviene utilizzare l’espressione (??),
∞
−∞
ωe−iω(te −c
−iω(t̄e −c−1 n̂·r(t̄e ))
= e
) n̂ × (n̂ × β ) dt
e
e
−1 n̂·r(t )
e
e−iωmT
I (ω, θ)
m
23
v.p.e. J. Schwinger, loc. cit. pag. 208, Sez. 38.2 Eq. (38.37).
228
Elettrodinamica
dove t̄e è un tempo di riferimento generico, mentre
T /2
I (ω, θ) =
ωe−iω(τ −c
−1 n̂·[r(t̄
e +τ )−r(t̄e )]
)
−T /2
× −ǫ⊥ sin (ω s (t̄e + τ )) + ǫ cos (ω s (t̄e + τ )) sin θ dτ ,
avendo introdotto i vettori di polarizzazione ǫ⊥ e ǫ = n̂× ǫ⊥ , con ǫ⊥ nel piano dell’orbita e
perpendicolare a β (t̄e ) . Il modulo |I (ω)|2 rappresenta l’inviluppo della densità di energia
delle armoniche lungo la direzione n̂.
Scegliendo un riferimento t̄e tale che β (t̄e ) risulti tangente alla proiezione di n̂ sul
piano dell’orbita, sviluppando fino al terzo ordine in τ = te − t̄e si ottiene
1
β
τ − n̂ · [r (t̄e + τ ) − r (t̄e )] = τ − ⊥ cos θ sin (ω s τ )
c
ωs
1
≃ (1 − β ⊥ cos θ) τ + β ⊥ cos θω 3s τ 3
6
Per β ⊥ prossimo ad 1 e θ sufficientemente piccolo risulta
1
1
τ − n̂ · [r (t̄e + τ ) − r (t̄e )] ≈
c
2γ 2
1 c2 γ 2 3
τ+
τ
3 rB
2 2
1+γ θ
3β ⊥ 1 + γ 2 θ2
=
4ω crit
3/2
1
ξ + ξ3
3
,
e
− ǫ⊥ sin (ω s (t̄e + τ )) + ǫ cos (ω s (t̄e + τ )) sin θ =
con ω crit frequenza critica (v. (6.78)) e ξ =
β⊥
− ǫ⊥ sin (ω s τ ) + ǫ cos (ω s τ ) sin θ ,
√γ
1+γ 2 θ2
T /2
a
−T/2
ω s τ . Per γ molto grande si possono
estendere i limiti di integrazione di I (ω, θ) =
τ ∈ (−∞, ∞) ottenendo così
"
#
β ⊥ 1 + γ 2 θ2
β ⊥ 1 + γ 2 θ2
I (ω, θ) ≃
−ǫ⊥
K2/3 (ς) + ǫ θK1/3 (ς) ,
γ
γ
3/2
con ς = 2ωωcrit β ⊥ 1 + γ 2 θ2
(v. Eqq. (1.23)).
Tenuto conto che
e K1/3 , K2/3 funzioni di Bessel modificate di seconda specie
∞
m=−∞
si ha
∞
m=−∞
2
−iωmT
e
sin ω M +
M→∞
sin ωT
2
e−iωmT = lim
sin2 ω M +
= lim
M→∞
sin2 ωT
2
1
2
T
=
1
2
T
∞
m=−∞
δ (ω − mω s ) .
Pertanto la densità spettrale dell’energia
eµ0
d2 E (n̂, ω)
=
2
dωd Ω
4π
2
ω 2 |I (ω, θ)|2
m
δ (ω − mω s )
6.7 Radiazione di sincrotrone
229
si compone di due contributi24
d2 E⊥ (n̂, ω) d2 E (n̂, ω)
d2 E (n̂, ω)
=
+
dωd2 Ω
dωd2 Ω
dωd2 Ω
δ (ω − mω s ) ,
m
dove
d2 E⊥/ (n̂, ω)
eµ0
= ζ −1
0
2
dωd Ω
4π
2
β⊥
γ
4
ω 2 1 + γ 2 θ2
2
ω
2ω crit
2
K2/3
/ 1/3
.
L’andamento di questi contributi è simile a quello illustrato nelle Figg. 10.2 e 10.3 con
un picco per ω = ω crit /2.
(f) Integrando le precedenti espressioni su d2 Ω si ha
d2 E⊥/ (ω)
=C F
dω
ω
2ω crit
+/−G
ω
2ω crit
.
dove25
√ 2
3e γ sin α
C =
2c
∞
ω
ω
F
=
K5/3 (ξ) dξ
2ω crit
ω crit ω/ωcrit
ω
ω
ω
G
=
K2/3
2ω crit
ω crit
ω crit
con K1/3 , K2/3 , K5/3 funzioni di Besel modificate di seconda specie (v. Eqq. (1.23)).
In definitiva la densità spettrale di potenza totale irradiata è data da
√ 3
ω
3e B sin α
P (ω) =
F
.
2
2πε0 mec
2ω crit
Integrando su ω si perviene alla formula di Ivanenko e Pomeranchuk 26
1 e4 2 2 2
P =
γ β ⊥ B sin2 α
2
6πε0 c m
B2
= cσ T γ 2 β 2⊥
sin2 α ,
µ0
(6.81)
con σ T sezione d’urto di Thomson (v. Eq. (??))
σT =
e2
4πε0 me c2
2
= 7.94 × 10−30 m2
Un osservatore verrà colpito da una sequenza di impulsi di durata dell’ordine di
1/γ 3 − volte il periodo di rivoluzione (v. Fig. (??b)). Se si assimila lo spettro a righe ad
24
25
26
v.p.e. F. Melia, Electrodynamics, The Univ. of Chicago Press, 2001, Eqq. (8.158-159)
v.p.e. F. Melia, loc. cit. pag. 229 Eqq. (8.168).
D. Ivanenko and A. A. Sokolov, Sov. Phys. Dokl. 59, 1551 (1948); A. A. Sokolov and I. M. Ternov,
”Synchrotron Radiation”, Akademie-Velag, Berlin, Pergamon Press, Oxford, 1968.
230
Elettrodinamica
uno continuo si può dimostrare con modesti sforzi, utilizzando l’Eq. (6.79) con l’espressione approssimata (6.76) che la densità spettrale I(ω) della radiazione emessa sul piano
dell’orbita è proporzionale a
I (ω) ∝
ω
ω crit
2
2
K2/3
ω
ω crit
(6.82)
con K2/3 (x) funzione di Bessel modificata di seconda specie di ordine 2/3 (v. Fig. (??)).
Esercizio 6.7.2 Confrontare la potenza emessa da una sorgente di luce di sincrotrone
fornita dalla formula di Ivanenko-Pomeranchuk
P =
1 e4 2 2 2
γ β ⊥ B sin2 α
6πε0 c m2e
con la formula di Larmor
P =
2 e2 a2
.
3 4πε0 c3
(6.83)
Esercizio 6.7.3 Nella macchina di luce di sincrotrone di Grenoble si utilizzano elettroni
da 6 GeV con correnti di 100 mA, che descrivono orbite con raggio di curvatura di 25 m.
Calcolare (a) la lunghezza d’onda critica, (b) l’energia critica, (c) l’angolo di emissione,
(d) la potenza irradiata su tutta l’orbita.
Esercizio 6.7.4 Rispondere ai quesiti del precedente problema per la macchina Elettra
di Trieste che presenta le seguenti caratteristiche: energia elettroni 2 GeV , raggio di
curvatura 5.5 m, corrente I = 400 mA.
6.7.1
Radiazione di ciclotrone
Esercizio 6.7.5 Analizzare la radiazione emessa da elettroni in orbite circolari con γ
non molto elevato
Soluzione: A differenza della radiazione di sincrotrone prodotta da particelle con β ⊥
prossimo ad 1 si dà il nome di radiazione di ciclotrone a quella prodotta per γ non molto
elevato. Il nome deriva dal ciclotrone, un acceleratore di particelle utilizzato soprattutto
in fisica nucleare. Il periodo delle orbite è indipendente dall’energia delle particelle, il
che consente al ciclotrone di operare ad una determinata frequenza ω c indipendentemente
dalle energie delle particelle. La radiazione di ciclotrone emessa dal plasma nello spazio
interstellare o attorno a buchi neri o qualunque altro corpo celeste dotato di campo magnetico dà importanti indizi sulle caratteristiche di questi campi magnetici extraterrestri;
nel sistema solare, in particolare, una grande sorgente di radiazione di ciclotrone è la
magnetosfera del pianeta Giove. La potenza media P emessa de ciascun elettrone è data
dalla formula di Ivanenko-Pomeranchuk (v. Eq. (6.81)):
2
dE
2 2 B
−
= P = cσ T γ β ⊥
sin2 α
dt
µ0
dove E è l’energia, t il tempo, σ T è la sezione d’urto di Thomson mentre α è l’angolo
formato da B col piano dell’orbita.
6.7 Radiazione di sincrotrone
231
La radiazione di ciclotrone sarebbe prodotta in esplosioni nucleari ad alta quota. I raggi
gamma prodotti dall’esplosione ionizzerebbero gli atomi nell’atmosfera superiore e tali
elettroni liberi interagirebbero con il campo magnetico terrestre producendo radiazione di
ciclotrone come impulso elettromagnetico.
Lo spettro della radiazione di ciclotrone ha un picco principale alla frequenza (v.
(6.77))
ω crit = γ 3 ω s = γ 2 ω c
che per γ prossimo ad 1 coincide con quella dell’orbita della particella e armoniche a
multipli interi di quest’ultima. Quando le particelle si muovono a velocità relativistiche
la radiazione di ciclotrone è chiamata radiazione di sincrotrone.
6.7.2
Radiazione di ondulatore
Esercizio 6.7.6 Calcolare (a) la radiazione emessa da un elettrone di energia γ che si
muove sul piano y-z attraverso un campo magnetico statico
B (r) = B cos
2π
z x̂
λw
(b) la densità spettrale d2 E/dωd2 Ω di energia irradiata
Soluzione: A r (t) definito in Eqq. (6.25) e (6.26) corrisponde una densità di corrente
pari a
J (r, t) = −e (ż (t) ẑ + ẏ (t) ŷ) δ(3) (r − r (t))
"
#
−1
λw dt
K sin ζ
= −e
ẑ +
ŷ δ (z − z (t)) δ (y − y (z)) δ (x − x0 )
2π dζ
1 − K2 sin2 ζ
Ne segue che
2πN
′ dt
J (r, t) eiωt(ζ ) dζ ′
dζ
0
"
#
2πN
λw
K sin ζ ′
i ωω K (ζ ′ ,K2 )
= −e
e 0
ẑ +
ŷ
2π 0
1 − K2 sin2 ζ ′
λw ′
ζ δ (y − y (ζ ′ )) δ (x − x0 ) dζ ′
×δ z −
2π
"
#
K sin ζ
i ωω K (ζ,K2 )
= −e ẑ +
ŷ
δ
(y
−
y
(ζ))
δ
(x
−
x
0) e 0
1 − K2 sin2 ζ
J̃ (r, ω) =
J (r, t) e
iωt
dt =
con
y (ζ) =
Proseguendo si ha:
λw
1+K
ln
2π K cos ζ + 1 − K2 sin2 ζ
232
Elettrodinamica
J̃ (r, ω) e−ir·k d3 r
J̃ (k, ω) =
2πN
λw
= −e e−ix0 kx
2π
i
ω
K
ω0
"
K sin ζ
ẑ +
0
1 − K2 sin2 ζ
cβ
cβ
− ω kz ζ
(ζ,K2 )− ω0 ky ln K cos ζ+√1+K
0
1−K2 sin2 ζ
×e
ω
ω
= J˜z
n̂, ω ẑ + J˜y
n̂, ω ŷ
c
c
In particolare per ky = k sin θ, kz = k sin θ e k0 = ω/c si ha
λw
J˜z (k0 n̂, ω) = −e
2π
λw
J˜y (k0 n̂, ω) = −e
2π
2πN
i ωω ψ(ζ)
e
0
2πN
dζ
K sin ζ
i ωω ψ(ζ)
1 − K2 sin2 ζ
0
e
0
1+K
ψ (ζ) = K ζ, K2 − β sin θ ln
K cos ζ + 1 − K2 sin2 ζ
Per K abbastanza piccolo si può approssimare K (ζ, K2 ) con
K ζ, K2 ≃ ζ +
ln
per cui
ψ (ζ) =
K cos ζ +
dζ
0
con
e porre
ŷ
#
1+K
ζ sin 2ζ
−
4
8
1 − K2 sin2 ζ
dζdζ
− β cos θζ
K2
≃ K (1 − cos ζ)
1
K2
1 + K2 − β cos θ ζ − β K (1 − cos ζ) sin θ −
sin 2ζ
4
8
Ne segue che
λw −i β
J˜y (k0 n̂, ω) = ie Ke ω0
4π
ω
2πN
K sin θ
e
i
ω
ω0
(1+ 14 K2 −β
cos θ)−1 ζ i ωω
e
0
0
i ωω
Ignorando pr il momento il fattore e
0
2
β K sin θ cos ζ− K8 sin 2ζ
si ha:
λw N −i ωω β K sin θ i ωω (1+ 14 K2 −β )−1 πN
J˜y (k0 n̂, ω) = ie
Ke 0
e 0
4
ω
1
sin c
1 + K2 − β cos θ − 1 πN
ω0
4
dove sinc (x) =
sin(x)
.
x
Quest’ultima espressione risulta massima per
ω0
ω0
ω = ωs =
≃
1 2
β
1 + 4 K − β cos θ
1 − β + 14 K2 + 2 θ 2
=
2γ 2 ω 0
1 + γ2
1 2
K
2
+ θ2
2
β K sin θ cos ζ− K8 sin 2ζ
dζ
6.7 Radiazione di sincrotrone
233
Si ritrova così l’espressione di ω s ricavata nell’Esercizio 6.1.14.
2
i2γ 2 K sin θ cos ζ− K8 sin 2ζ
Per esaminare il contributo di e
Jacobi (v. Eq. (1.21)), da cui segue che:
i2γ 2 K sin θ cos ζ
e
2
−iγ 2 K4
e
sin θ sin 2ζ
∞
=
n=−∞
∞
=
(−i)n Jn 2γ 2 K sin θ einζ
Jm γ 2
m=−∞
∞
2
i2γ 2 K sin θ cos ζ− K8 sin 2ζ
e
si può utilizzare l’identità di
K2
sin θ eim2ζ
4
Cq eiqζ
=
q=−∞
dove
∞
Cq =
m=−∞
(−i)q−2m Jq−2m 2γ 2 K sin θ Jm γ 2
K2
sin θ
4
Se ne deduce che
λw N −i ω β
J˜y (k0 n̂, ω) = ie
Ke 0
4
ω
sin c
ω
ω0
∞
K sin θ
i
Cq e
ω
ω0
(1+ 14 K2 −β )−1−q
πN
q=−∞
1
1 + K2 − β cos θ − 1 − q πN
4
ovvero J˜y ωc ẑ, ω è composto da una fondamentale a frequenza ω s e da armoniche multiple
di ω s . In particolare l’ampiezza di
C0 =
∞
m=−∞
(−1)m J2m 2γ 2 K sin θ Jm γ 2
≃ J0 2γ 2 K sin θ J0 γ 2
K2
sin θ
4
K2
sin θ
4
(b) Pertanto la densità spettrale d2 E/dωd2 Ω di energia irradiata lungo n̂ per unità di
angolo solido (6.65) è espressa da:
d2 E (n̂, ω)
4π µ20 ω 2
=
dωd2 Ω
ζ (8π)2
eλw NK
4
2
sin c
ω
ω0
1
1 + K2 − β cos θ − 1 πN
4
2
.
Indice analitico
plasma, 116
di modi
cavità elettromagnetica, 129
di probabilità ampiezza campo E, 131
densità spettrale
energia irradiata, 208, 233
potenza irradiata, 208
densita spettrale
energia irradiata
radiazione di bremsstrahlung, 221
enrgia irradiata
luce di sincrotrone, 223
potenza irradiata
istantanea luce di sincrotrone, 223
media luce di sincrotrone, 223
totale luce di sincrotrone, 229
deuterone, 82
diffusione
di una perturbazione, 152
pacchetti d’onda Gaussiani, 153
diffusione onde e.m.
scattering Compton, 58
scattering elettroni legati, 192
dispersione
pacchetti d’onda e.m., 151
adsorbimento, 100—102
angolo di Brewster, 140
approssimazione di onda rotante, 107
armonica sferica
vettoriale, 137
B Oersted, 105
Bloch F. [1905-1983], 107
bottiglia magnetica, 185
Bragg, W.H. [1862—1942], 56
campo irradiato da un elettrone
in un’orbità generica, 203
in un’orbita circolare, 223
campo locale, 92, 93, 99
campo magnetico H
A/m MKS Gauss CGS, 75
campo magnetico quadrupolare, 103
campo/i
4-tensore del campo e.m., 47
4-vettore d’onda, 117
e.m. in prossimità di un fuoco, 122, 194
integrale di Luneburg-Debye, 194
espansione in modi, 128
irradiato da un elettrone, 219, 227
locale, 143
onde piane, 47, 117
Chu S. [1948, ], 196
Cohen-Tannoudji C. [1933, ], 196
Compton A.H. [1892—1962], 57
connessione affine
coefficienti di connessione
simboli di Christoffel, 18, 69
derivata covariante, 18
Levi-Civita, 18, 68
coordinate
cilindriche, 19
ottiche, 122
sferoidali, 20
spazio-tempo, 37
corpo nero, 131
costante
di propagazione
modo e.m., 129
di schermaggio, 146
di Verdet, 150
dielettrica
di Lindhard, 146
metalli, 146
costante di Madelung, 93
costante di scambio, 105
effetto
Compton, 57
Mossbauer, 59
Sagnac, 68
Stewart-Tolman, 205
energia
elettromagnetica, 52
magnetica, 61
elettrone in movimento, 61
equazione
BMT, 61
d’onda
di Helmholtz, 128, 129
del moto
dello spin elettronico, 60
spin elettronico, 61
del moto BMT, 60
dell’energia di un elettrone in un campo e.m.,
155, 188
dell’iconale, 30
di Bessel, 31
di Bethe-Bloch, 218
di Bloch, 107
di continuità, 47
di Fresnel, 148
di Helmholtz, 118
di Laplace, 94
di Mathieu, 34
di Poisson, 89
di precessione dello spin, 61
di una geodesica, 69
Dehmelt H. G. [1922, ], 196
densità
di carica esterna ed indotta, 47
di energia elettromagnetica
mezzi dispersivi, 116
235
236
equazioni
di Bloch, 106
di Maxwell-Schiff, 63
equazioni di Bloch, 109
espansione
interazione coulombiana, 84
espansione campo e.m.
in modi, 128
int. coulombiana in armoniche sferiche, 76
esperimento
di Mossbauer, 59
Faraday M. [1791-1867], 151
fattore di Landè
nucleare, 106
Feynman R.P. [1918—1988], 191
fibre ottiche, 127
flusso induzione magnetica Weber, 75
formula
del boost, 43
di Abraham-Lorentz, 191
di Gaunt, 85
di Larmor, 209
formula della rotazione, 41
formule
di Fresnel, 140
forza
ponderomotiva, 192
particella polarizzabile, 193
forze
di van der Waals, 100
di Van der Walls, 100, 102
Fourier J.B.[1768 — 1830], 23
frequenza
di ciclotrone, 61
di Larmor, 61
di plasma, 139, 141
di Rabi, 106
funzione
di Airy, 29
di Bessel, 31
modificata, 34
di Green
equazione di diffusione, 152
scalare, 118, 120
tensoriale, 121
di Green scalare 2D, 218
di Hankel, 31
di Struve, 32
funzioni
di Struve, 211
funzioni d’onda
oscillatore armonico, 131
gauge di Lorentz, 49
geodesica, 69
nulla, 69
H Gauss, 105
Hansch T. [1941, ], 196
Helmholtz H [1821, 1894], 196
identità di Jacobi, 32, 223, 227, 233
idrogeno
moto a rosetta di Sommerfeld, 156
indice di rifrazione
metalli, 139
induzione magnetica B
Indice analitico
Tesla MKS Oersted CGS, 75
integrale
di Coulomb j’, 88
di Luneburg-Debye, 194
di sovrapposizione, 87
integrale di Coulomb, 88
elettroni conduzione, 90
molecole, 87
integrale di diffrazione
di Luneburg-Debye, 122
integrale di scambio, 88
elettroni conduzione, 90
molecole, 87
integrali
diretti (coulombiani), 84
integrali del moto
elettrone relativistico in un potenziale coulombiano, 158
integrali interazioni densità elettroniche
molecole, 87, 88
interazione radiazione-materia
laser cooling, 196
pinzette ottiche, 194
pressione di radiazione, 195
reazione di radiazione, 195
Kramers H.A. [1894-1952], 147
Kronig R. de L. [1904-], 147
lagrangiana
metrica di Schwarzschild, 71
relativistica elettrone, 156
lenti elettrostatiche, 159
cilindri coassiali, 162
lenti magnetiche, 165
Glazer, 170
lunghezza focale, 169
quadrupolo, 170
spira di corrente, 169
Lorentz H. A. [1853—1928], 38, 191
luce di sincrotrone
Elettra, 230
ESRF, 230
M emu/cc, 105
magnetismo
magnetizzazione, 46
magnetizzazione
saturazione, 105
magnetizzazione M
Tesla MKS emu/cc CGS, 75
magnetizzazione M
Tesla MKS emu/cc CGS, 75
magnetone di Bohr
nucleare, 106
materiali
fluoruro di bario (BaF2), 57
grafite (C), 57
ioduro di cesio (CsI), 57
molibdeno (Mo), 57
matrici di Pauli, 41
proprietà, 41
metodo
del cammino di massima pendenza, 27
dello steepest descent, 33
WKB, 29
metodo di integrazione
SP, 27
Indice analitico
metrica
di Schwarzschild, 70
microscopio
scansione campo vicino (SNOM), 123
modello
di Fermi-Thomas, 146
modi
campo e.m., 128, 129, 150
momento
di quadrupolo, 82
momento magnetico
elettrone con movimento a spirale, 183
Mossbauer R.L. [1929—...], 59
movimento precessione
nucleo, 106
multipoli elettrici e magnetici, 75
onde piane
espansione in onde sferiche, 32
ondulatore, 175
radiazione emessa, 231
traiettoria, 176
traiettorie elettroni, 175
operatore
di creazione e distruzione
bosoni, 131
oscillazioni di plasma, 139
pacchetti d’onda
e.m., 151
parametro di saturazione, 195
permeabilità magnetica Henry/m, 75
Phillips W. D. [1948, ], 196
pinzette ottiche, 194
polarizzabilità
atomica, 143
molecole, 142, 144
polarizzazione, 46
potenza irradiata
distribuzione di correnti, 208, 233
potenza irradiata da un elettrone, 209
potenziale
vettore
elettrone in moto uniforme, 218
potenziale medio di eccitazione, 218
potenziali e.m.
di Liénard-Wiechert, 202
potenziale 4-vettore, 46
precessione
di Thomas, 45
principio
indeterminazione di Heisenberg
campi e.m., 131
237
di ciclotrone, 230
potenza irradiata, 230
di ondulatore
potenza irradiata, 231
di sincrotrone, 227
densità spettrale, 227
densità spettrale potenza totale, 229
frequenza critica, 224
potenza irradiata totale, 229
sincrotrone
frequenza critica, 224
radiazione di sincrotrone, 230
raffreddamento
laser cooling, 196
raggi, 30
raggio
di Bohr, 58
di Compton, 58
raggio nucleo, 82
rapporto giromagnetico, 61
nucleo, 106
relazioni costitutive
indice di rifrazione, 117
mezzi girotropici, 151
relazioni di dispersione
di Kramers-Kronig, 147
fibre ottiche, 127
modi e.m., 129
riferimento proprio, 44
rilassamento
spin-reticolo, 107
spin-spin, 107
tempo rilassamento longitudinale
spin-reticolo, 107
tempo rilassamento trasversale
spin-spin, 107
quadricorrente, 47
tensore campo F, 47
tensore campo G, 47
vettore potenziale e.m., 117
vettore velocità, 38
quantizzazione
campo e.m., 131
oscillatori, 131
saddle point, 27
scatterring
di Rayleigh, 136
sezione d’urto
sfera, 136
simbolo 6j, 80
simmetrie
inversione temporale, 150
sistema riferimento
proprio, 38
tempo proprio, 38
sorgenti e.m., 46
specchio magnetico, 185
spettro
diffusione da elettroni legati, 192
radiazione di sincrotrone, 227
spettroscopia
gamma (Mossbauer), 59
meccanismi allargamento righe, 59
spin
4-vettore, 60
stabilità
equazione di Mathieu, 34
struttura iperfine
momento di quadrupolo del nucleo, 82
suscettività NMR, 108
radiazione
Cerenkov, 218, 219
deensità spettrale, 219
di bremsstrahlung, 221
tensore
metrico, 18
sforzi elettromagnetici, 52
teorema
238
addiizione armoniche sferiche, 77
di Poynting dominio frequenza, 116
di reciprocità, 115
energia e.m. per mezzi dispersivi , 116
Thomas L.H. [1903—1992], 45, 60
trasformate
di Fourier, 23
di Fourier-Bessel, 23
di Hilbert, 24
trasformazioni di Lorentz, 38
proprie, 38
unità cgs, 75
unità di misura
sistema SI, 46
unita’ di misura
Fermi, 82
velocitá
di gruppo, 152
vettore
d’onda, 117
di Poynting, 52
di Rabi, 107
induzione magnetica B, 46
magnetizzazione M, 76
Indice analitico